Download as pdf or txt
Download as pdf or txt
You are on page 1of 405

SIGNIFICANT AMENDMENTS FOR NOVEMBER 2021 EXAMINATION

PAPER 6D: ECONOMIC LAWS

THE FOREIGN EXCHANGE MANAGEMENT ACT, 1999


1. Amendment in the Foreign Exchange Management (Export of Goods & Services)
Regulations, 2015
Vide Notification No. FEMA 23(R)/(4)/2021-RB , dated January 08, 2021, the Foreign
Exchange Management (Export of Goods and Services) (Amendment) Regulations,
2021 has been enacted .
According to the amended Regulation, in exercise of the powers conferred by clause (a)
of sub-section (1), sub-section (3) of section 7 and clause (b) of sub-section (2) of section
47 of the Foreign Exchange Management Act, 1999, the Reserve Bank of India makes the
following amendments in the Foreign Exchange Management (Export of Goods & Services)
Regulations, 2015 through the enforcement of the Foreign Exchange Management (Export
of Goods and Services) (Amendment) Regulations, 2021.
In the Principal Regulations, in regulation 4, for sub-regulation (ea), the following shall
be substituted, namely:-
“(ea) re-export of leased aircraft/helicopter and/or engines/auxiliary power units (APUs),
either completely or in partially knocked down condition repossessed by overseas lessor
and duly de-registered by the Directorate General of Civil Aviation (DGCA) on the request
of Irrevocable Deregistration and Export Request Authorisation (IDERA) holder under
‘Cape Town Convention’ or any other termination or cancellation of the lease agreement
between the lessor and lessee subject to permission by DGCA/Ministry of Civil Aviation for
such export/s.”
2. The Prevention of Money Laundering Act, 2002
A. Vide Notification G.S.R. 798(E) [F. NO. P-12011/14/2020-ES CELL-DOR], Dated
28-12-2020, in exercise of the powers conferred by sub-clause (iv) of clause (sa) of
sub-section (1) of section 2 of the Prevention of Money-laundering Act, 2002 , the
Central Government hereby rescinds the notification of the Government of India,
Ministry of Finance, Department of Revenue, No. 8/2017, dated 15 November, 2017,
published in the Gazette of India, Part II, Section 3, Sub-section (ii), extraordinary,
vide GSR 1423 (E) dated the 16 November 2017, except as respects things done or
omitted to done before such recession and notifies the "Real Estate Agents", as a

© The Institute of Chartered Accountants of India


person engaged in providing services in relation to sale or purchase of real
estate and having annual turnover of Rupees twenty lakhs or above, as
"persons carrying on designated businesses or professions".
B. Vide Notification G.S.R. 799(E) [F. NO. P-12011/14/2020-ES CELL-DOR], Dated
28-12-2020, in exercise of the powers conferred by sub-clause (iv) of clause (sa) of
sub-section (1) of section 2 of the Prevention of Money-laundering Act, 2002, the
Central Government hereby notifies the dealers in precious metals, precious
stones as persons carrying on designated businesses or professions - if they
engage in any cash transactions with a customer equal to or above Rupees ten
lakhs, carried out in a single operation or in several operations that appear to
be linked.
C. Vide Notification G.S.R. 59(E) [F. NO. P-12011/24/2017-ES CELL-DOR-PART(1)],
Dated 28-1-2021, in exercise of the powers conferred by sub-section (1) of section
11A of the Prevention of Money-laundering Act, 2002 , the Central Government on
being satisfied that the reporting entities mentioned below comply with standards of
privacy and security under the Aadhaar (Targeted Delivery of Financial and Other
Subsidies, Benefits and Services) Act, 2016 (18 of 2016) and it is necessary and
expedient to do so, and after consultation with the Unique Identification Authority of
India established under sub-section (1) of section 11 of the Aadhaar (Targeted
Delivery of Financial and Other Subsidies, Benefits and Services) Act, 2016 and the
regulatory authority, namely the Reserve Bank of India, hereby notifies the
reporting entity specified below to undertake Aadhaar authentication service of
the Unique Identification Authority of India under section 11A of the Prevention
of Money-laundering Act, 2002, namely:—
"National Payments Corporation of India."
The Insolvency & Bankruptcy Code, 2016
(A) The Insolvency and Bankruptcy Code (Amendment) Ordinance, 2021
The President promulgated the Insolvency and Bankruptcy Code (Amendment)
Ordinance, 2021 on 4th April 2021. The Cabinet had approved on 31st March
2021 the proposal to make amendments in the Insolvency and Bankruptcy Code,
2016 (Code), through the Insolvency and Bankruptcy Code (Amendment)
Ordinance, 2021.
The amendments aims to provide an efficient alternative insolvency resolution
framework for corporate persons classified as micro, small and medium
enterprises (MSMEs) under the Code, for ensuring quicker, cost-effective and
value maximising outcomes for all the stakeholders, in a manner which is least

© The Institute of Chartered Accountants of India


disruptive to the continuity of MSMEs businesses and which preserves jobs. The
initiative is based on a trust model and the amendments honour the honest
MSME owners by trying to ensure that the resolution happens and the company
remains with them.
It is expected that the incorporation of Pre-Packaged insolvency resolution
process for MSMEs in the Code will alleviate the distress faced by MSMEs due
to the impact of the pandemic & the unique nature of their business, duly
recognizing their importance in the economy. It provides an efficient alternative
insolvency resolution framework for corporate persons classified as MSMEs for
timely, efficient & cost-effective resolution of distress thereby ensuring positive
signal to debt market, employment preservation, ease of doing business and
preservation of enterprise capital. Other expected impact and benefits of the
amendment in Code are lesser burden on Adjudicating Authority, assured
continuity of business operations for corporate debtor (CD), less process costs
& maximum assets realization for financial creditors (FC) and assurance of
continued business relation with CD and rights protection for operational
Creditors (OC).
The Amendment Ordinance seeks to amend sections such as 4, 5, 11, 33, 34,
61, 65, 77, 208, 239, 240 & insert new sections such as 11A, 67A, 77A and a
new chapter as IIIA on Pre-Packaged insolvency resolution process for MSMEs
in the Code based on recommendations made by the Insolvency Law Committee
(ILC).
Details of the amendments are given at under:
1. This Ordinance may be called the Insolvency and Bankruptcy Code
(Amendment) Ordinance, 2021. It shall come into force at once.
2. Amendment of Section 4- Application of this Part II of the Code
After the given proviso in the said section, the following proviso shall be
inserted, namely:—
“Provided further that the Central Government may, by notification, specify
such minimum amount of default of higher value, which shall not be more
than one crore rupees, for matters relating to the prepackaged insolvency
resolution process of corporate debtors under Chapter III-A.”.
3. Amendment of Section 5- Definitions covered under this part of the
Code.
(i) after clause (2), the following clause shall be inserted, namely:—

© The Institute of Chartered Accountants of India


‘(2A) “base resolution plan” means a resolution plan provided by
the corporate debtor under clause (c) of ub-section (4) of section
54A;’;
(ii) in clause (5), in sub-clause (b), after the words “corporate insolvency
resolution process”, the words “or the pre-packaged insolvency
resolution process, as the case may be,” shall be inserted;
(iii) in clause (11), after the words “corporate insolvency resolution
process”, the words “or prepackaged insolvency resolution process,
as the case may be” shall be inserted;
(iv) in clause (15), after the words, “process period”, the words “or by the
corporate debtor during the pre-packaged insolvency resolution
process period, as the case may be,” shall be inserted;
(v) in clause (19), after the words “for the purposes of”, the words and
figures “Chapter VI and” shall be inserted;
(vi) after clause (23), the following clauses shall be inserted, namely: —
(23A) “preliminary information memorandum” means a memorandum
submitted by the corporate debtor under clause (b) of sub-section (1)
of section 54G;
(23B)“pre-packaged insolvency commencement date” means the
date of admission of an application for initiating the pre-packaged
insolvency resolution process by the Adjudicating Authority under
clause (a) of sub-section (4) of section 54C;
(23C) “pre-packaged insolvency resolution process costs” means—
(a) the amount of any interim finance and the costs incurred in
raising such finance;
(b) the fees payable to any person acting as a resolution
professional and any expenses incurred by him for conducting the
pre-packaged insolvency resolution process during the prepackaged
insolvency resolution process period, subject to sub-section (6) of
section 54F;
(c) any costs incurred by the resolution professional in running the
business of the corporate debtor as a going concern pursuant to an
order under sub-section (2) of section 54J;

© The Institute of Chartered Accountants of India


(d) any costs incurred at the expense of the Government to facilitate
the pre-packaged insolvency resolution process; and
(e) any other costs as may be specified;
(23D) “pre-packaged insolvency resolution process period”
means the period beginning from the pre-packaged
insolvency commencement date and ending on the date on
which an order under sub-section (1) of section 54L, or sub-
section (1) of section 54N, or sub-section (2) of section 54-
O, as the case may be, is passed by the Adjudicating
Authority;’;
(vii) in clause (25), after the words, brackets and figures “of sub-section
(2) of section 25”, the words, figures and letter “or pursuant to section
54K, as the case may be” shall be inserted;
(viii) in clause (27), after the words “corporate insolvency resolution
process”, the words “or the prepackaged insolvency resolution
process (PPIRP), as the case may be,” shall be inserted.
4. Amendment of section 11- Persons not entitled to make application.
(i) in clause (a), after the words “corporate insolvency resolution process”,
the words “or a prepackaged insolvency resolution process” shall be
inserted;
(ii) after clause (a), the following clause shall be inserted, namely:–
“(aa) a financial creditor or an operational creditor of a corporate
debtor undergoing a prepackaged insolvency resolution process; or”;
(iii) after clause (b), the following clause shall be inserted, namely:—
“(ba) a corporate debtor in respect of whom a resolution plan has
been approved under Chapter III-A, twelve months preceding the date
of making of the application; or”.
5. After section 11 of the principal Act, following new section 11A shall
be inserted,.
“11A. (1) Where an application filed under section 54C is pending, the
Adjudicating Authority shall pass applications under section an order to
admit or reject such application, before 54C and under considering any
application filed under section 7 or section 7 or section 9 or section 10

© The Institute of Chartered Accountants of India


during the pendency of such section 9 or application under section 54C, in
respect of the same section 10. corporate debtor.
(2) Where an application under section 54C is filed within fourteen days of
filing of any application under section 7 or section 9 or section 10, which is
pending, in respect of the same corporate debtor, then, notwithstanding
anything contained in sections 7, 9 and 10, the Adjudicating Authority shall
first dispose of the application under section 54C.
(3) Where an application under section 54C is filed after fourteen days of
the filing of any application under section 7 or section 9 or section 10, in
respect of the same corporate debtor, the Adjudicating Authority shall first
dispose of the application under sections 7, 9 or 10.
(4) The provisions of this section shall not apply where an application under
section 7 or section 9 or section 10 is filed and pending as on the date of
the commencement of the Insolvency and Bankruptcy Code (Amendment)
Ordinance, 2021.”.
6. In section 33 of the principal Act, which deals with the initiation of
liquidation, in sub-section (3), after the words, “approved by the
Adjudicating Authority”, the words, figures, brackets and letter “under
section 31 or under sub-section (1) of section 54L,” shall be inserted.
7. Amendment of section 34- Appointment of liquidator and fee to be
paid.
In section 34 of the principal Act, in sub-section (1), after the words and
figures, “under Chapter II”, the words, figures and letter “or for the pre -
packaged insolvency resolution process under Chapter III-A” shall be
inserted.
8. After Chapter III of the principal Act, the following Chapter III-A, shall be
inserted, namely:—
CHAPTER III-A
PRE-PACKAGED INSOLVENCY RESOLUTION PROCESS
Corporate debtors eligible for pre-packaged insolvency resolution process.
54 A. (1) An application for initiating pre-packaged insolvency resolution process may be made
in respect of a corporate debtor classified as a micro, small or medium enterprise under sub-
section (1) of section 7 of the Micro, Small and Medium Enterprises Development 27 of 2006.
Act, 2006.

© The Institute of Chartered Accountants of India


(2) Without prejudice to sub-section (1), an application for initiating pre-packaged insolvency
resolution process may be made in respect of a corporate debtor, who commits a default referred
to in section 4, subject to the following conditions, that––
(a) it has not undergone pre-packaged insolvency resolution process or completed corporate
insolvency resolution process, as the case may be, during the period of three years preceding
the initiation date;
(b) it is not undergoing a corporate insolvency resolution process;
(c) no order requiring it to be liquidated is passed under section 33;
(d) it is eligible to submit a resolution plan under section 29A;
(e) the financial creditors of the corporate debtor, not being its related parties, representing
such number and such manner as may be specified, have proposed the name of the insolvency
professional to be appointed as resolution professional for conducting the pre-packaged
insolvency resolution process of the corporate debtor, and the financial creditors of the
corporate debtor, not being its related parties, representing not less than sixty -six per cent. in
value of the financial debt due to such creditors, have approved such proposal in such form as
may be specified:
Provided that where a corporate debtor does not have any financial creditors, not being its
related parties, the proposal and approval under this clause shall be provided by such persons
as may be specified;
(f) the majority of the directors or partners of the corporate debtor, as the case may be, have
made a declaration, in such form as may be specified, stating, inter alia,—
(i) that the corporate debtor shall file an application for initiating pre-packaged
insolvency resolution process within a definite time period not exceeding ninety days;
(ii) that the pre-packaged insolvency resolution process is not being initiated to
defraud any person; and
(iii) the name of the insolvency professional proposed and approved to be appointed as
resolution professional under clause (e);
(g) the members of the corporate debtor have passed a special resolution, or at least three -
fourth of the total number of partners, as the case may be, of the corporate debtor have passed
a resolution, approving the filing of an application for initiating pre-packaged insolvency
resolution process.
(3) The corporate debtor shall obtain an approval from its financial creditors, not being its
related parties, representing not less than sixty-six per cent. in value of the financial debt due

© The Institute of Chartered Accountants of India


to such creditors, for the filing of an application for initiating pre-packaged insolvency resolution
process, in such form as may be specified:
Provided that where a corporate debtor does not have any financial creditors, not being its
related parties, the approval under this sub-section shall be provided by such persons as may
be specified.
(4) Prior to seeking approval from financial creditors under sub-section (3), the corporate
debtor shall provide such financial creditors with —
(a) the declaration referred to in clause (f) of sub- section (2);
(b) the special resolution or resolution referred to in clause (g) of sub- section (2);
(c) a base resolution plan which conforms to the requirements referred to in section 54K,
and such other conditions as m ay be specified; and
(d) such other information and documents as may be specified.

Duties of resolution professional before initiation of pre-packaged insolvency resolution


process.
54B. (1) The insolvency professional, proposed to be appointed as the resolution professional,
shall have the following duties commencing from the date of the approval under clause (e) of
sub-section (2) of section 54A, namely:—
(a) prepare a report in such form as may be specified, confirming whether the corporate
debtor meets the requirements of section 54A, and the base resolution plan conforms
to the requirements referred to in clause (c) of sub-section (4) of section 54A;
(b) file such reports and other documents, with the Board, as may be specified; and
(c) perform such other duties as may be specified.
(2) The duties of the insolvency professional under sub-section (1) shall cease, if, —
(a) the corporate debtor fails to file an application for initiating pre-packaged insolvency
resolution process within the time period as stated under the declaration referred to
in clause (f) of subsection (2) of section 54A; or
(b) the application for initiating pre-packaged insolvency resolution process is admitted
or rejected by the Adjudicating Authority, as the case may be.
(3) The fees payable to the insolvency professional in relation to the duties perfo rmed under
sub-section (1) shall be determined and borne in such manner as may be specified and such

© The Institute of Chartered Accountants of India


fees shall form part of the prepackaged insolvency resolution process costs, if the application
for initiation of pre-packaged insolvency resolution process is admitted.
Application to initiate pre-packaged insolvency resolution process.
54C. (1) Where a corporate debtor meets the requirements of section 54A, a corporate applicant
thereof may file an application with the Adjudicating insolvency Authority for initiating pre -
packaged insolvency resolution resolution process.
(2) The application under sub-section (1) shall be filed in such form, containing such
particulars, in such manner and accompanied with such fee as may be prescribed.
(3) The corporate applicant shall, along with the application, furnish—
(a) the declaration, special resolution or resolution, as the case may be, and the approval
of financial creditors for initiating pre-packaged insolvency resolution process in terms
of section 54A;
(b) the name and written consent, in such form as may be specified, of the insolvency
professional proposed to be appointed as resolution professional, as approved under
clause (e) of sub-section (2) of section 54A, and his report as referred to in clause (a)
of sub-section (1) of section 54B;
(c) a declaration regarding the existence of any transactions of the corporate debtor that
may be within the scope of provisions in respect of avoidance of transactions under
Chapter III or fraudulent or wrongful trading under Chapter VI, in such form as may
be specified;
(d) information relating to books of account of the corporate debtor and such other
documents relating to such period as may be specified.
(4) The Adjudicating Authority shall, within a period of fourteen days of the receipt of the
application, by an order,––
(a) admit the application, if it is complete; or
(b) reject the application, if it is incomplete:
Provided that the Adjudicating Authority shall, before rejecting an application, give
notice to the applicant to rectify the defect in the application within seven days from
the date of receipt of such notice from the Adjudicating Authority.
(5) The pre-packaged insolvency resolution process shall commence from the date of
admission of the application under clause (a) of sub-section (4).

© The Institute of Chartered Accountants of India


Time-limit for completion of pre-packaged insolvency resolution process.
54D. (1) The pre-packaged insolvency resolution process shall be completed within a period of
one hundred and twenty days from the pre-packaged insolvency commencement date.
(2) Without prejudice to sub-section (1), the resolution professional shall submit the resolution
plan, as approved by the committee of creditors, to the Adjudicating Authority under sub-section
(4) or subsection (12), as the case may be, of section 54K, within a period of ninety days from
the pre-packaged insolvency commencement date.
(3) Where no resolution plan is approved by the committee of creditors within the time period
referred to in sub-section (2), the resolution professional shall, on the day after the expiry of
such time period, file an application with the Adjudicating Authori ty for termination of the pre-
packaged insolvency resolution process in such form and manner as may be
specified.
Declaration of moratorium and public announcement during prepackaged insolvency
resolution process
54E. (1) The Adjudicating Authority shall, on the pre-packaged insolvency commencement date,
along with the order of admission under section 54C —
(a) declare a moratorium for the purposes referred to in sub-section (1) read with sub-section
(3) of section 14, which shall, mutatis mutandis apply, to the proceedings under this Chapter;
(b) appoint a resolution professional —
(i) as named in the application, if no disciplinary proceeding is pending against him; or
(ii) based on the recommendation made by the Board, if any disciplinary proceeding is
pending against the insolvency professional named in the application.
(c) cause a public announcement of the initiation of the pre-packaged insolvency resolution
process to be made by the resolution professional, in such form and manner as may be
specified, immediately after his appointment.
(2) The order of moratorium shall have effect from the date of such order till the date on which
the prepackaged insolvency resolution process period comes to an end.
Duties and powers of resolution professional during pre-packaged insolvency resolution
process.
54F. (1) The resolution professional shall conduct the pre-packaged insolvency resolution
process of a corporate debtor during the pre-packaged insolvency resolution process period.
(2) The resolution professional shall perform the following duties, namely: —

© The Institute of Chartered Accountants of India


(a) confirm the list of claims submitted by the corporate debtor under section 54G, in
such manner as may be specified;
(b) inform creditors regarding their claims as confirmed under clause (a), in such manner
as may be specified;
(c) maintain an updated list of claims, in such manner as may be specified;
(d) monitor management of the affairs of the corporate debtor;
(e) inform the committee of creditors in the event of breach of any of the obligations of
the Board of Directors or partners, as the case may be, of the corporate debtor, under
the provisions of this Chapter and the rules and regulations made thereunder;
(f) constitute the committee of creditors and convene and attend all its meetings;
(g) prepare the information memorandum on the basis of the preliminary information
memorandum submitted under section 54G and any other relevant information, in
such form and manner as may be specified;
(h) file applications for avoidance of transactions under Chapter III or fraudulent or
wrongful trading under Chapter VI, if any; and
(i) such other duties as may be specified.
(3) The resolution professional shall exercise the following powers, namely: —
(a) access all books of accounts, records and information available with the corporate
debtor;
(b) access the electronic records of the corporate
debtor from an information utility having financial information of the corporate debtor;
(c) access the books of accounts, records and
other relevant documents of the corporate debtor available with Government
authorities, statutory auditors, accountants and such other persons as may be
specified;
(d) attend meetings of members, Board of
Directors and committee of directors, or partners, as the case may be, of the
corporate debtor;
(e) appoint accountants, legal or other professionals in such manner as may be
specified;
(f) collect all information relating to the assets,

© The Institute of Chartered Accountants of India


finances and operations of the corporate debtor for determining the financial position
of the corporate debtor and the existence of any transactions that may be within the
scope of provisions relating to avoidance of transactions under Chapter III or
fraudulent or wrongful trading under Chapter VI, including information relating to —
(i) business operations for the previous two
years from the date of pre-packaged insolvency commencement date;
(ii) financial and operational payments for the previous two years from the date of
prepackaged insolvency commencement date;
(iii) list of assets and liabilities as on the initiation date; and
(iv) such other matters as may be specified;
(g) take such other actions in such manner as may be specified.
(4) From the date of appointment of the resolution professional, the financial institutions
maintaining accounts of the corporate debtor shall furnish all information relating to the
corporate debtor available with them to the resolution professional, as and when required by
him.
(5) The personnel of the corporate debtor, its promoters and any other person associated with
the management of the corporate debtor shall extend all assistance and cooperation to the
resolution professional as may be required by him to perform his duties and exercise his powers,
and for such purposes, the provisions of sub-sections (2) and (3) of section 19 shall, mutatis
mutandis apply, in relation to the proceedings under this Chapter.
(6) The fees of the resolution professional and any expenses incurred by him for conducting
the prepackaged insolvency resolution process shall be determined in such manner as may be
specified:
Provided that the committee of creditors may impose limits and conditions on such fees and
expenses:
Provided further that the fees and expenses for the period prior to the constitution of the
committee of creditors shall be subject to ratification by it.
(7) The fees and expenses referred to in sub-section (6) shall be borne in such manner as
may be specified.
List of claims and preliminary information memorandum.
54G. (1) The corporate debtor shall, within two days of the pre-packaged insolvency
commencement date, submit to the resolution professional the following information, updated
as on that date, in such form and manner as may be specified, namely: —

© The Institute of Chartered Accountants of India


(a) a list of claims, along with details of the respective creditors, their security interests
and guarantees, if any; and
(b) a preliminary information memorandum containing information relevant for
formulating a resolution plan.
(2) Where any person has sustained any loss or damage as a consequence of the omission
of any material information or inclusion of any misleading information in the list of claims or the
preliminary information memorandum submitted by the corporate debtor, every person who —
(a) is a promoter or director or partner of the corporate debtor, as the case may be, at
the time of submission of the list of claims or the preliminary information memorandum
by the corporate debtor; or
(b) has authorised the submission of the list of claims or the preliminary information
memorandum by the corporate debtor,
shall, without prejudice to section 77A, be liable to pay compensation to every person
who has sustained such loss or damage.
(3) No person shall be liable under sub-section (2), if the list of claims or the preliminary
information memorandum was submitted by the corporate debtor without his knowledge or
consent.
(4) Subject to section 54E, any person, who sustained any loss or damage as a consequence
of omission of material information or inclusion of any misleading information in the list of claims
or the preliminary information memorandum shall be entitled to move a court having jurisdiction
for seeking compensation for such loss or damage.
Management of affairs of corporate debtor
54H. During the pre-packaged insolvency resolution process period,—
(a) the management of the affairs of the corporate debtor shall continue to vest in the Board
of Directors or the partners, as the case may be, of the corporate debtor, subject to such
conditions as may be specified;
(b) the Board of Directors or the partners, as the case may be, of the corporate debtor, shall
make every endeavour to protect and preserve the value of the property of the corporate
debtor, and manage its operations as a going concern; and
(c) the promoters, members, personnel and partners, as the case may be, of the corporate
debtor, shall exercise and discharge their contractual or statutory rights and obligations in
relation to the corporate debtor, subject to the provisions of this Chapter and such other
conditions and restrictions as may be prescribed.

© The Institute of Chartered Accountants of India


Committee of creditors
54-I. (1) The resolution professional shall, within seven days of the pre-packaged insolvency
commencement date, constitute a committee of creditors, based on the list of claims confirmed
under clause (a) of sub-section (2) of section 54F:
Provided that the composition of the committee of creditors shall be altered on the basis of the
updated list of claims, in such manner as may be specified, and any such alteration shall not
affect the validity of any past decision of the committee of creditors.
(2) The first meeting of the committee of creditors shall be held within seven days of the
constitution of the committee of creditors.
(3) Provisions of section 21, except sub-section (1) thereof, shall, mutatis mutandis apply, in
relation to the committee of creditors under this Chapter:
Provided that for the purposes of this sub-section, references to the “resolution professional”
under subsections (9) and (10) of section 21, shall be construed as references to “corporate
debtor or the resolution professional”.
Vesting management of corporate debtor with resolution professional
54J. (1) Where the committee of creditors, at any time during the pre-packaged insolvency
resolution corporate process, by a vote of not less than sixty-six per cent. of the voting shares,
resolves to vest the management of the corporate debtor with the resolution professional, the
resolution professional shall make an application for this purpose to the Adjudicating Authority,
in such form and manner as may be specified.
(2) On an application made under sub-section (1), if the Adjudicating Authority is of the opinion
that
during the pre-packaged insolvency resolution process—
(a) the affairs of the corporate debtor have been conducted in a fraudulent manner; or
(b) there has been gross mismanagement of the affairs of the corporate debtor,
it shall pass an order vesting the management of the corporate debtor with the
resolution professional.
(3) Notwithstanding anything to the contrary contained in this Chapter, the provisions of—
(a) sub-sections (2) and (2A) of section 14;
(b) section 17;
(c) clauses (e) to (g) of section 18;
(d) sections 19 and 20;

© The Institute of Chartered Accountants of India


(e) sub-section (1) of section 25;
(f) clauses (a) to (c) and clause (k) of sub- section (2) of section 25; and
(g) section 28,
shall, mutatis mutandis apply, to the proceedings under this Chapter, from the date
of the order under subsection (2), until the pre-packaged insolvency resolution
process period comes to an end.
Consideration and approval of resolution plan
54K. (1) The corporate debtor shall submit the base resolution plan, referred to in clause (c) of
sub-section (4) of section 54A, to the resolution professional within two days of the pre-
packaged insolvency commencement date, and the resolution professional shall present it to
the committee of creditors.
(2) The committee of creditors may provide the corporate debtor an opportunity to revise the
base resolution plan prior to its approval under sub-section (4) or invitation of prospective
resolution applicants under sub-section (5), as the case may be.
(3) The resolution plans and the base resolution plan, submitted under this section shall
conform to the requirements referred to in sub-sections (1) and (2) of section 30, and the
provisions of sub-sections (1), (2) and (5) of section 30 shall, mutatis mutandis apply, to the
proceedings under this Chapter.
(4) The committee of creditors may approve the base resolution plan for submission to the
Adjudicating Authority if it does not impair any claims owed by the corporate debtor to the
operational creditors.
(5) Where —
(a) the committee of creditors does not approve the base resolution plan under sub -
section (4); or
(b) the base resolution plan impairs any claims owed by the corporate debtor to the
operational creditors, the resolution professional shall invite prospective
resolution applicants to submit a resolution plan or plans, to compete with the base
resolution plan, in such manner as may be specified.
(6) The resolution applicants submitting resolution plans pursuant to invitation under sub -
section (5), shall fulfil such criteria as may be laid down by the resolution professional with the
approval of the committee of creditors, having regard to the complexity and scale of operations
of the business of the corporate debtor and such other conditions as may be specified.
(7) The resolution professional shall provide to the resolution applicants, —

© The Institute of Chartered Accountants of India


(a) the basis for evaluation of resolution plans
for the purposes of sub-section (9), as approved by the committee of creditors subject
to such conditions as may be specified; and
(b) the relevant information referred to in section 29, which shall, mutatis mutandis apply,
to the proceedings under this Chapter, in such manner as may be specified.
(8) The resolution professional shall present to the committee of creditors, for its evaluation,
resolution plans which conform to the requirements referred to in sub-section (2) of section
30.
(9) The committee of creditors shall evaluate the resolution plans presented by the resolution
professional and select a resolution plan from amongst them.
(10) Where, on the basis of such criteria as may be laid down by it, the committee of creditors
decides that the resolution plan selected under sub-section (9) is significantly better than
the base resolution plan, such resolution plan may be selected for approval under
subsection (12):
Provided that the criteria laid down by the committee of creditors under this sub -section
shall be subject to such conditions as may be specified.
(11) Where the resolution plan selected under sub-section (9) is not considered for approval or
does not fulfil the requirements of sub-section (10), it shall compete with the base
resolution plan, in such manner and subject to such conditions as may be specified, and
one of them shall be selected for approval under subsection (12).
(12) The resolution plan selected for approval under sub-section (10) or sub-section (11), as
the case may be, may be approved by the committee of creditors for submission to the
Adjudicating Authority:
Provided that where the resolution plan selected for approval under sub -section (11) is
not approved by the committee of creditors, the resolution professional shall file an
application for termination of the pre-packaged insolvency resolution process in such form
and manner as may be specified.
(13) The approval of the resolution plan under sub-section (4) or sub-section (12), as the case
may be, by the committee of creditors, shall be by a vote of not less than sixty-six per cent.
of the voting shares, after considering its feasibility and viability, the manner of distribution
proposed, taking into account the order of priority amongst creditors as laid down in sub -
section (1) of section 53, including the priority and value of the security interest of a secured
creditor and such other requirements as may be specified.

© The Institute of Chartered Accountants of India


(14) While considering the feasibility and viability of a resolution plan, where the resolution
plan submitted by the corporate debtor provides for impairment of any claims owed by the
corporate debtor, the committee of creditors may require the promoters of the corporate
debtor to dilute their shareholding or voting or control rights in the corporate debtor:
Provided that where the resolution plan does not provide for such dilution, the committee
of creditors shall, prior to the approval of such resolution plan under sub-section (4) or sub-
section (12), as the case may be, record reasons for its approval.
(15) The resolution professional shall submit the resolution plan as approved by the committee
of creditors under sub-section (4) or sub-section (12), as the case may be, to the
Adjudicating Authority.
Explanation I.––For the removal of doubts, it is
hereby clarified that, the corporate debtor being a resolution applicant under clause (25)
of section 5, may submit the base resolution plan either individually or jointly with any other
person.
Explanation II.––For the purposes of sub-
sections (4) and (14), claims shall be considered to be impaired where the resolution plan
does not provide for the full payment of the confirmed claims as per the updated list of
claims maintained by the resolution professional.
Approval of resolution plan
54L. (1) If the Adjudicating Authority is satisfied that the resolution plan as approved by the
committee of creditors under sub-section (4) or sub-section (12) of section 54K, as the case
may be, subject to the conditions provided therein, meets the requirements as referred to in
sub-section (2) of section 30, it shall, within thirty days of the receipt of such resolution plan, by
order approve the resolution plan:
Provided that the Adjudicating Authority shall, before passing an order for approval of a
resolution plan under this sub-section, satisfy itself that the resolution plan has provisions for its
effective implementation.
(2) The order of approval under sub-section (1) shall have such effect as provided under sub-
sections (1), (3) and (4) of section 31, which shall, mutatis mutandis apply, to the proceedings
under this Chapter.
(3) Where the Adjudicating Authority is satisfied that the resolution plan does not conform to
the requirements referred to in sub-section (1), it may, within thirty days of the receipt of such
resolution plan, by an order, reject the resolution plan and pass an order under section 54N.

© The Institute of Chartered Accountants of India


(4) Notwithstanding anything to the contrary contained in this section, where the Adjudicating
Authority has passed an order under sub-section (2) of section 54J and the resolution plan
approved by the committee of creditors under sub-section (4) or subsection (12), as the case
may be, of section 54K, does not result in the change in the management or control of the
corporate debtor to a person who was not a promoter or in the management or control of the
corporate debtor, the Adjudicating Authority shall pass an order —
(a) rejecting such resolution plan;
(b) terminating the pre-packaged insolvency resolution process and passing a liquidation
order in respect of the corporate debtor as referred to in subclauses (i), (ii) and (iii) of
clause (b) of sub-section (1) of section 33; and
(c) declaring that the pre-packaged insolvency resolution process costs, if any, shall be
included as part of the liquidation costs for the purposes of liquidation of the corporate
debtor.
Appeal against order under section 54L
54M. Any appeal from an order approving the resolution plan under sub-section (1) of section
54L, shall be on the grounds laid down in sub-section (3) of section 61.
Termination of pre-packaged insolvency resolution process.
54N. (1) Where the resolution professional files an application with the Adjudicating Authority,—
(a) under the proviso to sub-section (12) of section 54K; or
(b) under sub-section (3) of section 54D, the Adjudicating Authority shall, within thirty
days of the date of such application, by an order, —
(i) terminate the pre-packaged insolvency resolution process; and
(ii) provide for the manner of continuation of proceedings initiated for avoidance of
transactions under Chapter III or proceedings initiated under section 66 and
section 67A, if any.
(2) Where the resolution professional, at any time after the pre-packaged insolvency
commencement date, but before the approval of resolution plan under subsection (4) or sub -
section (12), as the case may be, of section 54K, intimates the Adjudicating Authority of the
decision of the committee of creditors, approved by a vote of sixty -six per cent. of the voting
shares, to terminate the pre-packaged insolvency resolution process, the Adjudicating Authority
shall pass an order under sub-section (1).
(3) Where the Adjudicating Authority passes an order under sub-section (1), the corporate
debtor shall bear the pre-packaged insolvency resolution process costs, if any.

© The Institute of Chartered Accountants of India


(4) Notwithstanding anything to the contrary contained in this section, where the Adjudicating
Authority has passed an order under sub-section (2) of section 54J and the pre-packaged
insolvency resolution process is required to be terminated under sub-section (1), the
Adjudicating Authority shall pass an order —
(a) of liquidation in respect of the corporate debtor as referred to in sub-clauses (i), (ii)
and (iii) of clause (b) of sub-section (1) of section 33; and
(b) declare that the pre-packaged insolvency resolution process costs, if any, shall be
included as part of the liquidation costs for the purposes of liquidation of the corporate
debtor.
Initiation of corporate insolvency resolution process.
54-O. (1) The committee of creditors, at any time after the pre-packaged insolvency
commencement date but before the approval of resolution plan under sub- section (4) or sub-
section (12), as the case may be, of section 54K, by a vote of sixty-six per cent. of the voting
shares, may resolve to initiate a corporate insolvency resolution process in respect of the
corporate debtor, if such corporate debtor is eligible for corporate insolvency resolution process
under Chapter II.
(2) Notwithstanding anything to the contrary contained in Chapter II, where the resolution
professional intimates the Adjudicating Authority of the decision of the committee of credit ors
under sub-section (1), the Adjudicating Authority shall, within thirty days of the date of such
intimation, pass an order to —
a. terminate the pre-packaged insolvency
resolution process and initiate corporate insolvency resolution process under Chapte r
II in respect of the corporate debtor;
b. appoint the resolution professional referred
to in under clause (b) of sub-section (1) of section 54E as the interim resolution
professional, subject to submission of written consent by such resolution profession al
to the Adjudicatory Authority in such form as may be specified; and
c. declare that the pre-packaged insolvency
resolution process costs, if any, shall be included as part of insolvency resolution
process costs for the purposes of the corporate insolvency resolution process of the
corporate debtor.
(3) Where the resolution professional fails to submit written consent under clause (b) of sub -
section (2), the Adjudicating Authority shall appoint an interim resolution professional by

© The Institute of Chartered Accountants of India


making a reference to the Board for recommendation, in the manner as provided under
section 16.
(4) Where the Adjudicating Authority passes an order under sub-section (2) —
(a) such order shall be deemed to be an order of
admission of an application under section 7 and shall have the same effect;
(b) the corporate insolvency resolution process shall commence from the date of such
order;
(c) the proceedings initiated for avoidance of
transactions under Chapter III or proceedings initiated under section 66 and section
67A, if any, shall continue during the corporate insolvency resolution process;
(d) for the purposes of sections 43, 46 and 50, references to “insolvency commencement
date” shall mean “pre-packaged insolvency commencement date”; and
(e) in computing the relevant time or the period for avoidable transactions, the time-
period for the duration of the pre-packaged insolvency resolution process shall also
be included, notwithstanding anything to the contrary contained in sections 43, 46
and 50.
Application of provisions of Chapters II, III, VI, and VII to this Chapter
54P. (1) Save as provided under this Chapter, sections 24, 25A, 26, 27, 28, 29A, 32A, 43 to 51,
and the provisions of Chapters VI and VII of this VI, and VII to Part shall, mutatis mutandis apply,
to the pre-packaged insolvency resolution process, subject to the following, namely:―
(a) reference to “members of the suspended Board of Directors or the partners” under
clause (b) of sub-section (3) of section 24 shall be construed as reference to
“members of the Board of Directors or the partners, unless an order has been passed
by the Adjudicating Authority under section 54J”;
(b) reference to “clause (j) of sub-section (2) of section 25” under section 26 shall be
construed as reference to “clause (h) of sub-section (2) of section 54F”;
(c) reference to “section 16” under section 27 shall be construed as reference to “section
54E”;
(d) reference to “resolution professional” in sub-sections (1) and (4) of section 28 shall
be construed as “corporate debtor”;
(e) reference to “section 31” under sub-section (3) of section 61 shall be construed as
reference to “sub-section (1) of section 54L”;

© The Institute of Chartered Accountants of India


(f) reference to “section 14” in sub-sections (1) and (2) of section 74 shall be construed
as reference to “clause (a) of sub-section (1) of section 54E”;
(g) reference to “section 31” in sub-section (3) of section 74 shall be construed as"
reference to “sub-section (1) of section 54L”.
(2) Without prejudice to the provisions of this Chapter and unless the context otherwise
requires, where the provisions of Chapters II, III, VI and VII are applied to the proceedings under
this Chapter, references to —
(a) “insolvency commencement date” shall be construed as references to “pre-packaged
insolvency commencement date”;
(b) “resolution professional” or “interim resolution professional”, as the case
may be, shall be construed as references to the resolution professional appointed
under this Chapter;
(c) “corporate insolvency resolution process” shall be construed as references to “pre -
packaged insolvency resolution process”; and
(d) “insolvency resolution process period” shall be construed as references to “pre -
packaged insolvency resolution process period.”.’.
(B) Vide Notification S.O. 4638 (E) [F. NO. 30/33/2020-INSOLVENCY], dated 22-12-2020
In exercise of the powers conferred by section 10A of the Insolvency and Bankruptcy Code,
2016 (31 of 2016), the Central Government hereby extended the period of suspension of
insolvency proceedings by further period of three months from the 25th December, 2020,
for the purposes of the said section.
(C) Vide MCA Notification S.O.1543(E) dated 9th April, 2021, in exercise of the powers
conferred by the second proviso to section 4 of the Insolvency and
Bankruptcy Code, 2016 (31 of 2016), as amended by the Insolvency and Bankruptcy Code
(Amendment) Ordinance, 2021, the Central Government hereby specifies ten lakh rupees
as the minimum amount of default for the matters relating to the pre -packaged insolvency
resolution process of corporate debtor under Chapter III-A of the Code.

© The Institute of Chartered Accountants of India


Final Course
(Revised Scheme of Education and Training)

Significant Case Laws

Elective Paper 6D
Economic Laws
(Relevant for May, 2021 examinations & Onwards)

BOARD OF STUDIES
THE INSTITUTE OF CHARTERED ACCOUNTANTS OF INDIA

© The Institute of Chartered Accountants of India


This material has been prepared by the faculty of the Board of Studies. The objective of the
material is to provide awareness to the students to enable them to have apprehension of
significant judgements. In case students need any clarifications or have any suggestions for
further improvement of the material contained herein, they may write to the Director of
Studies.
All care has been taken to provide interpretations and discussions in a manner useful for the
students. However, the material has not been specifically discussed by the Council of the
Institute or any of its Committees and the views expressed herein may not be taken to
necessarily represent the views of the Council or any of its Committees.
Permission of the Institute is essential for reproduction of any portion of this material.

© The Institute of Chartered Accountants of India

All rights reserved. No part of this book may be reproduced, stored in a retrieval system, or
transmitted, in any form, or by any means, electronic, mechanical, photocopying, recording, or
otherwise, without prior permission, in writing, from the publisher.

Edition : December, 2020

Website : www.icai.org

E-mail : bosnoida@icai.in

Committee/ : Board of Studies


Department

ISBN No. :

Price : `

Published by : The Publication Department on behalf of The Institute of


Chartered Accountants of India, ICAI Bhawan, Post Box No.
7100, Indraprastha Marg, New Delhi 110 002, India.

Printed by :

ii

© The Institute of Chartered Accountants of India


Preface

This booklet on Case laws is the collection of legal decisions given by Courts, Adjudicating
Authority and Tribunals. These Case laws reflect the interpretation of law by various competent
authorities under various circumstances. The competent authorities are using these pronounced
judicial cases to resolve ambiguities and fill gaps by adding new dimensions to the rules of
interpretation. These past decisions are called "case law", or “precedent”.
This booklet is comprising of some landmark and recent renowned cases. The most significant
cases are those that have had a lasting effect on the application of a certain law, often
concerning individual rights and liberties.
Law is dynamic subject and keeps on changing and has to be in alignment with the socio –
economic changes in the society. The issues under the law in the prevailing scenario can be
better understood through the relative judicial pronouncements or decided case laws. The Board
of Studies, through this release of booklet wishes to create awareness amongst the students
about the various significant judgements relating to various Economic Laws. As part of its
continuous endeavour towards enrichment of knowledge, Board of Studies, had decided to bring
out a booklet on Case Laws under Elective paper 6D: Economic Laws. Being an open book
examination, this initiative will lead to understanding of above on the interpretational, application
and analysis of the Laws covered therein. This publication contains the summarised version of
the facts with the marked legal issues and decisions of the Courts/Tribunals. The link of detailed
order or decision is also provided for the reference of the students to have appropriate
understanding of the comprehensive judicial pronouncements. Students, are, therefore, advised
to keep themselves updated about the latest judgements in order to interpret and apply them in
various circumstances.
This booklet is relevant for May 2021 Examination and onwards.

Wishing you happy reading!

iii

© The Institute of Chartered Accountants of India


CONTENT

1. Chapter – 1: Competition Act, 2002 .........................................................1.1 – 1.20

2. Chapter – 2: Real Estate (Regulation & Development) Act, 2016...............2.1 – 2.16

3. Chapter – 3: Insolvency & Bankruptcy Code, 2016 ...................................3.1 – 3.23

4. Chapter – 4: Prevention of Money Laundering Act, 2002...........................4.1 – 4.14

5. Chapter – 5: Foreign Exchange Management Act, 1999............................5.1 – 5.15

6. Chapter – 6: Prohibition of Benami Property Transactions Act, 1988 .........6.1 – 6.12

iv

© The Institute of Chartered Accountants of India


1
Competition Act, 2002

1. Mahindra Electric Mobility Limited & Ors. vs. CCI & Another
Citation: Delhi HC, W.P.(C) No. 11467/2018 dated 10.04.2019

Facts
♦ The complaint alleged that three car manufacturers, Honda Siel Cars India Ltd,
Volkswagen India Pvt. Ltd and Fiat India Automobiles Limited, restricted free availability
of spare parts in the open market, which caused a denial of market access for
independent repairers. This was in addition to other anti-competitive effects including
high prices of spare parts and repair and maintenance services for automobiles.
♦ After a detailed investigation by the Director General (DG) into the practices of 14 car
manufacturers (the Informant had only complained about three car manufacturers), the
CCI found that the car manufacturers had contravened provisions of Sections 3 and 4
of the Act and levied a penalty of 2% of the total turnover in India on each of the
manufacturers.
♦ As a consequence, some car manufacturers filed a writ before the Delhi HC challenging
the constitutional validity of certain provisions of the Act, which directly impacted the
validity of the CCI’s final order in the Auto Parts Case
Issue
1. Whether the CCI is a tribunal exercising judicial functions?
2. Whether the composition of the CCI is unconstitutional and violates the principle of
separation of powers?
3. Whether the ‘revolving door’ practice at the CCI vitiates any provisions of the Act and,
more specifically, if the manner for decision making provided under Section 22(3) of
the Act is unconstitutional?
4. Whether it is illegal to expand the CCI’s scope of inquiry?

© The Institute of Chartered Accountants of India


1.2 Economic Laws

Key Ratio Decidendi


1. Ruling on the first issue, the Delhi HC held that the CCI is in part administrative,
expert (when discharging advisory and advocacy functions) and quasi-judicial (while
issuing final orders, directions and penalties) and cannot be characterized as a
tribunal solely discharging judicial powers.
2. On the second issue, the Delhi HC dealt with each of the provisions of the Act that
were challenged by the petitioners and also undertook a comparison of regulatory
models of different specialized bodies/tribunals vis-à-vis the CCI. In particular, the
following were upheld to be valid:
(a) Section(s) 61 and 53T of the Act (which deal with exclusion of jurisdiction of
Civil Courts and High Courts, respectively).
(b) Section 9 (which provides for the selection procedure/committee for
members of the CCI).
(c) Section(s) 11, 55 and 56 (which deal with tenure of the members of the CCI
and the provision for supersession by the Central Government in the event
the CCI is unable to discharge its functions).
(d) Section 53D (which prescribes the composition and constitution of the
Appellate Tribunal).
(e) The Delhi HC declared Section 53E of the Act (which deals with composition
of the selection committee of the Appellate Tribunal), to be unconstitutional
subject to the decision of the Hon’ble SC in Central Administrative Tribunal v.
Union of India (wherein certain provisions of the Finance Act, 2017 have
been challenged).
3. Regarding the “revolving door policy”, the Delhi HC emphasised the principle of ‘who
hears must decide’ and stated that any violation of this rule would render any final
order void. It is also clarified that much would depend on the factual context and
merely resorting to the practice of “revolving door” would not render Section 22 of the
Act invalid or arbitrary. It is necessary that the party raising such objections must
have been prejudiced.
4. Further, in line with the decision of the Hon’ble SC in Excel Crop Care Limited v.
Competition Commission of India, the Delhi HC held that the CCI is well within its
power to expand the scope of inquiry to include other issues and parties. This is
because at the prima facie stage, the CCI may not have all information in respect of
the parties’ conduct.
Link to Order
http://lobis.nic.in/ddir/dhc/SRB/judgement/11-04-2019/SRB10042019CW114672018.pdf

© The Institute of Chartered Accountants of India


Competition Act, 2002 1.3

2. Competition Commission of India vs M/s Fast Way Transmission

Citation : Supreme Court, Civil Appeal No. 7215 of 2014 dated 24.01.2018

Facts
On August 1, 2010, the broadcasters of the news channel ‘Day & Night News’ (‘Broadcasters’)
entered into a channel placement agreement with the Multi System Operators (‘MSOs’),
forming part of the Fast Way group, for a period of one year. A notice of termination was
served on the Broadcasters, which was alleged to be an act of abuse of dominant position by
the MSOs in denying market access to the Broadcasters.
In its order, CCI found that the MSOs were dominant in the relevant market, having 85% of the
total subscriber share. CCI opined that the MSOs’ reason for termination, that the
Broadcasters had low television rating points (‘TRP’), and that the MSOs were facing
spectrum constraint, were insufficient and mere after thoughts put forth by the MSOs.
Accordingly, a penalty of ` 8,40,01,141 was imposed by CCI on the MSOs.
In appeal before the COMPAT, CCI’s order was reversed. The COMPAT held that a
broadcaster cannot be said to be a competitor of MSOs, and denial of market access can be
caused only by one competitor to another.
Issue
Whether abuse of dominant position as specified in the Competition Act is dependent on the
existence of, or effect on, competitors, or is it based on the abuse that the enterprise may
indulge in on the basis of its dominant position in the relevant market in any manner?
Proceedings Before the CCI and the COMPAT
When the MSOs unilaterally terminated the Channel Placement Agreement (CPA) with the
broadcaster, the broadcaster (Kansan News Private Limited) approached the CCI alleging
violation of the Competition Act. The CCI found that the group of MSOs were inter-related and
constituted a “group” in terms of the Competition Act. Additionally, the CCI found that the
group occupied a dominant position in the relevant market for the provision of cable TV
services in Punjab and Chandigarh.
The MSOs raised various preliminary arguments. They contended that the broadcaster had
suppressed the fact that it had already approached (without relief) the High Court of Punjab &
Haryana and the Supreme Court alleging breach of contract by the MSOs. Additionally, it was
pointed out that the broadcaster had also filed a case with the Telecom Disputes Settlement
and Appellate Tribunal (TDSAT) challenging the termination of the CPA. Most importantly, the
MSOs also challenged the jurisdiction of the CCI claiming that a dispute between a
broadcaster and an MSO falls within the exclusive jurisdiction of the TDSAT. On merits, the

© The Institute of Chartered Accountants of India


1.4 Economic Laws

MSOs contended that their conduct was based on technical and commercial reasons and,
thus, could not be considered an abuse of dominance.
Rejecting the contentions on merits, the CCI determined that the MSOs’ conduct amounted to
denial of market access and a violation of section 4(2)(c) of the Competition Act. The CCI,
thus, held that the MSOs were abusing their dominant position and imposed a penalty of INR
8,40,01,141 (INR 84 million) on the group of MSOs. It may be noted that the CCI, in its order,
did not deal with the issue of ‘jurisdiction’.
Against the CCI order, the MSOs approached the COMPAT reiterating their arguments on
jurisdiction and merits.
The COMPAT set aside the CCI order on the premise that denial of market access (under
section 4(2)(c) of the Competition Act) can be occasioned only by one competitor to another.
The COMPAT reasoned that since the broadcaster and the MSOs were not competitors, there
could not be denial of market access and, thus, set aside the CCI’s order and the penalty
imposed thereby.
Subsequently, the CCI approached the Supreme Court challenging the COMPAT’s decision on
the ground that abuse of dominant position as specified in the Competition Act is not
dependent on the existence of, or effect on, competitors, but is based on the abuse that the
enterprise may indulge in on the basis of its dominant position in the relevant market in any
manner.
Key Ratio Decidendi [CCI]
The Supreme Court partly agreed with both the CCI and the MSOs. The Supreme Court took
into consideration the Preamble of the Competition Act and its salient provisions to conclude
that the CCI has been vested with a positive duty to eliminate all practices that have an
adverse effect on competition. Although the Supreme Court did not directly deal with the issue
of jurisdiction, it specifically noted the non-obstante clause (section 60) in the Competition Act,
which expressly states that the provisions of the Competition Act shall have an overriding
effect notwithstanding anything inconsistent therewith contained in any other law.
Accordingly, the Supreme Court noted that once the existence of a dominant position is
proved, the question of whether the denial of market access is being done by a competitor or
not is irrelevant. The only relevant factor, according to the Supreme Court, is the denial of
market access due to unlawful termination of the CPA. Thus, the Supreme Court agreed with
the CCI and set aside the COMPAT’s decision in this regard.
The Supreme Court observed that CCI has a positive duty to eliminate all practices that lead
to an adverse effect on competition. Distinguishing from the opinion of the COMPAT, the
Supreme Court held that for there to be an abuse of dominant position, once dominance is
made out, it becomes irrelevant whether the parties are competitors or not. The Supreme
Court observed that Section 4(2)(c) of the Competition Act would be applicable for the simple

© The Institute of Chartered Accountants of India


Competition Act, 2002 1.5

reason that the Broadcasters were denied market access due to an unlawful termination of the
agreement between the Broadcasters and MSOs. The Supreme Court noted that the position
of dominance of the MSOs was clearly made out, owing to subscriber share of 85% enjoyed
by the MSOs in the relevant market of ‘Cable TV market in Punjab and Chandigarh’ and held
that the MSOs acted in breach of Section 4(2)(c) by terminating the agreement, but found that
the reasons for termination provided by the MSOs were justified, and therefore quashed the
penalty imposed by CCI.
Link to Order
https://sci.gov.in/supremecourt/2014/22445/22445_2014_Order_24-Jan-2018.pdf

3. Rajasthan Cylinders and Containers Limited vs UOI

Citation: Supreme Court, Civil Appeal No. 3546/2018 dated 01.10.2019

Facts
♦ A tender was floated by Indian Oil Corporation Ltd (IOCL) for the purchase of LPG
cylinders.
♦ An LPG cylinder manufacturer approached the CCI challenging the tender conditions
imposed by IOCL. However, while the case against IOCL was closed, during the
investigation of the aforesaid tender, the Director General (DG) noticed a similar
pattern in a bid submission by LPG cylinder manufacturers. This chain of events led the
CCI to initiate an inquiry, on its own motion, into the alleged cartelization and bid-
rigging by LPG cylinder manufacturers.
♦ Subsequent to the DG’s investigation and recommendations in its investigation report,
the CCI came to the conclusion that 45 LPG cylinder manufacturers, out of the 47
which were inquired into, had entered into an anti-competitive agreement in violation of
the Competition Act by rigging bids in the IOCL tender.
♦ This led to the CCI imposing penalties on those LPG cylinder manufacturers found in
contravention. Except for one party, 44 parties filed an appeal before the Competition
Appellate Tribunal (COMPAT) challenging the CCI order. While the COMPAT upheld
the CCI decision as to the existence of the contravention, it ordered reduction of the
penalties on the basis of the principle of “relevant turnover”.
Issue
Can enterprises be in violation of the Competition Act, 2002 (Competition Act) when prevailing
market conditions are themselves not conducive to a competitive market?

© The Institute of Chartered Accountants of India


1.6 Economic Laws

Key Ratio Decidendi


1. The Supreme Court reiterated that the purpose of the Competition Act is not merely to
eliminate anti-competitive practices but also to promote and sustain competition.
2. The Supreme Court held that there need not be direct evidence of cartelization since
such agreements are entered into in secret and the standard or proof required is that
of balance of probabilities. However, the Supreme Court held that the presumption of
anti-competitiveness attached to horizontal agreements is rebuttable by parties
through evidence.
3. While examining the market conditions prevailing in the LPG cylinders market, the
Supreme Court held that “those very factors on the basis of which the CCI has come
to the conclusion that there was cartelization, in fact, become valid explanations to
the indicators pointed out by the CCI”. The Supreme Court noted that the above
mentioned market condition led to a “situation of oligopsony that prevailed because of
limited buyers and influence of buyers in fixation of prices was all prevalent”.
4. On the basis of the above factors, the Supreme Court held that the LPG cylinder
manufacturers had discharged their onus by showing that the parallel behavior was
not a result of concerted practice but of the market conditions where IOCL was calling
the shots in so far as price control is concerned.
5. Thus, the Supreme Court held that at this stage it was up to the CCI to inquire further
in the case, which it failed to do. The Supreme Court also took note of the fact that
the CCI had failed to summon the IOCL before it, despite the IOCL having “full control
over the tendering process”.
6. Accordingly, the Supreme Court held that there was not “sufficient evidence” to hold
the LPG cylinder manufacturers in violation of the Competition Act and set aside the
COMPAT orders upholding the LPG cylinder manufacturers in violation of the
Competition Act.
Link to Order
https://sci.gov.in/supremecourt/2014/3644/3644_2014_Judgement_01-Oct-2018.pdf

4. Re: Umar Javeed and Google LLC

Citation: Case No. 39 of 2018, Competition Commission of India dated 16.04.2019

Facts
♦ The present case was filed under Section 19(1)(a) of the Act, alleging Google LLC and
Google India Private Limited, (collectively ‘Google’) of abuse of its dominant position in
the mobile operating systems market.

© The Institute of Chartered Accountants of India


Competition Act, 2002 1.7

♦ Google is leading in the operating systems’ market as majority of the users and the
developers use its operating system (OS) along with varied other Google’s proprietary
applications and services [Google Mobile Services (GMS)].
♦ The CCI noted that a wide range of applications like Google Maps, Google Chrome,
YouTube were only available through GMS on android phones which had to be
preinstalled by the manufacturer and couldn’t be availed directly by the end-users.
♦ In order to obtain rights to these applications, the manufacturers had to enter into
agreements with Google, namely, Mobile Application Distribution Agreement (“MADA”)
and Anti Fragmentation Agreement (“AFA”).
Issue
Whether Google had abused its dominant position?
Key Ratio Decidendi
1. The CCI recorded that to form a prima facie case of abusive conduct, it was
necessary to identify the relevant market and to define the dominance of Google in
such market.
2. Firstly, it observed that the primary relevant market will be the market for licensable
smart mobile device OS in India. The CCI identified that the mobile OS are different
from OS designed for desktop as they have additional handheld use features. Thus, it
differs in characteristics and on the basis of usage. Also, the CCI noted that other OS
like iOS and Windows are not available for licensing to third-party developers and
thus are excluded from the relevant market.
3. Secondly, the CCI held that Google appears to be dominant in the relevant market as
Android accounted for 80% of India’s mobile OS market. The European Commission’s
decision [Case AT.40099 – Google Android, Commission Decision of 18 July 2018.]
was also relied upon where the EC had found Google to dominant in the markets of
general internet search services and licensable smart mobiles OS.
4. Furthermore, the Commission held that the signing of MADA mandates pre-
installation of entire suite of Google apps. ‘Play Store’, a must have app, cannot be
pre-installed separately thereby compulsorily tying it with the other applications. In the
backdrop, the CCI noted that making pre-installation of ‘Play Store’ conditional,
Google reduced the ability and disincentivized the device manufacturers to develop
viable alternatives, thereby restricting technical development to the prejudice of
consumers violative of Section 4(2) of the Act.
5. While reading Section 4 with Section 32 of the Act, it is important to note that the
conduct of Google to tie or bundle applications and services is an attempt to eliminate
effective competition from the market. There exists an element of coercion as the

© The Institute of Chartered Accountants of India


1.8 Economic Laws

device manufacturers are coerced to purchase the GMS suite altogether which results
in consumer harm through reduction in choice of products
Link to Order
https://www.cci.gov.in/sites/default/files/39-of-2018.pdf

5. Re: House of Diagnostics LLP and Esaote Asia Pacific Diagnostic


Pvt. Ltd

Citation: Case No. 09 of 2016, Competition Commission of India dated 27.09.2018

Facts
♦ The information was filed by M/s House of Diagnostics LLP (‘HoD’), engaged in the
business of medical diagnostics and diagnostic imaging services.
♦ The allegations of HoD pertained to the purchase of three ‘Dedicated Standing/ Tilting
MRI machines’ manufactured by Esaote (‘G-Scan Machines’) for HoD’s diagnostic
centers. It was alleged that Esaote was dominant in the defined market and had abused
its dominance as: (i) Esaote misled HoD that new machines were being supplied to it;
(ii) acted unfairly by refusing to supply ‘See through Perforated RF Cage’ and ‘Head
Coils’ to HoD; (iii) insisted that the Comprehensive Maintenance Contract (‘CMC’) was
to be paid for each machine supplied; and (iv) limited the provision of services in the
after sale market and denied market access to third party service providers.
Issue
Had Esaote S.P.A and Esaote Asia Pacific Diagnostic Pvt. Ltd. acted in contravention to
Section 4(2) of the Act?
Key Ratio Decidendi
The DG Report came to following conclusions: (i) the relevant market was found to be the
market for ‘Dedicated Standing/ Tilting MRI machines in India’; (ii) Esaote was dominant in the
market for ‘Dedicated Standing/ Tilting MRI machines in India’; and (iii) Esaote abused its
dominance in the defined market.
In its analysis, CCI observed that G-scan Machines are a distinct product that can tilt the
patient up to 90 degrees which is not possible in conventional supine MRI machines. It was
also observed that conventional MRI machines can scan the whole body of a patient whereas
the dedicated standing/ tilting MRI is meant specifically for joint and spines. In addition, the
facts that there are diagnostic centers which have both types of MRI machines indicate that
the products are distinct. Therefore, CCI found the relevant market to be the ‘market for
dedicated standing/tilting MRI machines in India’ (‘Relevant Market’). Further, CCI found
Esaote to be dominant in the Relevant Market as Esaote had patent rights over its G-Scan

© The Institute of Chartered Accountants of India


Competition Act, 2002 1.9

Machines and due to the absence of other companies who manufacture such machines in the
Indian market; Esaote is able to operate independently of competitive forces.
On a reading of the relevant purchase order, CCI found Esaote to have abused its dominance
by insisting that the CMC was to be paid for each machine. Further, the exclusive rights given
to EAPD for supply of spare parts and for providing after sales services for G-Scan Machines
in India limited the provision of services in the after sale market and denied market access to
third party service providers. Based on the above, CCI found Esaote to have abused its
dominance under Section 4 of the Act and imposed a penalty of Rs 9.33 lakhs while also
directing Esaote to ‘cease and desist’ from the infringing conduct.
Dissenting Opinion: The CCI Chairperson wholly disagreed with the majority’s view on the
delineation of the Relevant Market. The dissent note observes that the market cannot be
narrowed to standing/ tilting MRI machines alone as any market delineation would have to
necessarily include all MRI machines. Once the market is defined in this manner, the behavior
of Esaote stands constrained by the presence of many other players as Esaote is a small
player in the market of all MRI machines. Therefore, there was no question of dominance or
abuse of dominance.
Link to Order
https://www.cci.gov.in/sites/default/files/case%20no.%2009%20of%202016%20%28Majority%
20Order%20P.%2001%20to%2033%29%20%2C%20Dissent%20Note%20by%20Chairperson
%20%28P.34%20to%2039%29.pdf

6. In Re: Anti competitive conduct in the Dry-Cell Batteries Market in


India

Citation: Suo Motu Case No. 03 of 2017, CCI dated 15.01.2019

Facts
(a) Panasonic was a manufacturer of zinc-carbon dry cell batteries (‘DCBs’), on the other
hand, Godrej did not have the capability to manufacture DCBs;
(b) Panasonic and Godrej entered into a product supply agreement (‘PSA’) for supply of
DCBs starting from early 2012 until late 2014;
(c) similar to customary contract manufacturing arrangements, the DCBs were procured
by Godrej from Panasonic in a ready to sell form i.e., the labelling and packaging of
the battery was also done by Panasonic;
(d) The DCBs procured by Godrej from Panasonic were sold by Godrej under its own
brand name, giving due credit to Panasonic as manufacturer of such batteries, on the
labelling of the DCB.

© The Institute of Chartered Accountants of India


1.10 Economic Laws

(e) In parallel, Panasonic entered into a price fixing arrangement with its competitors i.e.,
Eveready Industries India Ltd. (‘Eveready’) and Indo National Limited (‘Nippo’) during
the period between May, 2009 and August, 2016.
(f) Panasonic filed a leniency application before CCI disclosing this conduct and CCI
found Panasonic, Eveready and Nippo guilty of a cartel conduct.
Issue
Whether a "bilateral ancillary cartel" existed between Panasonic (India) and Godrej in the
institutional sales of dry cell batteries?
Key Ratio Decidendi
1. The CCI specifically notes that Panasonic used its knowledge of future prices fixed
with Eveready and Nippo and used ‘its position of leverage to extract higher
procurement price’ from Godrej, by indicating that the prices will go up.
2. CCI further held Godrej guilty of entering into a cartel with Panasonic on the basis of:
(i) a specific mutual comfort clause under the PSA effectively stating that the buyer
and seller would not act against each other’s market interests; and (ii) e-mails that
were exchanged between the officials of Panasonic and Godrej in relation to
procurement price of DCBs pursuant to the PSA.
3. CCI’s reasoning appears to be primarily based on the following premise: (i) Godrej
and Panasonic were identified as ‘independent principals’ under the agreement and
therefore, a mutual comfort clause was inherently anti-competitive; and (ii) none of
the e-mail exchanges used the term ‘procurement price’ and therefore, could not have
been viewed as negotiation between buyer and seller in relation to procurement price.
As a corollary, CCI reasoned that price related discussion on e-mails could only be
viewed as an anti-competitive price fixing of DCBs.
4. The Godrej Order appears to suggest that an agreement entered between a buyer
and seller on a principal-to-principal basis could be viewed as akin to an agreement
between independently operating competitors and therefore, could be scrutinized
under the ambit of Section 3(3) of the Competition Act. Section 3(3) of the
Competition Act (i.e., horizontal agreements) which precludes collusion between
competitors i.e., independent operating entities at the same level of the production
chain. In contrast, Section 3(4) (i.e., vertical agreements) of the Competition Act deals
with agreements between two or more enterprises that are at different stages of the
production chain. Vertical agreements manifest an interdependent relation between
(non-competing) enterprises as a result of being staged at different points of a value
chain and generally proscribe harm associated with unilateral conduct. On the other
hand, Section 3(3) of the Competition Act governs agreements between competitors,
who were otherwise required to act independently.
Link to Order
https://www.cci.gov.in/sites/default/files/Suo-Moto-03-of-2017_0.pdf

© The Institute of Chartered Accountants of India


Competition Act, 2002 1.11

7. In Re: Fx Enterprise Solutions India Pvt. Ltd. and Hyundai Motor


India Limited

Citation: Case No. 36 and 82 of 2014, CCI dated 14.06.2017

Facts
♦ The Dealership Agreement required dealers to seek consent prior to taking up
dealerships with a competing brand and to procure spare parts either directly from
Hyundai or through its pre-approved vendors and this amounted to an “exclusive supply
agreement” and “refusal to deal”.
♦ The enforcement of the “Discount Control Mechanism” (DCM) under the Dealership
Agreement by Hyundai resulted in Resale price maintenance (RPM). Further, it was
alleged that Hyundai perpetuates “hub-and-spoke” arrangements, wherein bilateral
vertical agreements between Hyundai and its dealers and horizontal agreements
between dealers through Hyundai, resulted in price collusion.
♦ It was alleged that Hyundai forces its customers to purchase passenger cars tied with
CNG Kits, Lubricants and Oils, and Insurance Services from specified vendors
Issue
Whether Hyundai Motor India Limited has acted in contravention of the provisions of Section 3
of the Act?
Key Ratio Decidendi [CCI]
1. Exclusive supply and refusal to deal: the CCI observed that merely because the
requirement of prior consent before entering into a dealership for a competitor brand,
did not amount to exclusivity.
2. On tie-in arrangements, the CCI reiterated its position in Sonam Sharma vs. Apple
and Ors. [Case No. 24 of 2011] to state that purchasers of Hyundai cars were not
“forced” to purchase CNG Kits, Lubricants and Oils, and Insurance Services from
specified vendors.
3. With respect to RPM allegations, the CCI observed that Hyundai restricts the
maximum discount which a dealer can give, thereby effectively fixing the minimum
resale price. Based on Hyundai's admission the CCI held that Hyundai enforces this
strictly by appointing mystery shopping agencies for policing its dealers and wherever
a dealer is found to be violating the discount control mechanism, a fine is imposed on
them.

© The Institute of Chartered Accountants of India


1.12 Economic Laws

Appeal to NCLAT
[Hyundai Motor India Ltd. Vs. CCI.
Competition Appeal (AT) No. 06 of 2017 dated 19.09.2018]
The NCLAT observed that the CCI failed to take into consideration the factors mentioned
under Section 19(6) and 19(7) of the Competition Act for the determination of the relevant
geographic market and product market, respectively. The NCLAT placed reliance on the
decision of the Supreme Court in Competition Commission of India v. Coordination Committee
of Artistes and Technicians of West Bengal Film and Television and Ors. to hold that the CCI
did not take into consideration the physical characteristics or end use of goods, including price
of goods or service; consumer preferences as required to be taken under sub-section (7) of
Section 19 for determination of 'relevant product market'. The factors such as the consumer
preferences, transport costs, distribution facilities are the corner stone for determining whether
a geography/area could be considered as a separate market from both consumer and the
producer's point of view.
While the CCI delineated the upstream market as the market of sale of passenger cars, the
downstream market was delineated as the market for dealership of Hyundai cars alone. As
such it is not clear from the analysis on the relevant market by the CCI, the reasons behind
not considering the substitutability of different passenger cars by the dealers.
Link to Order
CCI Order: https://www.cci.gov.in/sites/default/files/36%20and%2082%20of%202014.pdf
NCLAT Order:
https://nclat.nic.in/Useradmin/upload/6594600435ba2337253f81.pdf

8. In Re: Samsher Kataria v. Honda Siel Cars India Ltd. & Ors

Citation: Case No. 03 of 2011, CCI dated 27.07.2015

Facts
The complaint was filed against various automobile manufacturers or original equipment
manufacturers (OEMs) – concerns were restriction of availability of genuine spare parts of
automobiles manufactured by them in open market
Issue
Whether Original Equipment Manufacturers (OEM) were abusing their dominant position?

© The Institute of Chartered Accountants of India


Competition Act, 2002 1.13

Key Ratio Decidendi


1. It was found that –
(a) the relevant market would be market for spares of automobiles
(b) each automobile manufacturer was dominant in respect of its respective
spares
(c) a customer who purchases a car, cannot substitute spares of a different
manufacturer, are ‘locked into’ certain aftermarket suppliers.
2. The DG concluded that each car manufacturer was in a dominant position in the
supply of its spare parts for its own brand of cars
3. The effect of treating the market for cars and its spares as separate and distinct
markets is that a manufacturer is automatically in a dominant position in respect of its
own spares. The Commission agreed with the DG’s analysis of relevant market.
4. CCI concluded that manufacturers were unable to demonstrate that they could, or
consumers could, compute life-span costs and hence, rejected the systems market
contention. Each car manufacturer was the sole seller of its spares and diagnostic
tools and that car manufacturers restricted authorized dealers from making sales in
the open market. CCI thus rejected ‘unified systems market’ contention of the car
manufacturers and agreed with the distinct market for cars and a distinct market for
after sales service.
5. The CCI was also of the view that a ‘cluster market’ existed for all the spare parts for
each brand of car manufactured by the OEM.
6. Hence, it further held that the Indian car spare-parts market forms a separate market
from that of cars, inter alia, as the consumers do not engage in whole life costing
while buying cars, and the OEMs, who were generally the car manufacturers
monopolized this spare-parts market.
Link to Order
https://www.cci.gov.in/sites/default/files/03201127.pdf

9. CCI vs Bharti Airtel Ltd

Citation: Supreme Court, Civil Appeal No. 11843 of 2017 dated 05.12.2018

Facts
♦ In 2017, the CCI, acting on information filed by Reliance Jio Infocomm Limited (‘Jio’)
under Section 19(1) of the Competition Act, 2002 (‘Act’), ordered the Director
General, CCI (‘DG’) to investigate (‘CCI Order’) against the alleged cartelization by

© The Institute of Chartered Accountants of India


1.14 Economic Laws

Bharti Airtel Limited, Vodafone India Limited, Idea Cellular Limited and the Cellular
Operators Association of India (‘OPs’). It was alleged that OPs had cartelized to deny
Jio entry into the telecom sector by not providing it adequate Points of
Interconnection (‘POIs’), resulting in call failures between Jio and other networks. Jio
had also filed letters with the TRAI complaining against the conduct of the OPs.
♦ The Bombay High Court (‘BHC’) by way of an order dated September 21, 2017 (‘BHC
Order’) set aside the CCI Order and held that the telecom sector is governed,
regulated and controlled by certain special authorities, and the CCI does not have the
jurisdiction to deal with interpretation or clarification of any “contract clauses”, “unified
license”, “interconnection agreements”, “quality of services regulations”, etc., which
are to be settled by the TRAI/ Telecom Disputes Settlement and Appellate Tribunal
(‘TDSAT’). BHC further held that the powers of CCI are not sufficient to deal with the
technical aspects associated with the telecom sector, which solely arise out of the
Telecom Regulatory Authority of India Act, 1997 (‘TRAI Act’) and related regulations.
CCI and Jio, aggrieved by the BHC Order, approached the SC.
Issue
Whether CCI has jurisdiction to deal with the matter unless the issues are settled by
authorities under the TRAI Act?
Key Ratio Decidendi [CCI]
Dealing with the question of whether the CCI had jurisdiction to look into the allegations of
collusion amongst the incumbents, the Supreme Court held that the TRAI Act and the
Competition Act are both special Acts and primacy has to be given to the respective objectives
of both the regulators under their respective statutes.
The Supreme Court clarified that the jurisdiction of the CCI is not ousted by the TRAI Act.
However, the Court was cognizant that simultaneous exercise of jurisdiction by both could
lead to conflicting outcomes and uncertainties. Having determined that both require primacy
and cognizant of the implications of simultaneous exercise of jurisdiction by both, the Supreme
Court sought to maintain the balance by adopting the following approach:
The TRAI, being the expert regulatory body governing the telecom sector, has, in the
first instance, powers to decide contractual issues such as obligation to provide POIs,
reasonableness of demand for access to POIs, concepts of “subscriber”, “test period”,
“test phase and commercial phase rights and obligations”, “reciprocal obligations of
service providers” or “breaches of contract”.
Once these “jurisdictional aspects” are straightened and answered by the TRAI, the CCI
can exercise its jurisdiction under the Act. The CCI’s exercise of jurisdiction is as such,
not rejected but pushed to a later stage when the TRAI has undertaken the necessary
activity of determining the jurisdictional facts.

© The Institute of Chartered Accountants of India


Competition Act, 2002 1.15

The Supreme Court has recognised that CCI’s jurisdiction is not excluded by presence of
sectoral regulators and to that end, the CCI enjoys primacy with respect to issues of
competition law. However, to the extent the Supreme Court holds that despite such primacy,
the CCI is “ill equipped to proceed” on account of absence of the determination of
“jurisdictional aspects” by a sectoral regulator, the Supreme Court grants to the CCI a ‘follow-
on’ jurisdiction.
Link to Order
https://sci.gov.in/supremecourt/2017/40072/40072_2017_Judgement_05-Dec-2018.pdf

10. Harshita Chawla v. WhatsApp

CCI, Case No 15 of 2020, dated 18.08.2020

Facts
The present petition was filed in March before the CCI under Section 19(1)(a) of the
Competition Act, 2002 (“Act”), alleging contravention of provisions of Section 4 of the Act.
Harshita Chawla (“Informant”) filed a case against WhatsApp and Facebook Inc. (“Facebook”)
for abusing their dominant position in launching their payment app services.
Issue
Facebook-owned WhatsApp is leveraging its dominance in the internet-based instant
messaging apps, by bundling its messaging app with the payment system, i.e. WhatsApp Pay
in contravention of Section 4(2)(a)(i) of the Act. This can also be termed as ‘coercion’ and in
contravention of Section 4(2)(d) of the Act. Facebook is inherently a predatory entity known to
focus on buying out its competition.
Key Ratio Decidendi
Relevant Market: CCI disagreed with the relevant market proposed by WhatsApp and
observed that the first relevant market would be ‘market for Over-The-Top (OTT) messaging
apps through smartphones in India’ and the second relevant market for the purposes of
assessment of antitrust allegations would be ‘market for UPI enabled Digital Payments Apps in
India’. Although in terms of nomenclature the first relevant market seems to be different than
what was originally proposed by the Informant, the CCI was of the view that both the markets
are same in essence, largely covering the same set of players and competition dynamics.
While demarcating the relevant markets, the CCI also observed that WhatsApp and Facebook
are third-party apps providing internet-based consumer communication services which again
can be sub-segmented based on different parameters. Notably, while WhatsApp is primarily
an OTT messaging app, Facebook is a social networking app connecting many users
simultaneously. WhatsApp and Facebook falling in different markets with their services not

© The Institute of Chartered Accountants of India


1.16 Economic Laws

being substitutable, the CCI rejected the ‘market for user attention’ (as proposed by
WhatsApp) for the purposes of the investigation.
Dominant Position: CCI observed that, despite operating in separate markets, Facebook and
WhatsApp are group entities and therefore, WhatsApp’s market position needs to be assessed
based on such considerations. The CCI held WhatsApp to be prima facie dominant in the first
relevant market ‘market for OTT messaging apps through smartphones in India’. While
conclusively determining the dominance, the CCI placed reliance on its advantage of reaping
the benefits of network effect, increased switching costs, lack of interoperability between
platforms, lack of constraints among the major players in the market such as Facebook
Messenger, and on the indications of dominance from the market data despite having chances
of infirmities.
Abuse of Dominance: The CCI did not find merit in allegations with respect to abuse of
dominance as the mere existence of an App on the smartphone does not necessarily convert
into transaction/usage. Given that WhatsApp had categorically ensured the offering of full
discretion to the users with regard to the usage of its payment app services and provision of
separate registration requiring the users to accept the terms of services and privacy policy, the
CCI ruled in favour of WhatsApp. With regard to allegations of violation of Section 4(2)(d) of
the Act, the CCI differed in its views. CCI clarified that the term ‘tying’ actually refers to a
practice whereby the seller of the product/service (‘tying product’) requires the buyers to also
purchase another separate product or service (‘tied product’). On the other hand, bundling
usually refers to act of selling two products in fixed proportions in a specific price as a bundled
package. The following necessary conditions need to be fulfilled for a tying case to be made
out:
(a) The tying and tied products are two separate products;
(b) The entity concerned is dominant in the market for tying products;
(c) The consumer has no choice to buy one of the products: tying or tied;
(d) The tying is capable of restricting/foreclosing in the market.
Tested on these parameters, the CCI observed that the first two conditions along with the
dominance of the entity were met, especially when WhatsApp Messenger and WhatsApp Pay
are distinct products offering different services to the users. However, with regard to the other
two conditions, the CCI ruled in favour of WhatsApp. Due to the voluntary installation of
WhatsApp messenger (stated by WhatsApp) coupled with (i) no explicit coercion on the user
to use WhatsApp Pay exclusively or (ii) influencing the consumer choice implicitly in any other
manner, the third condition was not met. Notably, WhatsApp Pay received approvals to act as
a payment app in India only in February 2020 in beta version, and also seems to have
complied with the data localisation norms as stipulated by NPCI. CCI taking a pragmatic
approach noted that WhatsApp actual conduct is yet to manifest in the market, especially

© The Institute of Chartered Accountants of India


Competition Act, 2002 1.17

when the number of users being served under the beta version is limited to less than 1% of its
users in India.
Conclusion
In light of the above, CCI ruled out the unwarranted and implausible apprehensions of the
Informant on WhatsApp using its dominant position in the first relevant market to gain leverage
in the second relevant market. CCI did not find any contravention of the provisions of Section
4 of the Act against WhatsApp or Facebook and directed closure under Section 26(2) of the
Act.
Link
https://www.cci.gov.in/sites/default/files/15-of-2020.pdf

11. Samir Agrawal v. CCI & Ors.

NCLAT, Competition Appeal (AT) No.11 OF 2019 dated 29.05.2020

Facts
Mr. Samir Agrawal (“Appellant/Informant”), an independent law practitioner filed information
with the Commission alleging contravention of Section 3 of the Act by a “hub and spoke cartel”
formed between the cab aggregator brands (being the hub), namely; Ola and Uber and the
drivers (being the spokes) using these cab aggregators’ mobile application and sought
investigation. The Appellant alleged that the pricing algorithm used by these cab aggregators
for providing radio taxi services facilitated their collusion with the drivers and among drivers as
well. The Commission, vide its order dated 06.11.2018, observed that no agreement,
understanding or arrangement appeared to exist either between the cab aggregators and their
respective drivers or between the drivers inter se regarding the price-fixing. The Commission,
therefore, ruled that no prima facie case of contravention of Section 3 of the Act existed to
order an investigation by the Director General and the matter was closed in terms of Section
26(2) of the Act. Aggrieved thereof, the Appellant assailed the Commission’s order before
NCLAT.
Key Ratio Decidendi
While adjudicating on the Informant’s right to approach the Commission, NCLAT referred to
Section 19(1) of the Act and noted that the Act provided for three modes for the Commission
to take cognizance of an alleged contravention of provisions of the Act: (i) either on its own
motion, or; (ii) through information from ‘any person’, consumer or their association or trade
association or; (ii) through a reference made to the Commission by the appropriate
Government or statutory authority.

© The Institute of Chartered Accountants of India


1.18 Economic Laws

Interpreting the term ‘any person’, NCLAT opined that though PILs, class action, etc have
watered-down the concept of locus standi, given a statute like the Act provided for peculiar
modes to take cognizance of a matter, “it has necessarily to be construed as a reference to a
person who has suffered invasion of his legal rights as a consumer or beneficiary of healthy
competitive practices”. The NCLAT justified adopting this restrictive interpretation to prevent
“unscrupulous people” with “oblique motives” from targeting any enterprise under the
provisions of the Act.
Conclusion
The locus standi to approach Competition Commission of India under the Competition Act,
2002 lies only with a person “who is either a consumer of the goods/services in question or a
beneficiary of healthy competitive practices in a given market”.
Link
https://nclat.nic.in/Useradmin/upload/9123996565ed0ea3eec766.pdf

12. Monsanto Holdings Pvt. Ltd. and Ors. vs.CCI

Delhi HC, W.P.(C) 3556/2017 and CM Nos. 15578/2017, 15579/2017 &35943/2017


dated 20.05.2020

Facts
The case is related to the trait rate charged via MMBL and the alternative terms and situations
imposed by means of it for the usage of the generation for production Bt. Cotton Seeds.
Monsanto is a business enterprise engaged in growing and commercializing generation for
producing genetically modified seeds. It holds a portfolio of patents, emblems, and licenses. It
is said that Monsanto become the first business enterprise to broaden and commercialize Bt.
Cotton Technology (Bollgard-I). The era is geared toward genetically editing hybrid seeds to
instil a particular trait – resistance to bollworms.MMBL is an agency integrated in India and is
a part of the Monsanto organization inasmuch as it is a joint assignment business enterprise
among MHPL (that's a one hundred subsidiary of Monsanto) and Mahyco. Further, MHPL
additionally holds 26% fairness in Mahyco. The Nuziveedu Seeds Ltd. (‘NSL’), Prabhat Agri
Biotech Ltd. (‘PABL’), and Pravardhan Seeds Pvt. Ltd. (‘PSPL’) had filed Information beneath
section 19(1)(a) of the Competition Act earlier than CCI alleging contravention of the
Competition Act. The Informants had accused the Petitioner of abusing its role as the
dominant participant inside the marketplace of Bt Cotton Seeds by means of charging
unreasonably high trait fees.

© The Institute of Chartered Accountants of India


Competition Act, 2002 1.19

Issue
1. Whether no word may be issued to the Directors / Persons In-rate of the Company
until the CCI returns a finding in opposition to the Company that it has indulged in
anti-aggressive sports under Sections 3 and four of the Competition Act;
2. Whether Section 48 of the Competition Act, which offers for vicarious liability of folks
in-price and chargeable for the conduct of commercial enterprise of the Company, will
follow handiest on contravention of orders of CCI or DG under Sections 42 to 44 of
the Competition Act and no longer to contravention of Sections three and 4 of the
Competition Act.
Key Ratio Decidendi
On the decision passed in Telefonaktiebolaget L.M. Ericsson v. Competition Commission of
India & Another: W.P.(C) 464/2014
♦ Sections 60 and 62 of Competition Act give the Competition Act an overriding effect
and provide the Act to be in addition to and not in derogation of other Acts. This Court
in Telefonaktiebolaget L.M. Ericsson v Competition Commission of India & Another
provided that Section 60 would not reduce the weight of the Patents Act and Section 62
makes it clear that Competition Act is in addition to other laws.
♦ In Telefonaktiebolaget examining the two Acts, the Court observed that orders that can
be passed by the CCI under Section 27 of Competition Act relating to abuse of
dominant position are different from the remedies under provisions pertaining to
Compulsory License under the Patents Act.
♦ In Telefonaktiebolaget, the Court also observed that under certain circumstances the
prospective licensee can approach the Controller for a compulsory license but the
same would not be inconsistent with CCI passing an order under Section 27.
♦ CCI under Section 20 of the Competition Act has a power to make a reference to any
regulator where in course of proceedings the CCI proposes to take any decision which
may be contrary to provisions of any statute, the implementation of which has been
entrusted to any statutory authority.
♦ Section 21 of the Competition Act enables any statutory authority, charged with
administration of any statute to make a reference to CCI if it proposes to take any
decision, which may be contrary to the provisions of Competition Act.
♦ In Telefonaktiebolaget, the Court concluded that there was no irreconcilable
repugnancy or conflict between the Competition Act and the Patents Act, and, thus, the
jurisdiction of the CCI to investigate complaints regarding abuse of dominance in
respect to patent rights could not be excluded.

© The Institute of Chartered Accountants of India


1.20 Economic Laws

On Competition Commission of India v. Bharti Airtel Ltd. And Ors.: Civil Appeal No.
11843/2018
♦ In the said case, the Supreme Court did not hold that jurisdiction of CCI in respect of
matters, regulated by a specialised statutory body were excluded from the applicability
of the Competition Act.
♦ Role of TRAI as a regulator is materially different from that of a Controller and is more
pervasive.
♦ The Bombay High Court decision, upheld by the Supreme Court, after examining the
role of TRAI, held that TRAI’s role was different than the role of a Controller and thus
Telefonaktiebolaget L.M. Ericsson (supra) was not applicable.
On Interpretation canvassed by Petitioner of Section 3 (5) of the Competition Act, 2002
♦ Section 3(5)(i) of the Competition Act cannot be broken down in the manner as
suggested.
♦ The words “or to impose reasonable conditions” are placed between two commas and
thus must be interpreted as being placed in parenthesis that explains and qualifies
the safe harbour of Section 3(5). Plainly, the exclusionary provision to restrain
infringement cannot be read to mean a right to include unreasonable conditions that
far exceed those that are necessary, for the aforesaid purpose.
♦ While an agreement, which imposes reasonable condition for protecting Intellectual
Property Rights is permissible any agreement which imposes unreasonable
conditions is impermissible under the Competition Act.
♦ The Hon’ble Court found no reason to interfere with the Impugned Order of
Investigation. The Hon’ble Court further found the said order to be an administrative
order and held that unless the same is found to be arbitrary or unreasonable, it shall
not be interfered with.
Conclusion
The Hon’ble Judge upholding the jurisdiction of CCI and disregarding the writ petition of the
Petitioners, held that an order surpassed by using the CCI below Section 26(1) of the
Competition Act is an administrative order and, consequently, unless it's miles found that the
equal is bigoted, unreasonable and fails the Wednesbury test no interference would be
warranted. Since a evaluate on deserves was impermissible on the time of the choice of the
Hon’ble Judge, consequently, the Hon’ble Judge avoided examining the deserves of the
dispute.

© The Institute of Chartered Accountants of India


2
Real Estate (Regulation & Development)
Act, 2016

1. M/s M3M India Pvt. Ltd. & Anr. v. Dr. Dinesh Sharma & Anr

Citation: Delhi HC, CM(M)--1249/2019 dated 04.09.2019

Facts
These petitions under Article 227 of the Constitution involve a common question, viz. whether
proceedings under the Consumer Protection Act, 1986 can be commenced by home buyers
(or allottees of properties in proposed real estate development projects) against developers,
after the commencement of the Real Estate (Development and Regulation) Act, 2016
Issue
Whether proceedings under the Consumer Protection Act (CPA) could be commenced by
home buyers against developers, after the commencement of RERA?
Key Ratio Decidendi
The court was of the view that judgment in Pioneer Urban Land and Infrastructure Ltd. & Anr.
vs. Union of India was binding on the high court with regard to the issue in question in as
much as:
1. While it was correctly pointed out by the Respondent that the litigation before the
Supreme Court principally raised the question of remedies under IBC and RERA, the
issues arising out of CPA proceedings were also brought to the attention of the Court.
In fact, it had recorded that "Remedies that are given to allottees of flats/apartments
are therefore concurrent remedies and connected matters such allottees of
flats/apartments being in a position to avail of remedies under the Consumer
Protection Act, 1986, RERA as well as the triggering of the Code." Thus, it could not
be said that any of those conclusions are obiter dicta or made as passing
observations, and not intended to be followed.

© The Institute of Chartered Accountants of India


2.2 Economic Laws

2. The high court could not disregard the judgment of the Supreme Court as being per
incuriam based on its perception regarding the arguments considered therein.
Reliance was placed on Sundeep Kumar Bafna v. State of Maharashtra & Anr.,
wherein the Supreme Court gave a "salutary clarion caution to all courts, including the
High Courts, to be extremely careful and circumspect in concluding a judgment of the
Supreme Court to be per incuriam".
Thereby the court concluded that "remedies available to the respondents herein under CPA
and RERA are concurrent, and there is no ground for interference with the view taken by the
National Commission in these matters."
Link to Order
http://lobis.nic.in/ddir/dhc/PRJ/judgement/05-09-2019/PRJ04092019CMM12442019.pdf

2. Jatin Mavani vs Rare Township Pvt. Ltd.

Citation: Maharashtra RERA, Complaint No. CC00600000005501, Maharera No.


P51800000756 dated 14.12.2018

Facts
The complainant had booked a flat in Ghatkopar for `1.33 crores (approx.) and was assured
by the respondent that work on the project will start by August 31, 2011 and it will be
completed within 36 months. But the respondent failed to complete the project on time, he also
did not register the agreement.
Issue
Whether multiple Proceedings on same issue are Permissible under RERA 2016?
Key Ratio Decidendi
♦ The Maharashtra Real Estate Regulatory Authority (MahaRERA) has held that multiple
proceedings on the same issue cannot be allowed under the Real Estate (Regulation &
Development) Act, 2016.
♦ “The record shows that the complainant was also one of the members of the said
Association. The said fact has not been denied by the complainant. The said complaint was
filed with the MahaRERA by 'the allottees seeking directions from MahaRERA to allow
cancellation of the flat bookings by the allottees who want to do so without cancellation fees
and direct the respondent to refund the amount paid by the allottees along with interest and
compensation. In the said complaint, after hearing arguments of both sides, the MahaRERA
has already given verdict directing the respondent promoter to execute the registered
agreements for sale with the members of the Rising City Ghatkopar Association. Since the
complainant is also a party, to the said proceeding, he cannot separately agitate this

© The Institute of Chartered Accountants of India


Real Estate (Regulation & Development) Act, 2016 2.3

complaint before the MahaRERA, as it will amount to multiple proceedings on the same
issue, which is not permissible in the RERA Act, 2016.”
Link to Order
https://maharerait.mahaonline.gov.in/SearchList/SearchJudgements

3. Lavasa Corporation Limited vs Jitendra Jagdish Tulsiani

Citation: Bombay HC, Civil Second Appeal (Stamp) No.9717 of 2018 dated 07.08.2018

Facts
♦ The Respondents-Allottees were bonafide purchasers of their respective apartments in
the projects/buildings, in a township scheme of Lavasa in Pune. The allottees were
given flats not under a transaction of ‘sale’, but under ‘Agreements of Lease’ where in
the flats were leased out to the allotees for the period of 999 years. The Allottees had
paid consideration to Lavasa almost, to the extent of 80% of the sale price and the
lease rent was only ` 1 per annum. They also paid substantial amount towards the
stamp-duty and the registration charges.
♦ As per the ‘Agreements of Lease’ executed between the parties, the project was to be
completed and the possession of the apartments was to be handed over to the
Allottees within a period of 24 months. However, after waiting for 6 to 7 years for
getting the project completed and after making several enquiries with Lavasa about the
progress of the said project, the Allottees found that there were no chances of the
project being completed in a near future. Hence, after Lavasa registered itself with the
RERA, the Allottees approached the ‘Adjudicating Authority’ under the MahaRERA with
an application, under Section 18 of the RERA, for compensation with interest for every
month of the delay in handing over possession of the apartments and for various other
reliefs, to which they are entitled under the RERA.
Issue
1. Whether the provisions of the RERA would apply in case of an 'Agreement to Lease'?
2. Whether the definition of the term “Promoter”, as provided in Section 2(zk) in the
RERA, would include a 'Lessor', and 'whether the remedy provided to the 'Allottees'
under Section 18 of the RERA can be available only against the 'Promoter', or, in that
sense, also against a 'Lessor'?
Key Ratio Decidendi
♦ The Bombay High Court has held that provisions of the Real Estate (Regulation and
Development) Act, would apply in case of agreements styled as 'Agreement to Lease'

© The Institute of Chartered Accountants of India


2.4 Economic Laws

when the lease period is long (say 999 years) and when the ‘lessee’ has paid a
substantial amount as consideration.
♦ “In an 'Agreement of Lease', the 'Lessee' does not pay more than 80% of the
consideration amount towards the price of the said apartment. In an 'Agreement of
Lease', the rent cannot be `1/- per annum only, for such an apartment, market rate of
which is more than `40 lakhs. In an 'Agreement of Lease', parties do not pay the
registration charges and stamp duty on the market value of the said apartment. The
'Agreement of Lease' also cannot be for such a long term for '999 years'. This long
period of lease in itself is sufficient to hold that, it is not an 'Agreement of Lease', but, in
reality, an 'Agreement of Sale'.”
♦ The court observed that very object of the RERA is to protect the consumers, the
persons, who have invested their hard-earned money by entering into an 'Agreement',
which is in the nature of purchase of the apartment itself, mere nomenclature of the
document as 'Agreement of Lease' will not in any way take away the rights given to
them by the statute.
♦ “If the entire 'Agreement' is perused as such, then it becomes apparent on the face of it
also, that it cannot be termed or treated as an 'Agreement of Lease', but, in its real
purport, it is an 'Agreement of Sale'. The very fact that more than 80% of the entire
consideration amount is already paid by the Respondents to the Appellant and the
lease premium agreed is only of `1/- per annum, including the clause relating to the
period of lease of 999 years, are self-speaking to prove that, in reality, the transaction
entered into by the parties is an 'Agreement of Sale' and not an 'Agreement of Lease';
though it is titled as such. The law is well settled that the nomenclature of the document
cannot be a true test of its real intent and the document has to be read as a whole to
ascertain the intention of the parties. if the entire 'Agreement' is perused as such, then
it becomes apparent on the face of it also, that it cannot be termed or treated as an
'Agreement of Lease', but, in its real purport, it is an 'Agreement of Sale'. The very fact
that more than 80% of the entire consideration amount is already paid by the
Respondents to the Appellant and the lease premium agreed is only of `1/- per annum,
including the clause relating to the period of lease of 999 years, are self-speaking to
prove that, in reality, the transaction entered into by the parties is an 'Agreement of
Sale' and not an 'Agreement of Lease'; though it is titled as such. The law is well settled
that the nomenclature of the document cannot be a true test of its real intent and the
document has to be read as a whole to ascertain the intention of the parties.”
Link to Order
https://bombayhighcourt.nic.in/generatenewauth.php?auth=cGF0aD0uL2RhdGEvanVkZ2VtZW
50cy8yMDE4LyZmbmFtZT1DU0E5NzE3MTgucGRmJnNtZmxhZz1OJnJqdWRkYXRlPSZ1cGx
vYWRkdD0wNy8wOC8yMDE4JnNwYXNzcGhyYXNlPTA1MTAxOTEzMzY0OA==

© The Institute of Chartered Accountants of India


Real Estate (Regulation & Development) Act, 2016 2.5

4. Neelkamal Realtors Suburban Pvt. Ltd. and anr vs UOI

Citation: Bombay HC, Original Writ Petition No. 2737 of 2017 dated 06.12.2017

Facts
The petitioners are builders and developers who are aggrieved by the new provisions of the
said Act which contains specific provisions to tackle problems like delay in possession,
arbitrary interests levied on home buyers etc.
Petitioners had challenged Section 3, 5, 7, 8, 11(h), 14(3), 15, 16, 18, 22, 43(5),59,60,61,63
and 64 of the Real Estate (Regulations and Development) Act, 2016 and Rules 3(f),
4,5,6,7,8,18,19, 20 and 21 of the Registration of real estate projects, Registration of real
estate agents, rates of interest and disclosures on website Rules, 2017.
Issue
Whether Sections 18, 38, 59, 60, 61, 63 and 64 of the RERA Act, are violative of Articles 14,
19(1)(g) and 20(1) of the Constitution of India and amount to unreasonable restrictions?
Key Ratio Decidendi
♦ The builders had challenged Section 18 of the Act, under which they will have to return
monies received with interest, if they fail to hand over possession or complete the
project in a time bound manner, if the allottee wishes to withdraw from a project. To this
the Court said, “…. in case the allottee wishes to withdraw from the project, without
prejudice to any other remedy available, to return the amount received by him in
respect of that apartment with interest at such rate as may be prescribed in this behalf
including compensation. If the allottee does not intend to withdraw from the project he
shall be paid by the promoter interest for every month’s delay till handing over of the
possession. The requirement to pay interest is not a penalty as the payment of interest
is compensatory in nature in the light of the delay suffered by the allottee that has paid
for his apartment but has not received possession of it. The obligation imposed on the
promoter to pay interest till such time as the apartment is handed over to him is not
unreasonable. The interest is merely compensation for use of money.”
♦ In case the promoter establishes and the authority is convinced that there were
compelling circumstances and reasons for the promoter in failing to complete the
project during the stipulated time, the authority shall have to examine as to whether
there were exceptional circumstances due to which the promoter failed to complete the
project. Such an assessment has to be done by the authority on case to case basis
and exercise its discretion to advance the purpose and object of RERA by balancing
rights of both, the promoter and the allottee.

© The Institute of Chartered Accountants of India


2.6 Economic Laws

♦ The Bench, while upholding the provisions of RERA said, “RERA is not a law relating to
only regulating concerns of the promoters but its object is to develop the real estate
sector, particularly the incomplete projects, across the country.”
♦ Hence, the court held that the challenge to constitutional validity of first proviso
to Section 3(1), Section 3(2)(a), explanation to Section 3, Section 4(2)(l)(C),
Section 4(2)(l)(D), Section 5(3) and the first proviso to Section 6, Sections 7, 8, 18,
22, 38, 40, 59, 60, 61, 63, 64 of the Real Estate (Regulation and Development) Act,
2016 fails. These provisions are held to be constitutional, valid and legal.”
♦ However, Section 46 (b) of the Act was set aside as it included any officer who has held
the post of Additional Secretary to be eligible for membership of the two-member
tribunal. Court held that the majority of the total members of the tribunal should always
be judicial members.
Link to Order
https://bombayhighcourt.nic.in/generatenewauth.php?auth=cGF0aD0uL2RhdGEvanVkZ2VtZW
50cy8yMDE3LyZmbmFtZT1PU1dQMjAxMDE3LnBkZiZzbWZsYWc9TiZyanVkZGF0ZT0mdXBs
b2FkZHQ9MDYvMTIvMjAxNyZzcGFzc3BocmFzZT0wNTEwMTkxMzUxNTA=

5. Simmi Sikka v. M/s Emaar MGF Land Ltd.

Citation: Haryana RERA at Gurugram, RERA-GRG-7-2018 dated 10.07.2018

Facts
The Complainant had booked a shop bearing No. EPS-SF-019. She had paid an amount of
` 21,74,234 and ` 6,96,472 on 21.03.2014. On demand, an amount of ` 30,00,000/- was paid
in advance which was to be counted towards early rebate payment with interest @ 12% on
19.04.2014. The buyer agreement was signed on 09.05.2014. As per agreement, the time of
completion of project was 30 months from the date of execution of agreement.
The contention of the complainant that the respondents were not acting as per the agreement
and thus defaulted in regard to the schedule of construction because of infighting inter-alia
between them and no construction took place till the period of two years and the period of 30
months had expired on 9th November 2016. The interest accrued on the amount has been
calculated and claimed as ` 15,62,055 compounded up to September 2017.
Issue
Objection raised by Respondent:
1. Project of the respondent is not an ongoing project as per Rule 2(1)(o) of the Haryana
Real Estate (Regulation & Development) Rules, 2017.

© The Institute of Chartered Accountants of India


Real Estate (Regulation & Development) Act, 2016 2.7

2. That the respondent had received deemed occupation certificate prior to the
publication of the rules.
3. That the present complaint for compensation and interest under section 12, 14, 18
and 19 of the Act is maintainable only before the adjudicating officer under the Act
and not with this authority.
Key Ratio Decidendi
♦ The RERA Act, mentions nowhere that it is applicable only for the registered projects.
♦ The RERA Act, provides certain categories of projects which are not required to be
registered but these are within the ambit of the Act. These projects mentioned in
section 3(2) have been taken out of the registration requirement but not out of the
purview of other provisions of the Act.
♦ The provisions regarding registration and obligation during registration are applicable
only for the registered projects.
♦ The obligations of the promoter’s post expiry of the validity of the registration provided
in the Act are applicable to even the real estate projects exempted from registration.
♦ The projects which were completed and handed over during the last 5 years are
covered for the purpose of workmanship and structural defect liability. A complaint may
be filed by the allottee in such matter in case the possession of the real estate was
within 5 years prior to the date of the complaint.
♦ All real estate projects are covered for land title defect liability.
♦ A complaint pertaining to violation of the provisions of RERA Act, Haryana RERA
Rules, and Regulations thereunder, may be filed by any aggrieved person in respect of
any real estate project as per the definition given in section 2(zn) of RERA Act.
Link to Order
https://credai.org/assets/upload/judgements/resources/haryana-real-estate-regulatory-authority-
ms-simmi-sikka-vs--ms-emaar-mgf-land-limited.pdf

6. Keystone Realtors Pvt. Ltd. vs. Anil V Tharthare & Ors.

Citation : SC, Civil Appeal No.2435 of 2019 dated 03.12.2019

Facts
The construction area of the Project was expanded from 32,395.17 square meters to
40,480.88 square meters, the Developer did not comply with the procedure under para 7(ii) of

© The Institute of Chartered Accountants of India


2.8 Economic Laws

the Environmental Impact Assessment (“EIA”) Notification but rather sought an amendment to
the earlier environmental clearance.
Issue
1. What is exact interpretation of the EIA Notification?
2. Is there need for Fresh Environmental clearance for expansion beyond limits
approved by prior EC?
Key Ratio Decidendi
It is only with industrial, thermal power and other such related operations that one can decide
on parameters of pollution. Development projects like highways, airports and other
infrastructure projects which seek to expand might have a detrimental impact due to factors
such as change in land use despite this, the project proponent can certify that there is no
change in pollution load and hence expansion is to be allowed. The current process seeks a
detailed EIA report to determine whether impacts can be mitigated. If the amendment is
brought into force, it will simply do away with this critical and necessary step in the
environmental clearance process. Therefore, this amendment should not be allowed.
It was further noted that as on the date of the impugned order construction at the project site
had already been completed. A core tenet underlying the entire scheme of the EIA Notification
is that construction should not be executed until ample scientific evidence has been compiled
so as to understand the true environmental impact of a project. By completing the construction
of the project, the appellant had denied the third and fourth respondents’ ability to evaluate the
environmental impact and suggest methods to mitigate any environmental damage. At that
stage, only remedial measures could have been taken. The NGT which had already directed
the appellant to deposit Rupees one crore and has set up an expert committee to evaluate the
impact of the appellants project and suggest remedial measures. In view of these
circumstances, the court uphold the directions of the NGT and directed that the committee
continue its evaluation of the appellants project so as to bring its environmental impact as
close as possible to that contemplated in the EC dated 2 May 2013 and also suggest the
compensatory exaction to be imposed on the appellant.
Conclusion
As on the date of the impugned order, construction at the project site had already been
completed. A core tenet underlying the entire scheme of the EIA Notification is that
construction should not be executed until ample scientific evidence has been compiled so as
to understand the true environmental impact of a project. By completing the construction of the
project, the Appellant denied the third and fourth Respondents the ability to evaluate the
environmental impact and suggest methods to mitigate any environmental damage. At this
stage, only remedial measures may be taken. The NGT has already directed the Appellant to

© The Institute of Chartered Accountants of India


Real Estate (Regulation & Development) Act, 2016 2.9

deposit Rupees one crore and has set up an expert committee to evaluate the impact of the
Appellant's project and suggest remedial measures.
Link :
https://main.sci.gov.in/supremecourt/2019/7891/7891_2019_8_1501_18876_Judgement_0
3-Dec-2019.pdf

7. Mumbai Grahak Panchayat, Magicbricks, 99acres.com,


Makkan.com, Housing .com and others

Citation : MahaRera Order in the Suo Moto Enquiry No.17/2018 dated 03.10.2019

Facts
The application was filed by Mumbai Grahak Panchayat before the MahaRERA informing that
the web portals dealing with real estate projects facilitating sale/purchase of real estate plots,
apartment act as ‘real estate agent’ under RERA and in order to make them responsible, they
should be RERA registered.
Issue
1. Whether digital portals “introduce” seller with buyer in sale of real estate?
2. Whether digital portals negotiate in sale of real estate?
3. Whether digital portals “facilitate” sale of real estate sale?
4. Whether digital portals collect charges/ fees/ renumerations/ commission?
5. Whether digital portals can be exempted from the definition of real estate agent, they
being intermediary under the IT Act.
Key Ratio Decidendi
Issue 1:
Portals when they collect the details of the viewer and share them with advertiser/seller and
also disclose the information of promoters to buyers, they introduce the parties to the sale
transaction.
Issue 2:
If the portal simply provide the information about the real estate project, its offering for sale to
the public at large, then they are simply the agencies engaged for advertisement and when an
individual is targeted by contacting and persuading him by the portals for sale and purchase of
listed properties they come under the legal definition of negotiation.

© The Institute of Chartered Accountants of India


2.10 Economic Laws

Issue 3:
Web Portals introduce the buyer and seller with each other, they provide the information of the
project to the buyer, they arrange virtual tour of the project and also provide other information
useful for taking an informative decision. Hence, they facilitate the sale of the real estate
project.
Issue 4:
Once any monitory gain is derived for the purpose of performing any act of the real estate
agent by whichever name it amounts the receipt of the fees under the RERA.
Issue 5:
The Parliament has not carved out any exceptions to the applicability of the provisions of
RERA, Hence, we hold that RERA overrides section 79 of the IT Act.
Conclusion
1. Portals whose activities are simply confined to advertisement defined by section 2 (b)
of RERA, need not register themselves as real estate agent, provided in disclaimer
they declare that they are simply advertising agencies and advise the viewers to
cross check the information from other sources including RERA website.
2. Other portals which carry the function of real estate agent as explained above need
registration. Such portals are directed to register with MahaRera within two months, if
their activities are spread within the territorial jurisdiction of it.
3. Portals will have to register themselves with real estate regulatory authority of a state
where their activities are going on until the registration at national level is made
permissible.
Link
https://maharera.mahaonline.gov.in/Upload/PDF/Suo%20moto%20Grahak%20Panchayat.pdf

8. Ravinder Kaur Grewal vs Manjit Kaur and others

Citation : SC, Civil Appeal No. 7764 of 2014, dated 07.08.2019

Facts
The predecessor of the appellants herein ­ Harbans Singh, son of Niranjan Singh, resident of
Sangrur, Punjab against his real brothers Mohan Singh (original defendant No. 1) and Sohan
Singh (original defendant No. 2) for a declaration that he was the exclusive owner in respect of
land admeasuring 11 kanals 17 marlas comprising khasra Nos. 935/1 and 935/2 situated at
Mohalla Road and other properties referred to in the Schedule. He asserted that there was a
family settlement with the intervention of respectable persons and family members,

© The Institute of Chartered Accountants of India


Real Estate (Regulation & Development) Act, 2016 2.11

whereunder his ownership and possession in respect of the suit land including the
constructions thereon (16 shops, a samadhi of his wife – Gurcharan Kaur and one service
station with boundary wall) was accepted and acknowledged. Structures were erected by him
in his capacity as owner of the suit land. It is stated that in the year 1970 after the purchase of
suit land, some dispute arose between the brothers regarding the suit land and in a family
settlement arrived at then, it was clearly understood that the plaintiff – Harbans Singh would
be the owner of the suit property including constructions thereon and that the name of Mohan
Singh (original defendant No. 1) and Sohan Singh (original defendant No. 2) respectively
would continue to exist in the revenue record as owners to the extent of half share and the
plaintiff would have no objection in that regard due to close relationship between the parties.
However, the defendants raised dispute claiming half share in respect of which Harbans Singh
(plaintiff) was accepted and acknowledged to be the exclusive owner and as a result of which
it was decided to prepare a memorandum of family settlement incorporating the terms already
settled between the parties, as referred to above. The stated memorandum was executed by
all parties on 10.3.1988. However, after execution of the memorandum of family settlement
dated 10.3.1988, the defendants once again raised new issues to resile from the family
arrangement. As a result, Harbans Singh (plaintiff) decided to file suit for declaration on
9.5.1988, praying for a decree that he was the owner in possession of the land admeasuring
11 kanals 17 marlas comprising of khasra Nos. 935/1 and 935/2 situated at Mohalla Road. An
alternative plea was also taken that since plaintiff was in possession of the whole suit property
to the knowledge of the defendants openly and adversely for more than twelve years, he had
acquired ownership rights by way of adverse possession.
Issue
1. Whether a person claiming the title by virtue of adverse possession can maintain a
suit under Article 65 of Limitation Act for declaration and permanent injunction.
2. Whether Article 65 of the Limitation Act only enables a person to set up a plea of
adverse possession as a defendant and cannot protect possession as a plaintiff?
Key Ratio Decidendi
A person in possession cannot be ousted by another person except by due procedure of law
and once 12 years’ period of adverse possession is over, even owner’s right to eject him is
lost and the possessory owner acquires right, title and interest possessed by the outgoing
person/owner as the case may be against whom he has prescribed. Once the right, title or
interest is acquired it can be used as a sword by the plaintiff as well as a shield by the
defendant within ken of Article 65 of the Act and any person who has perfected title by way of
adverse possession, can file a suit for restoration of possession in case of dispossession.

© The Institute of Chartered Accountants of India


2.12 Economic Laws

Conclusion
Article 65 of Limitation Act, 1963 not only enables a person to set up a plea of adverse
possession as a shield as a defendant but also allows a plaintiff to use it as a sword to protect
the possession of immovable property or to recover it in case of dispossession.
Link
https://main.sci.gov.in/supremecourt/2008/4680/4680_2008_33_1501_23189_Judgement_
31-Jul-2020.pdf

9. Bikram Chatterji & Ors. vs. Union of India & Ors.

Citation : SC, Writ Petition Civil No.940/2017, dated 23.07.2019

Facts
Amrapali Group of Companies proposed to construct 42,000 flats by assuring delivery of
possession in 36 months to the home buyers on the land which was given on lease by
Noida/Greater NOIDA Authority (“Authorities”). Later, Amarpali group were found in serious
breach of their obligation to deliver the projects and the payment due to the Authorities and
the Banks.
Issue
1. Whether the charges levied by officials, banks, home purchasers and development
agencies are valid?
2. Whether the Amrapali Group’s RERA registration be cancelled?
Key Ratio Decidendi
The Supreme Court ordered a forensic audit to look into the affairs of the Amrapali Group. The
forensic report confirmed that
(i) there had been diversion of funds by the Group by incorporating shell/dummy
companies;
(ii) the promoters had created a web of more than 150 companies for routing of funds
and creating assets;
(iii) the homebuyer’s funds along with the loans from the banks were diverted to other
companies/directors, such funds were used by the promoters to acquire personal
assets, properties and applied towards other business ventures.
The Supreme Court also observed that the mortgage created in favour of the lenders required
an NOC from the Authorities which was issued subject to certain conditions such as full/ timely
payment of the lease rents/premiums to the Authorities. The Court held that in the eyes of law,

© The Institute of Chartered Accountants of India


Real Estate (Regulation & Development) Act, 2016 2.13

no valid mortgage had been created in favour of the banks on account of the conditional NOC
which had not been fulfilled.
In light of the observations made and the findings of the forensic report, the Apex Court issued
the following orders:
I The RERA registrations of the various projects of the Group were cancelled and the
National Building Construction Corporation (NBCC) was assigned the task of
completing the projects.
II The Court Receiver has been given the right of the lessee and is authorised to
execute the tripartite agreement and ensure that the title is passed on to the home
buyers, free from any encumbrances.
III The Supreme Court further directed that the Authorities and the banks will have to
recover their dues from other properties and assets of the Group which have been
attached.
IV The homebuyers have been directed to deposit the outstanding amount as per the
payment schedule under the builder buyer agreement with the promoters/developers
in a court administered bank account within three months. The amount deposited by
the homebuyers will be disbursed by the Court order as per the stage-wise
completion by NBCC.
V Further, the Court advised appropriate action to be taken against the leaseholders of
similar projects not only in Noida and Greater Noida but in other cities as well. Central
Govt. Ministries and State Govt. Agencies have been further directed to ensure
completion of other projects in a time-bound manner as contemplated in RERA and
ensure that the home buyers are not defrauded.
VI Lastly, the Noida and Greater Noida Authorities were further directed to issue
completion certificate and registered conveyance deed to be executed within one
month concerning the projects where the homebuyers were already residing.
Conclusion
RERA Amrapali Group registration under RERA Act shall be revoked and NBCC (India) Ltd is
finalizing various projects.
The separate lease agreements issued for projects under consideration in favour of Amrapali
Group Authorities are revoked and all the rights will now be vested in the Court Receiver who
has authority to alienate, lease out or take any decision to raise funds. The Court Receiver will
pay money raised to NBCC will complete the project with 8% profit margin (senior Adv., Shri
R. Venkataramani).

© The Institute of Chartered Accountants of India


2.14 Economic Laws

The Authorities and Banks do not have the right to sell the property of the property buyers or
the land leased for payment of their dues. They have to receive all their charges from the
selling of other assets attached to the Amrapali Group.
The right of the lessee shall be enshrined in the Court Receiver (formerly with the Amrapali
Group) and shall, by means of an authorized person on his behalf, conclude a tripartite
agreement and perform all other acts as may be necessary and shall also make sure that the
title is handed over to the home-buyers and that the possession is handed over to them.
Link
https://main.sci.gov.in/supremecourt/2017/30157/30157_2017_4_1501_15413_Judgement_23
-Jul-2019.pdf

10. Sushil Ansal Vs. Ashok Tripathi

Citation : NCLAT, Company Appeal (AT) (Insolvency) No. 452 of 2020 dated
14.08.2020

Facts
The two (2) homebuyers in this case, had jointly booked two (2) units under the real estate
project at Sushant Golf City, Lucknow with APIL. In September 2014, builder buyer
agreements had been executed and as per the agreements, APIL was required to complete
construction within two (2) years from the date of commencement of construction. However,
APIL had failed to complete the projects within the stipulated time period and the homebuyers
accordingly approached RERA. RERA in this case issued recovery certificates to the
homebuyers against APIL. UP RERA then granted a decree to the homebuyers for an amount
of about Rupees Seventy Three (73) Lakhs, after APIL failed to make the payments to the
homebuyers. The homebuyers then as financial creditors approached the NCLT under Section
7 of the Insolvency and Bankruptcy Code, 2016 ("I&B Code"). The homebuyers approached
the NCLT against the default of financial debt committed by APIL, on account of non-payment
of the principal amount along with the penalty as decreed by the UP RERA besides the
recovery certificate. Instead of seeking execution of the decree under civil law, the
homebuyers approached the NCLT to initiate insolvency proceedings against APIL. The NCLT
thereafter admitted the application and declared a moratorium on the assets of APIL and
appointed an interim resolution professional who received claims from more than two hundred
and fifty (250) allottees. APIL's director Mr. Sushil Ansal, thereafter, approached the NCLAT
contesting the order passed by NCLT.
It is pertinent to note that during the pendency of the appeal at the NCLAT, APIL and the two
(2) homebuyers filed a joint application before the NCLAT for withdrawal of the application

© The Institute of Chartered Accountants of India


Real Estate (Regulation & Development) Act, 2016 2.15

under Section 7 of the I&B Code on the basis of a settlement deed having been executed
between the parties on June 1, 2020 by invoking Rule 11 of the NCLAT Rules, 2016.
Issue
1. Whether this is a fit case for invoking Rule 11 of the NCLAT Rules to allow the parties
to settle the dispute?
2. Whether the application filed by the homebuyers under Section 7 of the I&B Code
was not maintainable?
Key Ratio Decidendi
♦ A corporate debtor is permitted to seek exit from Corporate Insolvency Resolution
Process ("CIRP") at the pre-admission stage. It can also seek exit at the post
admission stage before constitution of the committee of creditors. The NCLAT further
states that while a party to CIRP can approach the NCLT directly for exercise of its
inherent powers under Rule 11 of the NCLT, Rules 2016 for withdrawal of application
under Section 7 of the I&B Code or disposal of such application based on the
settlement worked out by the parties, however, such power can be exercised by the
NCLT only before constitution of the committee of creditors. Further, exercise of such
inherent powers is discretionary and must only be invoked to meet the ends of justice
or prevent abuse of process of court. Accordingly, on analysing the facts and
circumstances surrounding the present case, the NCLAT refused to invoke Rule 11
and held that while the committee of creditors had not been constituted, invoking such
settlement would be detrimental to the interests of the other claimants including the
other allottees and would not be in consonance with the object of the I&B Code. The
NCLAT held that in a case where interests of the majority of stakeholders are in
serious jeopardy, it would be inappropriate to allow settlement with only two (2)
creditors which may amount to perpetrating of injustice. Exercise of inherent powers
is such cases would be a travesty of justice.
♦ It is clear that an application for initiation of CIRP by allottees under a real estate
project is required to be filed jointly by not less than one hundred (100) of such
allotees or not less than ten percent (10%) of the total number of allotees thereunder.
Thus, it is clear that an application at the instance of a single allottee or by a group of
allottees falling short of the prescribed threshold limit would not be maintainable.
♦ The NCLAT observes that the case set up by the homebuyers in the present case is
not on the strength of a transaction having the commercial effect of a borrowing
thereby clothing the status of 'financial creditor' but instead is on the strength of being
'decree-holders', which owed its genesis to the recovery certificate issued by the UP
RERA. In view thereof, the NCLAT then proceeded to determine whether in the
homebuyers projected capacity as 'decree-holders', they could maintain an
application under Section 7 of the I&B Code. The NCLAT observed that a 'decree-

© The Institute of Chartered Accountants of India


2.16 Economic Laws

holder' is undoubtedly covered by the definition of 'creditor' under the I&B Code but
would not fall within the ambit of 'financial creditor' unless the debt was disbursed
against the consideration for time value of money or falls within any of the clauses
enumerated under the definition of 'financial debt' under the I&B Code. In the present
case, the RERA had conducted recovery proceedings at the instance of the
homebuyers against APIL which culminated in the issuance of the recovery certificate
and passing of an order under Section 40 of the Real Estate (Regulation and
Development) Act, 2016, directing the concerned authority to recover an amount of
about Rupees Seventy Three (73) Lakh from APIL as an arrear of land revenue. In
view thereof, the NCLAT held that the answer to the question as to whether a decree-
holder would fall within the definition of 'financial creditor' has to be an empathetic
'No' as the amount claimed under the decree is an adjudicated amount and not a debt
disbursed against the consideration for the time value and does not fall within the
ambit of any of the clauses enumerated under the definition of 'financial debt' under
Section 5(8) of the I&B Code.
Conclusion
The NCLAT held that on their own showing, the homebuyers were decree holders seeking
execution of money due under the recovery certificate which is impermissible within the ambit
of Section 7 of the I&B Code. Clearly the homebuyers application for triggering of CIRP would
not be maintainable as allottees. Further, in the present case, 'decree-holder' although
included in the definition of 'creditor' would not fall within the definition of 'financial creditor'
and thus the homebuyers could not seek initiation of CIRP on that basis. Accordingly, the
NCLAT set aside all orders passed by the NCLT on March 17, 2020 in relation to the
insolvency proceedings initiated against APIL.
Link
https://nclat.nic.in/Useradmin/upload/386586195f364d7da63bc.pdf

© The Institute of Chartered Accountants of India


3
Insolvency & Bankruptcy Code, 2016

1. Swiss Ribbons Pvt. Ltd. & Anr. V. Union of India & Ors.

Citation: Supreme Court, WP(C) No.99 of 2018, dated 25.01.2019

Facts
The constitutional validity of the Insolvency and Bankruptcy Code, 2016 was challenged in
various petitions before the Hon’ble Court.
Issue
Whether IBC was discriminatory and unfair to operational creditors as compared to financial
creditors?
Key Ratio Decidendi
1. The court held that financial creditors are clearly different from operational creditors
and therefore, there is obviously an intelligible differentia between the two which has
a direct relation to the objects sought to be achieved by the Code. Referring to the
Code, the bench explained the difference between Financial Creditor and Operational
Creditor. It said: "A perusal of the definition of financial creditor and financial debt
makes it clear that a financial debt is a debt together with interest, if any, which is
disbursed against the consideration for time value of money. It may further be money
that is borrowed or raised in any of the manners prescribed in Section 5(8) or
otherwise, as Section 5(8) is an inclusive definition. On the other hand, and
'operational debt' would include a claim in respect of the provision of goods or
services, including employment, or a debt in respect of payment of dues arising under
any law and payable to the Government or any local authority."
2. "The main thrust against the provision of Section 12A is the fact that ninety per cent
of the committee of creditors has to allow withdrawal. This high threshold has been
explained in the ILC Report as all financial creditors have to put their heads together
to allow such withdrawal as, ordinarily, an omnibus settlement involving all creditors
ought, ideally, to be entered into. This explains why ninety per cent, which is
substantially all the financial creditors, have to grant their approval to an individual

© The Institute of Chartered Accountants of India


3.2 Economic Laws

withdrawal or settlement. In any case, the figure of ninety per cent, in the absence of
anything further to show that it is arbitrary, must pertain to the domain of legislative
policy, which has been explained by the Report (supra). Also, it is clear, that under
Section 60 of the Code, the committee of creditors do not have the last word on the
subject. If the committee of creditors arbitrarily rejects a just settlement and/or
withdrawal claim, the NCLT, and thereafter, the NCLAT can always set aside such
decision under Section 60 of the Code. For all these reasons, we are of the view that
Section 12A also passes constitutional muster."
3. Under the Code, the resolution professional is given administrative as opposed to
quasi-judicial powers. Even when the resolution professional is to make a
'determination' under Regulation 35A, he is only to apply to the Adjudicating Authority
for appropriate relief based on the determination. "Unlike the liquidator, the resolution
professional cannot act in a number of matters without the approval of the committee
of creditors under Section 28 of the Code, which can, by a two-thirds majority, replace
one resolution professional with another, in case they are unhappy with his
performance. Thus, the resolution professional is really a facilitator of the resolution
process, whose administrative functions are overseen by the committee of creditors
and by the Adjudicating Authority."
4. “Even the categories of persons who are ineligible under Section 29A, which includes
persons who are malfeasant, or persons who have fallen foul of the law in some way,
and persons who are unable to pay their debts in the grace period allowed, are
further, by this proviso, interdicted from purchasing assets of the corporate debtor
whose debts they have either wilfully not paid or have been unable to pay. The
legislative purpose which permeates Section 29A continues to permeate the Section
when it applies not merely to resolution applicants, but to liquidation also.
Consequently, this plea is also rejected.”
5. It will be seen that the reason for differentiating between financial debts, which are
secured, and operational debts, which are unsecured, is in the relative importance
of the two types of debts when it comes to the object sought to be achieved by the
Insolvency Code. We have already seen that repayment of financial debts infuses
capital into the economy in as much as banks and financial institutions are able,
with the money that has been paid back, to further lend such money to other
entrepreneurs for their businesses. This rationale creates an intelligible differentia
between financial debts and operational debts, which are unsecured, which is
directly related to the object sought to be achieved by the Code. In any case,
workmen's dues, which are also unsecured debts, have traditionally been placed
above most other debts. Thus, it can be seen that unsecured debts are of various
kinds, and so long as there is some legitimate interest sought to be protected,

© The Institute of Chartered Accountants of India


Insolvency & Bankruptcy Code, 2016 3.3

having relation to the object sought to be achieved by the statute in question,


Article 14 does not get infracted.
Link to Order
https://www.sci.gov.in/supremecourt/2018/4653/4653_2018_Judgement_25-Jan-2019.pdf Also
Read: https://www.ibbi.gov.in/webadmin/pdf/whatsnew/2019/Jan/Swiss%20Ribbons_2019-01-
27%2019:08:24.pdf

2. Pioneer Urban Land and Infrastructure Ltd and Anr vs Union of


India

Citation: Supreme Court, WP(C) No.43 of 2019, dated 09.08.2019

Facts
The challenge was primarily to the explanation added to Section 5(8)(f) of the Code that "any
amount raised from an allottee under a real estate project shall be deemed to be an amount
having the commercial effect of a borrowing". Amounts having the commercial effect of
borrowing are treated as 'financial debt' as per Section 5(8)(f). Therefore, any amount invested
by a person in a real estate project for allotment of apartments will be deemed as "financial
debt".
Issue
♦ Whether treating homebuyers as financial creditors amounts to treating unequal’s as
equals?
♦ Whether homebuyers have a separate remedy under the RERA Act for their grievance
redressal?
♦ Whether giving advance payment for flat allotment can be regarded as 'financial
lending'?
Key Ratio Decidendi
1. What is unique to real estate developers vis-à-vis operational debts, is the fact that, in
operational debts generally, when a person supplies goods and services, such person
is the creditor and the person who has to pay for such goods and services is the
debtor. In the case of real estate developers, the developer who is the supplier of the
flat/apartment is the debtor in as much as the home buyer/allottee funds his own
apartment by paying amounts in advance to the developer for construction of the
building in which his apartment is to be found. Another vital difference between
operational debts and allottees of real estate projects is that an operational
creditor has no interest in or stake in the corporate debtor, unlike the case of an
allottee of a real estate project, who is vitally concerned with the financial

© The Institute of Chartered Accountants of India


3.4 Economic Laws

health of the corporate debtor, for otherwise, the real estate project may not be
brought to fruition. Also, in such event, no compensation, nor refund together
with interest, which is the other option, will be recoverable from the corporate
debtor. One other important distinction is that in an operational debt, there is no
consideration for the time value of money – the consideration of the debt is the goods
or services that are either sold or availed of from the operational creditor. Payments
made in advance for goods and services are not made to fund manufacture of such
goods or provision of such services.
2. The expression "borrow" is wide enough to include an advance given by the
home buyers to a real estate developer for "temporary use" i.e. for use in the
construction project so long as it is intended by the agreement to give
"something equivalent" to money back to the home buyers. The "something
equivalent" in these matters is obviously the flat/apartment." Also of importance is the
expression "commercial effect". "Commercial" would generally involve transactions
having profit as their main aim. Piecing the threads together, therefore, so long as an
amount is "raised" under a real estate agreement, which is done with profit as the
main aim, such amount would be subsumed within Section 5(8)(f) as the sale
agreement between developer and home buyer would have the "commercial effect" of
a borrowing, in that, money is paid in advance for temporary use so that a
flat/apartment is given back to the lender.
3. Referring to Section 88 of the RERA Act, the Court said that it was an additional
remedy, which will not bar other remedies available to a homebuyer.
Link to Order
https://sci.gov.in/supremecourt/2019/1348/1348_2019_5_1501_15816_Judgement_09-Aug-
2019.pdf

3. Macquarie Bank Limited vs Shilpi Cable Technologies Ltd

Citation: Supreme Court, Civil Appeal No. 15135 of 2017, dated 15.12.2017

Facts
♦ Hamera International Private Limited executed an agreement with the appellant,
Macquarie Bank Limited, Singapore, on 27.7.2015, by which the appellant purchased
the original supplier’s right, title and interest in a supply agreement in favour of the
respondent.
♦ The respondent entered into an agreement dated 2.12.2015 for supply of goods worth
US $ 6,321,337.11 in accordance with the terms and conditions contained in the said
sales contract. The supplier issued two invoices dated 21.12.2015 and 31.12.2015.

© The Institute of Chartered Accountants of India


Insolvency & Bankruptcy Code, 2016 3.5

Payment terms under the said invoices were 150 days from the date of bill of ladings
dated 17.12.2015/19.12.2015.
♦ Since amounts under the said bills of lading were due for payment, the appellant sent
an email dated 3.5.2016 to the contesting respondent for payment of the outstanding
amounts. Several such emails by way of reminders were sent, and it is alleged that the
contesting respondent stated that it will sort out pending matters.
♦ Ultimately, the appellant issued a statutory notice under Sections 433 and 434 of the
Companies Act, 1956. A reply dated 5.10.2016 denied the fact that there was any
outstanding amount.
♦ After the enactment of the Code, the appellant issued a demand notice under Section 8
of the Code on 14.2.2017 at the registered office of the contesting respondent, calling
upon it to pay the outstanding amount of US$6,321,337.11. By a reply dated 22.2.2017,
the contesting respondent stated that nothing was owed by them to the appellant. They
further went on to question the validity of the purchase agreement dated 27.7.2015 in
favour of the appellant.
♦ On 7.3.2017, the appellant initiated the insolvency proceedings by filing a petition under
Section 9 of the Code. On 1.6.2017, the NCLT rejected the petition holding that Section
9(3)(c) of the Code was not complied with, in as much as no certificate, as required by
the said provision, accompanied the application filed under Section 9. It, therefore, held
that there being non-compliance of the mandatory provision of Section 9(3)(c) of the
Code, the application would have to be dismissed at the threshold.
Issue
Whether an advocate/lawyer can issue a notice under Section 8 on behalf of the operational
creditor?
Key Ratio Decidendi
1. On a conjoint reading of Section 30 of the Advocates Act, 1961 and Sections 8 and 9
of the Code together with the Adjudicatory Authority Rules and Forms thereunder, a
notice sent on behalf of an operational creditor by a lawyer would be in order. “The
expression “an operational creditor may on the occurrence of a default deliver a
demand notice…..” under Section 8 of the Code must be read as including an
operational creditor’s authorized agent and lawyer.”
2. The expression “shall” in Section 9(3) does not take us much further when it is clear
that Section 9(3)(c) becomes impossible of compliance in cases like the present. It
would amount to a situation wherein serious general inconvenience would be caused
to innocent persons, such as the appellant, without very much furthering the object of
the Act… would have to be construed as being directory in nature.

© The Institute of Chartered Accountants of India


3.6 Economic Laws

3. Hence, the court concluded that a lawyer on behalf of the operational creditor can
issue a demand notice of an unpaid operational debt. The court also held that the
provision contained in Section 9(3)(c) of the Code is not mandatory for initiating
insolvency proceedings.
Link to order
https://sci.gov.in/supremecourt/2017/29095/29095_2017_Judgement_15-Dec-2017.pdf

4. B.K. Educational Services Pvt. Ltd. vs. Parag Gupta and


Associates

Citation: Supreme Court, Civil Appeal No.23988 of 2017, dated 11.10.2018

Issue
Whether the Limitation Act, 1963 will apply to applications that are made under Section 7
and/or Section 9 of the Code on and from its commencement on 01.12.2016 till 06.06.2018?
Key Ratio Decidendi
The Bench observed that since the Limitation Act is applicable to applications filed under
Sections 7 and 9 of the Code from the inception of the Code, Article 137 of the Limitation Act
gets attracted.
“The right to sue”, therefore, accrues when a default occurs. If the default has occurred over
three years prior to the date of filing of the application, the application would be barred under
Article 137 of the Limitation Act, save and except in those cases where, in the facts of the
case, Section 5 of the Limitation Act may be applied to condone the delay in filing such
application.”
Hence, the Limitation Act is applicable to applications filed under Sections 7 and 9 of
Insolvency and Bankruptcy Code from the inception of the Code.
Link to order
https://www.sci.gov.in/supremecourt/2017/41322/41322_2017_Judgement_11-Oct-2018.pdf

© The Institute of Chartered Accountants of India


Insolvency & Bankruptcy Code, 2016 3.7

5. State Bank of India vs. V. Ramakrishnan

Citation: Supreme Court, Civil Appeal No. 3595 of 2018 dated 14.08.2018

Facts
♦ When the SARFAESI Proceedings were pending, the Corporate Debtor initiated the
corporate insolvency resolution process against itself. Moratorium was imposed
statutorily invoking Section 14 of the Code.
♦ In these proceedings, the Personal Guarantor, the Managing Director of the Corporate
Debtor, filed an application contending that Section 14 of the Code would apply to the
personal guarantor as well, as a result of which proceedings against the personal
guarantor and his property would have to be stayed.
♦ National Company Law Tribunal allowed his plea observing that, since under Section
31 of the Code, a Resolution Plan made thereunder would bind the personal guarantor
as well, and since, after the creditor is proceeded against, the guarantor stands in the
shoes of the creditor, Section 14 would apply in favour of the personal guarantor as
well. This view was upheld by National Company Law Appellate Tribunal.
Issue
Whether section 14 moratorium would apply to a personal guarantor of a corporate debtor?
Key Ratio Decidendi
1. Section 60 of the Code, in sub-section (1) thereof, refers to insolvency resolution and
liquidation for both corporate debtors and personal guarantors, the Adjudicating
Authority for which shall be the National Company Law Tribunal, having territorial
jurisdiction over the place where the registered office of the corporate person is
located. This sub-section is only important in that it locates the Tribunal which has
territorial jurisdiction in insolvency resolution processes against corporate debtors. So
far as personal guarantors are concerned, we have seen that Part III has not been
brought into force, and neither has Section 243, which repeals the Presidency-Towns
Insolvency Act, 1909 and the Provincial Insolvency Act, 1920. The net result of this is
that so far as individual personal guarantors are concerned, they will continue to be
proceeded against under the aforesaid two Insolvency Acts and not under the Code.
Indeed, by a Press Release dated 28.08.2017, the Government of India, through the
Ministry of Finance, cautioned that Section 243 of the Code, which provides for the
repeal of said enactments, has not been notified till date, and further, that the
provisions relating to insolvency resolution and bankruptcy for individuals and
partnerships as contained in Part III of the Code are yet to be notified. Hence, it was
advised that stakeholders who intend to pursue their insolvency cases may approach

© The Institute of Chartered Accountants of India


3.8 Economic Laws

the appropriate authority/court under the existing enactments, instead of approaching


the Debt Recovery Tribunals.
2. Section 31(1), in fact, makes it clear that the guarantor cannot escape payment as the
Resolution Plan, which has been approved, may well include provisions as to
payments to be made by such guarantor. This is perhaps the reason that Annexure
VI(e) to Form 6 contained in the Rules and Regulation 36(2) referred to above,
require information as to personal guarantees that have been given in relation to the
debts of the corporate debtor. Far from supporting the stand of the Respondents, it is
clear that in point of fact, Section 31 is one more factor in favour of a personal
guarantor having to pay for debts due without any moratorium applying to save him.
3. Hence, Section 14 of the Insolvency and Bankruptcy Code, 2016, which
provides for a moratorium for the limited period mentioned in the Code, on
admission of an insolvency petition, would not apply to a personal guarantor of
a corporate debtor.
Link to order
https://www.sci.gov.in/supremecourt/2018/11958/11958_2018_Judgement_14-Aug-2018.pdf

6. Mobilox Innovations Private Limited vs. Kirusa Software Private


Limited.

Citation: Supreme Court, Civil Appeal No. 9405 of 2017 dated 21.09.2017

Facts
♦ Kirusa issued a demand notice to Mobilox as an Operational Creditor under the Code,
demanding payment of certain dues. Mobilox issued a reply to the demand notice
(“Mobilox Reply”) inter alia stating that there exists certain serious and bona fide
disputes between the parties and alleged a breach of the terms of a non-disclosure
agreement by Kirusa. Kirusa filed an application under Section 9 of the Code
(“Application”) before the National Company Law Tribunal, Mumbai (“NCLT”) for
initiation of the corporate insolvency resolution process (“CIRP”) against Mobilox. This
was dismissed by the NCLT, which expanded the scope of an ‘existing dispute’ under
the Code to hold that a valid notice of dispute had been issued by Mobilox.
♦ Kirusa filed an appeal before the National Company Law Appellate Tribunal (“NCLAT”),
which allowed Kirusa’s appeal and inter alia, held that the notice of dispute does not
reveal a genuine dispute between the parties. Mobilox filed an appeal before the
Supreme Court impugning the order of the NCLAT.

© The Institute of Chartered Accountants of India


Insolvency & Bankruptcy Code, 2016 3.9

Issue
Whether and to what extent can the NCLT go into the question of “existence of a dispute”?
Key Ratio Decidendi
The Supreme Court held that once an operational creditor has filed an Application, which is
otherwise procedurally complete, the Adjudicating Authority has to consider the following;
1. Whether there is an “operational debt”, as defined under the Code, which exceeds
INR 100,000;
2. Whether the documentary evidence furnished with the application shows that the
aforesaid debt is due and payable and has not yet been paid; and
3. Whether there is existence of a dispute between the parties or the record of the
pendency of a suit or arbitration proceeding filed before the receipt of the demand
notice of the unpaid operational debt in relation to such dispute;
While determining the third point above, the Supreme Court stated that the Adjudicating
Authority must see is whether there is a plausible contention which requires further
investigation and that the “dispute” is not a patently feeble argument or an assertion of fact
unsupported by evidence. On the basis of the above, the Supreme Court allowed the appeal
and set aside the order of the NCLAT.
Link to Order
https://www.sci.gov.in/supremecourt/2017/20386/20386_2017_Judgement_21-Sep-2017.pdf

7. K. Sashidhar vs. Indian Overseas Bank & Ors

Citation: Supreme Court, Civil Appeal No.10673 of 2018 dated 05.02.2019

Facts
• In the case of the corporate debtor KS&PIPL, the resolution plan, when it was put to
vote in the meeting of CoC held on 27th October, 2017, could garner approval of only
55.73% of voting share of the financial creditors and even if the subsequent approval
accorded by email (by 10.94%) is taken into account, it did not fulfill the requisite vote
of not less than 75% of voting share of the financial creditors. On the other hand, the
resolution plan was expressly rejected by 15.15% in the CoC meeting and later
additionally by 11.82% by email.
• Similarly, in the case of corporate debtor IIL, the resolution plan received approval of
only 66.57% of voting share of the financial creditors and 33.43% voted against the
resolution plan. This being the indisputable position, NCLAT opined that the resolution

© The Institute of Chartered Accountants of India


3.10 Economic Laws

plan was deemed to be rejected by the CoC and the concomitant is to initiate
liquidation process concerning the two corporate debtors.
• The Managing Director of the corporate debtor (KS&PIPL) appeared before the
adjudicating authority (NCLT) on 6th November, 2017, and also filed a memo on 17th
November, 2017, inter alia submitting that for the financial creditor who choose not to
participate in the voting, the votes and the majority be counted without their vote.
Issue
What is the scope of NCLT jurisdiction to enquire into justness of rejection of the resolution
plan?
Key Ratio Decidendi
“Neither the Adjudicating Authority (NCLT) nor the Appellate Authority (NCLAT) has been
endowed with the jurisdiction to reverse the commercial wisdom of the dissenting financial
creditors and that too on the specious ground that it is only an opinion of the minority financial
creditors. The fact that substantial or majority percent of financial creditors have accorded
approval to the resolution plan would be of no avail, unless the approval is by a vote of not
less than 75% (after amendment of 2018 w.e.f. 06.06.2018, 66%) of voting share of the
financial creditors. To put it differently, the action of liquidation process postulated in Chapter-
III of the I&B Code, is avoidable, only if approval of the resolution plan is by a vote of not less
than 75% (as in October, 2017) of voting share of the financial creditors. Conversely, the
legislative intent is to uphold the opinion or hypothesis of the minority dissenting financial
creditors. That must prevail, if it is not less than the specified percent (25% in October, 2017;
and now after the amendment w.e.f. 06.06.2018, 44%). The inevitable outcome of voting by
not less than requisite percent of voting share of financial creditors to disapprove the
proposed resolution plan, de jure, entails in its deemed rejection"
The term “may” occurring in Section 30(4) of the Code, is ascribable to the discretion of the
CoC to approve the resolution plan or not to approve the same. What is significant is the
second part of the said provision, which stipulates the requisite threshold of "not less than
seventy five percent of voting share of the financial creditors" to treat the resolution plan as
duly approved by the CoC. That stipulation is the quintessence and made mandatory for
approval of the resolution plan. Any other interpretation would result in rewriting of the
provision and doing violence to the legislative intent.
"The amendment under consideration pertaining to Section 30(4), is to modify the
voting share threshold for decisions of the CoC and cannot be treated as clarificatory in
nature. It changes the qualifying standards for reckoning the decision of the CoC
concerning the process of approval of a resolution plan. The rights/obligations
crystallized between the parties and, in particular, the dissenting financial creditors in
October 2017, in terms of the governing provisions can be divested or undone only by a
law made in that behalf by the legislature. There is no indication either in the report of

© The Institute of Chartered Accountants of India


Insolvency & Bankruptcy Code, 2016 3.11

the Committee or in the Amendment Act of 2018 that the legislature intended to undo
the decisions of the CoC already taken prior to 6th day of June, 2018. It is not possible
to fathom how the provisions of the amendment Act 2018, reducing the threshold
percent of voting share can be perceived as declaratory or clarificatory in nature. In
such a situation, the NCLAT could not have examined the case on the basis of the
amended provision. For the same reason, the NCLT could not have adopted a different
approach in these matters. Hence, no fault can be found with the impugned decision of
the NCLAT."
Link to order
https://www.sci.gov.in/supremecourt/2018/39315/39315_2018_Order_05-Feb-2019.pdf

8. M. Ravindranath Reddy vs. G. Kishan and Ors

Citation NCLAT, Company Appeal (AT) (Insolvency) No. 331 of 2019 dated
17.01.2020

Facts
The Appellant was the Corporate Debtor of the Respondents/ Petitioners, Lessors of industrial
Premises. That tenancy of the Appellant was yearly, and the rent payable for the period from
July 2011 to June 2017 was Rs. 85,67,290/- and the Corporate Debtor stopped making the
payment from January 2017, after the last part payment was made, which was adjusted
towards rental dues. The Adjudicating Authority held that the Corporate Debtor had taken the
property of the Petitioners on rent and they were paying rent up to June 2017.
Issue
1. Whether a landlord by providing lease, will be treated as providing services to the
corporate debtor, and hence, an operational creditor within the meaning of Section
5(20) read with Section 5(21) of the Insolvency and Bankruptcy Code, 2016?
2. Whether the petition filed under Section 9 of the Insolvency and Bankruptcy Code
2016 is not maintainable on account of ‘pre-existing dispute’?
Key Ratio Decidendi
The NCLAT held that the Code recognises two types of debt, on the basis of which creditors
may make an application for initiating insolvency proceedings against the corporate debtor,
namely, financial debt and operational debt. It was stated that if there is a debt, other than a
financial debt or an operational debt, the creditor will not qualify to make an application under
Section 7 or Section 9, as the case may be and therefore, the determination of nature of
claim/type of debt is an important step while considering the admission of an application under
the Code.

© The Institute of Chartered Accountants of India


3.12 Economic Laws

The NCLAT opined that there is a rationale behind restricting this right to initiate corporate
insolvency resolution process (CIRP) only to operational creditors, apart from financial
creditors. It was stated that default committed in relation to operational creditors, in payment
of their debt, connotes that the corporate debtor is not even in a position to service the regular
payments and operational expenses, as required for the day-to-day functioning of the
corporate debtor.
It was further observed that since the Code does not define goods or services, one has to rely
upon the general usage of the terms in law, with due regard to the context in which the same
have been used. The NCLAT noted that even though the Bankruptcy Law Reforms Committee
recommended the treatment of lessors/landlords as operational creditors, the same was not
included in the Code and only the claims in respect of goods and services had been retained
in the definition of operational creditor and operational debt under Sections 5(20) and 5(21) of
the Code. The NCLAT held that definition of operational creditor does not give scope to
interpret rent dues as operational debt.
Conclusion
The NCLAT stated that for a debt to be classified as an operational debt under Code, the
following conditions should be satisfied:
I. The debt amount should fall within the definition of "claim" as defined under Section
3(6) of the Code;
II. Such a claim should fall within the confines of the definition of a "debt" as defined
under Section 3(11) of the Code, meaning thereby that it should arise out of a liability
or an obligation due from any person; and
III. Such a "debt" should fall strictly within the scope of an "operational debt" as defined
under Section 5(21) of the Code, i.e. the claim should arise either in respect of
provision of goods or services including employment or in respect of the repayment of
dues arising under any law for the time being in force and payable either to the
Central Government, any State Government or local authority.
Link
https://nclat.nic.in/Useradmin/upload/5845699855e228da21841a.pdf

© The Institute of Chartered Accountants of India


Insolvency & Bankruptcy Code, 2016 3.13

9. Maharasthra Seamless Limited vs. Padmanabhan


Venkatesh & Ors.

Citation :SC, Civil Appeal No. 4242 of 2019 dated 22.01.2020

Facts
o The present application was filed by Financial Creditor Under Section 7 of IBC. The
total debt of the corporate debtor was Rs. 1897 crores, out of which Rs. 1652 crores
comprised of term loans from two entities DB International (Asia) Limited and
Deutsche Bank AG, Singapore Branch. As per the process embedded in the Code for
CIRP all the requisite actions were carried out by Interim Resolution Professional and
Resolution Professional (RP).
♦ The Resolution Plans were placed before COC (Committee of Creditors). The plan
submitted by Maharashtra Seamless Limited was approved by majority of the COC by
87.10% of the votes.
♦ In the present case the application was filed by Resolution Professional for the
approval of Resolution Plan for Corporate Debtor under section 30(6) and 31 of IBC,
2016 along with regulation 39 (4) of IBBI (Insolvency Resolution for Corporate
Persons) and Rule 11 of NCLT Rules, 2016. The approval was in respect with the
Resolution Plan submitted by Resolution Applicant – M/s Maharashtra Seamless
Limited.
♦ It was averred by the suspended Board of Directors that the COC meeting and the
approval of Resolution Plan was in contravention to the Code. The Tribunal passed
detailed order to re-determine the liquidation value of the Corporate Debtor and then
placed the revised Resolution Plan before COC for voting which was not duly followed
by the RP. The other Resolution Applicants were not given the opportunity to submit
their revised plans. Therefore, it was prayed that the Resolution Plan be rejected.
♦ The Resolution Plan of MSL which was being approved by the COC in majority
aggrieved by the order of tribunal to re-determine the liquidation value and the fair
value appealed NCLAT. The Appellate Authority directed the Adjudicating Authority to
pass order under section 31 of IBC, 2016 uninfluenced by the previous order.
♦ The RP filed an application under section 30 and 31 of IBC, 2016 along with
regulation 39 (4) of IBBI (Insolvency Resolution for Corporate Persons). The RP also
informed the tribunal about the order of the Adjudicating Authority to re determine the
liquidation value was complied, which was now at Rs. 597.54 crores as compared to
previous amount of Rs. 439.92 crores.

© The Institute of Chartered Accountants of India


3.14 Economic Laws

♦ Further suspended Board of Directors were permitted to attend the COC meeting and
express their views which were recorded. The Resolution Plan of MSL was approved
by 87.10% of the votes. The bid of the MSL of Rs. 477 crores were far more below
the liquidation value of Rs. 597.54 crores.
♦ The RP contended that the Adjudicating Authority cannot sit in appeal over the
commercial wisdom of the COC members in approving resolution plan.
♦ The Tribunal held that as per the orders of the Hon’ble NCLAT it has to pass order
uninfluenced by the previous order and it has to test resolution plan in conformity with
the provisions of section 30(2) of the IBC, 2016. The question regarding approval
from CCI was also answered that it has already been discussed by the COC and it
was came to the conclusion that it is not required, even if it is required then the RA
has a time period of one year to get the desired permissions.
The Resolution plan submitted by M/s MSL was approved.
Issue
1. Whether the scheme of the Insolvency and Bankruptcy Code, 2016 ("Code")
contemplates that the sum forming part of the resolution plan should match the
liquidation value?
2. Whether Section 12-A is the applicable route through which a successful Resolution
Applicant can retreat
Key Ratio Decidendi
Issue 1:
♦ It has been held that the Adjudicating Authority cannot interfere on merits with the
commercial decision taken by the Committee of Creditors. The COC should make
sure that the corporate debtor needs to keep going as a going concern because the
rationale being that during resolution, the corporate debtor remains a going concern,
whereby the financial creditors will have the opportunity to lend further money, the
operational creditors will have a continued business and the workmen and employees
will have job opportunities; that it needs to maximise the value of its assets; and that
the interests of all stakeholders including operational creditors has been taken care of
during the insolvency resolution process.
♦ If the Adjudicating Authority finds the abovementioned parameters have not been
taken care of, it may send a resolution plan back to the COC. If the adjudicating
authority has been satisfied that the COC has taken care of the parameters
mentioned then only it has to pass the resolution plan. Further the reasons given by
the Committee of Creditors while approving a resolution plan may thus be looked at
by the Adjudicating Authority.

© The Institute of Chartered Accountants of India


Insolvency & Bankruptcy Code, 2016 3.15

♦ The Supreme Court held that no provision in the Code or Regulations has been
brought to notice under which the bid of any Resolution Applicant has to match
liquidation value arrived at in the manner provided in clause 35 of the Insolvency and
Bankruptcy Board of India (Insolvency Resolution Process for Corporate Persons)
Regulations, 2016.
♦ While it may seem that release of assets at a value below its liquidation value is
inequitable, the Court ought to rely on the commercial wisdom of the creditors.
Further the main objective of the Code is maximisation of value of assets of
stakeholders, and to balance the interests of all the stakeholders of the corporate
debtor, the court observed that resolution of the corporate debtor should be given
preference over liquidation of the corporate debtor.
♦ It was held that the object behind prescribing such valuation process is to assist the
COC to take decision on a resolution plan properly. Once, a resolution plan is
approved by the COC, the statutory mandate on the Adjudicating Authority under
section 31(1) of the Code is to ascertain that a resolution plan meets the requirement
of sub-sections (2) and (4) of section 30 thereof. Further in the present case AA has
not found breach of these provision.
♦ Section 31(1) of the Code lays down that for final approval of a resolution plan, the
Adjudicating Authority has to be satisfied that the requirement of sub-section (2) of
section 30 of the Code has been complied with. The proviso to section 31(1) of the
Code stipulates the other point on which an Adjudicating Authority has to be satisfied.
The scope of interference by the Adjudicating Authority in limited judicial review has
been laid down in the case of Essar Steel. The Appellate Authority ought not to have
interfered with the order of the Adjudicating Authority in directing the successful
Resolution Applicant to enhance their fund inflow upfront.
Issue 2:
The Supreme Court held that the exit route prescribed in Section 12A is not applicable to a
Resolution Applicant. The procedure envisaged in the said provision only applies to applicants
invoking Sections 7, 9 and 10 of the Code. Accordingly, the Resolution Professional is
directed to take physical possession of the assets of the corporate debtor and hand it over to
the MSL (appellant) within a period of four weeks. The police and administrative authorities
are directed to render assistance to the Resolution Professional to enable him to carry out
these directions
Conclusion
There is no provision in the Code, or regulations which prescribe that the bid of any resolution
applicant has to match the liquidation value arrived at, in the manner provided in Clause 35 of
the Insolvency and Bankruptcy Board of India (Insolvency Resolution Process for Corporate
Persons) Regulations, 2016.

© The Institute of Chartered Accountants of India


3.16 Economic Laws

10. Bijay Kumar Agarwal, Ex-Director of M/s Genegrow Commercial


Pvt. Ltd. vs. State Bank of India & Anr.

Citation : NCLAT, Company Appeal (AT) Insolvency No. 993 of 2019 dated 23.01.2020

Facts
The appellant Ex-Director of M/s. Genegrow Commercial Pvt. Ltd. had initiated the instant
appeal as an “Aggrieved Person” in respect to the order passed by the NCLT, Kolkata Bench
in August,2018 wherein the application was filed by the 1st Respondent/Financial Creditor/
Bank. A deed of guarantee was issued between the M/s Genegrow Commercial Pvt. Ltd.
acting as the ‘Corporate Guarantor’ for the ‘Principal Borrower’ ‘Gee Pee Infotech Pvt. Ltd.’
against the bank for a loan of Rs. 162 Crores (Amount plus Interest). In January 2014, it was
issued that the account of the Corporate Debtor’ was declared ‘Non-Performing Asset’ and so
the 1st Respondent/Bank filed an application against ‘Gee Pee Infotech Pvt. Ltd.’ before the
Debt Recovery Tribunal-1, Kolkata filing for a recovery of the balance sum of Rs. 84.5 lakhs.
The first Respondent had filed an application under Section 7 of the IBC,2016 to initiate CIRP
before the Adjudicating Authority (NCLT) against the Principal Borrower (Gee Pee Infotech
Pvt. Ltd.) as well as the Appellant for Claim and Default primarily committed by the Principal
Borrower. The NCLT Kolkata bench admitted the Claim based on acceptance from the
Principal Borrower of the claim made and had no defense and so Corporate Insolvency
Resolution Process was initiated against both the Principal Borrower and Corporate
Guarantor. The Corporate Guarantor hence, filed an appeal to the NCLAT, New Delhi against
the order passed by NCLT Kolkata Bench claiming that the later had failed to appreciate that
the liability of the Principal Borrower and Guarantor is co-extensive for recovery.
Issue
Whether a financial creditor is permitted to initiate CIRP proceedings under Section 7 of the
IBC against the principal debtor as well as the guarantor, for the same set of claims?
Key Ratio Decidendi
There is no bar in the ‘I&B Code’ for filing simultaneously two applications under Section 7
against the ‘Principal Borrower’ as well as the ‘Corporate Guarantor(s)’ or against both the
‘Guarantors’. However, once for same set of claim application under Section 7 filed by the
‘Financial Creditor’ is admitted against one of the ‘Corporate Debtor’ (‘Principal Borrower’ or
‘Corporate Guarantor(s)’), a second application by the same ‘Financial Creditor’ for same set
of claim and default cannot be admitted against the other ‘Corporate Debtor’ (the ‘Corporate
Guarantor(s)’ or the ‘Principal Borrower’).

© The Institute of Chartered Accountants of India


Insolvency & Bankruptcy Code, 2016 3.17

Conclusion
The NCLAT observed that there is no fetter in the Code for projecting simultaneously two
applications under section 7 against the principal borrower, as well as the corporate
guarantor(s). However, for the same set of claims, if an application filed by the Financial
Creditor is admitted against either the principal borrower or the corporate guarantor, a second
application filed by the same Financial Creditor for the same set of claims cannot be admitted
against the other. It clarified that a creditor cannot sue the principal borrower and claim the
guarantor’s insolvency at the same time.
Link
https://nclat.nic.in/Useradmin/upload/20485369485e2a76361b6e1.pdf

11. Flat Buyers Association, Winter Hills – 77, Gurgaon vs. Umang
Realtech Pvt. Ltd. and Ors.

Citation NCLAT, Company Appeal (AT) Insolvency No. 926 of 2019 dated 04.02.2020

Facts
Mrs. Rachna Singh and Mr. Ajay Singh (Allottees) had moved an application under Section 7
of the Insolvency and Bankruptcy Code, 2016 (‘IBC’) for initiation of CIRP of M/s. Umang
Realtech Pvt. Ltd., a real estate company involved in constructing flats/ apartments and the
said application was admitted by National Company Law Tribunal (‘NCLT’), Principal Bench,
New Delhi. Once an application for CIRP of the Corporate Debtor is admitted and an IRP is
appointed, it is the duty of the IRP/RP to keep the Corporate Debtor as a going concern.
However, in case of a real estate infrastructure company, the only business is to build and
complete flats/projects and thus, maintaining the Corporate Debtor as a going concern is a
hurdle that is almost impossible to cross. Given the grave situation, most real estate
companies are pushed into liquidation and the same does not serve any benefit to the
company or its creditors.
Issue
Whether CIRP proceedings initiated by a flat buyer in relation to one project of a real estate
company will affect the other group projects of the company.
Key Ratio Decidendi
♦ Corporate Insolvency Resolution Process if initiated against a real estate
infrastructure company by the allottees or financial institutions/banks/NBFCs or
operational creditors of one project, the said CIRP shall be restricted to that particular
project and cannot affect any other project(s) of the same real estate infrastructure
company in other places where separate plan(s) have been approved by different

© The Institute of Chartered Accountants of India


3.18 Economic Laws

authorities. The assets of that particular project only can be maximised for balancing
the interest of the creditors of that project and any other allottees or other financial
creditors or operational creditors of other projects cannot file any claim before the
IRP/RP.
♦ A secured creditor such as ‘financial institutions/banks’ cannot be provided with
flats/apartments being the assets of the Corporate Debtor by preference over the
allottees (unsecured financial creditors) for whom the project has been approved.
However, while satisfying the allottees, they may agree to opt for another
flat/apartment, or another tower and their agreements can be modified to this extent
by the IRP/RP with the counter signature of the promoter and allottee.
♦ The prayer of allottees asking for refund cannot be allowed by the NCLT or NCLAT.
However, after offering allotment, it is open to the allottee to request the
IRP/RP/Promoter, whoever is in-charge to find a third party to purchase the allotted
flat/apartment and thus, receive back the money invested by the said allottee. It is
also possible for the allottee to arrive at an agreement with the promoter (not
corporate debtor) for refund of the amount upon completion of the flat/project or
during completion of the project.
Conclusion
CIRP against a real estate company is limited to a project as per the resolution plan approved
by the competent authority, and not to other projects which are separate at other places for
which separate resolution plans have been approved. It was stated that if the same real estate
company has any other project, all such projects cannot be clubbed together.
Link
https://nclat.nic.in/Useradmin/upload/18011332575e3d0b157e29a.pdf

12. Savan Godiwala vs. Mr. Apalla Siva Kumar

Citation NCLAT, Company Appeal (AT) (Insolvency) No. 1229 of 2019 dated
11.02.2020

Facts
The Liquidator filed an appeal before the Hon’ble National Company Law Appellate Tribunal
against the Order of the National Company Law Tribunal which had observed and ordered that
the Liquidator could not avoid the liability to pay gratuity to the employees on the ground that
the Corporate Debtor didn’t have separate funds for payment of gratuity and further directed
the Liquidator to provide sufficient provision for payment of gratuity, according to the eligibility
of the employees.

© The Institute of Chartered Accountants of India


Insolvency & Bankruptcy Code, 2016 3.19

Issue
Whether a liquidator can be directed to make payment of gratuity to the employees of the
company, in case no fund is created by a company, in violation of the statutory provision of the
Payment of Gratuity Act, 1972
Key Ratio Decidendi
The NCLAT held that it is a settled position of law that the provident fund, the pension fund
and the gratuity fund, do not come within the purview of 'liquidation estate' for the purpose of
distribution of assets under Section 53 of the Code. The NCLAT stated that the only inference
which can be drawn is that pension fund, gratuity fund and provident fund cannot be utilised,
attached or distributed by the liquidator, to satisfy the claim of other creditors.
The NCLAT further stated that Section 36(2) of the Code provides that the liquidator shall hold
the liquidation estate in fiduciary capacity, for the benefit of all the creditors. It has also held
that the liquidator has no domain to deal with any other property of the corporate debtor, which
does not form a part of the liquidation estate.
The NCLAT opined that even in a case where no fund is created by a company, in violation of
the statutory provision of Section 4 of the Gratuity Act, the liquidator cannot be directed to
ensure payment of gratuity to the employees, since the liquidator does not have the power to
deal with properties of the corporate debtor, which do not form part of the liquidation estate.
Conclusion
The PF, pension fund and the gratuity fund do not form part of the liquidation estate and
therefore, the liquidator, who holds liquidation estate in fiduciary capacity, has no authority to
deal with such funds.
Link
https://nclat.nic.in/Useradmin/upload/4527439455e428cedc81d0.pdf

13. JSW Steel Ltd. Vs. Mahender Kumar Khandelwal & Ors.

Citation NCLAT, Company Appeal (AT)(Insolvency) No. 957 of 2019, dated


17.02.2020

Facts
In the CIRP of Bhushan Power & Steel Limited, the Resolution Plan submitted by JSW Steel
Limited (Resolution Applicant) has been approved by the Adjudicating Authority by impugned
Judgment dated 5th September, 2019 with certain conditions. After the approval of plan when
Monitoring Committee was monitoring the change of management, on 10th October, 2019, the
Directorate of Enforcement of Central Government attached assets of Bhushan Power & Steel

© The Institute of Chartered Accountants of India


3.20 Economic Laws

Limited under Section 5 of the Prevention of Money Laundering Act, 2002. JSW Steel Limited
is Successful Resolution Applicant, in its appeal has sought for setting aside/ modification of
conditions imposed in paragraph 128 sub paras (e), (f), (g), (i), (j), (k) of the impugned order
dated 5th September, 2019. It has also raised objection and challenged the jurisdiction of
Directorate of Enforcement to attach the properties of the Bhushan Power & Steel Limited-
(Corporate Debtor), after change of hands.
Issue
Whether after approval of a resolution plan under Section 31 of the Code, it is open to the
Directorate of Enforcement to attach the assets of a corporate debtor on the alleged ground of
money laundering by erstwhile promoters of the corporate debtor.
Key Ratio Decidendi
The NCLAT observed that a plain reading of Section 32A (1) and (2) clearly suggests that the
Directorate of Enforcement/ other investigating agencies do not have the powers to attach
assets of a corporate debtor, once the resolution plan stands approved and the criminal
investigations against the corporate debtor stands abated. It was stated that Section 32A of
the Code does not in any manner suggest that the benefit provided thereunder is only for such
resolution plans which are yet to be approved.
The NCLAT further held that the requirement set out in Section 32A(1)(b) of the Code is that
the investigating agency must have reason to believe that the resolution applicant had abetted
or conspired in commission of the offence, on the basis of material in its possession as on that
date. The phrase "on the basis of material in its possession" along with the usage of the words
"has" and "reason to believe that he had abetted or conspired.." has to necessarily be
construed as, the material in the possession of investigating agency as on the date when such
agency is called to provide its confirmation/ certification with respect to Section 32A (1) (b) of
the Code. It was further specified that if the investigating agency is permitted to keep such
confirmation in abeyance till the investigation is complete in all respects, then the object and
purpose of introducing Section 32A (1) (b) will be defeated and no resolution applicant would
come forward to implement its resolution plan for fear of the possibility of assets of the
corporate debtor being attached.
The NCLAT held that the intent and purpose behind insertion of Section 32A is to provide
certainty to the resolution applicant that the assets of the corporate debtor, as represented to
him, and for which he proposes to pay value/ consideration in terms of the resolution plan,
would be available to him in the same manner as at the time of submission of the resolution
plan.
Conclusion
Upon a perusal of Section 32A(1) (a) read with the definition of “Related Party” Section 5(24)
of the Code, it is an ex facie evident that JSW is not an associate company/ related party of

© The Institute of Chartered Accountants of India


Insolvency & Bankruptcy Code, 2016 3.21

the Corporate Debtor. The Union of India had unequivocally stated that after the completion of
the corporate insolvency resolution process, there cannot be any threat of criminal
proceedings against the Corporate Debtor or attachment or confiscation of its assets by any
investigating agency after approval of the resolution plan. After taking into consideration the
submissions, NCLAT stayed the order of attachment passed by the ED and also prohibited it
from attaching property of Corporate Debtor without seeking prior approval of the NCLAT.
NCLAT also directed the that the property already attached by the ED be realised in favour
of the resolution professional immediately.
Link :
https://ibbi.gov.in//uploads/order/21db96b041a8c94300d9c73a8912
8265.pdf

14. Sagufa Ahmed Vs. Upper Assam Plywood Products Pvt. Ltd.

Citation SC, Civil Appeal Nos.3007-3008 of 2020 dated 18.09.2020

Facts
♦ Challenging an order passed by the NCLAT dismissing an application for condonation
of delay as well as an appeal as time barred, the appellants have come up with the
above appeals.
♦ The appellants herein together claim to hold 24.89% of the shares of a company by
name Upper Assam Plywood Products Private Limited, which is the first respondent
herein. The appellants moved an application before the Guwahati Bench of the NCLT
for the winding up of the company. The said petition was dismissed by the NCLT by
an order dated 25.10.2019.
♦ According to the appellants, the certified copy of the order dated 25.10.2019 passed
by the NCLT was received by their counsel on 19.12.2019, pursuant to the copy
application made on 21.11.2019. Though the appellants admittedly received the
certified copy of the order on 19.12.2019, they chose to file the statutory appeal
before NCLAT on 20.07.2020. The appeal was filed along with an application for
condonation of delay.
♦ By an order dated 04.08.2020, the Appellate Tribunal dismissed the application for
condonation of delay on the ground that the Tribunal has no power to condone the
delay beyond a period of 45 days. Consequently the appeal was also dismissed. It is
against the dismissal of both the application for condonation of delay as well as the
appeal, that the appellants have come up with the present appeals.
♦ The contentions raised by the learned counsel for the appellants are two­fold namely (i) that
the Appellate Tribunal erred in computing the period of limitation from the date of the order

© The Institute of Chartered Accountants of India


3.22 Economic Laws

of the NCLT, contrary to Section 421(3) of the Companies Act, 2013, and (ii) that the
Appellate Tribunal failed to take note of the lockdown as well as the order passed by this
Court on 23.03.2020 in Suo Motu Writ Petition (Civil) No.3 of 2020, extending the period of
limitation for filing any proceeding with effect from 15.03.2020 until further orders.
Issue
1. The counsel for the appellants, Mr Gunjan Singh, contended that the limitation for
filing an appeal ( 45 days) would only start from the day when a party receives the
order of a court.
2. Mr Gunjan Singh also contended that due to the COVID 19 pandemic and in
accordance with SC order 23.03.2020 which was “limitation prescribed under the
general law or Special Laws whether condonable or not shall stand extended w.e.f.
15th March 2020 till further order” so their application for condonation of delay should
be allowed.
Key Ratio Decidendi
♦ The period of limitation of 45 days prescribed in Section 421(3) of Companies Act,
2013 would start running only from the date on which a copy of the order of the
Tribunal is made available to the person aggrieved.
♦ The law of limitation finds its root in two latin maxims, one of which is Vigilantibus Non
Dormientibus Jura Subveniunt which means that the law will assist only those who
are vigilant about their rights and not those who sleep over them.
♦ The principle forming the basis of Section 10(1) of the General Clauses Act, also finds
a place in Section 4 of the Limitation Act, 1963.
♦ The words “prescribed period” appear in several Sections of the Limitation Act, 1963.
Though these words “prescribed period” are not defined in Section 2 of the Limitation
Act, 1963, the expression is used throughout, only to denote the period of limitation.
♦ The expression “prescribed period” appearing in Section 4 of the Limitation Act, 1963
cannot be construed to mean anything other than the period of limitation. Any period
beyond the prescribed period, during which the Court or Tribunal has the discretion to
allow a person to institute the proceedings, cannot be taken to be “prescribed period”.
♦ From 19.12.2019, the date on which the counsel for the appellants received the copy
of the order, the appellants had a period of 45 days to file an appeal. This period
expired on 02.02.2020. The appellants did not file the appeal on or before
18.03.2020, but filed it on 20.07.2020. To get over their failure to file an appeal on or
before 18.03.2020, the appellants relied upon SC order dated 23.03.2020.

© The Institute of Chartered Accountants of India


Insolvency & Bankruptcy Code, 2016 3.23

What was extended by the above order of this Court was only “the period of limitation”
and not the period upto which delay can be condoned in exercise of discretion
conferred by the statute.
Conclusion
The Hon’ble SC held that the Appellate Tribunal did not err in computing the period of
limitation from the date of the order of NCLT and that it was the failure of appellants
themselves to file an appeal on or before the stipulated period of time got over. With respect
to the second contention, SC held that the order passed by the SC on 23.03.2020 was only for
extension of period of limitation and not the extension of period upto which delay can be
condoned in exercise of discretion conferred by the statute.
Link
https://main.sci.gov.in/supremecourt/2020/17038/17038_2020_31_1501_23998_Judgement_1
8-Sep-2020.pdf

© The Institute of Chartered Accountants of India


4
Prevention of Money Laundering, 2002

1. Directorate of Enforcement v. Deepak Mahajan

Citation: SC, Criminal Appeal No. 537 of 1990 dated 31.01.1994

Facts
♦ Deepak Mahajan, the respondent was arrested by the officers of the Enforcement
Directorate for an offence punishable under the provisions of FERA and taken before
the Additional Chief Metropolitan Magistrate, New Delhi on the next date as per the
mandate of sub-section (2) of Section 35 of the said Act.
♦ An application under Section 167(2) of the Code of Criminal Procedure Code was
moved by the Enforcement Officer seeking petitioner’s ‘judicial remand’ on the ground
that it was necessary to complete the investigation. On the very same day, the
respondent unsuccessfully moved the court for bail.
♦ The Magistrate remanded the first respondent to judicial custody for fourteen days and
subsequently extended the detention period. The first respondent challenged the
jurisdiction of the Magistrate in authorising the detention (remand) and the subsequent
consecutive extensions. But his plea was rejected on the basis of the decision in Gupta
case.
♦ The case ultimately reached the Supreme Court.
Issue
1. Whether the Special Leave Petition is maintainable?
2. Whether the Magistrate before whom a person arrested under subsection (1) of Section
35 of the Foreign Exchange Regulation Act of 1973 which is in pari materia with sub-
section (1) of Section 104 of the Customs Act of 1962, is produced under sub-section
(2) of Section 35 of the Foreign Exchange Regulation Act, has jurisdiction to authorize
detention of that person under Section 167(2) of the Code of Criminal Procedure?
3. Whether Enforcement Directorate under FERA (now FEMA) or Custom’s Act are
competent person to take judicial remand of an arrested person?

© The Institute of Chartered Accountants of India


Prevention of Money Laundering Act, 2002 4.2

Rule of Law
♦ Section 167(2)[iii] of Criminal Procedure Code
♦ Section 35 of Foreign Exchange Regulation Act
♦ Section 104 of Customs Act
♦ Section 35 of Indian Penal Code
♦ Article 136 of Constitution of India
Key Ratio Decidendi
1. The petition is maintainable under Article 136 of Constitution. Article 136 provides that
the aggrieved party requires a special permission to be heard by the Apex Court in
appeal against any judgement or order of any court /tribunal in the territory of India.
However the Supreme Court may, in its discretion, grant special leave to appeal from
any judgment, decree or order made by any court or tribunal in India. Art. 136 can only
be applicable in special cases only and gross violation of principles of natural justice,
gross miscarriage of justice, decision shocking the conscience of the court, when
concerned point of law cannot be decided by ordinary law and other forums are the
examples of such extraordinary and special circumstances. Even in the instant case
there may be a grave violation of principles of natural justice and gross miscarriage of
justice if the Magistrate does not have power to try cases under FERA and Customs
Act Further another important question that has been raised under this case is whether
the Directorate of Enforcement or Customs Officer fall within the definition of ‘Police
Officer’ under Section 167(2) of CrPC. Since such important Issue has to be answered,
hence the petition is validly brought under Special Leave Appeal.
2. The question is not what the words in the relevant provision mean but whether there
are certain grounds for inferring that the legislature intended to exclude jurisdiction of
the courts from authorizing the detention of an arrestee whose arrest was effected on
the ground that there is reason to believe that the said person has been guilty of an
offence punishable under the provisions of FERA or the Customs Act which kind of
offences seriously create a dent on the economy of the nation and lead to hazardous
consequences. Further the Supreme Court stated that to invoke Section 167(1), it is not
an indispensable pre- requisite that in all circumstances, the arrest should have been
effected only by a police officer and no one else and that there must necessarily be
records of entries of a case diary. Hence the Supreme Court stated that the
Enforcement Officer or Custom Officer can be termed as ‘police officer’ for the purpose
of arrest.
3. The Supreme Court held that “sub-sections (1) and (2) of Section 167 are squarely
applicable with regard to the production and detention of a person arrested under the
provisions of Section 35 of FERA and Section 104 of Customs Act and that the

© The Institute of Chartered Accountants of India


4.3 Economic Laws

Magistrate has jurisdiction under Section 167(2) to authorize detention of a person


arrested by any authorized officer of the Enforcement under FERA and taken to the
Magistrate in compliance of Section 35(2) of FERA.”
Link to order
https://main.sci.gov.in/jonew/judis/11668.pdf

2. M/s. PMT Machines Ltd. vs The Deputy Director, Directorate of


Enforcement, Delhi

Citation: Appellate Tribunal for SAFEMA, FEMA, PMLA, NDPS & PBPT Act, FPA -
PMLA -2792/DLI/2019 dated 16.09.2019

Facts
♦ PMT Machines is a part of SBL group companies, acquired by the promoters of SBL
Nitin Sandesara, Chetan Sandesara and Deepti Sandesara, in 1993. Before the
provisional attachment order was passed by the ED, NCLT Mumbai had already
initiated Corporate Insolvency Resolution Process (CIRP) against PMT Machines under
the Insolvency and Bankruptcy Code, 2016 (IBC), which is still under process.
♦ The ED had conducted a 'search and seizure' under the provisions of FEMA and
Income Tax Act at Mumbai, Vadodara premises of Sterling Group, its Promoters and
Promoters' Companies in 2017. Later, a provisional attachment order was issued
against the SBL group companies in 2018. In this order, immovable properties and
other movables, vehicles etc. belonging to the PMT Machines Ltd., were also
provisionally attached.
Issue
The appeal before the appellate authority under PMLA was filed by the RP, seeking release of
the assets of the PMT Machines. The RP contended that moratorium under Section 14 of the
IBC was applicable to the properties of PMT Machines, and the attachment order had
impacted the resolution process.
Rule of Law
♦ Section 5 of the Prevention of Money Laundering Act, 2002
Key Ratio Decidendi
1. The Appellate Authority of the Prevention of Money Laundering Act, 2002 (PMLA) has
upheld the prevalence of the IBC over the provisions of PMLA.
2. The PMLA Appellate Tribunal, distinguished between the objectives of the PMLA and
IBC, and was of the view that “the objective of the PMLA was to deprive the offender

© The Institute of Chartered Accountants of India


Prevention of Money Laundering Act, 2002 4.4

from enjoying the 'illegally acquired' fruits of crime by taking away his right over
property acquired through such means. The Bench opined that the IBC's objective on
the other hand was maximization of value of assets, to promote entrepreneurship,
availability of credit and balancing the interest of all the stakeholders.”
3. The Appellate Bench observed that, if the attachment in this case were lifted, the RP
would be able to take steps to get a viable Resolution Plan. It was noted that the
attachment order was passed in relation to mortgaged properties in favour of banks,
which were not purchased from "proceeds of crime", as they were purchased and
mortgageD with the banks prior to the crime period.
Link to order
http://atfp.gov.in/writereaddata/upload//Judgement/Judgement4TGTLOAE06_917201922824M.PDF

3. B.K. Singh vs Suraj Pal @ Chacha

Citation: In the court of Ms. Santosh Snehi Mann, Special Judge (PC Act), CBi – 08,
CCS No. 04/15 and CIS No. 21/2019 dated 09.09.2019

Facts
♦ The three accused persons were apprehended by a joint team of the Delhi Police Crime
Branch, Maharashtra Forest Department and the Wildlife Crime Control Bureau, on the
basis of information received on movement of wildlife offenders. After search and
seizure, a sack containing uncured trophies of tiger parts, tiger nails and total cash
worth Rs. 52,60,000 were recovered from the accused persons.
♦ Pursuant to this, a complaint was filed against the three accused persons for offences
under sections 39, 44, 49B and 51 of Wildlife Protection Act. Since, these offences also
fall in the category of Scheduled Offences under the Prevention of Money Laundering
Act, a complaint was also registered at the Delhi Zonal Office of Enforcement Director.
After recording the statements under section 50 of PMLA, the accused persons were
charged with sections 3 and 4 of the said Act.
Issue
Whether the accused are guilty under Section 3 of PMLA, punishable under Section 4 of
PMLA?
Rule of Law
♦ Section 3 of the Prevention of Money Laundering Act, 2002
♦ Section 4 of the Prevention of Money Laundering Act, 2002
♦ Section 50 of the Prevention of Money Laundering Act, 2002

© The Institute of Chartered Accountants of India


4.5 Economic Laws

Key Ratio Decidendi


1. The court while examining the evidence opined that in proving the criminal activity
under Wildlife Protection Act, there's no need to examine the informer. Moreover,
Wildlife Crime Investigation Handbook prohibits the examination of informer as a
witness in court to protect his identity. Therefore, the claim of the defence regarding
non-examination of the informer was refuted by the court.
2. The court also opined that physical production of the wildlife contraband during trial is
not essential to prove the recovery.
3. It was also accepted by the court that the monies recovered from the custody of the
accused persons were the proceeds of crime under PMLA. The court, while convicting
two of three accused persons under section 3 of PMLA, also ordered the recovered
monies to be confiscated by the Central Government under sections 8(5) and 8(7) of
the aforementioned anti-money laundering legislation.
Link to order
https://services.ecourts.gov.in/ecourtindia_v4_bilingual/cases/display_pdf.php?filename=/orders/2019/2
02400000212019_32.pdf&caseno=CT%20Cases/21/2019&cCode=5&appFlag=web&normal_v=1

4. Chhagan Chandrakant Bhujbal vs. Union of India and Ors

Citation: Bombay HC, Criminal Writ Petition No. 3931 of 2016 dated 14.12.2016

Facts
♦ The Petitioner being former PWD Minister of Maharashtra was accused of generating
huge illicit funds to the tune of Rs.840.16 crores that were money laundered.
♦ While holding official position as the PWD Minister, the Petitioner allegedly awarded
contracts of public works for self-gains.
♦ As per the provisions laid down in the PMLA, 2002 the Petitioner was arrested and the
Special Court took cognizance of the offence and passed a detailed Order sending him
to custody.
♦ The Petitioner moved the Hon'ble Bombay High Court under Article 226 of the
Constitution of India seeking Writ of Habeas Corpus.
Issue
1. The amendment of Section 45 of the Prevention of Money Laundering Act in 2005
made all offences under the Act non-cognizable and therefore procedure under section
155 (2) of the Criminal Procedure Code should have been followed. Unless cognizance
of the offence is taken by the Magistrate or the Special Court, the arrest of the

© The Institute of Chartered Accountants of India


Prevention of Money Laundering Act, 2002 4.6

Petitioner could not have been effected. Therefore, requisite procedure for arrest of the
Petitioner was not followed.
2. The grounds of arrest were not mentioned in writing in the Arrest Warrant.
3. The Assistant Director, being an authority established under section 48(c) of the
Prevention of Money Laundering Act, without any notification issued by Central
Government under section 49(3) imposing any conditions or limitations on his powers,
cannot be held as 'competent' to exercise its powers under section 19 of the Act.
Therefore, the Assistant Director, Directorate of Enforcement was not competent and
had no authority to arrest the Petitioner.
Rule of Law
♦ Section 19 of the Prevention of Money Laundering Act, 2002
Key Ratio Decidendi
1. The Assistant Director's power to arrest under section 19 does not depend upon the
question as to whether offence is cognizable or non-cognizable. It was pertinently noted
that while amending section 45 of the Act, the Legislature had not changed the
heading, thereby giving clear indication that it did not intend to make the offence "non-
cognizable" but only wanted to clear the conflict between the power of the Police
Officer, who can arrest, in cognizable offence, without warrant and the authority
established under Section 19 of the PML Act, who can arrest on conditions being
satisfied, as laid down in the Act.
2. It was further held that Section 19 of the PMLA does not contemplate either registration
of FIR, on receipt of information relating to cognizable offence or of obtaining
permission of the Magistrate in case of non-cognizable offence before taking
cognizance or before effecting arrest of the accused in respect of any offence
punishable under this Act. The only conditions, which are laid down under Section 19 of
the Act, pertain to the reasonable belief of the authority, which is on the basis of the
material in its possession.
3. Further, sections 48 and 49 of the PMLA give the officers of the Directorate of
Enforcement powers to investigate cases of money laundering. The enlisted officers
have also been authorised to arrest and initiate proceedings for attachment of property
and to launch prosecution in the designated Special Court for the offence of money
laundering. It was held that the law is well settled that the definition given in the Rules
must be read in conformity with the provisions of Section 19 of PML Act and hence
appropriate interpretation would be that as far as Directors, Deputy Directors and
Assistant Directors are concerned, no authorization of the Central Government is
required; whereas, in respect of other officers, such authorization may be necessary.

© The Institute of Chartered Accountants of India


4.7 Economic Laws

Link to order
https://bombayhighcourt.nic.in/generatenewauth.php?auth=cGF0aD0uL2RhdGEvanVkZ2VtZW50c
y8yMDE2LyZmbmFtZT1DUldQMzkzMTE2LnBkZiZzbWZsYWc9TiZyanVkZGF0ZT0mdXBsb2FkZH
Q9MTQvMTIvMjAxNiZzcGFzc3BocmFzZT0yNjEyMTkxODE2MDU=

5. Dalmia Cement Bharat Ltd. Vs State of AP, Hyderabad

Citation: Telangana HC, Writ Petition No. 36838 of 2014 dated 29.02.2016

Facts
♦ The first petitioner is a company registered under the Companies Act and the second
petitioner is its Managing Director. Petitioners state that the Central Bureau of Investigation
(CBI) filed a charge sheet before the Special Court for CBI Cases.
♦ Based on the allegations in the said charge sheet, the first respondent registered a case on
30.08.2011 for scheduled offences under the Prevention of Money-Laundering Act, 2002 in
which the first petitioner is shown as accused at Sl. No. 26, alleging that the same, prima
facie, discloses an offence under Section 3 of the PMLA.
Issue
Whether a Statement made before Enforcement Directorate (PMLA) is binding on the Accused
without proof/as Admission?
Rule of Law
♦ Section 50 of the Prevention of Money Laundering Act, 2002
♦ Section 171-A of the Sea Customs Act.
Key Ratio Decidendi
1. Under PMLA, a person is required to give truthful statement if such person is summoned by
the Director. This power to the director is given under section 50(2) of PMLA which provides
that Director (or additional director, joint director, deputy director or assistant director) has
the power to summon any person whose attendance he considers necessary whether to
give evidence or to produce any records during the course of any investigation or
proceeding. All such summoned persons are bound to state the truth or make statements,
and produce such documents as may be required [(Section 50(3) of PMLA)].
2. Statements made under Section 171-A of the Sea Customs Act are not confessions
recorded by a Magistrate under Section 164 of the Code of Criminal Procedure but are
statements made in answer to a notice under sec.171-A of the Sea Customs Act. As they
are not made subject to the safeguards under which confessions are recorded by
Magistrates they must be specially scrutinized to finding out if they were made under threat

© The Institute of Chartered Accountants of India


Prevention of Money Laundering Act, 2002 4.8

or promise from someone in authority. If after such scrutiny they are considered to be
voluntary, they may be received against the maker and in the same way as confessions are
received
3. Furthermore, the court held that “The protection under Article 20(3) of the Constitution of
India is available at the stage of investigation the court held that the provisions of Section 50
of PMLA are required to be read down so as to ensure that petitioners are not prejudiced in
the CBI case as well as under PMLA.”
Link to order
http://tshcstatus.nic.in/hcorders/2014/wp/wp_36838_2014.pdf

6. Financial Intelligence Unit-IND vs Corporation Bank

Citation: Delhi HC, CRL.A. 877/2017 dated 04.09.2019

Facts
♦ A sting operation was conducted by online media portal named "Cobrapost.com" on various
banks. During the sting, undercover reporters approached employees of various banks
representing themselves to be customers who required opening of accounts to deposit
black money belonging to a Minister and for laundering the same. The video indicated that
officials of the banks had expressed willingness to accept deposits of black money.
♦ Consequently, FIU issued letters to the respondent banks asking them to provide certain
information under Section 12(a) of the Prevention of Money-Laundering Act, 2002 (the Act),
in reference to the sting operation and held hearings, all of which culminated into issuance
of show cause notices under Section 13 of the Act, alleging non-compliance of the
provisions of Section 12 of the Act read with Prevention of Money Laundering (Maintenance
and Records) Rules, 2005 (the Rules). Section 12 of the Act envisages reporting obligations
of banks against 'suspicious transactions'.
♦ The FIU proceeded on the basis that the conversations recorded in the sting operation
constituted 'suspicious transactions' within the meaning of Rule 2(g) of the Rules, and
imposed monetary fines under Section 13 of the Act.
♦ This order of the FIU was modified by the Appellate Tribunal, PMLA stating that violation of
the reporting obligations on part of the respondent banks warranted issuance of a warning
in writing under Section 13(2)(a) of the Act, instead of a monetary penalty as imposed under
Section 13(2)(d) of the Act.
♦ Hence, the present appeal titled "Financial Intelligence Unit- Ind v. Corporation Bank" along
with other appeals was filed under Section 42 of the Act.

© The Institute of Chartered Accountants of India


4.9 Economic Laws

Issue
Whether the Appellate Tribunal could modify the order passed by the Director, FIU by
reducing the penalty imposed.
Rule of Law
♦ Section 13 of the Prevention of Money Laundering Act, 2002
Key Ratio Decidendi
1. The court noted that there was nothing on record to establish that the sting operation
had been conducted prior to 15.02.2013. Therefore, FIU's contention that unamended
provisions were applicable to the Banks was bereft of any factual foundation.
2. Thus the only question for consideration was whether the amended provisions of
Section 13 of the Act, which provide for a lesser punitive measure, were applicable
retrospectively.
3. In this regard, the court relied on T. Barai v. Henry Ah Hoe & Anr., wherein the
Supreme Court had explained that insofar as a new enactment creates new offences or
enhances punishment for a particular type of offence, no person can be convicted by
such ex post facto law nor can the enhanced punishment prescribed by the
amendment, be imposed. However, if a punishment for an offence is reduced, "there is
no reason why the accused should not have the benefit of the reduced punishment"
4. FIU's contention in this regard that in the present cases the respondent banks had
suffered a civil liability and the aforesaid precedent was applicable only to criminal laws
was rejected by the court. It was held that "Even if it is assumed that the liability
imposed on the respondent banks is a civil liability, no distinction can be drawn on the
aforesaid ground so as to deprive the respondents of the rule of beneficial
construction".
5. Reliance was placed upon Commissioner of Tax (Central)-I, New Delhi v. Vatika
Township Pvt. Ltd., wherein the Supreme Court had held that if a legislation confers the
benefit on some persons without inflicting a corresponding detriment on some other
person or where it appears that the intention of legislature is to confer such benefit, the
rule of purposive construction would be applicable and the said legislation would be
construed as applicable with retrospective effect.
6. In these circumstances, the court held that "even if it is assumed that the sting
operation was conducted prior to 15.02.2013, there is no infirmity in the decision of the
Appellate Tribunal to modify the punishment from a monetary fine to a warning in
writing, in terms of Section 13(2)(a) of the Act".
7. Hence, the rule that the enactment must be construed as prospective is not applicable
in cases of a beneficial legislation. In such cases, the same must be construed

© The Institute of Chartered Accountants of India


Prevention of Money Laundering Act, 2002 4.10

retrospectively. It would be unfair to impose a higher punishment then as prescribed


under a statute as currently in force, merely because the person visited with such
punishment has committed the offence / default prior to the legislation being enacted.
Link to order
http://delhihighcourt.nic.in/dhcqrydisp_o.asp?pn=209546&yr=2019

7. Smt. K. Sowbaghya vs Union of India

Citation: Karnataka HC, Writ Petition No. 14649 of 2014 dated 28.01.2016

Facts
♦ The petitioner's husband and son are politicians and businessmen. The Karnataka
Lokayukta Police are said to have registered a case in Crime no. 57/2010 against the
petitioner's husband, her son and several others alleging offences punishable under
Sections 7, 8, 12, 13 (1) & (2) of the Prevention of Corruption Act, 1988, read with
Sections 419, 420, 465, 468,471 read with Section 120 B of the Indian Penal Code,
1860 (hereinafter referred to as 'the IPC', for brevity).
♦ The petitioner contends that inspite of a restraint order passed in the first of these
petitions, restraining the respondent from proceeding to take action pursuant to the
Amendment no. 2 / 2013 of the PML Act, the respondent having instituted a complaint
before the Special Court, before the restraint order could be served on the respondent,
and the Special Court having taken cognizance of the offences alleged, has sought to
file the second of these petitions, with an additional prayer seeking the quashing of the
order of the Special Court, taking cognizance.
Issue
The petitioners has challenged the validity of Sections 17, 18 and 19, of the Act, which provide for
Search and seizure, Search of persons and Arrest, respectively.
Rule of Law
♦ Section 24 of PMLA
♦ Section 44 of PMLA
Key Ratio Decidendi
1. The Court held that “Money laundering is an independent stand-alone offence.” This was on
the reasoning that, although, under sections 3 & 4 of the PML Act, it is not possible to
envision an offence under PML Act as a ‘stand-alone’ offence without the guilt of the
offender in the Scheduled offence being established, the expression “proceeds of crime”
has been defined to include “property” of all kinds as defined under clause (v) of Section

© The Institute of Chartered Accountants of India


4.11 Economic Laws

2(1). Hence, it is possible to extend the definition of ‘proceeds of crime’ to property used in
the commission of an offence under the Act or any of the Scheduled offences. Thus, money
laundering can also be treated as a ‘stand-alone’ offence, de hors, a scheduled offence, if
circumstances warrant.
2. Furthermore, merely because the provisions contemplate measures relating to search,
seizure and arrest, the same cannot be considered draconian.
3. The Court held that “the provisions of the Act which clearly and unambiguously enable
initiation of proceedings for attachment and eventual confiscation of property in possession
of a person not accused of having committed an offence under Section 3 as well, do not
violate the provisions of the Constitution including Articles 14, 21 and 300-A and are
operative proprio vigore [of or by its own force independently.]”
4. Section 24 as amended by the Amendment Act of 2013 is held to be constitutionally valid.
5. And so far as Section 44 of the Act is concerned it is sought to be contended that Section
44 mandates that the offence under this Act shall be triable by a Special Court. The entire
scheme of this section is vague, violates the right to speedy trial and also is ambiguous,
vague oppressive, arbitrary, discriminatory, unconstitutional and offending Articles, 14, 20,
21 and Article 300 of the Constitution of India and is ultra-vires.
Link to order
http://judgmenthck.kar.nic.in/judgmentsdsp/bitstream/123456789/98927/1/WP14649-14-28-01-
2016.pdf

8. B. Rama Raju v. Union of India

Citation: Andhra Pradesh HC, Writ Petition No. 10765 of 2010 dated 04.03.2011

Facts
In this case, a writ petition was filed challenging certain provisions of the Prevention of Money
Laundering Act, 2002 including its amendments. The provision of attachment and confiscation
under Section 2(1) of the PMLA 2002 was challenged.
Issue
♦ Whether property owned by or in possession of person, other than person charged of
having committed a scheduled offence is liable to attachment and confiscation
proceedings? And if so whether Section 2(1)(u) was invalid?
♦ The issue was whether provisions of second proviso of Section 5 were applicable to
property acquired prior to enforcement of this provision and if so, whether provision is
invalid for retrospective penalization?

© The Institute of Chartered Accountants of India


Prevention of Money Laundering Act, 2002 4.12

♦ Whether provisions of Section 8 were invalid for procedural vagueness and for
exclusion of mens rea of criminality in acquisition of such property and for enjoining
deprivation of possession of immovable property even before conclusion of
guilt/conviction in prosecution for an offence of money laundering?
♦ Whether presumption enjoined by Section 23 was unreasonably restrictive, excessively
disproportionate?
♦ Whether shifting/imposition of the burden of proof, by Section 24 is arbitrary and invalid
and was applicable only to trial of offence under Section 3?
Rule of Law
♦ Section 3 of PMLA
♦ Section 5 of PMLA
♦ Section 8 of PMLA
♦ Section 23 of PMLA
Key Ratio Decidendi
1. It was held that “object of Act is to prevent money laundering and connected activities
and confiscation of "proceeds of crime" and preventing legitimizing of money earned
through illegal and criminal activities by investments in movable and immovable
properties often involving layering of money generated through illegal activities.
Therefore, the Act defines expression "proceeds of crime" expansively to sub-serve
broad objectives of Act. Thus property owned or in possession of a person, other than a
person charged of having committed scheduled offence was equally liable to
attachment and confiscation proceedings under Chapter III”.
2. It was held that “Parliament has authority to legislate and provide for forfeiture of
proceeds of crime which is a produce of specified criminality acquired prior to
enactment of Act as well. It has also authority to recognize degrees of harm such
conduct has on fabric of society and to determine appropriate remedy. Thus provisions
of second proviso to Section 5 were applicable to property acquired even prior to
coming into force of this provision and even so were not invalid for retrospective
penalization.”
3. It was held that “considering object and scheme of Act, provisions of Section 8 could
not be held invalid for vagueness; incoherence as to onus and standard of proof;
ambiguity as regards criteria for determination of nexus between property targeted for
attachment/confirmation and offence of money-laundering; or for exclusion of mens rea/
knowledge of criminality in acquisition of such property. Section 8(4) which enjoined
deprivation of possession of immovable property pursuant to order confirming

© The Institute of Chartered Accountants of India


4.13 Economic Laws

provisional attachment and before conviction of accused for offence of money-


laundering, was valid.”
4. It was held that “Section 23 enjoins a rule of evidence and rebuttable presumption
considered essential and integral to effectuation of purposes of Act in legislative
wisdom. Thus, validity of provision was upheld.”
5. It was held that “where property is in ownership, control or possession of person not
accused of having committed an offence under Section 3 and where such property is
part of inter-connected transactions involved in money laundering, then and in such
event presumption enjoined in Section 23 comes into operation and not inherence of
burden of proof under Section 24 of the Act. Therefore person other than one accused
of having committed offence under Section 3 is not imposed the burden of proof
enjoined by Section 24. On person accused of offence under Section 3, burden applies,
also for attachment and confiscation proceedings.”
Link to order
https://www.casemine.com/judgement/in/56ea714d607dba369a6ee772

9. J. Sekar and others v. ED

Citation: Delhi HC, Writ Petition (C) No. 5320 of 2017 dated 11.01.2018

Facts
All the writ petitions under Article 226 of the Constitution of India in the present case, only one
prayer is for a declaration that the second proviso to Section 5 (1) of the Prevention of Money-
Laundering Act, 2002 (PMLA) is ultra vires to Article 14 of the Constitution of India
Issue
Constitutional validity of proviso to Section 5(1) of the PMLA
Rule of Law
♦ Section 3 of PMLA
♦ Section 5 of PMLA
♦ Section 8 of PMLA
♦ Section 23 of PMLA
Key Ratio Decidendi
1. The second proviso to Section 5(1) of the PMLA is not violative of Article 14 of the
Constitution of India; the challenge in that regard in these petitions is hereby negatived.

© The Institute of Chartered Accountants of India


Prevention of Money Laundering Act, 2002 4.14

2. The expression reasons to believe has to meet the safeguards inbuilt in the second
proviso to Section 5(1) of PMLA read with Section 5(1) of PMLA.
3. The expression reasons to believe' in Section 8(1) of PMLA again has to satisfy the
requirement of law as explained in this decision.
4. There has to be a communication of the ‘reasons to believe' at every stage to the
noticee under Section 8(1) of PMLA.
5. The noticee under Section 8(1) of PMLA is entitled access to the materials on record
that constituted the basis for reasons to believe subject to redaction in the manner
explained hereinbefore, for reasons to be recorded in writing.
6. If there is a violation of the legal requirements outlined hereinbefore, the order of the
provisional attachment would be rendered illegal.
7. There can be single-member benches of the AA and the Administrative Tribunal under
the PMLA. Such single-member benches need not mandatorily have to be Judicial
Member and can be Administrative Members as well.
Link to order
http://delhihighcourt.nic.in/dhcqrydisp_o.asp?pn=286829&yr=2018

© The Institute of Chartered Accountants of India


5
Foreign Exchange Management
Act, 1999

1. IDBI Trusteeship Services Limited v. Hubtown Ltd

Citation: Supreme Court, Civil Appeal No.10860 of 2016 dated 15.11.2016

Facts
♦ FMO, a non-resident foreign entity, made an investment into an Indian company, Vinca
Developer Private Limited (Vinca) by way of compulsorily convertible debentures
(CCPS) and equity shares.
♦ The CCPS were to convert into 99% of the voting shares of Vinca. The proceeds of the
investment were further invested by Vinca in its wholly owned subsidiaries, Amazia
Developers Private Limited (Amazia) and Rubix Trading Private Limited (Rubix) by way
of optionally convertible debentures (OPCDs) bearing a fixed rate of interest.
♦ IDBI Trusteeship Services Ltd. (Debenture Trustee) was appointed as a debenture
trustee in relation to the OPCDs, acting for the benefit of Vinca. Hubtown Limited
(Hubtown) also issued a corporate guarantee in favour of the Debenture Trustee to
secure the OPCDs.
♦ Amazia and Rubix defaulted on the OPCDs and Hubtown, being called upon to pay
under the guarantee, failed to do so. Accordingly the Debenture Trustee filed a
summary suit against Hubtown in the Bombay High Court.
Issue
Under which circumstances a defendant may be granted leave to defend in a suit for summary
judgment?
Bombay HC
1. The Bombay High Court granted Hubtown an unconditional leave to defend in the
summary suit.

© The Institute of Chartered Accountants of India


Foreign Exchange Management Act, 1999 5.2

2. The order of the High Court proceeded on the premise that the various transactions
including the CCPS and OPCDs should be construed as a whole, since the court was of
the view that they constituted a colourable device for providing assured return to the
foreign investor, which according to the High Court was violative of the FEMA
guidelines.
3. The Bombay High Court found that since Vinca would be owned by FMO upon
conversion of the CCDs, by virtue of the fixed return on the OPCDs, FMO was indirectly
obtaining a fixed return on its investment (which is not allowed under certain
circumstances in the FEMA guidelines).
4. The court further held that the investors, having participated in the illegality, could not
seek the assistance of the court to recover amounts invested illegally.
Supreme Court
1. The court noted that the investment was made by FMO in Vinca for subscription to
shares as well as compulsorily convertible debentures. This transaction was not
violative of the FEMA regulations.
2. The investment made by Vinca in Amazia and Rubix by way of OPCDs was also prima
facie in compliance with FEMA regulations. Further, once the corporate guarantee was
invoked, a payment made under the corporate guarantee would be a transaction
between residents. At this stage also prima facie again, there was no infraction of the
FEMA regulations.
3. The court further held since FMO was to become a 99% holder of Vinca after the
requisite time period had elapsed, FMO would at that stage have the ability to utilise
the funds received pursuant to the overall structure in India. Again prima facie there
would have been no breach of FEMA regulations. At the stage that FMO wishes to
repatriate such funds, RBI permission would be necessary. If RBI permission were not
granted (and therefore the amounts were retained in India), then again there would be
no infraction of FEMA regulations.
4. The Supreme Court further examined the different categories of defences that a judge
should keep in mind before granting an unconditional leave to defend in a summary
suit. The court held that in the present case, the defence of the defendant was in the
realm of ‘plausible but improbable’ as the defendants had initially serviced the OPCDs
before occurrence of the default and also there was no prima facie breach of the FEMA
regulations. The court accordingly directed Hubtown to deposit the principal amount
claimed under the guarantee as a precondition to defend the suit.
Link to Order
https://sci.gov.in/supremecourt/2015/33244/33244_2015_Judgement_15-Nov-2016.pdf

© The Institute of Chartered Accountants of India


5.3 Economic Laws

2. Cruz City I Mauritius Holdings v. Unitech Limited

Citation: Delhi High Court, EX.P.132/2014, dated 11.04.2017

Facts
♦ Cruz City 1 Mauritius Holdings (Cruz City) filed a petition in the Delhi High Court for
enforcement of an arbitral award rendered under the rules of the London Court of
International Arbitration (Award).
♦ This required Unitech Limited (Unitech) and Burley Holding Limited (Burley), a wholly
owned subsidiary of Unitech, to pay Cruz City the pre-determined purchase price of all
of Cruz City’s equity shares in a joint venture (incorporated in Mauritius) pursuant to:
1. A “put option” exercised by Cruz City against Burley.
2. A keep-well agreement (which was in the nature of a guarantee) whereby
Unitech was to make the necessary financial contribution in Burley to enable it to
meet its obligations.
♦ However, when that award was sought to be enforced before the Delhi High Court,
Unitech claimed that the enforcement of award is impressible under FEMA as it is
against public policy.
Issue
Whether violation of any regulation or any provision of FEMA would ipso jure offend the public
policy of India?
Key Ratio Decidendi
The Delhi High Court critically examined the scope of the “public policy” exception under
section 48 of the Arbitration & Conciliation Act, 1996 and considered “whether violation of any
regulation or any provision of FEMA would ipso jure offend the public policy of India”.
The court held that “the width of the public policy defense to resist enforcement of a foreign
award, is extremely narrow. And the same cannot be equated to offending any particular
provision or a statute.”
“It plainly follows from the above that a contravention of a provision of law is insufficient to
invoke the defence of public policy when it comes to enforcement of a foreign award.
Contravention of any provision of an enactment is not synonymous to contravention of
fundamental policy of Indian law. The expression fundamental Policy of Indian law refers to
the principles and the legislative policy on which Indian Statutes and laws are founded. The
expression "fundamental policy" connotes the basic and substratal rationale, values and
principles which form the bedrock of laws in our country. The expression "fundamental policy

© The Institute of Chartered Accountants of India


Foreign Exchange Management Act, 1999 5.4

of law" must be interpreted in that perspective and must mean only the fundamental and
substratal legislative policy and not a provision of any enactment.”
Hence, foreign arbitral award can be enforced in India pertaining to put options, exit at
assured return, and guarantee arrangements and the provisions of FEMA and related
regulations cannot be claimed as defense by Indian parties
Link to Order
http://lobis.nic.in/ddir/dhc/VIB/judgement/12-04-2017/VIB11042017EX1322014.pdf

3. NTT Docomo Inc. v. Tata Sons Ltd

Citation: Delhi High Court, O.M.P.(EFA)(COMM.) 7/2016, dated 28.04.2017

Facts
♦ In 2009, NTT Docomo Inc. (Docomo), Tata Sons Ltd. (Tata) and Tata Teleservices Ltd.
(TTSL) entered into a shareholders' agreement. Under the agreement, Tata was
required to find a buyer for Docomo's shares in TTSL, in the event that TTSL failed to
meet certain performance parameters.
♦ The sale price was required to be the higher of (i) the fair value of the shares; or (ii)
50% of the price at which Docomo had purchased the shares. The clause was intended
to give Docomo downside protection on its investment.
♦ In 2014, Docomo exercised this right and called upon Tata to find a buyer for its shares
in TTSL. Tata argued that it was not under an unconditional obligation, and that it had
the option to decide whether to find a buyer or to buy the shares itself. Having chosen
to buy the shares itself, the 'special permission' of the RBI was required, since the
value of the shares had fallen. Tata argued that since the 'special permission' of the
RBI was not forthcoming, it was not liable to purchase the shares under the contract.
♦ In a unanimous award, a three-member arbitral tribunal rejected this argument and held
that Tata was under an unqualified obligation to perform. It held that the impediment to
Tata's performance was factual rather than legal, and that the contract could be
performed even without the special permission of the RBI. The tribunal then proceeded
to award damages to Docomo to the extent of USD 1.17 billion along with interest and
costs.
♦ The amount of damages represented the amount that Docomo would have received
had Tata performed the contract. Since Tata did not pay the amounts awarded,
Docomo filed an enforcement petition in the Delhi High Court.
♦ While Tata resisted the enforcement initially, the parties ultimately reached a
compromise, and filed consent terms with the Court, essentially giving effect to the

© The Institute of Chartered Accountants of India


5.5 Economic Laws

award. During the course of the proceedings, the RBI impleaded itself in these
proceedings, and argued that neither the award nor the consent terms should be given
effect since it would lead to a violation of foreign exchange regulations.
Issue
The primary issue in dispute before the Delhi High Court was the legitimacy of RBI’s
objections to the Award’s enforcement.
Delhi High Court
1. The Court analysed Section 48(1) along with Section 2(h) of the Arbitration and
Conciliation Act, 1996 (the “Arbitration Act”) and concluded that there is no provision
envisaged under the Arbitration Act which permits intervention by an entity that is not a
party to the award, to oppose enforcement of an arbitral award.
2. The Court then considered Order XXIII Rule 3 of the CPC which provides that a
compromise must be lawful. The Court held that the mere fact that a statutory body’s
power and jurisdiction might be discussed in an adjudication or an Award will not
confer locus standi on such body or entity to intervene in those proceedings. At the
same time, the RBI will, just as any other entity, be bound by an award interpreting the
scope of its powers and any of its regulations subject to it being upheld by a Court
when challenged by a party to the award. It also held that the RBI does not have the
locus to challenge the decision of a court / arbitral tribunal interpreting Indian
regulations in a contractual dispute. Thus, for instance, if an arbitral tribunal determined
that the RBI's permission was not required for a particular payment, it was not open to
the RBI to challenge such determination.
3. The Court then went on to examine the validity of the arbitral award. It agreed with the
interpretation of the tribunal in holding that Tata's obligations were capable of
performance without the special permission of the RBI. It also held that the tribunal's
interpretation of FEMA was not improbable, and did not violate Indian public policy. The
Court opined that there were no provisions in FEMA that absolutely prohibited a
contractual obligation from being performed. It only envisaged a grant of special
permission of the RBI.
4. The Court held that it was correctly observed by the AT that Clause 5.7.2 of SHA was
legally capable of performance even without the special permission of the RBI because
such permission could be generally obtained under sub-regulation 9(2) of FEMA where
shares of an Indian company are transferred between two non-resident entities. With
regard to the legality of the Award, the Court agreed with the AT and stated that it was
rightly pointed by the AT that the clauses of SHA were in consonance with the
provisions of Indian law and therefore the grounds under Section 48 of Arbitration Act
could also not be attracted. Docomo invested US $2.5 billion and would just receive
half of that amount as Award, thus making it neither perverse nor improbable.

© The Institute of Chartered Accountants of India


Foreign Exchange Management Act, 1999 5.6

5. Lastly, the Court examined the validity of the compromise terms agreed between the
parties, and held that they were enforceable as well. In coming to this finding, the Court
placed considerable emphasis on the importance of giving effect to contracts entered
into by Indian entities while attracting international investors and building goodwill in the
international arena.
Link to order
http://lobis.nic.in/ddir/dhc/SMD/judgement/28-04-2017/SMD28042017OMPENFCOMM72016.pdf

4. Venture Global v. Tech Mahindra

Citation: Supreme Court, Civil Appeal No. 17753-17755 of 2017 dated 01.11.2017

Facts
The case arose out of an international commercial arbitral award rendered in London in 2006,
pursuant to a Joint Venture Agreement (JV) between Venture Global, an American company
and Tech Mahindra. In keeping with the terms of this JV, the award held that there was an
‘event of default’ at Venture Global’s behest and directed that it transfer its 50% share in the
JV to Tech Mahindra at book value. Following enforcement proceedings at the High Court
level, the matter was taken up on appeal by the Supreme Court. Two main issues were framed
– whether the patent illegality standard was to be applied in this case and whether a violation
of the Foreign Exchange Management Act (FEMA) arising out of the share transfer directed by
the arbitral tribunal would render the award unenforceable in India on public policy grounds.
Issue
What is the applicability of the “patent illegality” limb of public policy to international
commercial arbitrations?
Whether an award that merely violates Indian municipal law would be rendered unenforceable
under this ground?
Key Ratio Decindi
The 2015 Amendment to the Arbitration Act which provided that the patent illegality standard
would not apply to international commercial arbitrations did not apply to this case, since
proceedings commenced much before the Amendment (which applies only prospectively).
With the patent illegality standard therefore applicable to the foreign award in this case, the
Associate Builders standard (that is only applicable to domestic awards post-2015) was
applied in this case.
Violation of certain provisions of the Foreign Exchange Management Act (‘FEMA’), the Indian
Penal Code (‘IPC’) and the Companies Act themselves would also render the award
unenforceable on public policy grounds raises alarm bells.

© The Institute of Chartered Accountants of India


5.7 Economic Laws

The court relied on the judgement of Renusagar v. General Electric wherein it was held that
any FERA violation, however technical, would render an international commercial arbitral
award unenforceable on public policy grounds.
Link to order
https://sci.gov.in/supremecourt/2013/29119/29119_2013_Judgement_01-Nov-2017.pdf

5. Mr. S. Bhaskar vs Enforcement Directorate FEMA

Citation: Karnataka High Court, M.F.A. No. 4546/2004 (FEMA) dated 17.03.2011

Facts
On 9.10.2002, the appellant was found in illegal possession of US $20,000/-. The Deputy
Director, Enforcement Directorate after holding an enquiry found the appellant guilty of
contravening Section 3(a) of the Act and accordingly passed the order dated 29.01.2003
referred to above by imposing a penalty of ` 50,000/- on the appellant under Section 13(1) of
the Act; the penalty of ` 50,000/- imposed was directed to be adjusted from the seized US
$20,000/- and the balance amount was ordered to be released to the appellant. The Deputy
Director thought it fit not to exercise the power under Section 13(2) of the Act to confiscate the
foreign currency involved in the offence.
Being aggrieved by the aforesaid order, the respondent-Enforcement Directorate carried the
matter to the Appellate Tribunal for Foreign Exchange, New Delhi by filing a Revision Petition
under Section 19(6) of the Act. The Appellate Tribunal, on consideration of the matter, found
that the power exercised by the Deputy Director was contrary to law and accordingly by the
order impugned herein has set aside the order of the Deputy Director in so far as it related to
release of the foreign currency by ordering confiscation of the seized currency of US $20,000/-
after adjustment of the penalty of ` 50,000/- imposed by the Deputy Director
Issue
Whether the Appellate Tribunal was justified in law in modifying the order of the Deputy
Director dated 29.01.2003 by directing confiscation of the foreign currency of US $20,000/-
after adjusting the penalty of ` 50,000/- therefrom?
Key Ratio Decidendi
Section 13 of the Act speaks of penalties; both the sub-Sections provide for imposition of
different kinds of penalty. A plain reading of the Section would show that imposition of a
penalty under sub-Section (1) will not bar exercise of power under sub-Section (2) to
confiscate any currency in respect of which the contravention has taken place. The power of
confiscation conferred under sub-Section (2) is in addition to the power to impose penalty
under sub-Section (1). Therefore, it is perfectly open to the Adjudicating Authority to exercise

© The Institute of Chartered Accountants of India


Foreign Exchange Management Act, 1999 5.8

power under sub-Section (2) in addition to exercise of power under sub-Section (1). In other
words, it is open to the Adjudicating Authority to impose any penalty as provided under sub-
Section (1) as well as directing confiscation of currency/security/money or property in respect
of which the contravention has taken place.
On the facts of the case, the Deputy Director was not right in exercising his discretion in
ordering release of the seized foreign currency. It is relevant to state that possession of the
foreign currency of US $20,000/- by the appellant was admittedly illegal; he had not traced his
possession of the foreign currency to any legitimate source of acquisition.
Link to order
http://scconline.com/DocumentLink/Rz6U2qqb

6. Union of India & Ors vs M/s Premier Limited

Citation: Supreme Court, Civil Appeal No. 3529 of 2008, dated

Facts
♦ On 01.05.1991, a memorandum to show cause notice was issued by the Special
Director to respondent Nos. 2, 3 and 4, namely, M/s. Godrej Industries Ltd and its two
Directors (R-3 and R-4) for allegedly committing contravention of Sections 9(1)(a),
9(1)(c) and Section 16(1) of the Foreign Exchange Regulation Act, 1973 (hereinafter
referred to as “FERA”) in respect of imports and exports of certain commodities made
with two foreign parties, viz., M/s. Fingrain, S.A., Geneva and M/s. Continental Grain
Export Corporation, New York during the year 1977-78.
♦ During the pendency of the proceedings, FERA was repealed with effect from
01.06.2000. It was, however, replaced by Foreign Exchange Management Act, 1999
(hereinafter referred to as “FEMA”).
♦ On 05.12.2003, an adjudication order was passed by the Deputy Director of
Enforcement under FEMA read with FERA in relation to the show cause notice dated
01.05.1991. By this order, penalty of ` 15,50,000/- was imposed on M/s. Godrej
Industries Ltd. and its two Directors for contravening the provisions of Sections 9(1)(a)
and 9(1)(c) read with Section 16(1) of FERA.
♦ On 15.01.2004, the respondent Nos. 2 to 4 felt aggrieved by the adjudication order
dated 05.12.2003 and filed appeal before the Special Director (Appeals) under Section
17 of FEMA.
♦ On 08.09.2004 and 08.11.2004, the Special Director (Appeals) dismissed the appeals
as being not maintainable. He held that the Special Director (Appeals) has no

© The Institute of Chartered Accountants of India


5.9 Economic Laws

jurisdiction to hear the appeals against the adjudication order passed under Section 51
of FERA.
♦ Respondent Nos. 2 to 4 felt aggrieved by orders dated 08.09.2004 and 08.11.2004 and
filed writ petitions before the High Court of Bombay at Mumbai. By impugned common
order, the High Court allowed the writ petitions and quashed the orders of the Special
Director (Appeals). The High Court held that the appeals filed by respondent Nos. 2 to
4 before the Special Director (Appeals) against the adjudication order dated 05.12.2003
were maintainable in as much as the Special Director (Appeals) possessed the
jurisdiction to decide the appeals on merits.
Issue
If the Adjudicating Officer has passed an order after the repeal of FERA in the proceedings
initiated prior to 01.06.2000, whether an appeal against such order will lie before the “Special
Director (Appeals)” under Section 17 of FEMA or before the “Appellate Tribunal” under Section
19 of FEMA.
Key Ratio Decidendi
Section 49(5)(b) of FEMA deals with repeal and saving in relation to the action taken and to be
taken under FERA, 1973. Reading of this Section shows that the legislature has equated the
Appellate Board constituted under FERA with the Appellate Tribunal constituted under FEMA
for disposal of the appeals filed under Section 52(2) of FERA against an order passed under
Section 51 of FERA which were pending before the Appellate Board as on 01.06.2000. Such
appeals stood transferred from the Appellate Board to the Appellate Tribunal for their disposal
in accordance with law.
The reason as to why a specific provision for transfer of such pending appeals was made for
their disposal from the Appellate Board to the Appellate Tribunal was that the Appellate Board
constituted under FERA stood dissolved by Section 49(1) of FEMA with effect from
01.06.2000.
It is this dissolution of the Appellate Board, which necessitated the legislature to make a
corresponding provision in the new Act (FEMA) so that the consequences arising out of the
dissolution of the Appellate Board constituted under FERA is taken care of by another
appellate authority constituted under the FEMA and all pending appeals are automatically
transferred to the Appellate Board for their disposal under FEMA.
 So far as Section 49(5)(b) of FEMA is concerned it specifically provides that the
appeals filed under Section 52(2) of FERA against the order passed under Section 51
of FERA will be decided by the Appellate Tribunal under FEMA.
 So far as Section 81(c) of FERA, 1973 is concerned, it deals with Repeal and Saving of
FERA, 1947. Clause (c) of Section 81 specifically provides that all the appeals filed
under Section 23 of FERA, 1947, whether pending on the date of Repeal or/and those

© The Institute of Chartered Accountants of India


Foreign Exchange Management Act, 1999 5.10

filed after the repeal of FERA, 1947, shall be disposed of by the Appellate Board
constituted under FERA, 1973.
While Section 49(5)(b) of FEMA is not worded alike Section 81(c) of FERA, yet, in our view, it
shows the intention of the legislature that all such appeals have to be heard by the Appellate
Board under the FERA. The legislative intent contained in Section 81(c) can be taken into
account for interpreting the relevant provisions of FERA and FEMA for deciding the question
which is the subject matter of this appeal.
Link to Order
https://sci.gov.in/supremecourt/2007/7781/7781_2007_Judgement_29-Jan-2019.pdf

7. Vodafone International Holding (VIH) v. Union of India (UOI)

Citation: Supreme Court, Civil Appeal No. 733 of 2012 dated 20.01.2012

Facts
Vodafone International Holding (VIH) and Hutchison telecommunication international limited or
HTIL are two non-resident companies. These companies entered into transaction by which
HTIL transferred the share capital of its subsidiary company based in Cayman Island i.e. CGP
international or CGP to VIH. VIH or Vodafone by virtue of this transaction acquired a
controlling interest of 67 percent in Hutch is on Essar Limited or HEL that was an Indian Joint
venture company (between Hutchinson and Essar) because CGP was holding the above 67
percent interest prior to the above deal. The Indian Revenue authorities issued a show cause
notice to VIH as to why it should not be considered as “assesse in default” and thereby sought
an explanation as to why the tax was not deducted on the sale consideration of this
transaction. The Indian revenue authorities thereby through this sought to tax capital gain
arising from sale of share capital of CGP on the ground that CGP had underlying Indian
Assets. VIH filed a writ petition in the High Court challenging the jurisdiction of Indian revenue
authorities. This writ petition was dismissed by the High Court and VIH appealed to the
Supreme Court which sent the matter to Revenue authorities to decide whether the revenue
had the jurisdiction over the matter. The revenue authorities decided that it had the jurisdiction
over the matter and then matter went to High Court which was also decided in favour of
Revenue and then finally Special Leave petition was filed in the Supreme Court.
Issue
Whether the Indian revenue authorities had the jurisdiction to tax an offshore transaction of
transfer of shares between two non-resident companies whereby the controlling interest of an
Indian resident company is acquired by virtue of this transaction?

© The Institute of Chartered Accountants of India


5.11 Economic Laws

Key Ratio Decidendi


Corporate structures
 Multinational companies often establish corporate structures or affiliate subsidiaries or
joint ventures for various business and commercial purposes and these are primarily
aimed to yield better returns to the investors and help in progress of the company.
 And therefore the burden is entirely upon the revenue to show that such incorporation,
consolidation, restructuring has been affected for fraudulent purpose so as to defeat
the law or evade the taxes.
Overseas companies
 Many overseas companies invest in countries like Mauritius, Cayman Island due to
better opportunities of investment and these are undertaken for sound commercial and
sound legitimate tax planning and not to conceal their income or assets from home
country tax jurisdiction and India have recognised such structures.
 These offshore transactions or these offshore financial centres do not necessarily lead
to the conclusion that these are involved in tax evasion.
Holding and Subsidiary Companies
 The companies act have recognized that subsidiary company is a separate legal entity
and though holding company control the subsidiary companies and respective business
of the company within a group but it is settled principle that business of subsidiary is
separate from the Holding company.
 The assets of subsidiary companies can be kept as collateral by the parent company
but still these two are distinct entities and the holding company is not legally liable for
the acts of subsidiaries except in few circumstances where the subsidiary company is a
sham.
 The Holding company and subsidiary companies may form pyramid of structures
whereby the subsidiary company may hold controlling interest in other companies
forming parent company.
Shares and controlling interest
 The transfer of shares and shifting of controlling interest cannot be seen as two
separate transactions of transfer of shares and transfer of controlling interest.
 The controlling interest is not an identifiable or a distinct capital asset independent of
holding of shares and is inherently a contractual right and not property right and cannot
be considered as transfer of property and capital assets unless the Statue stipulates
otherwise.

© The Institute of Chartered Accountants of India


Foreign Exchange Management Act, 1999 5.12

 The acquisition of shares may carry acquisition of controlling interest which is purely
commercial concept and tax is levied on transaction and not on its effect.
Role of CGP
 CGP was already part of HTIL corporate structure and sale of CGP share was a
genuine business transaction and commercial decision taken interest of investors and
corporate entity and not a dubious one.
The site of shares of CGP
 Shares of CGP were registered in Cayman Island and law of Cayman also does not
recognize multiplicity of registers and hence site of shares and transfer of shares is
situated in Cayman and shall not shift to India.
Extinguishment of rights of HTIL in HEL
 The transfer of CGP share automatically resulted in host of consequences that included
transfer of controlling interest and controlling interest cannot be dissected from CGP
share without legislative intervention.
 Upon transfer of shares of the holding Company, the controlling interest may also pass
on to the purchaser along with the shares and this Controlling interest might have
percolated down the line to the operating companies but that controlling interest is still
inherently remains contractual and not a property right unless otherwise is provided by
the statue.
 The acquisition of shares may carry the acquisition of controlling interest and this is
purely a commercial concept and the tax can be levied only on the transaction and not
on its effect and hence, consequently, on transfer of CGP share to Vodafone, Vodafone
got control over eight Mauritian Company and this does not mean that the site of CGP
share has shifted to India for the purpose of charging capital gains tax.
Hence, Sale of CGP share by HTIL to Vodafone or VIH does not amount to transfer of capital
assets within the meaning of Section 2 (14) of the Income Tax Act and thereby all the rights
and entitlements that flow from shareholder agreement etc. that form integral part of share of
CGP do not attract capital gains tax.
Link to order
https://sci.gov.in/jonew/judis/39003.pdf

© The Institute of Chartered Accountants of India


5.13 Economic Laws

8. Kanwar Natwar Singh vs Director Of Enforcement & Anr.

Citation: Supreme Court, Civil Appeal No. 8601 of 2010 dated 05.10.2010

Facts
Natwar Singh & Jagat Singh were alleged to have dealt in and acquired Foreign Exchange
totaling US $8,98,027 in respect of some Iraq oil contracts in contravention of FEMA.
A notice was issued asking Natwar Singh to show-cause why an inquiry should not be held
against them. In response, Natwar Singh demanded that the Adjudicating Authority furnish
“copies of all documents in … possession in respect of the instant case, including the 83000
documents allegedly procured by one Virender Dayal”.
The Adjudicating Authority furnished copies of the documents as were relied upon by it but
declined to furnish copies of other documents and decided to hold an inquiry in accordance
with FEMA. This non-furnishing of “all documents” was challenged by Natwar Singh in the
Delhi High Court which dismissed the challenge.
Issue
Whether a noticee served with show cause notice under Rule 4(1) of the Foreign Exchange
Management (Adjudication Proceedings and Appeal) Rules, 2000 (hereinafter referred to as
‘the Rules’) is entitled to demand to furnish all the documents in possession of the
Adjudicating Authority including those documents upon which no reliance has been placed to
issue a notice requiring him to show cause why an inquiry should not be held against him?
Key Ratio Decidendi
The extent of applicability of principles of natural justice depends upon the nature of inquiry,
the consequences that may visit a person after such inquiry from out of the decision pursuant
to such inquiry. The right to fair hearing is a guaranteed right. Every person before an
Authority exercising the adjudicatory powers has a right to know the evidence to be used
against him. However, the principles of natural justice do not require supply of
documents upon which no reliance has been placed by the Authority to set the law into
motion. Supply of relied on documents based on which the law has been set into motion
would meet the requirements of principles of natural justice.
The concept of fairness is not a one way street. The principles of natural justice are not
intended to operate as roadblocks to obstruct statutory inquiries. Duty of adequate disclosure
is only an additional procedural safeguard in order to ensure the attainment of the fairness and
it has its own limitations. The extent of its applicability depends upon the statutory framework.
The only object of Natwar Singh’s unreasonable insistence for supply of all documents was
obviously to obstruct the proceedings and he has been able to achieve that object as is

© The Institute of Chartered Accountants of India


Foreign Exchange Management Act, 1999 5.14

evident from the fact that the inquiry initiated as early as in the year 2006 still did not even
commence.
Furthermore, observations of Courts are not to be read as Euclid’s theorems nor as provisions
of the statute. The observations must be read in the context in which they appear. A line or a
word in a judgment cannot be read in isolation or as if interpreting a statutory provision to
impute a different meaning to the observations.
Link to Order
https://sci.gov.in/jonew/judis/36937.pdf

9. S.K. Sinha, Chief Enforcement Officer vs Videocon


International Ltd.

Citation: Supreme Court, Appeal (crl.) 175 of 2007 dated 25.01.2008

Facts
The Foreign Exchange Regulation Act, 1973 (FERA) was repealed with effect from 01.06.2000
on coming into force of the Foreign Exchange Management Act, 1999 (FEMA). Section 49(3)
of FEMA says that “notwithstanding in any other law…., no Court shall take cognizance of an
offence under the repealed Act……after expiry of a period of two years from the date of
coming into force of FEMA.”
A complaint under FERA was filed, taken cognizance of on May 24, 2002 and issue of
summons also was ordered on the same day making the process returnable on 07.02.2003.
Process, however was issued on February 3, 2003.
On these facts the High Court equated taking cognizance with issue of process and held the
complaint to be barred under Section 49(3) of FEMA
Issue
Whether issuance of process in a criminal case is one and the same thing or can be equated
with taking cognizance by a Criminal Court? And if the period of initiation of criminal
proceedings has elapsed at the time of issue of process by a Court, the proceedings should
be quashed as barred by limitation?
Key Ratio Decidendi
The expression “cognizance” has not been defined in the Code. But the word (cognizance) is
of indefinite import. It has no esoteric or mystic significance in criminal law. It merely means
“become aware of” and when used with reference to a court or a Judge, it connotes “to take
notice of judicially”. It indicates the point when a court or a Magistrate takes judicial notice of

© The Institute of Chartered Accountants of India


5.15 Economic Laws

an offence with a view to initiating proceedings in respect of such offence said to have been
committed by someone.
The Supreme Court further held that “taking cognizance” does not involve any formal action of
any kind. It occurs as soon as a Magistrate applies his mind to the suspected commission of
an offence. Cognizance is taken prior to commencement of criminal proceedings. Taking of
cognizance is thus a sine qua non or condition precedent for holding a valid trial. Cognizance
is taken of an offence and not of an offender. Whether or not a Magistrate has taken
cognizance of an offence depends on the facts and circumstances of each case and no rule of
universal application can be laid down as to when a Magistrate can be said to have taken
cognizance.
In the case on hand, it is amply clear that cognizance of the offence was taken by the Chief
Metropolitan Magistrate, Mumbai on May 24, 2002, i.e., the day on which the complaint was
filed, the Magistrate, after hearing the counsel for the department, took cognizance of the
offence and passed the necessary order. Undoubtedly, the process was issued on February 3,
2003. In our judgment, however, it was in pursuance of the cognizance taken by the Court on
May 24, 2002 that a subsequent action was taken under Section 204 under Chapter XVI.
Taking cognizance of offence was entirely different from initiating proceedings; rather it was
the condition precedent to the initiation of the proceedings. Order of issuance of process on
February 3, 2003 by the Court was in pursuance of and consequent to taking cognizance of an
offence on May 24, 2002.
The High Court, in our view, therefore, was not right in equating taking cognizance with
issuance of process and in holding that the complaint was barred by law and criminal
proceedings were liable to be quashed. The order passed by the High Court, thus, deserves to
be quashed and set aside.
Link to Order
https://sci.gov.in/jonew/judis/30187.pdf

© The Institute of Chartered Accountants of India


6
Prohibition of Benami Property
Transactions Act, 1988

1. Mangathai Ammal (Died) Through Lrs vs Rajeswari

Citation: Supreme Court, Civil Appeal no. 4805 of 2019 dated 09.05.2019

Facts
The Supreme Court was considering an appeal against Trial Court and High Court orders which
held that the suit properties are benami transactions as the part sale consideration was paid by
another person (Narayanasamy Mudaliar) at the time of the purchase of the property. It was also
found that the stamp duty at the time of the execution of the Sale Deed was purchased by
Mudaliar.
Issue
♦ Whether the transactions/Sale Deeds in favour of defendant no.1 can be said to be benami
transactions or not?
♦ The defendants also contended was that since by Benami Amendment Act, 2016, Section 3
(2) of the Benami Transaction Act, 1988 the statutory presumption, which was rebuttable,
has been omitted, the plea of statutory transaction that the purchase made in the name of
wife or children is for their benefit would not be available in the present case?
Key Ratio Decidendi
“While considering a particular transaction as benami, the intention of the person who
contributed the purchase money is determinative of the nature of transaction. The intention of
the person, who contributed the purchase money, has to be decided on the basis of the
surrounding circumstances; the relationship of the parties; the motives governing their action
in bringing about the transaction and their subsequent conduct etc."
“To hold that a particular transaction is benami in nature these six circumstances can be taken
as a guide:
1. The source from which the purchase money came;

© The Institute of Chartered Accountants of India


6.2 Economic Laws

2. the nature and possession of the property, after the purchase;


3. Motive, if any, for giving the transaction a benami colour;
4. Position of the parties and the relationship, if any, between the claimant and the alleged
benamidar;
5. Custody of the title deeds after the sale;
6. Conduct of the parties concerned in dealing with the property after the sale.”
"Furthermore, the court held that the Benami Transaction (Prohibition) Act would not be
applicable retrospectively"
Link to Order
https://sci.gov.in/supremecourt/2016/23345/23345_2016_Judgement_09-May-2019.pdf

2. Smt. P.Leelavathi vs V.Shankarnarayana Rao

Citation: Supreme Court, Civil Appeal No. 1099 of 2008 dated 09.04.2019

Facts
• Late G. Venkata Rao was an Estate Agent and he was doing money lending business
in his name and also in the names of his sons and he was purchasing properties in the
names of his sons, though his father was funding those properties.
• According to the plaintiff, at the time of his death, G. Venkata Rao was in possession of
a large estate comprising of immoveable properties, bank deposits etc.
• It was the case that the suit schedule properties were as such joint family properties
and/or they were purchased in fact by their late father G. Venkata Rao and the same
was funded by their father.
• That, the plaintiff was entitled to 1/4th share in all the said properties belonging to her
father.
Issue
Whether the transactions can be said to be benami in nature merely because some financial
assistance has been given by the father (Late G. Venkata Rao) to the sons (defendants) to
purchase the properties, subject matter of the suit (filed by his daughter, claiming share in
these properties)
Key Ratio Decidendi

© The Institute of Chartered Accountants of India


Prohibition of Benami Property Transactions Act, 1988 6.3

In the case of Binapani Paul v. Pratima Ghosh the court had held that “the source of money
had never been the sole consideration, and is only merely one of the relevant considerations
but not determinative in character.”
In Valliammal v. Subramaniam, the court had delineated six circumstances to check whether
the transaction is benami or not. These are: (a) The source from which the purchase money
came; (b) the nature and possession of the property, after the purchase; (c) Motive, if any, for
giving the transaction a benami color; (d) Position of the parties and the relationship, if any,
between the claimant and the alleged benamidar; (e) Custody of the title deeds after the sale;
and (f) Conduct of the parties concerned in dealing with the property after the sale.
"It is true that, at the time of purchase of the suit properties, some financial assistance was
given by Late G. Venkata Rao. However, as observed by this Court in the aforesaid decisions,
that cannot be the sole determinative factor/circumstance to hold the transaction as benami in
nature. The plaintiff has miserably failed to establish and prove the intention of the father to
purchase the suit properties for and on behalf of the family, which were purchased in the
names of defendant Nos. 1 to 3.......Therefore, the intention of Late G. Venkata Rao to give
the financial assistance to purchase the properties in the names of defendant Nos. 1 to 3
cannot be said to be to purchase the properties for himself and/or his family members and,
therefore, as rightly observed by the High Court, the transactions of purchase of the suit
properties – Item Nos. I(a) to I(c) in the names of the defendant Nos. 1 to 3 cannot be said to
be benami in nature. The intention of Late G. Venkata Rao was to provide the financial
assistance for the welfare of his sons and not beyond that.”
Link to Order
https://sci.gov.in/supremecourt/2008/1432/1432_2008_Judgement_09-Apr-2019.pdf

3. G. Mahalingappa vs G.M. Savitha

Citation: Supreme Court, Civil Appeal no. 2867 of 2000 dated 09.08.2005

Facts
• The appellant was the father of the respondent. The appellant purchased a suit property in
the name of the respondent by a registered sale deed dated 24th of August, 1970 when the
respondent was a minor of seven years of age.
• After her marriage, the respondent asked for vacation of the suit property not only from the
appellant and his family but also from the tenants who were also defendants in the suit and
for payment of rent to her.
• The respondent filed a suit or declaration of title and recovery of possession in respect of
the suit property on the averment that since the suit property stood in her name and the
same was purchased for the benefit of the respondent and as a security for her marriage

© The Institute of Chartered Accountants of India


6.4 Economic Laws

she was entitled to a decree for declaration and possession after the appellant and the
tenants refused to move and to pay rents to her.
Issue
1. Does the plaintiff prove that she is the owner of the suit property?
2. Is she entitled to possession of the suit property as contended by her?
3. Is she entitled for damages as claimed by her?
4. To what relief the plaintiff was entitled, if any?
Key Ratio Decidendi
The Supreme Court noted the essential features of a Benami transaction:
♦ The real owner should have purchased the property in the name of the ostensible owner;
♦ The property should have been purchased by the benamidar for his own benefit - in the
case in hand the fact that the real owner, the father of the benamidar daughter, bought the
property for his own benefit supported the inference that the father was the real owner;
♦ The fact that the father and mortgaged the property to raise a loan also supported this
inference;
♦ That he had let out the property also shows that he was in control of the same.
Hence, the court concluded that the circumstances surrounding the transaction,
relationship of the parties and the circumstances surrounding the transaction, and
subsequent conduct of the appellant tend to show that the transaction was Benami in
nature.
Link to Order
https://sci.gov.in/jonew/judis/27105.pdf

4. Meenakshi Mills, Madurai v. CIT

Citation: Supreme Court, Civil Appeal No. 124 of 1954 dated 26.09.1956

Facts
The assessee’s Managing Agents are the firm of Messrs K. R. Thyagaraja Chettiar and Co.,
whose partners are Mr. Thyagaraja Chettiar and his two sons. It carries on business in the
manufacture and sale of yarn, and for the purpose of that business it purchases cotton and
occasionally sells it. Its profits arise for the most part from the sale of yarn and to some extent
from the re-sale of cotton. According to the account books of the company, its profits from
business for the account year 1941-42 were Rs. 9,25,364, for 1942-43 Rs. 24,09,832 and for

© The Institute of Chartered Accountants of India


Prohibition of Benami Property Transactions Act, 1988 6.5

1943-44 Rs. 29,13,88 1. In its returns, the appellant showed these amounts as its income
chargeable to tax for the respective years. The Department contended that the Company had
earned more profits than were disclosed in its accounts, and that it had contrived to suppress
them by resort to certain devices. The contention of the Department was that the amounts
shown as profits made by the intermediaries [Meenakshi and Co., Sivagami and Co.,
Mangayarkarasi and Co., and Alagu and Co.] represented in fact the profits actually earned by
the appellant, and that they should be added to the figures shown in its accounts as its profits.
Issue
Whether on the facts and in the circumstances of the case there is any legal evidence to
support the finding that the four firms, Meenakshi and Co., Sivagami and Co., Mangayarkarasi
and Co., and Alagu and Co., were benamidars for the appellant and that the profits made by
these firms were profits made by the appellant?
Key Ratio Decidendi
In this connection, it is necessary to note that the word 'benami' is used to denote two classes
of transactions which differ from each other in their legal character and incidents. In one
sense, it signifies a transaction which is real, as for example, when A sells properties to B but
the sale deed mentions X as the purchaser. Here the sale itself is genuine, but the real
purchaser is B, X being his benamidar. This is the class of transactions which is usually
termed as benami. But the word 'benami' is also occasionally used, perhaps not quite
accurately, to refer to a sham transaction, as for example, when A purports to sell his property
to B without intending that his title should cease or pass to B. The fundamental difference
between these two classes of transactions is that whereas in the former there is an operative
transfer resulting in the vesting of title in the transferee, in the latter there is none such, the
transferor continuing to retain the title notwithstanding the execution of the transfer deed. It is
only in the former class of cases that it would be necessary, when a dispute arises as to
whether the person named in the deed is the real transferee or B, to enquire into the question
as to who paid the consideration for the transfer, X or B. But in the latter class of cases, when
the question is whether the transfer is genuine or sham, the point for decision would be, not
who paid the consideration but whether any consideration was paid.
Link to Order
https://sci.gov.in/jonew/judis/722.pdf

© The Institute of Chartered Accountants of India


6.6 Economic Laws

5. Sh. Amar N. Gugnani Vs. Naresh Kumar Gugnani (Through


Legal Heirs)

Citation: Delhi High Court, CS(OS) No. 478/2004 dated 30.08.2005

Facts
♦ The plaintiff and Defendant are brothers. The Plaintiff was living with his father, mother
and younger brother as well as sisters in a rented house.
♦ That in February/March 1969, the Plaintiff visited India and gave substantial funds to
his father to keep it by way of deposit in India for the benefit of the Plaintiff. Again in the
year September 1970, the Plaintiff handed over substantial funds to his father to keep
in deposit in trust for and on behalf of the Plaintiff and for his benefit.
♦ That after marriage of the Plaintiff, the Plaintiff’s father suggested that as the Plaintiff’s
substantial funds are in deposit with him and he is doing well for himself in USA, he
should purchase a plot of land to build a house thereon in New Delhi.
♦ That on 4th May, 1973 the Plaintiff came to India and handed over further funds to his
father for acquiring the plot that had already been identified on perpetual lease. The
said deposit was made so that including the funds deposited from time to time, the
Plaintiff’s father had sufficient funds for the acquisition, registration of lease deed and
incidental expenses.
♦ That in view of the said, understanding the Plaintiff’s father in his capacity as trustee
obtained perpetual lease of the aforesaid plot benami in his name to endure to the
exclusive benefits of the Plaintiff. All the funds in purchase of the plot were availed by
the Plaintiff’s father from the money deposited with father and given to him from time to
time. The possession of the plot was obtained by the Plaintiff’s father for and on behalf
of Plaintiff in his capacity as a trustee and, a perpetual lease deed was executed by the
DDA, which was registered.
♦ The Plaintiff entrusted the title deed of the land in question to his father for safe custody
in his capacity, as a benami and the real ownership always vested in the Plaintiff.
Issue
Whether the claim in the suit is barred by the provisions of Benami Transactions Prohibition Act,
1988?
Key Ratio Decidendi
♦ "I would at this stage refer to a judgment delivered by this Court in the case of J M
Kohli Vs. Madan Mohan Sahni & Anr in RFA No.207/2012 decided on 07.05.2012. In
this judgment this Court has had an occasion to consider the intendment of the passing

© The Institute of Chartered Accountants of India


Prohibition of Benami Property Transactions Act, 1988 6.7

of the Benami Act as reflected from Section 7 of the Benami Act. Section 7 of the
Benami Act repealed the provisions of Sections 81, 82 and 94 of the Indian Trusts Act,
1882 (in short 'the Trusts Act') and which provisions of the Trusts Act gave statutory
recognition and protection to the benami transactions by calling such transactions
protected by a relationship of a trust. It bears note that benami transactions were very
much legal within this country before the passing of the Benami Act and the relationship
of a benamidar to the owner was in the nature of a trust/fiduciary relationship because
it was the Trusts Act which contained the provisions of Sections 81, 82 and 94 giving
statutory recognition to the benami ownership of the properties being in the nature of
trust."
♦ The expression “fiduciary relationship” and a relationship of a trustee cannot be so
interpreted so as to in fact negate the Benami Act itself because all benami
transactions actually are in the nature of trust and create a fiduciary relationship and if
the expression “trustee” or “fiduciary relationship” is interpreted liberally to even include
within its fold a typical benami transaction, then it would amount to holding that there is
no Benami Act at all
Link to Order
http://lobis.nic.in/ddir/dhc/VJM/judgement/10-08-2015/VJM30072015S4782004.pdf

6. Pawan Kumar Gupta Vs. Rochiram Nagdeo

Citation: Supreme Court, Civil Appeal No. 2369 of 1999 dated 20.04.1999

Facts
♦ Respondent was the tenant of the suit building (consisting of a shop room and godown
premises) which belonged to one Narain Prasad. As per a sale deed, Narain Prasad
transferred his rights in the suit building to the appellant.
♦ On its footing appellant filed Civil Suit for eviction of the respondent under Section 12(1)(a)
of the M.P. Accommodation Act, 1961 (for short "the Act") on the ground that respondent
has not paid rent to the appellant.
♦ That suit was contested by the respondent raising the contention that the building was
actually purchased by Pyarelal (father of the appellant) as per Ext.P11-sale deed and
appellant is only a name-lender therein, and hence appellant is not entitled to get the
eviction order or the rent of the building.
♦ In that suit the court found that appellant is the real owner of the building pursuant to sale-
deed and that he was entitled to receive rent of the building. However, the suit was
dismissed as the respondent deposited the arrears of rent in court during pendency of the

© The Institute of Chartered Accountants of India


6.8 Economic Laws

suit but appellant was permitted to withdraw the arrears of rent so deposited by the
respondent as per the judgment rendered in that suit.
Issue
1. Whether the plaintiff is owner of the suit premises?
2. Whether the defendant is tenant of plaintiff of disputed premises @ Rs.210/- p.m.?"
Key Ratio Decidendi
Section 2(a) of the Benami Act defines benami transaction as "any transaction in which
property is transferred to one person for a consideration paid or provided by another
person." The word "provided" in the said clause cannot be construed in relation to the
source or sources from which the real transferee made up funds for buying the sale
consideration. The words "paid or provided" are disjunctively employed in the clause
and each has to be tagged with the word "consideration". The correct interpretation
would be to read it as "consideration paid or consideration provided". If
consideration was paid to the transferor then the word provided has no application as
for the said sale. Only if the consideration was not paid in regard to a sale transaction the
question of providing the consideration would arise. In some cases of sale transaction ready
payment of consideration might not have been effected and then provision would be made for
such consideration. The word "provided" in Section 2(a) of Benami Act cannot be
understood in a different sense. Any other interpretation is likely to harm the interest of
persons involved in genuine transactions, e.g., a purchaser of land might have availed himself
of loan facilities from banks to make up purchase money.
We are, therefore, not inclined to accept the narrow construction of the word "provided" in
Section 2(a) of the Benami Act. So even if appellant had availed himself of the help rendered
by his father Pyarelal for making up the sale consideration that would not make the sale deed
a benami transaction so as to push it into the forbidden area envisaged in Section 3(1) of the
Benami Act.
Link to Order
https://sci.gov.in/jonew/judis/16871.pdf

7. Bhim Singh v. Kan Singh

Citation: Supreme Court, Civil Appeal 626 of 1971 dated 01.01.1970

Facts
♦ Plaintiff no. 1 and plaintiff no. 2 were father and son while defendant was the brother of
plaintiff no. 1.

© The Institute of Chartered Accountants of India


Prohibition of Benami Property Transactions Act, 1988 6.9

♦ The plaintiffs in their suit against the defendant claimed that the suit house in which the
defendant was living, belonged to them by virtue of a patta issued in their names.
♦ They alleged that the deceased brother of plaintiff no. 1, who remained a bachelor till
his death, loved plaintiff no. 2 as his son and had thought of adopting plaintiff no. 2 but
since he died all of a sudden it could not be done.
♦ The defendant on the other hand claimed that he and his deceased brother lived as
members of a joint family after the partition of their family that as a result of the joint
efforts of himself and his deceased brother the Maharaja, of Bikaner sanctioned sale of
the house to them, that the purchase money was paid out of their joint income but that
the patta was granted in the names of the plaintiffs due to political reasons and
therefore the plaintiffs were at the most benamidars.
Issue
The principle issue which arises for consideration relates to the ownership of the suit house.
Key Ratio Decidendi
♦ Two kinds of benami transactions are generally recognised in India. Where a person
buys a property with his own money but in the name of another person without any
intention to benefit such other person, the transaction is called benami. In that case the
transferee holds the property for the benefit of the person who has contributed the
purchase money, and he is the real owner.
♦ The second case which is loosely termed as benami transaction is a case where a
person who is the owner of the property executes a conveyance in favour of another
without the intention of transferring the title to the property thereunder. In this case, the
transferor continues to be the real owner.
♦ The difference between the two kinds of benami transactions referred to above lies in
the fact that whereas in the former case there is an operative transfer from the
transferor to the transferee though the transferee holds the property for the benefit of
the person who has contributed the purchase money, in the latter case there is no
operative transfer at all and the title rests with the transferor notwithstanding the
execution of the conveyance.
♦ One common feature, however, in both these cases is that the real title is divorced from
the ostensible title and they are vested in different persons.
♦ The question whether a transaction is a benami transaction or not mainly depends upon
the intention of the person who has contributed the purchase money in the former case
and upon the intention of the person who has executed the conveyance in the latter
case.

© The Institute of Chartered Accountants of India


6.10 Economic Laws

♦ The principle underlying the former case is also statutorily recognised in section 82 of
the Indian Trusts Act, 1882, which provides that where property is transferred to one
person for a consideration paid or provided by another person and it appears that such
other person did not intend to pay or provide such consideration for the benefit of the
transferee, the transferee must hold the property for the benefit of the person paying or
providing the consideration
Link to Order
https://sci.gov.in/jonew/judis/4636.pdf

8. Valliammal (D) By Lrs vs Subramaniam & Ors

Citation: Supreme Court, Civil Appeal 5142 of 1998 dated 31.08.2004

Facts
• The land measuring 10.37 \0261/2 acres (suit land) belonged to Malaya Gounder,
plaintiff and his younger brother, Marappa Gounder.
• Marappa Gounder stood guarantee for his Uncle Chinnamalai Gounder in a loan
transaction advance by one Samasundaram Chettiar who was a money-lender for a
sum of Rs. 200/-.
• Marappa Gounder died in the year 1923 and was succeeded to by his brother Malaya
Gounder, as the legal representative of Marappa Gounder.
• Suit was decreed against the debtor as well as the guarantor. They were made jointly
liable. Suit land was sold on 1.8.1927 in the auction to satisfy the decree passed in OS
No. 338 of 1925.
• Land was purchased by one Chockalingam Chettiar. Chockalingam Chettiar could not
get physical possession of the land, however, he was given the symbolical possession.
• The suit land was purchased by Ramayee Ammal wife of Malaya Gounder, original
Plaintiff, for a consideration of Rs. 500/- on 5.12.1933. Ramayee Ammal executed a
registered will in favour of her daughters the defendants/respondents herein. Ramayee
Ammal died on 2.1.1979.
Issue
Whether the courts below have wrongly cast the onus of proving the benami nature of the sale
on the defendants and further more whether they have failed to apply the various tests laid
down by the Supreme Court for determination of the question whether the sale in favour of
Ramayee was a benami transaction?

© The Institute of Chartered Accountants of India


Prohibition of Benami Property Transactions Act, 1988 6.11

Key Ratio Decidendi


♦ There is a presumption in law that the person who purchases the property is the owner
ofthe same. This presumption can be displaced by successfully pleading andproving that
the document was taken benami in name of another person for some reason, and the
person whose name appears in the document is not the real owner, but only a benami.
Heavy burden lies on the person who pleads that recorded owner is a benamiholder.
♦ It is well established that burden of proving that a particular sale is benami lies on
the person who alleges the transaction to be a benami. The essence of a benami
transaction is the intention of the party or parties concerned and often, such intention is
shrouded in a thick veil which cannot be easily pierced through. But such difficulties do not
relieve the person asserting the transaction to be benami of any part of the serious onus
that rests on him, nor justify the acceptance of mere conjectures or surmises, as a
substitute for proof.
♦ After saying so, this Court spelt out following six circumstances which can be taken as a
guide to determine the nature of the transaction:
1. the source from which the purchase money came;
2. the nature and possession of the property, after the purchase;
3. motive, if any, for giving the transaction a benami colour;
4. the position of the parties and the relationship, if any, between the claimant and the
alleged benamidar;
5. the custody of the title deeds after the sale; and
6. the conduct of the parties concerned in dealing with the property after the sale.
Link to Order
https://sci.gov.in/jonew/judis/26384.pdf

9. Niharika Jain W/o Shri Andesh Jain Vs Union of India

Citation: Rajasthan High Court, Civil Writ Petition No. 2915/2019 (SB), dated
12.07.2019

Facts
♦ There was a search action under Section 132 of the Income Tax Act, 1961 on the petitioner
before the amendment in Benami Act i.e. before 01/11/2016. During the course of search
action, various incriminating documents were seized indicating several benami transactions.

© The Institute of Chartered Accountants of India


6.12 Economic Laws

♦ Accordingly, show cause notice under Section 24(1) of the Amended Benami Act was
issued on the petitioner calling to show cause as to why the transactions found during the
search action should not be treated as “Benami Transaction” under Section 2(9) of the
Amended Benami Act and should not be liable for punishment under Section 3 of the
Amended Act. The Initiating officer ignoring the submission of the petitioner treated the
transactions as benami liable for punishment under Section 3 of the Amended Benami Act.
The said matter travelled to the High Court.
♦ Before the High Court, the petitioner took the view that sub-section (3) of section 3 of
Amended Benami Act talks of punishment in respect of transaction entered into after
amendment. Since the transactions were entered into before the amendment, the said
transactions fall outside the ambit of said sub-section.
Issue
Whether amendments in section 3 of Prohibition of Benami Transaction are “retrospective” or
“prospective” in nature?
Key Ratio Decidendi
The Rajasthan High Court agreeing with the contention of the petitioner held that sub-section
(3) to section (3) of the Amended Benami Act talks about punishments in respect of benami
transactions entered into after amendment in Benami Act and is thus, prospective. While
holding the said sub-section (3) as prospective, the Rajasthan High Court observed that :
1. Unless a contrary intention is reflected, a legislation is presumed and intended to be
prospective;
2. Where an amendment affects rights or imposes obligations or castes a new duties or
attached a new disability is to be treated as prospective ; and
3. Benami Amendment Act, 2016 neither appears to be Clarificatory nor curative.
Accordingly, the Rajasthan High Court threw the entire transactions entered into by the
petitioner before amendment out of the purview of Benami Act.
Link to Order
https://services.ecourts.gov.in/ecourtindiaHC/cases/display_pdf.php?filename=%2FE3WiyNU
WFIaR1oBGE62Whacq%2FGpQYzdIxN%2BwvtBO5EKm0nNnCnELUP2ElZ2VwbO&caseno=
CW/2915/2019&cCode=1&appFlag=

© The Institute of Chartered Accountants of India


ELECTIVE PAPER 6D: ECONOMIC LAWS

CASE STUDY 1

Mr. Bhanu Pratap Taneja is a leading real estate developer based in Delhi. In the
last decade, his company Garvit Bhoomi Developers Pvt. Limited having registered
office in Bhikaji Cama Place, New Delhi, had successfully developed four housing
projects – two in Gurgaon and one each in Jaipur and Lucknow. They had a robust
management team. Having been the name behind developing more than five
thousand luxurious apartments with modern amenities, they had the reputation of
delivering the projects well within the promised time.
In the beginning of the year 2015, they launched another project in Indirapuram, UP
by the name Omega Capetown Residency in which 1000 residential units consisting
of 2BHK and 3BHK apartments were to be developed. They were to be completed in
all respects by January 2018 and delivered to the consumers by that date. This
project was being carried on smoothly when the Real Estate (Regulation and
Development) Act, 2016 came to be enacted w.e.f. 1 st May, 2016. Section 3, which
was enacted later w.e.f. 1 st May 2017. Since Omega Capetown Residency consisted
of 1,000 residential units, it was required to be registered and so the company
submitted the requisite documents with concerned authorities.
As the application for registration was found to be complete in all respects, the
Omega Capetown Residency Project was granted registration by RERA (UP) within
the statutory period and was provided with a registration number including a log-in
ID for assessing the website of the Authority and to create webpage.
In the meantime, Mr. Taneja was approached by some of the influential developers
that an understanding had been reached among them to control the price of
apartments to be built by them. However, because of legal tangles such
understanding could not be brought into writing and it was also not intended to be
enforced by legal proceedings. Mr. Taneja did not agree to the proposal because
even though the understanding was not in writing and it was not intended to be
enforceable by legal proceedings, it was still illegal as per the Competition Act, 2002.
This revelation made by Mr. Taneja discouraged the intending developers and they
desisted from being a party to this proposal.
Mr. Taneja’s son Garvit, who was a commerce graduate and holder of law degree,
had a college friend Rohit whose father Mr. Dev Kumar dealt in sale, purchase and
renting of properties under the title ‘Dev Property Dealers’ from the Yusuf Sarai
market. Since Rohit had joined his father’s business, it was thought prudent to
convert the existing proprietary business into a registered partnership firm titled as
‘Dev & Sons Property Dealers’. Because of the enactment of Real Estate Act, Rohit
consulted his friend Garvit regarding its implications in case of real estate agents.
Accordingly, the firm was got registered as real estate agent with the help of Garvit’s
legal advisor.
Further, Garvit made a proposal to Rohit and his father that they could associate
themselves with his Omega Capetown Residency, a registered RERA project in
Indirapuram for facilitating sale of apartments which they readily accepted. Garvit
also cautioned them that as per the Act, since their firm was now a registered real
estate agent they were not supposed to facilitate sale/purchase of any plot,
© The Institute of Chartered Accountants of India
apartment or building in a real estate project being sold by the promoter in any
planning area, if such project was not registered with RERA of the concerned State.
In addition, Garvit’s legal advisor told them that as required by Section 10, a
registered real estate agent would maintain and preserve proper books of accounts
and other necessary documents. Further, such agent would not involve himself in
any unfair trade practice like making a false statement regarding services to be
provided by him. He would also not permit the publication of any advertisement
whether in any newspaper or otherwise of services that were not intended to be
offered. Besides, the agent would also have to help the intending buyers in getting
the required information and documents to which they were entitled, at the time of
booking of any property.
Rohit had a friend Tarun whose father Dr. Sreenivas Sharma was a surgeon in a
government hospital and was residing in a rented government flat in the hospital
campus itself. He had an intense desire to have a luxurious flat of his own. Tarun
had joined IBM after doing MBA from IIFT, New Delhi. So, with the combined salary
of both, they decided to buy a flat. Tarun contacted Rohit to help him in searching a
suitable apartment for his family. In turn, Rohit informed him that one particular 3BHK
flat at an ideal location was available in Omega Capetown Residency in Indirapuram
as the original allottee had withdrawn from the scheme; otherwise the booking under
this project was already full. Dr. Sharma got interested in the information and went
to the Omega Capetown Residency along with his family to see the concerned
apartment. He liked its strategic location and gathered more information regarding
sanctioned plan, layout plan along with the other specifications, etc. He then asked
for stage-wise time schedule of completion of the project and also enquired regarding
provision of water, sanitation and other amenities. Since, Rohit personally knew
Garvit and his father Mr. Taneja - the promoters of the project - Dr. Sharma and his
family had a lively and fruitful meeting with them. Subsequently, he and his son jointly
entered into an agreement for sale with the promoters of the project and made
payment of 75% of the cost of the apartment, while remaining 25% of the cost was
to be paid at the time when the apartments were ready for occupation.
A few months after booking the apartment, Dr. Sharma got a notice from the
promoters of Omega Capetown Residency that due to unforeseen circumstances
they were not in a position to complete the project and needed the allottees’ consent
for transferring of their majority rights and liabilities to another reputed developer
M/s. Sai Developers Pvt. Limited of New Delhi. In case any of the allottees was not
agreeable to this proposal he could get his money refunded. Since Dr. Sharma was
very much attached to the location of the flat, he accepted the proposal after
enquiring with Rohit and his father. He also learnt that 95% of the allottees had
already given their written permission. Further, the Authority had given its written
approval to the proposal for transfer and completion of Project by M/s. Sai
Developers Pvt. Limited. Dr. Sharma was also assured by Mr. Bhanu Pratap Taneja,
the erstwhile promoter with whom he had earlier interacted satisfactorily, that all the
pending obligations would be fulfilled by the new developer and in no case the date
of completion of the project would be extended; otherwise it would attract penalty. It
was also disclosed by Mr. Taneja that the new promoter would rectify any structural
defect if it occurred within a period of five years from the date of handing over the
possession of the apartments. Dr. Sharma, thus felt relieved.
© The Institute of Chartered Accountants of India
M/s. Sai Developers completed the project on time and received Completion
Certificate from the Competent Authority. As per the agreement for sale, Dr. Sharma
made payment of the remaining 25% of the cost. Thereafter, he received Occupancy
Certificate and took physical possession of the apartment well before two months
since the allottees were supposed to take physical possession within statutory period
of two months from the issue of Occupancy Certificate. He was also given other
necessary documents and plans, including that of common areas. He also became
a member of the RWA formed by the allottees. In the meantime, the promoter
executed a registered conveyance deed in favour of each of the allottees along with
the undivided proportionate title in the common areas to the RWA.
I. Multiple Choice Questions
1. Registration of a real estate project shall not be required –
(a) where the area of land proposed to be developed does not exceed
five hundred square meters or the number of apartments proposed to
be developed does not exceed eight.
(b) where the area of land proposed to be developed does not exceed
five thousand square meters or the number of apartments proposed
to be developed does not exceed eighty.
(c) where the area of land proposed to be developed does not exceed
two hundred fifty square meters or the number of apartments
proposed to be developed does not exceed four.
(d) where the area of land proposed to be developed does not exceed
three hundred square meters or the number of apartments proposed
to be developed does not exceed three.
2. Who is required to submit a copy of duly obtained approvals and
commencement certificate for getting the project registered with RERA:
(a) Allottee
(b) Promoter
(c) Real Estate Agent
(d) None of the above
3. A registered real estate agent shall -
(a) Facilitate the sale/purchase of any plot, apartment or building, being
sold by the promoter in any planning area, which is registered with
the Authority;
(b) maintain and preserve prescribed books of account, records and
documents;
(c) not involve himself in any unfair trade practices
(d) All of the above.

© The Institute of Chartered Accountants of India


4. The promoter is required to rectify any structural defect if it occurs within a
period of ----- years from the date of handing over the possession of the
apartments to allottees –
(a) Two years
(b) Three years
(c) Four years
(d) Five years
5. Registration of on-going project for which completion certificate is yet to be
received is mandatory -
(a) Yes, if the area of land (developed or to be developed) exceeds five
hundred square meters or the number of apartments (developed or
to be developed) exceeds eight.
(b) No, irrespective of the area of land or the number of apartments
(c) Can’t say
(d) None of the above
6. A real estate developer can leave the project mid-way by selling that project
to another developer if he has taken a written approval of ---------- of allottees
along with approval of the Authority.
(a) 2/3rd
(b) 1/3rd
(c) 3/4th
(d) 1/4 th
7. The time limit within which an allottee is required to take physical possession
of the apartment after issue of occupancy certificate is -
(a) one month
(b) two months
(c) three months
(d) four months
8. A certificate certifying that the real estate project has been developed
according to the sanctioned plan, layout plan and specifications as approved
by the competent authority under the local laws is called -
(a) Commencement Certificate
(b) Completion Certificate
(c) Occupancy Certificate
(d) None of the above

© The Institute of Chartered Accountants of India


9. The flat purchased by Dr. Sharma jointly with his wife Mrs. Neelima Sharma
though funded by him would be held as ‘benami transaction’ under the
Prohibition of Benami Property Transactions Act, 1988 -
(a) Yes
(b) No
(c) Can’t say
(d) None of the above
10 As per the Competition Act, 2002 ‘Agreement’ includes any arrangement or
understanding or action in concert:
(a) Whether or not, such arrangement, understanding or action is formal
or in writing; or
(b) Whether or not such arrangement, understanding or action is
intended to be enforceable by legal proceedings.
(c) Whether or not, such arrangement, understanding or action is formal
or in writing; or whether or not such arrangement, understanding or
action is intended to be enforceable by legal proceedings.
(d) None of the above.
II. Descriptive Questions
1. (i) Examine the following given aspects with reference to the allottee in
the situation given in the case study :
(a) Rights exercised by Dr. Sharma as an allottee.
(b) Duties fulfilled by Dr. Sharma as an allottee.
(c) Right which was not exercised by him and duty which was not
required to be fulfilled by Dr. Sharma.
(ii) The promoters of Omega Capetown Residency transferred majority
of rights and liabilities to Sai DeveIopers Pvt Ltd. for the completion
of the project. Advise as to the validity of such transfer of a real estate
project to a Sai Developer’s Pvt Ltd in the case study?
2. In the given case study Omega Capetown Residency has got itself registered
under the Real Estate Regulatory Authority, as it consisted of 1,000
residential units. However, if Omega Capetown Residency consisted of only
250 residential units, then was it necessary to get itself registered under the
Real Estate (Regulation and Development) Act, 2016: if yes, name the
various important documents and declarations which are required to be
submitted by a ‘real estate developer’ while registering a project with the
Real Estate Regulatory Authority (RERA) having only 250 residential units
and not 1,000 residential units.
3. Mr. Bhanu Pratap Taneja was approached by some of the influential
developers to join their association so as to reach an understanding whereby
they could inflate the price of the apartments built by them. Even though the
deal was in favour of Mr. Bhanu Pratap Taneja, he rejected the proposal from
other developers. In the light of the provisions of the Competition Act, 2002,
discuss whether the decision of Mr. Bhanu Pratap Taneja is lawful?
© The Institute of Chartered Accountants of India
ELECTIVE PAPER 6D – ECONOMIC LAWS
SUGGESTED SOLUTION – CASE STUDY 1

I. ANSWERS TO OBJECTIVE TYPE QUESTIONS

1. (a) [Hints: Refer Section 3 of the Real Estate (Regulation and Development) Act, 2016]

2. (b) [Hint: Refer Section 4 (2) (c) of the Real Estate (Regulation and Development) Act,
2016]

3. (d) [Hint: Refer Section 10 of the Real Estate (Regulation and Development) Act, 2016]

4. (d) [Hint: Section 14 (3) of the Real Estate (Regulation and Development) Act, 2016]

5. (a); [Hint: Refer Section 3 of the Real Estate (Regulation and Development) Act, 2016.
It may not be mandatory in a particular State if the State has granted exemption to
such on-going project]

6. (a); [Hint: Section 15 of the Real Estate (Regulation and Development) Act, 2016. It is to
be noted that if a consumer or his family holds more than one unit in the project then
he will be considered as one consumer only]

7. (b); [Hint: Refer Section 19 (10) of the Real Estate (Regulation and Development) Act,
2016]

8. (b); [Hint: Refer Section 2 (q) of the Real Estate (Regulation and Development) Act,
2016]

9. (b); [Hint: Refer Section 2 (9) (iii) of the Prohibition of Benami Property Transactions Act,
1988]

10. (c). [Hint: According to Section 2 (b) of the Competition Act, 2002, ‘Agreement’ includes
any arrangement or understanding or action in concert:

(i) Whether or not, such arrangement, understanding or action is formal or in


writing; or

(ii) Whether or not such arrangement, understanding or action is intended to be


enforceable by legal proceedings.]

© The Institute of Chartered Accountants of India


1
II. ANSWERS TO DESCRIPTIVE QUESTIONS

1. (i) (a) With reference to Section 19 of the Real Estate (Regulation and
Development) Act, 2016, Dr. Sharma, as an allottee, exercised the following
rights:

(I) Obtained the information relating to sanctioned plans, layout plans


along with the specifications as approved by the competent
authority.

(II) Demanded to know stage-wise time schedule of completion of the


project, including the provisions for water, sanitation, electricity and
other amenities.

(III) Claimed physical possession of the said apartment.

(IV) Obtained the necessary documents and plans, including that of


common areas, after getting the physical possession of the
apartment from the promoter.

(b) With reference to Section 19 of the Real Estate (Regulation and


Development) Act, 2016, Dr. Sharma, as an allottee, fulfilled the following
duties:

(i) Made necessary payments within the time as specified in the


agreement for sale.

(ii) Became a member of the RWA formed by the allottees.

(iii) Took physical possession of the apartment within a period of two


months from the issue of Occupancy Certificate.

(iv) Participated towards registration of the conveyance deed of the


apartment.

(c) (i) With reference to Section 19 of the Real Estate (Regulation and
Development) Act, 2016, Dr. Sharma, as an allottee, did not exercise
the following right:

The right to claim the refund of amount paid along with prescribed
rate of interest. It was so because the promoter was able to give

© The Institute of Chartered Accountants of India


2
possession of the apartment in accordance with the terms of
agreement for sale.

(ii) With reference to Section 19 of the Real Estate (Regulation and


Development) Act, 2016, Dr. Sharma, as an allottee, was not
required to fulfill the following duty:

The duty to pay interest at prescribed rate for delay in making any
payment. It was so because he had made the payments in
accordance with the terms of agreement for sale.

(ii) As per section 15 of the Real Estate (Regulation and Development) Act, 2016, a
promoter is permitted to transfer his majority rights and liabilities in respect of a real
estate project to a third party.

The provisions given below are to be adhered to by the promoter for transfer:

(a) Obtain prior written consent from two-third of allottees. Such consent will not
include the consent given by the promoter.

(b) Also obtain prior written approval of the Authority.

Note: It is to be ensured that such transfer shall not affect the allotment or sale of
the apartments, plots or buildings, as the case may be, in the real estate project
developed by the promoter.

(i) After obtaining the required consent of both allottees and the Authority, the
new promoter shall be required to independently comply with all the pending
obligations under the provisions of the Act or the rules and regulations made
thereunder.

(ii) The new promoter is also required to comply with the pending obligations as
per the agreement for sale entered into by the erstwhile promoter with the
allottees.

(iii) Further, the new promoter must note that any transfer so permitted shall not
result in extension of time to him to complete the real estate project.

Note: In case of default, he shall be liable to the consequences for delay, as per the
provisions of the Act or the rules and regulations made thereunder.

© The Institute of Chartered Accountants of India


3
Since in the given case study , 95% of the allottees had already given their written
permission. Further, the Authority had given its written approval to the proposal for
transfer and completion of Project by M/s. Sai Developers Pvt. Limited in compliance
with the requirements given in the said provisions. Such transfer of a real estate
project to a Sai Developer’s Pvt Ltd. is valid.

2. According to proviso to section 3 of the Real Estate (Regulation and Development) Act, 2016,
projects that are on going on the date of commencement of the Act, and for which the
completion certificate has not been issued, the promoter of the project are required to make
and application to the concerned Authority for the registration of the said project within a
period of 3 months from the date of commencement of the Act.

Further, the section provides that no registration of real estate project shall be required where
the area of land proposed to be developed does not exceed 500 square meters or the number
of the apartments proposed to be developed does not exceed 8 inclusive of all phases.

Hence, the Act requires registration of on-going projects where completion certificate was
yet to be obtained as well as new projects, if the area to be developed exceeded 500 sq.
mtrs. or apartments to be built under the project exceeded eight. Thus, registration of Omega
Capetown Residency was must with the Real Estate Regulatory Authority of UP (RERA, UP),
as consisted of 1,000 residential units.

Further, even if Omega Capetown Residency consisted of only 250 residential units (i.e.
more than 8 units), it will be compulsory to get itself registered under the Act. The process
of registering a project with the Real Estate Regulatory Authority (RERA) which consists of
1,000 units or 250 units is same which is given under section 4 of the Act.

With reference to Section 4, various important documents and declaration required to be


submitted while registering a project with RERA are as under:

• Details of the project such as name, address, type, names and photographs of the
promoters, etc.

• Details of the project which were already launched by the real estate developer in
the preceding 5 years and their present status.

• Approvals and commencement certificates obtained from the competent authority for
each phase of the project separately.

© The Institute of Chartered Accountants of India


4
• Sanctioned layout plan, the development plan for the project and details of basic
facilities being made available like drinking water, electricity etc.

• Proforma of allotment letter, agreement for sale and conveyance deed to be signed
with the consumers.

• Location of the project with clear demarcation of the land for the project.

• Number, type and carpet areas of units to be sold.

• The details of open areas if any like terraces, balconies etc.

• Details of associated engineers, contractors, architects and intermediaries in the


project.

• a declaration, duly supported by an affidavit, stating the following important matters:

o that the promoter has a legal title to the land and it is free from all encumbrances
along with legally valid documents;

o the time period required for completion of the project;

o that seventy per cent. of the amount realised from the allottees, from time to
time, shall be deposited in a separate escrow account and shall be used only
for the purpose of completion of project;

o that the promoter shall get his accounts audited within six months after the end
of every financial year by a chartered accountant in practice; and shall take all
the pending approvals on time from the competent authorities; etc.

3. According to section 2(b) of the Competition Act, 2002, ‘Agreement’ includes any
arrangement or understanding or action in concert:

(i) Whether or not, such arrangement, understanding or action is formal or in writing, or

(ii) Whether or not such arrangement, understanding or action is intended to be


enforceable by legal proceedings.

Further, section 2(c) of the Competition Act, 2002, "Cartel" includes an association of
producers, sellers, distributors, traders or service providers who, by agreement amongst
themselves, limit, control or attempt to control the production, distribution, sale or price of,
or, trade in goods or provision of services.

© The Institute of Chartered Accountants of India


5
An association for the welfare of the trade or formed for any other purpose not mentioned in
the aforesaid definition will not be a cartel Thus, it is only when an association, by agreement
amongst themselves, limits control or attempts to control the production, distribution, sale or
price of, or, trade in goods or provision of services, that it will be a cartel.

Hence, an agreement which prohibits an enterprise or person or their association for entering
into an agreement in respect of production, supply, distribution, storage, acquisition or
control of goods or services, which causes or is likely to cause an appreciable adverse affect
on competition. Such agreements entered in contravention of the above are void. These
agreements are presumed to have an appreciable adverse affect on competition.

Here, in the given situation, the agreement between Mr. Bhanu Pratap Taneja and other
builders would have fallen into the ambit of section 2(b) and 2(c) of the Competition Act,
2002 as the aim of the association was to increase the price of the apartments. Thus, such
an association would be void.

© The Institute of Chartered Accountants of India


6
ELECTIVE PAPER 6D: ECONOMIC LAWS

CASE STUDY - 2

Mr. Manohar Mehta, renowned builder of Mumbai, owns a reputed building construction company known
as “Sri Ram Building Construction and Real Estate”. Due to his vast business empire, he is known as the
“King of the Property World”.

On the personal front, Mr. Mehta in his family has an elderly mother, wife, and three children. His father,
Mr. Sri Ram Mehta, had recently expired after prolonged sickness. Mr. Manohar Mehta’s mother, Mrs.
Rama Devi, is a religious lady always dedicating her time in worship and holy works. Wife, Urvashi, is a
home maker and a socialite. She is mostly involved in all the social activities and runs her own NGO
named “AAWAZ”. Mr. Mehta regularly give financial contribution to his wife’s NGO. These minor donations
gave Mr. Mehta a Noble man tag in the social circle and better business prospects.

Sonia, eldest daughter of Mr. Mehta, is married and well settled with her husband in Australia. She has
recently started her own import export business in Australia with the help of her father. Mr. Mehta would
transfer the amount to his daughter and she regularize the amount in the books of accounts of her
business.

Recently, a project was started by Mr. Mehta in Marol area of Mumbai. The project was named as “Shubh
Appartment”. Under this project a 5 storey building comprising of 2 flats (2 BHK) at each floor were
constructed. The actual construction cost of each flat was Rs. 50 lakh. The flat was sold at Rs. 60 Lakh.
The advance booking charges of Rs. 6 lakh for each flat was collected from the buyers by cheque. Proper
receipt was issued to all the buyers for the advance payment. Out of the 10 flats, 4 flats were sold at an
increased price of Rs. 62 Lakh. Rs. 2 lakh each was taken in cash from the 4 buyers. Therefore, he earned
in total Rs. 8 Lakh for these 4 flats. This amount of Rs. 8 lakh was send to Sonia via an independent agent.
Sonia utilized this amount in her business and taken into record via some entry in her books of accounts.

Mr. Manohar Mehta has two sons, Rohit and Sorav. Sorav is the youngest son. He is pursuing his
graduation from one of the best universities of Chicago. For his education, Mr. Mehta remitted foreign
exchange of USD 2, 00,000 through authorized person. During course of his studies, Sorav was caught
with the seasonal influenza, so there he required an emergency medical treatment. Mr. Mehta transmitted
additional amount of USD 70,000 for treatment through authorized person who was well known to him for
hassle free transfer.

Rohit, the elder son, after successful completion of his M.B.A. Finance degree, is now actively supporting
his father in his real estate business. To give a start to his career, Mr. Mehta handed over the project

1
© The Institute of Chartered Accountants of India
“Royal Aashiana” to be constructed in Kharghar. The said project was proposed to be developed in 1000
sq. mts. Rohit was working on the project under guidance of Mr. Mehta. He marketed about the said
project and invited persons to purchase the flats in the Royal Ashiana. It was an ongoing project, Rohit
without registration of the project made an agreement to sell some of the flats.

As per Mr. Mehta’s regular morning routine, he one day read his favorite column “Property for Sale” in the
newspaper. He came across one advertisement regarding the sale of the residential plot in Panvel district
of Maharashtra. He discussed about the advertisement with his manager, Mr. Shyam Pareekh. He asked
his manager to visit the actual site of the mentioned property.

Mr. Pareekh called the land owner, Mr. R. Thakker, and took the appointment for the visit. He went to
Panvel to meet the owner and see the property. It was a 10,000 square feet plot near the city area. Mr.
Thakker quoted a price of Rs. 1crore for selling his property. After two rounds of meeting the final
negotiation with the land owner was done and deal was locked for Rs. 90 Lakh. On mutual consensus
between them, down payment of Rs. 20 lakh was made to Mr. Thakker in cash. Further, a payment of Rs.
70 Lakh was done by cheque and the property was registered in the name of his (Mr. Mehta) mother.
Being a sacred woman, she was not interested in all such types of transactions or arrangements made on
her name by Mr. Mehta.

After few months, Mr. Manohar Mehta from his sources came to know that an agricultural land is on sale by
a farmer, Mr. Bhima Singh. The farmer’s 5 acres of agricultural land was located in Thane district of
Mumbai. Mr. Manohar Mehta thought it would be a great deal to buy the agricultural land around the lush
green vicinity of the Thane district. He further thought that he can resale this property after converting it to
farm houses to the potential buyers.

After the detail discussion with his management regarding the purchase of land, Mr. Manohar Mehta went
to Thane to see the agricultural land. The land was just 500 meters away from the highway. After visiting
the land Mr. Mehta became keen to buy the property. They had a talk with the farmer, Mr. Bhima Singh.
The farmer being illiterate hardly knew about the legal sale/purchase of the land. Mr. Manohar Mehta and
Mr. Pareekh negotiated and finalized the deal in Rs. 80 Lakh.

Since Mr. Manohar Mehta required funds for purchasing the Thane property, he decided to sell his Panvel
plot which was in the name of his mother. He retransferred the Panvel property (Which was in the name of
his mother) to himself and then sold the Panvel plot for Rs. 1.10 Crore. He took partial amount by cheque
and rest by cash. This way he safeguarded himself from showing the capital gain on financial record. Mr.
Mehta received Rs. 80 Lakh in cheque and rest Rs. 30 Lakh in cash.

2
© The Institute of Chartered Accountants of India
Whereas, Mr. Mehta induced Bhima Singh, and paid him Rs. 50 Lakh through cheque and Rs. 30 lakh
through cash.

Mr. Manohar Mehta was still having Rs. 30 Lakh out of sale of 1.10 Crore panvel property, at his disposal.
Mr. Mehta decided to deposit Rs. 2,000 each to his wife, two sons and mother, saving accounts every
month. He would continue deposition of Rs. 2000 each per month for next couple of years.

During one of the corporate parties while having a discussion, Mr. Mehta’s friend advised him to invest the
remaining amount in the shell company outside India. Mr. Mehta liked the suggestion and decided to send
Rs. 10 Lakh to invest in the shell company in Singapore via Hawala. He learned about Mr. Varun Das who
runs a business of hawala under the veil of running a financial company. Mr. Mehta contacted Varun Das
who agreed to transfer the fund via Hawala on 1% commission basis. In this way Mr. Mehta managed to
circulate the amount in the shell company outside India.

Mr. Manohar Mehta also donated Rs. 50,000 in cash to his wife’s NGO ÄAWAZ”.

After few months, Mr. Mehta decided to buy a new car, worth Rs. 50 Lakh. He did the down payment of Rs.
5 Lakh via cheque. For the remaining Rs. 45 lakh he took 3 years auto loan, so that he can deposit the
monthly installment in the bank. Hence in this way the remaining Rs. 10 Lakh, which he gained from the
sale of the Panvel property, was utilized..

Due to frequent transactions of hefty amount and his conduct of other financial activities in a year, Mr.
Mehta bank accounts and his family members account of transactions were in the scrutiny of the Income
Tax Department.

On further investigation it was discovered that Mr. Mehta, Mr. Thakkar, Rohit and Sonia being guilty for
different offences punishable under the different Acts.

I. Multiple Choice Questions

1. Sonia plans to make investment in India. She was permitted to do so as per the FEM
regulation in -

a. Trading in transferable development rights

b. Real Estate business and construction of farm houses

c. Agricultural or plantation activities

d. Chit funds subscribed through banking channel and on non-patriation basis

3
© The Institute of Chartered Accountants of India
2. Sonia ordered exports of goods from India for her business. The amount (export value) of
good shall be released and repatriated to India within period -

a. 3 Months from date of export

b. 6 Months from date of export

c. 9 Months from date of export

d. 9 Months from date of invoice covering such export

3. Amount released for the real estate project from allottees in separate account can be
withdrawn by promoter after it is certified by-

a. Cost accountant and an Architect

b. Engineer, and a Chartered Accountant

c. an Architect and an Engineer

d. Engineer, an Architect, and a Chartered Accountant in practice

4. "Who according to the Provision of Prevention of Money Laundering Act is/ are held to be
liable in dealing of Panvel property-

a. Mr. Manohar Mehta


b. Mr. Thakkar
c. Both (a) and (b)
d. Mr. Shyam Pareekh
5. Who among the following is liable for an offence of money laundering as per the Part C of
the Schedule given in the Prevention of money laundering Act-

a. Mr. Mehta
b. Mr. Thakkar
c. Mr. Atul
d. Ms. Sonia
6. How much amount of penalty Mr. Mehta has to pay on illegal remittance of money
transferred to Sorav

a. USD 6,00,000
b. USD 1,80,000
c. USD 60,000

4
© The Institute of Chartered Accountants of India
d. USD 50,000
7. Suppose any project started by Mr. Mehta was not completed within preferred time due to
force majeure. Remedy available to Mr. Mehta in this situation

a. Registration may be interim cancelled by Authority


b. Registration need to be freshly applied
c. Registration granted may be extended for period not exceeding 1 year on
application
d. Registration may be extended for reasonable period on application.
8. Mr. Mehta sells a flat of Royal Aashiana to Mr. X for the amount Rs. 75 lakh. Mr. X made
the advance payment. The correct value of the advance payment is

a. Rs. 7.5 Lakhs


b. Rs. 8 Lakhs
c. Rs. 8.5 Lakhs
d. Rs. 9 Lakhs
9. In case of dispute between Mrs. Rama Devi and Mr. Mehta, can Mr. Mehta legally claim
her right over the Panvel Plot?

a. Yes, because he is the beneficial owner in the transaction


b. No, because the transaction is Null and void
c. Yes, because he paid consideration for the transaction
d. None of the above
10. Mr. Mehta files an application for initiation of voluntary liquidation proceeding of his Real
Estate construction company. Mr. X, a home buyer of a flat in one of the project of Mr.
Mehta claimed for the re-fund of paid amount or demanded for handover of possession of
flat. Which amongst following is not incorrect statement

a. X cannot claim the amount due to pending of Insolvency process


b. X can file a suit for the default committed by Mr. Mehta under the Insolvency &
Bankruptcy code
c. X’s right & interest is protected after execution of an agreement to sale till the
conveyance of the flat
d. None of the above

5
© The Institute of Chartered Accountants of India
II. Descriptive Question

1. What would be the consequences in the following given situations:

(a) Where if Mr. Mehta remitted Foreign Exchange USD 2,00,000 and USD 70,000 as
educational and medical expenses to Sorav .

(b) Sorav used USD 20,000 out of the remitted medical expenses (i.e., USD 70,000)
and used remaining amount to purchase immovable property jointly with Mr.
Mehta in Chicago.

2. Suppose Royal ashiana, is a 15 year old building of Mr . X. It was purchased by Mr .


Mehta in January 2016. He planned to re-develop the said building into residential
apartments. He launched the project in end of January 2016. During the course, the
Government enacted the Real estate(Regulation and Development) Act, 2016.Mr. Mehta
seek advise of his legal consultant on the issues related to the registration of the said
Project-

(1) Is the Registration for the re-development of society is mandatory?

(2) If he plans to develop a new society under new name with new allotments . Is the
registration mandatory of the project?

(3) State the position where project have been completed and obtained the certificate
of completion before the commencement of RERA?

(4) Where if the project is to be devolved into phases ?

(5) Where if the authority has not taken any decision on application for the registration
within the prescribed period.

3. (i) What remedy is available to Mr. Mehta, in case he want to compound for the
commission of illegal remittance to Sorav under FEMA Act 1999?

(ii) According to the case study the property bought by Mr. Mehta in the name of his
mother, Mrs. Rama Devi, is a Benami Transaction and will be confiscated.
Suppose the property rightfully belonged to Mrs. Rama Devi then what measures
she can take to save the property from confiscation. Explain?

6
© The Institute of Chartered Accountants of India
ELECTIVE PAPER 6D – ECONOMIC LAWS
SUGGESTED SOLUTION – CASE STUDY 2

I. ANSWERS TO OBJECTIVE TYPE QUESTIONS

1. (d) [Hint: As per regulation 4 (b) explanation (ii) of the FEMA (permissible capital
account transaction) Regulation 2000.]

2. (c) [As per regulation of FEMA (Export of goods and services) regulation 2016]

3. (d) [Hint: Section 4 (2), proviso to (D) of clause (l) of the RERA, 2016]

4. (c) [Hint: Refer section 3]

5. (d) [Hint: Refer Part C of the Schedule to the Prevention of Money Laundering Act, 2002]

6. (c) [Hint: Refer Section 13 of Foreign Exchange Management Act 1999 along with
Schedule III of the FEM (Permissible Current Account Transactions) Regulations,
2000]

7. (c) [Hint: Refer Section 6 of RERA, 2016]

8. (a) [Hint: As per Section 13 of RERA, 2016]

9. (b) [Hint: Section 6 Prohibition of Benaim Property Transactions Act, 1988 ]

10. (c) [Hint: Section 11 (4) read with section 89 of the RERA, 2016]

II. ANSWERS TO DESCRIPTIVE QUESTIONS

1 (a) As per section 13 (1) of the FEMA, 1999, If any person contravenes any provision of
this Act, or contravenes any rule, regulation, notification, direction or order issued in
exercise of the powers under this Act, or contravenes any condition subject to which
an authorisation is issued by the Reserve Bank, he shall, upon adjudication, be liable
to a penalty up to thrice the sum involved in such contravention where such amount
is quantifiable, or up to two lakh rupees where the amount is not quantifiable.

Any Adjudicating Authority adjudging any contravention to above provisions, may, if


he thinks fit in addition to any penalty which he may impose for such contravention
direct that any currency, security or any other money or property in respect of which
the contravention has taken place shall be confiscated to the Central Government

© The Institute of Chartered Accountants of India


1
and further direct that the foreign exchange holdings, if any of the persons
committing the contraventions or any part thereof, shall be brought back into India
or shall be retained outside India in accordance with the directions made in this
behalf.

According to the above provisions, Mr. Mehta will be penalized thrice of the extra
amount (USD, 20,000) remitted above the prescribed limit (USD 2, 50,000). Hence
liable to pay a penalty of USD 60,000 to the Government.

(b) The second issue is related to sections 13(1A), 13(1C) & 37A of the FEMA Act, 1999
read with Regulation 5 of the FEM(Acquisition & transfer of immovable property
outside India)Regulation , 2015.

As per section 13(1A), if any person is found to have acquired any foreign exchange,
foreign security or immovable property, situated outside India, of the aggregate value
exceeding the threshold prescribed under the proviso to sub-section (1) of section
37A, he shall be liable to a penalty up to three times the sum involved in such
contravention and confiscation of the value equivalent, situated in India, of the
foreign exchange, foreign security or immovable property.

13(1C) of FEMA says that if any person is found to have acquired any foreign
exchange, foreign security or immovable property, situated outside India, of the
aggregate value exceeding the threshold prescribed under the proviso to sub-section
(1) of section 37A, he shall be, in addition to the penalty imposed under sub-section
(1A), punishable with imprisonment for a term which may extend to five years and
with fine.

According to Section 37A of the FEMA, upon receipt of any information or otherwise,
if the Authorised Officer prescribed by the Central Government has reason to believe
that any foreign exchange, foreign security, or any immovable property, situated
outside India, is suspected to have been held in contravention of section 4, he may
after recording the reasons in writing, by an order, seize value equivalent, situated
within India, of such foreign exchange, foreign security or immovable property:

Provided that no such seizure shall be made in case where the aggregate value of
such foreign exchange, foreign security or any immovable property, situated outside
India, is less than the value as may be prescribed.

© The Institute of Chartered Accountants of India


2
As per regulation 5 of the FEM (Acquisition & transfer of immovable property outside
India) Regulation, 2015, a person resident in India may acquire immovable property
outside India jointly with a relative who is a person outside India. Provided there is
no outflow of funds from India.

Since in the given case, Mr. Mehta remitted Foreign exchange to Sorav in excess to
the limit prescribed under the FEMA. Sorav partially used USD 20,000 for medical
treatment and rest USD 50,000 to purchase property outside India jointly with Mr.
Mehta. So Both Mr. Mehta and his son Sorav will be liable under sections 13(1),
13(1A), 13(1C) of the FEMA, 1999.

2. (1) According to section 3(2) of the Real Estate (Regulating ) Authority Act, 2016, no
registration of the real estate project shall be required for the purpose of renovation
or repair or re-development which does not involve marketing, advertising selling or
new allotment of any apartment, plot or building, as the case may be, under the real
estate project. So Registration for the re-development of society (Royal Ashiana)
was not required.

(2) According to the above provision no registration is required when any project is
renovated or repair or re-development and it does not involve marketing, advertising
selling or new allotment of any apartment, plot or building, as the case may be, under
the real estate project. However, in the given situation in the question, Mr. Mehta
plans to develop it as a new society under new name with new allotments. So
registration of the said project was necessaited as the Act.

(3) As per the proviso to section 3(1) of the RERA, projects that are ongoing on the date
of commencement of this Act and for which the completion certificate has not been
issued, the promoter shall make an application to the Authority for registration of the
said project within a period of three months from the date of commencement of this
Act. In the given case, where the project have been completed and obtained the
certificate of completion before the commencement of RERA, such project shall not
require registration.

(4) As per the explanation to section 3 of the RERA, where the real estate project is to
be developed in phases, every such phase shall be considered a stand alone real
estate project, and the promoter shall obtain registration under this Act for each

© The Institute of Chartered Accountants of India


3
phase separately. Therefore if the said project is to be developed in phases , it needs
separate registration for each phase.

(5) As per Section 5 of the RERA, the Authority has to decide on the application within
30 days of its receipt. It further provides that in case the Authority fails to take a
decision within the said period of 30 days the project shall be deemed to be
registered.

3. (i) According to provision of Section 15 (1)

(1) Any contravention under section 13 may, on an application made by the


person committing such contravention, be compounded within one hundred
and eighty days from the date of receipt of application by the Director of
Enforcement or such other officers of the Directorate of Enforcement and
officers of the Reserve Bank as may be authorised in this behalf by the
Central Government in such manner as may be prescribed."

(2) Where a contravention has been compounded under sub-section (l), no


proceeding or further proceeding, as the case may be, shall be initiated or
continued, as the case may be, against the person committing such
contravention under that section, in respect of the contravention so
compounded.

As per the above mention provision Mr. Mehta will submit the application to the
concerned authority for compounding of the offences committed in contravention to
the FEMA Act.

(ii) The Section 5 of PMLA authorizes the Director or any other officer not below the
rank of Deputy Director to attach the property. Section 8 of PMLA lays down an
elaborate procedure for adjudication of complaint under Section 5 of PMLA. It calls
for a show cause notice to be issued to the offender/ person from whom property
has been seized, so as to give the person an opportunity to make a case against
attachment. Such a person in order to avoid confiscation, can demonstrate the
legitimate source of his income/earning or assets, out of which or by means of which
he has acquired the property attached. The evidence on which he realized and other
relevant information and particulars, and to basically convince the authority about
the property which should not be declared to be the property involved in money
laundering.

© The Institute of Chartered Accountants of India


4
If the authority reached the conclusion that the offense has not taken place it shall
order release of such property to the person entitled to receive it.

Hence as per the above provision, Mrs. Rama Devi aggrieved by the provisional
attachment may file her objection before the adjudicating authority.

© The Institute of Chartered Accountants of India


5
FINAL COURSE
ELECTIVE PAPER 6D: ECONOMIC LAWS

CASE STUDY - 3

Mr. Krishnakant Mathur was a lecturer in agricultural college of Pushkar. After 25 years of his service he retired
from his job last month. To utilise his time, Mr. Mathur keeps the accounts of his society and has also started a
coaching classes for the needy and poor students. Mr. Mathur has three children, two daughters Jaya & Lata and
son Neeraj. His wife Asha Devi is a house wife. Neeraj is eldest of all the three children. Jaya and Lata are still
studying in Class 12th and 10th respectively. At the time of retirement of Mr. Mathur, Neeraj has completed his
engineering degree.
During his work tenure, Mr. Mathur purchased one property (area 2000 Sq feet), at Mansarover colony, Jaipur in
the name of his son Neeraj Mathur. The property already has a constructed house over it. Mr. Mathur also owns
an ancestral property in his village Titari. Since he was posted at Jaipur, so he resided at Mansarover colony house
with his family.
Neeraj got a good job in Maharashtra State Electricity Board (MSEB) as a Junior Engineer. He was posted at Pune.
He was very desperate to earn more money and become rich. For clearing the contractors billing and giving
necessary approvals he started giving favours to the contractors and vendors. Various projects were in the hands
of MSEB, they authorised Neeraj to finalise the tenders related to supply of required articles, goods and services
for attainment of the government projects. Many contractors pleased him by cash or kind for acceptance of their
tenders for the projects. Neeraj used his position in the said department by manipulating the bidding process.
Soon, Neeraj got promotion. He decided to celebrate his grand success and planned for a trip to Hongkong with
his friend. There in Hongkong, he came to know about XBL Company, which was a joint ventures abroad of Indian
Companies. He thought to make an equity investment in XBL Company. He talked with the Indian Service Providers
for the investment of funds to Securities Exchange of Hongkong for buying equity in XBL Company. Neeraj applied
for drawl of foreign exchange, but due to legal compliances, authorised dealer denied for the said transaction.
Neeraj, through known authorise dealer, on payment of commission exported the foreign exchange for the equity
investment in XBL Company. So in this way, Neeraj made an overseas investments.
In the meantime, Neeraj, while residing in Pune, met a girl in his office named Shalini. They both fell in love with
each other. After couple of months they both decided to marry without informing any of the family members as they
both are of different caste. As a witness Rahul, Neeraj’s friend, was the only one who knew about his marriage.
Rahul is located in Dubai and works as a Senior Manager with Al-Aadil Works LLC.
Rahul insisted him to come to Dubai. So Neeraj and Shalini planned their visit to Dubai. Shalini discussed about
her trip with her friend. Her friend told that Dubai is a beautiful place and is a hub for Gold and electronics. Since
Shalini had a keen interest in buying gold and Neeraj had a restriction in carrying foreign currency, he came up
with an idea. He gave Rahul’s family Rs. 1 Lakh in cash, in India and took the equivalent foreign currency from
Rahul in Dubai. They both had a good time in Dubai. They visited the Gold Souq in Deira. Shalini purchased gold
jewellery worth Rs. 1.5 Lakh. Rahul also took them around to see some latest Electronic items. Neeraj found that
the price of the Smart Television was less and the shopkeeper proposed further discount without a bill. So Neeraj
also bought a new Television worth Rs. 60000. After their return, they did not declare it to the Indian customs and
passed through the Green channel. Custom officials on the matter of doubt withheld them at the airport and
interrogated. After compliance with the required formalities under the legal prospects, the matter was sorted.
In 2016, during the period of demonetisation, Neeraj was holding a cash of Rs. 10 Lakh. He deposited Rs. 3 lakh
is his and his wife’s account within the permissible limit imposed by the government. To settle the rest of the amount
he took the help of his friend Jai Bakshi. Jai is a renowned hotelier based in Mumbai. As per the discussion Jai
told that he could manage Rs. 2 Lakh by depositing it in his account. Further Jai told that he will charge 10 percent

1
© The Institute of Chartered Accountants of India
of Rs. 2 Lakh to accommodate the amount in his record books. Also, he will return this amount only after six months
due to scrutiny of income tax department. Neeraj found that the market rate of changing the demonetised 500 and
1000 rupees notes were 50% of the given amount. So he thought it is better to agree to the terms and conditions
of Jai and gave him Rs. 2 Lakh. With the remaining Rs. 5 Lakh, he booked a flat in Shubh Laxmi apartment near
Badlapur Mumbai in the name of his wife Shalini. Neeraj on record showed above that paid amount of Rs. 5 lakh
as a loan taken from Shalini’s uncle.
Whereas, Shubh Laxmi Apartment, constructed by J.K. Builders was registered with the Maharashtra Real Estate
Regulatory Authority. The flat was sold by Himmat Chand to Neeraj. In the meantime J.K. Builders filed an
application for bankruptcy and shed of their responsibilities as to the completion and handover of the possession
of the flats to the buyers. Neeraj approached to the office of J.K. Builders and they denied from their responsibilities
by saying that the said flat was sold by Himmat Chand, so he owns the responsibility. Whereas Himmat chand
took the plea that ultimate responsibility lies with the J.K. Builders, being a promotor. Neeraj went to the consumer
forum for the relief.
In the meantime, due to the tip off received from unknown sources and on grounds of noticing the suspicious
activities of Neeraj, Income Tax Department issued a scrutiny notice to him. According to the Notice, Neeraj was
asked to clarify the mode of payment for the loan amount from his uncle as there was no entry of cash credited to
Shalini’s account. Neeraj and Shalini, however, managed and came safely out of a situation on benefit of doubt.
After few months, Neeraj went to Jaipur with his wife Shalini to meet his family. Mr. Mathur was extremely angry
and shattered to see his son married without his consent. Mr. Mathur and his son had a heated argument and he
turned both of them out of his house. Neeraj told that this house is in his name and legally belongs to him. Hence
his father has no right to throw them away.
Mr. Mathur files a case for claiming ownership over the property as he does not want to give the possession of his
house to Neeraj. As per plaint allegation, Mr. Mathur has purchased the property in 18/2/2015 in the name of his
son Neeraj Mathur. As per plaint allegation, he has purchased the property out of his own income. He has claimed
relief that he be declared as owner of the property and Neeraj should be permanently restrained from interfering
in possession of the property. Plaint clearly reveals that Mr. Mathur has purchased the property out of his own
income in the name of his son Neeraj. It was his own property and he had claimed declaration of ownership right
over the property. He has also prayed for permanent injunction against his son. Entire plaint allegation does not
whisper that it was joint Hindu family property or purchased by Mr. Mathur for joint Hindu family.
On the other hand, Neeraj claimed that he has purchased the property from his own income, his father was not
having any right over the property.
During pendency of the suit, Mr. Mathur died. According to the procedure of the Tribunal in case of death of one
of several plaintiffs or of sole plaintiff, the right to sue survives. The Tribunal, on an application made in that behalf
shall cause the legal representative of the deceased plaintiff to be made a party and shall proceed with the suit.
So, his wife and two daughters were impleaded as legal representatives of the deceased. His legal representatives
have amended the plaint and have claimed declaration that the property is Joint Hindu family property. By detail
amendment, made in the plaint, his legal representatives pleaded that Mr. Mathur was “Karta” of Joint Hindu
Family.
The Tribunal held that in the present case, as per the claim of Mr. Mathur he purchased the property in the name
of defendant i.e. the defendant was Benamidar. So, Mr. Mathur was not entitled to claim any right over such
property in the light of Section 4 of the Prohibition of Benami Property Transactions Act, 1988.
Further, the Tribunal held that Late Mr. Mathur or legal representatives of the plaintiff have neither pleaded nor
proved that the defendant was holding the property as a trustee or in a fiduciary capacity for the benefit of other
persons for whom he was a trustee or was standing in a fiduciary capacity, inter alia, as per original pleading and
amended pleading, the property was owned by Mr. Mathur as his self-acquired property or property was owned by
Joint Hindu Family. Nothing has been pleaded by original plaintiff or his legal representatives that Neeraj was a

2
© The Institute of Chartered Accountants of India
trustee or standing in a fiduciary capacity for the present legal representatives, original plaintiff and for others. Inter
alia, it has been specifically pleaded that the owner was the plaintiff and he has purchased the property in the
name of the present defendant.
Objective Type Questions:
1. In the given case study, Neeraj and Shalini bought gold jewellery worth Rs. 1.5 Lakh from Dubai. They have
custom clearance through green channel. State whether the given act will constitute an offence under the
Prevention of Money Laundering Act, 2002?
a. Yes, because the gold bought is beyond the permissible limit
b. Yes, because of an evasion of duty chargeable thereon goods on an & above the permissible limit.
c. No, because they are carrying the original bill of the purchased gold
d. No, because gold bought is within the permissible limit
2. Will Shalini’s uncle be liable for punishment under the Money Laundering Act, if he lends the loan amount
from his known sources of income?
a. Yes, because he is knowingly associating in the crime
b. No, because he is not a party to the crime
c. Yes, because he is actually involved in the process
d. Not sure
3. During the period of demonetisation Neeraj deposited Rs. 2 Lakh in Jai’s account. who is the beneficial person
in the light of the Prevention of Money-Laundering Act, 2002?
a. Neeraj
b. Jai
c. Both a and b
d. Neither a nor b
4. Suppose Mr. Mathur acquired a property from undisclosed & unaccounted sources of funds. Later, he created
a trust of his entire property for his family benefit and appointed Neeraj as his trustee. Will Neeraj be held
liable for such transaction made by Mr. Mathur-
a. Yes
b. No
c. Partially Liable
d. Not Sure
5. Neeraj in the course of his duty took commission in clearing the bill. This act can be termed as?
a. Money laundering through Prevention of Corruption Act, 1988
b. Money laundering through unlawful activities (Prevention) Act, 1967
c. Money laundering through customs
d. Money laundering through Indian Penal Code, 1860
6. Can Neeraj resale his house located in Jaipur to his father?
a. No, it is prohibited under Benami Transaction

3
© The Institute of Chartered Accountants of India
b. Yes, Because Neeraj has the right to sale the property
c. Yes, because Neeraj is an owner
d. Both b and c
7. State which statement as to the drawl of the foreign exchange by Neeraj for transaction related to equity
investment in XBL Company is correct –
a. Neeraj can do such transaction through authorised dealer by providing commission on export for equity
investment in XBL Company.
b. Neeraj can do such transaction by directly buying equity in XBL Company
c. Neeraj cannot transact for equity investment in XBL Company
d. Drawl of foreign exchange by Neeraj for payment of commission on exports towards equity investment
in XBL Company, is prohibited.
8. Neeraj can claim relief for the completion and handover of the possession of the flat purchased in Shubh
Lakshmi Apartment against-
a. J.K. Builders
b. Himmat Chand
c. Both (a) & (b)
d. Maharashtra Real Estate Authority
9. When any transaction cannot said to be benami transaction-
a. If a person deposits old currency in account of another person in an understanding that account
holder will return the money in new currency
b. Individual held property in joint name with his grand children with the consideration paid by an
individual.
c. Company raising share capital through fictitious shareholders
d. Person takes a loan and not able to prove the genuinity of the lender
10. Suppose there are 3 bidders X, Y, & Z in a tender process initiated for MSEB. X & Y bidders were removed
by neeraj unethically showing them incompetent to make bid in the tender process for supply of goods to
MSEB for its project. This relates to-
a. Regulation of combinations
b. Abuse of dominant position
c. Controlling affairs or management
d. All of the above
Descriptive Questions:
1. a. Neeraj without consulting his Chief Engineer Authority, affixes his sign and seal to a document
certifying Mr. X (Fictitious Client) as a registered dealer of electronic goods. He there by obtains
his share for certifying the said document.
Mr. X misappropriates this forged document for his business to obtain various projects of
government for supply of electric goods.
Examine the given situation as to the nature of offence committed under the Prevention of Money
Laundering Act, 2002.

4
© The Institute of Chartered Accountants of India
b. Neeraj While returning to India from Dubai trip gave wrong declaration about his gold and electronic
purchase at the airport. State the nature of liability of Neeraj for the commission of the above act?
2. a. Neeraj was given an offer by a company vendor to disclose him the lowest bid quoted by other
vendors. Neeraj accessed the computer of his Executive Director and passed on the lowest
quotation to the vendor and thus helped him in quoting the lowest among all the bids . Examine and
analyse the situation and conclude how Neeraj will be held liable under PMLA?
b. If suppose Neeraj with one of his friend registers a company and quotes bid to get a tender of
MSEB to get an extra income. Neeraj being at the back foot helps his friend unofficially to get
maximum tenders allotted to their company. They need to raise a share capital of the company. So
Neeraj decided to invest from his unknown sources of his income in the name of fictitious
shareholders.
Determine in the given scenario, the liability of Neeraj being covered under which Act?
3. (i) The Competition Commission of India (CCI) has received a complaint from a State Government
alleging that Shubh Limited and Mangal Limited have entered into an informal agreement, not
enforceable at law, to limit or control production, supply and market, to determine the sale price of
their products. Such an action of these companies has an appreciable effect on competition.
Examining the provisions of the Competition Act, 2002:
(A) Decide whether the above agreement has appreciable effect on competition.
(B) What factors shall the Competition Commission of India consider while taking the above
decision?
(C) What orders can the Competition Commission of India pass on completion of the inquiry?
(ii) Explain the restrictions, if any, under Foreign Exchange Management Act, 1999 in respect of the
following issue and transfer of shares:
Issue of Equity Shares of ` 1 crore at face value accounting for 45 percent of post-issue capital to
non-resident Indians in U.S.A. on non-repatriation basis. The shares are issued by M/s ABC
Knitwear Limited to finance the modernization of its plant.

5
© The Institute of Chartered Accountants of India
ELECTIVE PAPER 6D- ECONOMIC LAWS

SOLUTION – CASE STUDY 3

I. ANSWERS TO OBJECTIVE TYPE QUESTIONS

1. (b)

[Hint: As per Schedule I of PMLA, 2002 it is a predicate offence related to custom Act]

2. (b)

[Hint: Section 3 of the Prevention of Money-Laundering Act, 2002 states of commission of an


offence of money-laundering.—Whosoever directly or indirectly attempts to indulge or knowingly
assists or knowingly is a party or is actually involved in any process or activity connected with
the proceeds of crime and projecting it as untainted property shall be guilty of offence of money-
laundering. Since Shalini’s uncle has given the loan from the known sources of his income so
he is not a part of this crime.]

3. (b)

[ Hint: 2(fa) of PMLA, 2002 ]

4. (b)

[ Hint: As per exception of Section 2 (9) (A) Neeraj is not liable for Benami Transaction as he
stand in a fiduciary capacity for the benefit of other person.]

5. (a)

[Hint: Refer the schedule of PMLA, 2002]

6. (a)

[Hint: Refer Section 6 of the Prohibition of Benami Property Transactions Act, 1988]

7. (d)

[Hint: Refer Schedule I of FEM (Current Account Transaction ) Rules, 2000]

8. (c)

[Hint: Refer section 2(v) of the RERA]

9. (b)

[Hint: Refer section 2(9) of the Prohibition of Benami Property Transactions Act, 1988]

© The Institute of Chartered Accountants of India


10. (b)

[Hint: Refer section 4 of the Competition Act, 2002]

II. ANSWERS TO DESCRIPTIVE QUESTIONS

1. a. According to provision of the PMLA, 2002 the money earned by Neeraj is not from the
legitimate sources. Since Neeraj forged the sign and seal of his Chief Engineer so the
money earned by him is proceed of crime. According to Schedule Part A of Para 1 of
PMLA, Neeraj has committed an offence under section 472 and 473 of Indian Penal
Code. These Section deals with the offence of making or possessing counterfeit seal,
etc., with intent to commit forgery. Whoever makes or counterfeits any seal, plate or
other instrument for making an impression, intending that the same shall be used for
the purpose of committing any forgery which would be punishable under section 467 of
this Code and under any other section under this Code, or, with such intent, has in his
possession any such seal, plate or other instrument, knowing the same to be counterfeit,
shall be punished with imprisonment for life, or with imprisonment of either description
for a term which may extend to seven years, and shall also be liable to fine.

b. Neeraj is liable under the provision of PMLA Act. His act is covered under the provision of
Part B of the Schedule. Part B of the Schedule refers to offence under the Customs Act,
1962. Section 132 of the Customs Act states that whosoever makes sign, or use or cause
to be made, sign or use any declaration, statement or document in relation to customs
knowingly or having reasons to believe that such declaration statement etc. is false shall be
punishable for a term which may extend to two years or fine or both.

2. a. Neeraj has contravened the Prevention of Money Laundering Act under Part A Para 22
of the Information Technology Act 2000. According to the provision of Section 72 of
Information of Technology Act 2000, if any person who, in pursuance of any of the
powers conferred under this Act, rules or regulations made thereunder, has secured
access to any electronic record, book, register, correspondence, information, document
or other material without the consent of the person concerned discloses such electronic
record, book, register, correspondence, information, document or other material to any
other person shall be punished with imprisonment for a term which may extend to two
years, or with fine which may extend to one lakh rupees, or with both.

Neeraj in the given case, without the consent of his Executive Director accessed the
electronic records and passed on the official information to the vendor without permission.
This information can produce large profits and legitimize the ill-gotten gains through money

© The Institute of Chartered Accountants of India


laundering. Hence it is punishable under the Section 72 of the Information of Technology
Act, 2000.

b. The given issues falls within the ambit of the Prohibition of Benami Property Transactions
Act, 1988. According to Section 2 (26) "Property" means assets of any kind, whether
movable or immovable, tangible or intangible, corporeal or incorporeal and includes any
right or interest or legal documents or instruments evidencing title to or interest in the
property and where the property is capable of conversion into some other form, then the
property in the converted form and also includes the proceeds from the property.

According to Section 2 (10) "benamidar" means a person or a fictitious person, as the case
may be, in whose name the benami property is transferred or held and includes a person
who lends his name.

According to Section 2 (9) (B), a transaction or an arrangement in respect of a property


carried out or made in a fictitious name; or (D) a transaction or an arrangement in respect
of a property where the person providing the consideration is not traceable or is fictitious.
Hence, according to all the above provisions, Neeraj has done Benami transactions. He has
done the investment in the share as a Benamidar as he holds the share in the fictitious
name of shareholders. Hence this is a Benami transaction and is liable for punishment
under the Benami Transaction Act.

3. (i) (A) The term ‘agreement’ as defined in section 2 (b) of the Competition Act, 2002,
includes any arrangement or understanding or action in concert.

(i) whether or not such arrangement, understanding or action is formal or in


writing, or

(ii) whether or not such arrangement, understanding or action is intended to


be enforceable by legal proceedings.

Thus an agreement between Shubh Ltd. and Mangal Ltd. satisfies the above
ingredients of an agreement as per section 2 (e) of the Act, so agreement has
appreciable effect on competition.

(B) Factors to be considered:

(1) creation of barriers to new entrants in the market.

(2) driving existing competitors out of the market.

(3) foreclosure of competition by hindering entry into the market.

(4) accrual of benefits to consumers.

© The Institute of Chartered Accountants of India


(5) improvements in production or distribution of goods or provision of
services.

(C) Orders of CCI: If after enquiry by the Director General, the Commission finds the
agreement entered into by Shubh Ltd. and Mangal Ltd. are in contravention of
section 3, it may pass all or any of the following orders:

(1) direct Shubh Ltd. and Mangal Ltd. to discontinue and not to re-enter such
agreement.

(2) impose such penalty as it may deem fit which shall not be more than 10% of
the average of the turnover for the last 3 preceding financial years, upon each
of such person or enterprises which are parties to such agreement or abuse;

(3) direct that agreement shall stand modified to the extent and in the manner
as may be specified in the order by the commission

(4) direct Shubh Ltd. and Mangal Ltd to abide by such other orders as the
commission may pass and comply with the directions including payment
of cost, if any.

(5) pass such other orders or issue such directions as it may deem fit.

(ii) Issue of equity shares to NRI’s and transfer of shares by NRIs are capital account
transactions.

RBI may in consultation with the Central Government specify any class or classes of
transactions which are permissible [Section 6(2)(a)].

According to Regulation 3(1) of the Foreign Exchange Management (Permissible capital


Account Transactions) Regulations, 2000 issued by RBI, Investment in India by a person
resident outside India is a permissible capital account transactions (Schedule II).

Further RBI is empowered under Section 6(3)(b) to prohibit, restrict or regulate, by


regulations, transfer or issue of any security by a person resident outside India.

According to Regulation 5(3)(ii) of the said regulations a NRI may purchase shares of an
Indian Company which is not engaged in Print Media Sector on non-repatriation basis
without any limit (para 2 of Schedule 4). The shares may be issued by the company either
by public issue or private placement. The only condition is that the amount of consideration
for purchase of shares shall be paid by way of inward remittance through normal banking
channels from abroad or out of funds held in NRE/FCNR/NRO/NRSR/N&NR account
maintained with an authorized dealer or as the case may be with an authorised bank in India
(Para 3 of Schedule 4).

© The Institute of Chartered Accountants of India


FINAL COURSE
ELECTIVE PAPER 6D: ECONOMIC LAWS
CASE STUDY - 4
Albert John, right from his childhood had high dreams and aspirations. His hobbies included travelling, scuba
diving, gliding, trekking, and other adventurous sports. To go around the globe was his biggest wish.
Therefore, after his schooling, to satisfy his prime interest in travelling, he opted for a two years’ Diploma and
thereafter, a 18 months’ Post Graduate Diploma in Tourism and Travel Industry Management, from the
University of Mumbai.
After post-graduation he had a lot of options like to become a travel agent or join any immigration and customs
services, travel agencies, airline catering or laundry services, etc. Initially, however, he got a chance to join
a reputed travel agency where he gathered on-hand experience and continued with the job for about two
years but his innermost desire was to do his own business and therefore, he opened up a proprietary travel
agency under the name of ‘John Travel Agents’ in Mumbai. Within a period of about five years he could earn
substantially from his business. He used to arrange foreign currency also through his contacts for needy
tourists who did not want to use banking channels for this purpose.
In the meantime, he got married to Neelima George and was comfortably settled in his 2BHK flat in Mumbai.
Keeping in view the future expansion of his travel agency business, he decided to form a private limited
company by the name ‘John and George Travel Agency Pvt. Ltd. having its registered office in Mumb ai. After,
it was got registered with ROC, Mumbai with the authorised capital of Rs. 10 lacs, Albert wrapped up its
business conducted so far under ‘John Travel Agents’.
His business was flourishing. By now he had seen most of the touring destinations of E urope and East Asia
by taking his clients around. One of such clients was Chimanbhai Patel, a leading and famous exporter of
Mumbai. He was a rich and dynamic businessman dealing in gold, diamonds and precious as well as semi -
precious stones. He had three companies i.e. Shining Gold Jewellery Pvt. Ltd., which dealt in gold jewellery;
Red Star Pvt. Ltd. which dealt in diamonds and jewellery studded with diamonds; and Blue Sapphire Pvt. Ltd.
which dealt in precious and semi-precious stones. He was the owner of two palatial bungalows situated at
Bandra and Juhu.
Once he took Chimanbhai, his wife and both of their married daughters as well as sons -in-law to Europe for
a family vacation trip. He arranged for them Royal Caribbean’s cruise liner ‘Harmony of th e Seas’ which was
like a five star hotel at sea, for a ‘seven night’ cruise starting from Barcelona. While on board, Chimanbhai
proposed him a business deal which required him to deliver gold biscuits worth Rs. one crore to one of his
close friends on his next visit to Hong Kong and for accomplishing this job he would get some hefty
commission. After some hesitation he agreed to do the assigned work and the deal was done. Albert managed
to somehow pass on the tainted wealth as directed by Chimanbhai. In return he got rich kickbacks in the form
of commission; and also admired this new way of earning quick money. The bond between Albert and Patel
grew intense and he accomplished many such assignments including converting of Indian currency
representing black money into foreign currency and delivering it outside India to a safe haven.
This way slowly and gradually Albert entered into money laundering activities. Time passed on. He was a rich
person now. He purchased a new 3BHK flat in the same locality and rented out his old flat. He also acquired
properties in Uttrakhand and Rajasthan and at the same time invested additional funds in purchasing gold
jewellery and diamonds including buying a rust coloured Mahindra XUV500. Though the current line of activity
helped him in fulfilling his high dreams and aspirations but in actuality he was converting proceeds of crime
to make them appear as legitimate money. He was a changed person now keeping the moral ethics at bay.

© The Institute of Chartered Accountants of India


Once, while travelling in a Vayu Airways flight from Hong Kong to Mumbai he was impressed by the hospitality
provided by Neetu Bhatia, a member of the cabin crew. An idea clicked him. Albert knew very well that flight
attendants had access to secure venues at airports. At times, they did not require baggage screening. The
cabin crews underwent minimum security check and therefore, needle of suspicion and surveillance was also
minimal in their case. Thinking so, Albert befriended her and through his mesmerizing talks he could gather
that she was a resident of Vile Parle, Mumbai and a regular employee on this route. He developed a story
which reflected his persona as that of a business tycoon. Neetu was highly impressed by the sweet talks and
manners exhibited by Albert and they exchanged mobile numbers.
A few days later, Chimanbhai sent requisite Indian currency to Albert for conversion into around one lac US
Dollars and its deliverance to his business associate in Hong Kong. This time Albert thought of Neetu, called
her and they met at a high profile restaurant ‘Green Tea Day’ in Worli. Albert disclosed her about delivering
of USD one lac in exchange for handsome commission to one of his known and trusted business associates
who would get the money lifted from the Hong Kong International Airport itself. Initially, Neetu was a little bit
hesitant but the desire to earn some quick cash without putting in much effort prevailed over her good senses
and she relented. Both of them, however, knew that it was a criminal conspiracy.
Albert had drawn a very simple modus operandi for her. He wrapped the stacks of dollars in aluminium foils
and carbon sheets to dodge x-ray machine at the Mumbai airport. After placing them in her suitcase he put
her make-up kit and clothes over them. At the security check, as he had anticipated, the foils were passed
off as chocolates. After landing at Hong Kong Airport, Albert’s local conduit picked up the cash from there.
As promised, Neetu got the commission for deliverance. Not being caught in her first operation, Neetu’s
confidence level rose to a considerable extent. A few other consignments, delivered through her, were a no
glitch operation but in the seventh one Albert’s luck ran out and this operation was spoiled by Enforcement
of Directorate (ED) officials who caught her before the plane could take off from the Chhatrapati Shivaji
International Airport, Mumbai.
After being caught, Neetu got frightened and spilled the beans. She was taken under custody by Directorate
of Enforcement Officers and her thorough investigation revealed the involvement of Albert. Her offence of
carrying foreign currency on the behest of Albert was considered to be a Scheduled Offence falling in Part A
of the Schedule to the PML Act (i.e. criminal conspiracy involving Section 120B of the IPC). After following
due procedures including filing a complaint before the jurisdictional Magistrate for taking cognizance of the
scheduled offence, her residence was also searched and gold jewellery worth Rs. 21 lacs was recovered and
Dy. Director duly authorised by the Director took steps to provisionally attach the recovered jewellery in the
presence of two independent witnesses.
Simultaneously, following due procedures, a search team headed by Dy. Director raided the house of Albert.
By the time the officers of ED entered his house, Albert was almost ready to go on a trip to Dubai as a part
of routine job but with a special mission. The officers could smell a rat and took him to his rust coloured
Mahindra XUV500. Immediate search of his car gave way to the recovery of 24 kgs. of gold which was going
to be smuggled out of India through various conduits. It transpired from Albert tha t the gold belonged to
Chimanbhai Patel, a famous exporter of Mumbai.
From the search of Albert’s residence, various incriminating documents were also recovered. In one of the
almirahs, there was a hidden bottom drawer but the hawk eyes of ED officers were able to detect it. Albert
was asked to open it but he did not oblige giving lame excuse that the keys were misplaced. This compelled
the officers to break open it. When opened forcibly, this secret drawer contained five silver pouches where
narcotic drugs were securely kept. On further enquiry it was found that he had two lockers in two different
banks. A search of the lockers gave way to the recovery of fixed deposits receipts worth Rs. 1.25 crores, hard
cash Rs. 50 lacs and property papers showing properties in Uttrakhand and Rajasthan. In both the properties
his name was not registered as the owner. The title documents of residential property at Uttrakhand contained

© The Institute of Chartered Accountants of India


the name of Raj Karan, his driver and the property in Rajasthan was a farm house wh ich was in the name of
Sanju who was the husband of the full time maid-servant Rani working at his house. Both the properties
seemed to be benami properties. Recovery of a green diary from one of the lockers confirmed the name of
Chimanbhai Patel and the various transactions Albert had with him. Time was up for both of them.
Since dealing in drugs was a Scheduled offence under the PML Act, the ED Officers, filed a complaint before
the jurisdictional Magistrate for taking cognizance of the scheduled offence. Thereafter, following the property
attachment procedures, the Dy. Director duly authorised by the Director provisionally attached and seized all
the movable and immovable properties as well as records. Identification marks were placed and an inventory
was made in respect of seized property and records. This was done in the presence of two independent
witnesses.
At the time when Albert revealed the involvement of Chimanbhai, immediately, a search team under the
supervision of duly authorised Dy. Director was sent to the palatial bungalow of Chimanbhai Patel situated at
Bandra. It was found that the bungalow was spread over 5000 sq.ft. approximately. A search of the basement
of his bungalow revealed presence of narcotic drugs and psychotropic substances. In between the wooden
partitions used in the basement, they also found counterfeit Indian currency valuing Rs. 40 crores. It was a
Scheduled offence falling in Part A of the Schedule to the PML Act where amount involved had no
consideration. It was alleged by the Dy. Director that Chimanbhai Patel possessed proceeds of crime but
tried to project the same as untainted property and therefore he was guilty of offence of money-laundering
under Section 3 of the PML Act.
The other - Jal Tarang Mahal residence - a 7 BHK villa of Chimanbhai Patel at Juhu was also raided
concurrently by the ED officials. It was really a humongous, palatial sea-facing bungalow covering
approximately 25,000 sq.ft. area and fully done up with imported and handpicked interiors. The building had
basement, ground floor and a first floor. In a two-day long search, the officials seized, inter-alia, thirty diamond
rings worth Rs. 30 crores; fifty watches worth Rs. five crores; the choicest of rare paintings by M. F. Hussain,
Hebbar, Tyeb Mehta and Amrita Shergil valuing approximately Rs. 21.5 crores which were displayed in a
special air-tight hall so that moisture in the air could not damage them; high end and antique jewellery valued
at Rs. 46 crores; high end cars which included Rolls Royce Ghost, Mercedes Benz, Porsche Panamera, Ford
Mustang, Toyota Fortuner and Innova. The total attachment and seizure of diamonds, gold, precious and
semi-precious stones and other movable and immovable assets stood at Rs. 6562 crores. His various
companies were also searched and a number of incriminating documents, files, computers, etc. were seized.
The intensive search revealed that Chimanbhai used to bring his own black money from about twenty shell
companies based at Hong Kong and Dubai into the accounts of his three main companies in India as foreign
direct investment. He subsequently diverted these funds into the accounts of various shell companies
describing transfers as unsecured loans from where the funds were siphoned off through various means
including cash withdrawals.
An investigation was also conducted under FEMA, 1999 for alleged violations of Sections 3 and 4 of FEMA
for dealing in and acquiring and holding foreign exchange in his account with United Royal Bank of
Switzerland whose value in Indian currency was approximately Rs. 3,600 crores.
Being a Scheduled offence falling in Part A, it was required of authorised ED Officers, to file a complaint
before the jurisdictional Magistrate for taking cognizance of the offence which was done immediately.
Thereafter, following the provisions of Section 17, the ED Officers seized all the movable and immovable
properties as well as records in the presence of two independent witnesses. As in the case of Albert,
identification marks were placed and an inventory of the seized property was also made.
All of them were arrested by the authorised ED Officers, since ED officers, on the basis of material in their
possession, had reason to believe that they were guilty of an offence punishable under PML Act. I mmediately

© The Institute of Chartered Accountants of India


after their arrest, the officers forwarded a copy of the order along with the material in their possession to the
Adjudicating Authority in a sealed envelope, in the prescribed manner. Further, the guilty persons were, within
24 hours, taken to the jurisdictional Magistrate.
As we have noted earlier, keeping in view Section 5 (1), in all the above cases, the ED Officers, through
written orders provisionally attached the properties because it was suspected that they were derived from the
proceeds of crime. The ED Officers knew that the maximum period of attachment would be limited to 180
days from the date of the order.
Thereafter, the ED Officers forwarded the copies of the orders provisionally attaching the properties of Neetu,
Albert and Chimanbhai Patel along with the various documents in his possession to the Adjudicating Authority
in a sealed envelope.
The ED Officers also filed complaints stating the facts of such provisional attachments before the Adjudicating
Authority within thirty days of such attachments.
The Adjudicating Authority served on Neetu, Albert and Chimanbhai Patel notices to explain in not less than
30 days their source of income, earning or assets out of which they had acquired the attached property.
The attachment of the properties was confirmed by the Adjudicating Authority bearing in mind that such properties
were involved in money laundering being obtained through the proceeds of crime. However, such confirmation was
made only after considering the replies of the aggrieved persons as well as after hearing them.
In terms of confirmation order passed by the Adjudicating Authority, the ED Officers, forthwith took the
possession of the attached properties.
The trial of the above money laundering offences is being done by the jurisdictional Special Cou rt. The Central
Government in consultation with the Chief Justice of the High Court is empowered to designate one or more
Courts of Sessions as Special Court or Special Courts for trial of offence of money laundering .
Under Section 4, if Neetu, Albert and Chimanbhai Patel are found to have committed the offence of money-
laundering, then they shall be punishable with rigorous imprisonment which shall be minimum three years
and maximum seven years and shall also be liable to fine. In case it is proved that th e proceeds of crime
involved in money-laundering relate to any offence specified under paragraph 2 of Part A of the Schedule,
the maximum punishment may extend to ten years instead of seven years.
On conclusion of a trial, if the Jurisdictional Special Court finds that the offence of money-laundering has
been committed, it shall order that the properties involved in the money laundering shall stand confiscated to
the Central Government.
If on conclusion of a trial, the Special Court finds that the offence of money laundering has not taken place,
it shall order release of such property to the person entitled to receive it.
Required (MCQ of 2 marks each)
Select the most appropriate answer from the options given for each question:
1. Whether the 2BHK flat owned by Albert but rented out can be considered to have been derived from the
proceeds of crime:
(a) Yes it can be considered because Albert, the owner, is involved in money laundering activities;
(b) No, it cannot be considered because Albert did not purchase it from funds obtained through money
laundering activities;
(c) No, it cannot be considered because Albert has rented it out;
(d) None of the above.

© The Institute of Chartered Accountants of India


2. Adjudicating Authority may serve a notice of not less than --------- on Neetu, Albert and Chimanbhai
Patel who are believed to have committed offence of money laundering to explain their source of income,
earning or assets out of which they had acquired the attached property.
(a) 14 days
(b) 30 days
(c) 60 days
(d) None of the above
3. After provisional order of attachment is confirmed by the Adjudicating Authority, the Director shall
forthwith -------------.
(a) confiscate the attached properties;
(b) take the possession of the attached properties;
(c) seize the attached properties;
(d) None of the above
4. Provisional attachment of property of Chimanbhai Patel suspected to be involved in money laundering
ensures that he is prohibited to:
(a) transfer the attached property;
(b) convert the attached property;
(c) dispose of the attached property;
(d) All of the above
5. A complaint with the Adjudicating Authority is to be filed within a period of --------- days by the Director
who provisionally attaches the property involved in money laundering.
(a) 15
(b) 20
(c) 30
(d) 60
6. If on conclusion of a trial by the Jurisdictional Special Court, the guilt of Neetu, Albert and Chimanbhai
Patel is proved, it shall make an order to --------
(a) Freeze the attached property;
(b) Confiscate the attached property;
(c) Seize the attached property;
(d) None of the above.
7. Chimanbhai Patel was found to have been in possession of counterfeit Indian currency which is a
Scheduled offence belonging to:
(a) the Unlawful Activities (Prevention) Act, 1967;
(b) the Indian Penal Code;
(c) the Prevention of Corruption Act, 1988;
(d) None of the above.
8. An offence specified in Part B of the Schedule shall be considered as Scheduled offence under PML Act
only if the total value involved in such offence is --------.
5

© The Institute of Chartered Accountants of India


(a) Rs. 30.00 lacs or less;
(b) Rs. 50.00 lacs or less;
(c) Rs. 1.00 crore or more;
(d) None of the above.
9. Both Raj Karan and Sanju are to be considered as Benamidar because they are the:
(a) fictitious persons who have not made any payment for purchase of properties;
(b) persons in whose name the benami properties are held without making any payment by them;
(c) persons who have lent their names to be owners of the properties without making any payment by
them;
(d) All of the above.
10. Any property found to be involved in money laundering cannot be provisionally attached by the Director,
ED for more than ---------------.
(a) 30 days
(b) 60 days
(c) 90 days
(d) 180 days
Required (Descriptive Questions of 10 marks each)
1. As per the facts, Albert through laundered money purchased 3 BHK. Suppose if the said flat is purchased
by him jointly on his and his wife’s name, Neelima Goerge. Examine in the light of the Prevention of
Money Laundering Act 2002, the following situtaions:
(a) Will Neelima be also liable for holding of the such joint property.
(b) If property is claimed by a person, other than whom the notice has been issued. Discuss the legal
position of the person claiming the property.
2. (a) Suppose Mr. X, a non-resident Indian, purchases a flat of Albert in India, for Rs. 50,00,000 and
paid 30,00,000 in by account payee cheque of his own account and rest in cash. The registry was
done at a value of Rs 30,00,000 which was paid by cheque. Discuss the nature of the transac tion.
(b) (i) Albert was assigned by Chiman Bhai to deliver counterfeit currency-notes to one of his close
friends to Honkong for which hefty commission was fixed by the Chiman Bhai. Discuss,
whether the said act can be considered as money laundering. Who shall be liable for the
commission of the money Laundering.?
(ii) State whether maintenance of foreign currency accounts in India and outside India by Albert
is permittable in FEMA, due to his nature of business.
3. (i) Chiman bhai is a person resident in India. He has different business units as to manufacturing &
designing of jewellary in Hongkong which is owned by him. How will you determine whether a
particular business units of Chiman bhai is a ‘person resident in India’ under the Foreign Exchange
Management Act, 1999?
(ii) Suppose if ‘Blue Sapphire Pvt. Ltd.” is a Singapore based company having several business units
all over the world. It has a unit for cutting & manufacturing precious and semi-precious stones in
the form to be used for the jewellary with its Headquarters in Mumbai. It has a Branch in Dubai
which is controlled by the Headquarters in Mumbai. What would be the residential status under the
FEMA, 1999 of units of Blue Sapphire Pvt. Ltd in Mumbai and that of Dubai branch?

© The Institute of Chartered Accountants of India


ELECTIVE PAPER 6D- ECONOMIC LAWS

SOLUTION – CASE STUDY 4

I. ANSWERS TO OBJECTIVE TYPE QUESTIONS

1. (b)

[ Hint: Based on Section 2 (1) (u) of the Prevention of Money-Laundering Act, 2002]

2. (b);

[Hint: Refer Section 8 of the Prevention of Money-Laundering Act, 2002].

3. (b);

[Hint: Refer section 8 (4) of the Prevention of Money-Laundering Act, 2002].

4. (d);

[Hint: Based on Section 2 (1) (d) of the Prevention of Money-Laundering Act, 2002]

5. (c);

[Hint: Refer Section 5 (5) of the Prevention of Money-Laundering Act, 2002].

6. (b);

[Hint: Refer Section 8 (3) and 8 (6) of the Prevention of Money-Laundering Act, 2002].

7. (b);

[Hint: Refer Schedule to the Prevention of Money-Laundering Act, 2002]

8. (c);

[Hint: Refer Section 2 (1) (y) of the Prevention of Money-Laundering Act, 2002].

9. (d);

Hint: Refer Section 2 (10) of the Prohibition of Benami Property Transactions Act, 1988.

10. (d):

[Hint: Refer Section 5 (1) of the Prevention of Money-Laundering Act, 2002].

II. ANSWERS TO DESCRIPTIVE QUESTIONS

1. According to section 8 of the Prevention of Money Laundering Act, 2002, on receipt of a


complaint or applications, if the Adjudicating Authority has reason to believe that any person

© The Institute of Chartered Accountants of India


has committed an offence of money laundering or is in possession of proceeds of crime, he may
serve a notice of not less than thirty days.

Such person shall be called upon to indicate the sources of his income, earning or assets, out
of which or by means of which he has acquired the property so or, seized or frozen.

However, where a notice specifies any property as being held by a person on behalf of any other
person, a copy of such notice shall also be served upon such other person. Where if, such
property is held jointly by more than one person, such notice shall be served to all persons
holding such property.

The Adjudicating Authority shall, after hearing the aggrieved person and the Director or any
other officer authorised by him in this behalf, and taking into account all relevant materials
placed on record before him, by an order, record a finding whether all or any of the properties
referred to in the notice issued , are involved in money-laundering. Provided that if the property
is claimed by a person, other than a person to whom the notice had been issued, such person
shall also be given an opportunity of being heard to prove that the property is not involved in
money-laundering.

According to the above stated provisions, following are the answers:

(a) Since in the given case, Alberts holds the property jointly in his and his wife’s name i.e.
Neelima George. As per the above law, such notice shall be served to all persons holding
such property. So accordingly, Neelima will also be served the notice, and being heard.
Taking into account all relevant materials placed on record before him, by an order, record
a finding whether all or any of the properties referred to in the notice issued , are involved
in money-laundering, then in such case Neelima will also be liable for holding of the joint
property.

(b) If property is claimed by a person, other than whom the notice has been issued therein,
such person shall also be given an opportunity of being heard to prove that the property is
not involved in money-laundering.

2. (a) According to the Explanation given to section 2(9) of the Prohibition to Benami
Transaction Act, Benami transaction shall not include any transaction involving the
allowing of possession of any property to be taken or retained in part performance of a
contract, where—

(i) consideration for such property has been provided by the person to whom
possession of property has been allowed but the person who has granted
possession thereof continues to hold ownership of such property;

(ii) stamp duty on such transaction or arrangement has been paid; and

© The Institute of Chartered Accountants of India


(iii) the contract has been registered

Since the property is in the name of Mr. X and not in others name and it is registered on
duly paid stamp duty, it is not a Benami Transaction.

(b) (i) As per the Prevention of Money Laundering Act, 2002, whosoever directly or
indirectly attempts to indulge or knowingly assists or knowingly is a party or is
actually involved in any process or activity connected with the proceeds of crime
including its concealment, possession, acquisition or use and projecting or
claiming it as untainted property shall be guilty of offence of money laundering
(Section 3).

“Proceeds of crime” means any property derived or obtained, directly or indirectly,


by any person as a result of criminal activity relating to a scheduled offence or the
value of any such property [Section 2(1)(u)].

Every Scheduled Offence is a Predicate Offence. The occurrence of the scheduled


Offence is a pre requisite for initiating investigation into the offence of money
laundering.

In the given case , Chiman Bhai assigned Albert to deliver counterfeit currency
notes to be given to his friends in Hongkong , which is an offence falling within the
purview of scheduled offence in Part A of the PMLA, 2002 under section 489B of
the IPC. This section deals with the using as genuine, forged or counterfeit
currency-notes or bank-notes. According to the section whoever sells to, or buys
or receives from, any other person, or otherwise traffics in or uses as genuine, any
forged or counterfeit currency-note or bank-note, knowing or having reason to
believe the same to be forged or counterfeit, shall be liable under the Prevention of
Money Laundering Act.

Hence, Albert, Chiman Bhai and his friends in Hongkong, all are said to be liable
under the Prevention of Money Laundering Act.

(ii) The Foreign Exchange Management (Permissible Capital Account Transactions)


Regulations, 2000 specifies list of transactions, which are permissible in respect of
persons resident in India in Schedule I.

Schedule I states the list of permissible classes of transactions made by persons


resident in India. List specifies, maintenance of foreign currency accounts in India
and outside India by a person resident in India. Accordingly, maintenance of foreign
currency accounts in India and outside India by Albert is permittable in FEMA.

© The Institute of Chartered Accountants of India


3. (i) Person resident in India

Section 2(v) of FEMA, 1999 defines the term “person resident in India”. According to Section
2(v) (iii), all business units in India will be “resident in India” even though these units are
owned or controlled by a person resident outside India.

Similarly all business units outside India will be ‘resident in India’ provided the business
units are either owned or controlled by a person resident in India [Section 2(v) (iv)].

It is necessary to determine the residential status of the person (i.e.,Chiman bhai) who
owns or controls the business units in outside India.

(ii) Blue Sapphire Pvt. Ltd., being a Singapore based company would be person resident
outside India [(Section 2(w)]. Section 2 (u) defines ‘person’ under clause (vii) thereof, as
person would include any agency, office or branch owned or controlled by such person. The
term such person appears to refer to a person who is included in clause (i) to (vii).
Accordingly Blue Sapphire Pvt. Ltd. unit in Mumbai, being a branch of a company would be
a ‘person’.

Section 2(v) defines a person resident in India. Under clause (iii) thereof person resident in
India would include an office, branch or agency in India owned or controlled by a person
resident outside India. Blue Sapphire Pvt. Ltd unit in Mumbai is owned or controlled by a
person resident outside India, and hence it, would be a ‘person resident in India.’

However, Dubai Branch though not owned, is controlled by Blue Sapphire Pvt. Ltd. unit in
Mumbai which is a person resident in India. Hence prima facie, it may be possible to hold a
view that the Dubai Branch is a person resident in India.

© The Institute of Chartered Accountants of India


ELECTIVE PAPER 6D: ECONOMIC LAWS

Case Study 5
Mrs. Shakuntala Bisht was a dynamic woman entrepreneur running her factory of manufacturing designer candles
and other items made of wax as a proprietary concern in Dehradun (Uttrakhand) titled as M/s. Bisht Designer
Candles since 2003. She had appointed a number of dealers pan India for selling her designer products.
She was residing in a bungalow on Subhash Road in Dehradun along with her family. Her husband Mr. O. P. Bisht
was joint-secretary in Uttrakhand Sachivalaya. Her son Varun had done his B.E. (Bachelor of Engineering) from
IIT, Kharagpur and thereafter MBA from IIM, Kolkata in the year 2013. Her daughter Latika was pursuing B.Sc.
(Hons.) in Physics from DIT University, Dehradun.
Varun, being a brilliant student, secured a job in Accenture through Campus placement. He attended a three
months’ residential training programme and joined as Assistant Manager (Operations) in Pune branch of the
company. He took a one BHK flat on rent at Hinjewadi locality, purchased some furniture and other daily household
items and got himself settled in the new atmosphere. He was happy and content as the package offered to him
was very lucrative.
Mrs. Bisht had high aspirations and was desirous of expanding her business further. Therefore, in the year 2013,
she thought of exporting her products to various countries crossing the borders of India. After discussing with her
family members, she decided to convert her proprietary concern into a private limited company. Accordingly, she
got registered her company under the title Bisht Designer Candles Pvt. Ltd. in which she and her daughter were
directors while all of the four family members were shareholders. Thereafter, she completed various formalities
required for exporting her product which, inter-alia, included obtaining a ten digit importer-exporter code (IEC)
number from Directorate General of Foreign Trade (DGFT).
In the year 2015, she sent her first export consignment of designer candles to a foreign buyer in Berlin, Germany.
The order amounted to € 20800 and the importer was required to make payment in three months after shipment.
As per the terms and conditions a Letter of Credit (L/C) was opened by the Deutsche Bank on behalf of the importer.
Before shipping goods, Mrs. Bisht had to fill requisite export declaration form since the consignment did not fall in
exempted category as mentioned in Regulation 4 of the FEM (Export of Goods and Services), Regulations, 2015.
After shipment of goods, she submitted the documentary bill of exchange drawn under L/C to Syndicate Bank,
Dehradun and got it discounted under her sanctioned bills discounting limit. On the due date Syndicate Bank
received the export payment and squared off her liability.
Subsequently, she explored candle market in the USA and came in contact with M/s. Williams’ Art Gallery in Boston
which had a five storey departmental store. In this store, one of the floors was meant only for designer candles and
other items made of wax. After due negotiations with the CEO Mr. Williams, she managed to get an advance of
50,000 USD being 50% of the total export value. It was well within her knowledge that in case an advance was
received against export to a foreign buyer, the shipment of goods was to be made within one year of receipt of
advance and the export documents were required to be routed through the same authorised dealer which received
the advance on her behalf. She shipped the goods much before one year and also got payment well within the
statutory period of nine months from the date of export.
On the of successful settlement of her first export consignment to M/s. Williams’ Art Gallery of Boston, she took
steps to complete another export order from the same party for USD 1,00,500. However, this time no advance
payment was made by the importer and on the basis of his firm order, she dispatched the consignment of designer
candles. After shipment of goods, she submitted the documentary B/E to Syndicate Bank, Dehradun for
discounting. As per the agreement, the importer was to make payment on the completion of five months from the

© The Institute of Chartered Accountants of India


receipt of consignment at his godown. However, by the time five months were over, the importer could make
payment of only 40% of the total export value.
Being in need of funds, she started raising and collecting funds from various sources. In one of the cases she had
given an unsecured loan of Rs. 5 lacs to a private limited company in which a distant relative of her husband was
a director. However, when she demanded her loan back from the company, it was transpired that the company
was under liquidation process before the National Company Law Tribunal under Insolvency and Bankruptcy Code,
2016.
Varun was doing his job at Pune to the complete satisfaction of his superiors. In the next three years’ time after
joining Accenture, Varun could save a lot of money as he was a man of few needs. One day, a casual talk with the
local grocer Ajay Gupta gave him an idea to buy a flat in a housing society. Ajay gave him the phone number of a
known property dealer, Mr. Rajnikant. Thereafter, a meeting was fixed in the office of Mr. Rajnikant where he
noticed a Certificate of Registration hanging on the wall of his office. On enquiry, he was told that now it was
mandatory for the property agents to get themselves registered under Real Estate (Regulation and Development)
Act, 2016. After seeing the certificate Varun could conclude that he was dealing with a genuine person. After due
negotiations, a ground floor 2BHK apartment was finalised in Vayudoot Apartments at a cost of Rs. 62.35 lacs. He
himself arranged Rs. 30 lacs out of his savings; obtained a housing loan of Rs. 20 lacs from Axis Bank while the
remaining amount of Rs. 12.35 lacs was given by his father out of his personal savings. The title deeds got
registered in his name after making payment of stamp duty and other statutory dues. On an auspicious day Varun
shifted to his new flat.
About after a month of shifting to his own flat, Varun’s boss called him and informed that recognizing his hard work
and devotion towards the company he was being transferred to Boston, USA on promotion as Manager
(Operations). He was beaming with happiness and thanked his boss from the bottom of his heart. He was supposed
to join within next one month.
He went back to Dehradun, completed various formalities including obtaining of visa, packed his belongings and
bade goodbye to his family. On the advice of his father he leased out his flat on rent to a reputed private company
and then flew to Boston and joined his job. Over there, he was provided with a furnished apartment by the company
in the suburbs of Boston. As daily commuting was a bit difficult, he purchased a second-hand SUV. Slowly and
gradually he settled in his new home, new office and new country.
Here in Dehradun, Mrs. Bisht was pursuing vigorously to obtain export payment from M/s. Williams’ Art Gallery
because the statutory period of nine months was over long back and the remaining payment was yet to be received.
In the meantime, the authorities at Syndicate Bank also started pressurizing Mrs. Bisht to get the foreign exchange
realised at the earliest since the statutory period of nine months was already over. They opined that in cases of
default the Reserve Bank of India may also issue appropriate directions for the purpose of securing the payment if
the goods were sold in USA or if they were still unsold to get them re-imported into India within the specified period.
Though the Reserve Bank had not so far issued any directions but according to her bankers, omission on the part
of RBI to give directions did not absolve her from the consequences of committing the contravention. Therefore,
she was duty bound to realise the export payment as early as possible.
Besides taking various steps, she also persuaded her son Varun who was already in Boston to follow the matter
vigourously and advised him to meet Mr. Williams personally and settle the case. A meeting was fixed and during
conversation, it was transpired that though Mr. Williams had sold whole of the consignment, the purchaser was yet
to make payment because of some mismanagement. However, on the vigourous persuasion of Varun, Mr. Williams
exerted pressure on the local purchasers and within next one month, remaining payment along with interest was
realised and repatriated to India.

© The Institute of Chartered Accountants of India


Varun had a school friend Raman Verma in India who had done MBA from Symbiosis, Pune after his graduation
from Dehradun and had joined sales team of LIC at Shimla. From time to time after joining Accenture in Boston,
Varun was persuading him to visit Boston and nearby areas along with his wife Vaishnavi Verma. At last, Raman
and his wife agreed for the foreign visit and both of them obtained visa.
Raman approached Canara Bank, Shimla for purchase of USD 12,000 for a private visit to the USA. The bank
without much formalities gave him the required amount in foreign currency since it was well within USD 2,50,000,
i.e. an amount which could be remitted by a resident individual in a financial year under Liberalised Remittance
Scheme (LRS). Moreover, the foreign currency was not required to be remitted for any prohibited current account
transaction [mentioned in Schedule I to the FEM (Current Account Transactions) Rules, 2000] like participation in
lottery schemes or lottery like schemes existing under different names like money circulation scheme or
remittances for the purpose of securing prize money/awards, etc. He was asked to submit a simple letter containing
the basic information, viz., his name, address and that of beneficiary (i.e. self), SB account number, amount to be
remitted and the purpose of remittance along with a cheque of equivalent amount in rupees. In no time, both of
them reached the USA.
Varun received Raman and his wife with open heart at the Logan International Airport, Boston and all of them
drove to his residence. The next ten days were full of fun and frolic. They visited a number of famous sites which
included John F. Kennedy Presidential Museum & Library, Boston Public Library which was opened in 1852 as the
first free publicly-supported municipal library in America, Museum of Fine Arts having world's most comprehensive
art collections, Boston Public Garden famous for its Swan Boats and having over 600 varieties of trees, Old North
Church & Historic Site where the two famous signal lanterns were hung launching the American Revolution, New
England Holocaust Memorial where its six glass towers represented the six million Jews who perished in the
holocaust, Bunker Hill Monument, etc. In between, they had an overnight stay at New York as well.
Varun helped them in purchasing some nice dresses, chocolates, perfumes, cosmetic items and also some
souvenirs for their relatives and friends in India. They enjoyed their trip to USA to the fullest and flew back to India
with nice memories.
Raman still had with him unspent amount of USD 3500. On enquiry with his bankers regarding surrender of this
amount he was informed that he could surrender to the bank any unspent foreign exchange within a period of 180
days from the date of his return to India. Even if he approached the bank after this period, the bank would not
refuse to purchase unspent foreign exchange merely because the prescribed period of 180 days had expired. He
was further informed that he was permitted to retain with him foreign currency notes up to USD 2000 and foreign
coins without any ceiling beyond 180 days and he could utilize this amount for his subsequent visit abroad.
Varun wanted to be inform regarding sale of his flat in Pune if he was to settle down in the USA permanently since
his family at Dehradun was not that much inclined to keep the flat. He once again approached Mr. Rajnikant and
enquired whether he, as NRI, could sell his flat. Mr. Rajnikant after obtaining necessary information from one of
his lawyer friends, informed him that he was permitted to sell his flat in India to a person resident in India. Further,
he could also sell the flat (since it was not an agricultural or plantation property or farm house) to a person resident
outside India who is an Indian citizen or to a person of Indian origin resident outside India. Such permission was
available under Regulation 3 of FEM (Acquisition and Transfer of Immovable Property in India), Regulations, 2000.
As regards purchase of immovable property at Boston, Varun was informed that FEMA did not restrict such
acquisition by a non-resident Indian and he had to follow local laws in this respect. However, if his family members
in India remitted to him funds under the Liberalised Remittance Scheme (LRS) for purchasing immovable property
outside India, then the said property should be in the name of all the members who made the remittances. Even
as per Section 6(4) of the FEMA, if he becomes a person resident in India in future, he would be allowed to hold,
own or transfer the immovable property situated outside India because such property was acquired by him when
he was resident outside India.
3

© The Institute of Chartered Accountants of India


I. Objective Type Questions (2 marks each)
Select the most appropriate answer from the options given for each question:
1. Which of the following remittance would require prior approval of the Reserve Bank of India?
(a) Donation exceeding 0.5% of foreign exchange earning during the previous three financial years or USD
40,00,000, whichever is less for contribution to funds promoted by educational institutes,
(b) Commission per transaction to agents abroad for sale of commercial plots in India of USD 20,000 or 4%
of the inward remittance whichever is more,
(c) Remittance exceeding USD 10,00,000 per project for other consultancy services procured from outside
India.
(d) Remittance of 4% of investment brought into India or USD 90,000 whichever is higher, by an entity in
India by way of reimbursement of pre-incorporation expenses.
2. Mr. O. P. Bisht’s name does not appear in the registration papers relating to Pune apartment purchased by
Varun though he contributed Rs. 12.35 lacs towards the cost of the apartment.
(a) It is a benami transaction to the extent of Rs. 12.35 lacs.
(b) It is wholly a benami transaction.
(c) It is not a benami transaction
(d) None of the above
3. Export of the following goods/software would require furnishing of the declaration under FEMA, 1999?
(a) Goods imported free of cost on re-export basis
(b) Publicity material supplied free of payment
(c) By way of gift of goods accompanied by a declaration by the exporter that they are of six lakh rupees in
value
(d) Unaccompanied personal effects of travellers
4. An exporter receiving advance payment against exports from the foreign buyer is required to make the
shipment of the goods within ---------- of receiving advance payment, if export agreement does not mention
anything to the contrary regarding time period:
(a) 6 months
(b) 9 months
(c) One year
(d) One and a half years
5. An Indian citizen resident outside India is permitted to transfer his agricultural property in India to:
(a) any person resident in India
(b) any person resident outside India if he is a citizen of India or a person of Indian origin.
(c) Neither (a) nor (b)
(d) both (a) and (b)
6. Foreign exchange purchased from an authorised dealer by a resident individual, if remains unspent, needs
to be surrendered to the authorised dealer within ------------ of purchase or date of his return to India:
(a) 60 days
4

© The Institute of Chartered Accountants of India


(b) 90 days
(c) 120 days
(d) 180 days
7. In case of goods valuing up to Rs. 5,00,000 as declared by the exporter and sent by way of gift to an importer
in a foreign country:
(a) an export declaration need to be furnished
(b) an export declaration need not be furnished
(c) furnishing of export declaration depends upon the discretion of the authorised dealer who handles
export documents
(d) furnishing of export declaration depends upon the discretion of the Custom authorities
8. The term 'Moratorium' in the Insolvency and Bankruptcy Code, means-
(a) A temporary prohibition on an activity by the competent authority.
(b) A period declared by the NCLT, during which no action can be taken against the Company or the
assets of the Company.
(c) Suspension order of the Board on the debtor's operations.
(d) Order issued by the NCLT prohibiting an action against the creditor.
9. Is it possible for a non-resident Indian to acquire immovable property outside India:
(a) No, it is not possible
(b) Yes, it is possible
(c) Yes, it is possible but subject to the permission of RBI
(d) None of the above
10. As per the Insolvency and Bankruptcy Code, 2016, an Interim Resolution professional approved by the
committee of Creditors:
(a) Can never be replaced until the conclusion of the resolution process
(b) Has a fixed term of 180 days
(c) Can be replaced with 75% voting in favour of the decision and approval of the Board
(d) Can be replaced with 75% voting in favour of the decision.

II Descriptive Questions (10 marks each)


1. Analyse the following situations under the Foreign Exchange Management Act, 1999:
(i) Forex Dealers Ltd. is an Authorised Person within the meaning of Foreign Exchange Management Act,
1999. Reserve Bank of India issued certain directions to the said Authorised Person to file certain returns,
which it failed to file. You are required to state the penal provisions to which the said Authorised Person has
exposed itself.
(ii) Mr. Shekhar resided for a period of 150 days in India during the Financial year 2016-2017 and thereafter
went abroad. He came back to India on 1 st April, 2017 as an employee of a business organization. What
would be his residential status during the financial year 2017-2018?

© The Institute of Chartered Accountants of India


(iii) ‘Printex Computer’ is a Singapore based company having several business units all over the world. It has
a unit for manufacturing computer printers with its Headquarters in Pune. It has a Branch in Dubai which is
controlled by the Headquarters in Pune. What would be the residential status under the FEMA, 1999 of printer
units in Pune and that of Dubai branch?
2. Examine with reference to the provisions of the Foreign Exchange Management Act, 1999 whether there
are any restrictions in respect of the following:-
(i) Drawal of Foreign Exchange for payments due for depreciation of direct investment in the ordinary
course of business.
(ii) A person, who is resident of U.S.A. for several years, is planning to return to India permanently. Can
he continue to hold the investment made by him in the securities issued by the co mpanies in U.S.A.?
(iii) A person resident outside India proposes to invest in the shares of an Indian company engaged in
construction of farm houses.
(iv) A person, who is resident of Canada, is planning to acquire an immovable property in Mumbai.
3. Analyze the following situations under the Real Estate (Regulation and Development) Act, 2016:
(i) Mr. Ram booked a 4 BHK flat under the Gateways project. The project is under supervision of Mr.
Pankaj. Mr. Pankaj without telling the allottees reduced the number of rooms from 4 to 3 himself.
Whether this is allowed under the Act and what remedies does the Allottees have.
(ii) Mr. Vivaan booked a 4 BHK flat under the Flower Valley project for a total cost of Rs. 2 Crore. The
project is under supervision of Mr. Shyam. Mr. Shyam put a condition to pay Rs. 50 Lakhs as an
application fee before entering into a written agreement for sale with Mr. Vivaan. Decide whether the
contention of Mr. Shyam is valid?

© The Institute of Chartered Accountants of India


ELECTIVE PAPER 6D: ECONOMIC LAWS

SOLUTION

Case Study 5

ANSWER 1
I ANSWERS TO OBJECTIVE TYPE QUESTIONS

1. (c) [Hints: Refer Regulation 2 of Schedule III of Foreign Exchange Management (Current
Account Transactions) Rules, 2000]
2. (c) [Hint: Refer Section 2 (9) of the Prohibition of Benami Property Transactions Act, 1988.
It is not a benami transaction because all statutory dues have been paid and his father
knew about the transaction. Therefore, it falls under exempted category. The amount so
contributed can be a loan or gift to the son.]
3. (c) [Hint: Refer Regulation 4 of the Foreign Exchange Management (Export of Goods and
Services) Regulations, 2015]
4. (c) [Hint: Refer Regulation 15 of the FEM (Export of Goods and Services), Regulations, 2000]
5. (a) [Hint: Refer Regulation 3 of FEM (Acquisition and transfer of immovable property in India)
Regulations, 2000]
6. (d) [Hint: Refer Regulation 7 of FEM (Realisation, Repatriation and surrende r of Foreign
Exchange) Regulations, 2015]
7. (b) [Hint Refer Regulation 4 the FEM (Export of Goods and Services), Regulations, 2015
which has exempted such export transaction from furnishing of export declaration]
8. (b) [Hint: Section 14 of the Insolvency and Bankruptcy Code, 2016, describes moratorium. It
is an order passed by the adjudicating authority (NCLT) declaring a moratorium on the
debtor's operations for the period of the Insolvency Resolution Process, during which no
action can be taken against the Company or the assets of the Company. This operates
as a 'calm period' during which no judicial proceedings for recovery, enforcement of
security interest, sale or transfer of assets, or termination of essential contracts can take
place against the debtor.]
9. (b) [Hint: FEMA does not impose any restriction on acquisition of immovable property outside
India by a non-resident Indian. Further, when at a future date the person concerned
becomes a person resident in India, Section 6(4) even permits him to hold, own or transfer
immovable property situated outside India since such property was acquired by him when
he was resident outside India]
10. (c) [Hint: As per section 22 of the Insolvency and Bankruptcy Code, 2016, an Interim
Resolution professional approved by the Committee of Creditors can be replaced with
75% voting in favour of the decision and approval of the Board.
II. ANSWERS TO DESCRIPTIVE QUESTIONS
1. (i) Section 11(3) of the Foreign Exchange Management Act, 1999 states that where any
Authorised person contravenes any direction given by the Reserve Bank of India under the
said Act or fails to file any return as directed by the Reserve Bank of India, the Reserve Bank
of India may, after giving reasonable opportunity of being heard, impose on Autho rised
Person, a penalty which may extend to ten thousand rupees and in the case of continuing

© The Institute of Chartered Accountants of India


contraventions with an additional penalty which may extend to two thousand rupees for every
day during which such contravention continues.
Since as per the facts given in the question, the Authorised person, namely, Forex Dealers
Ltd., has failed to file the returns as directed by the Reserve Bank of India. According to the
above provisions, it has exposed itself to a penalty which may extend to ten thousand rupees
and in the case of continuing contraventions in the nature of failure to file the returns, with an
additional penalty which may extend to two thousand rupees for every day during which such
contravention continues.
(ii) According to the provisions of section 2(v) of the Foreign Exchange Management Act, 1999,
a person in order to qualify for the purpose of being treated as a "Person Resident in India"
in any financial year, must reside in India for a period of more than 182 days duri ng the
preceding financial year. In the given case, Mr. Shekhar has resided in India for a period of
only 150 days, i.e., less than 182 days, during the financial year 2016-2017. Hence, he cannot
be considered as a "Person Resident in India" during the financial year 2017-2018 irrespective
of the purpose or duration of his stay.
(iii) Printex Computer being a Singapore based company would be person resident outside India
[(Section 2(w)]. Section 2 (u) defines ‘person’ under clause (vii) thereof, as person would
include any agency, office or branch owned or controlled by such person. The term such
person appears to refer to a person who is included in clause (i) to (vii). Accordingly printex
unit in Pune, being a branch of a company would be a ‘person’.
Section 2(v) defines a person resident in India. Under clause (iii) thereof person resident in
India would include an office, branch or agency in India owned or controlled by a person
resident outside India. Printex unit in Pune is owned or controlled by a person resident outside
India, and hence it, would be a ‘person resident in India.’
However, Dubai Branch though not owned is controlled by Printex unit in Pune which is a
person resident in India. Hence prima facie, it may be possible to hold a view that t he Dubai
Branch is a person resident in India.
2. Capital Account Transactions: All the transactions referred to in the question are capital account
transactions.
Section 6(2) of FEMA, 1999 provides that the Reserve Bank may in consultation with the Centra l
Government specify the permissible capital account transactions and the limit upto which foreign
exchange will be allowed for such transactions.
(i) Depreciation of direct investments: According to proviso to section 6(2), the Reserve bank
shall not impose any restriction on the drawal of foreign exchange for certain transactions.
One such transaction is drawal of foreign exchange for payment due for depreciation of direct
investment in the ordinary course of business. Hence this transaction is permissi ble without
any restrictions.
(ii) Person resident in USA returning permanently to India: When the person returns to India
permanently, he becomes a resident in India. Section 6(4) provides that a person resident in
India may hold, own, transfer or invest in foreign currency, foreign security, etc. if such
currency, security or property was acquired, held or owned by such person when he was
resident outside India or inherited from a person who was resident outside India. In view of
this, the person who returned to India permanently can continue to hold the foreign security
acquired by him when he was resident in U.S.A.
(iii) Investment in shares of Indian company by non-resident: Reserve Bank issued Foreign
Exchange Management (Permissible Capital Account Transactions) Regulations, 2000.
Regulation 4(6) of the said Regulations prohibits a person resident outside India from making
investment in India, in any form, in any Company or partnership firm or proprietary concern
or any entity, whether incorporated or not, which is engaged or proposes to engage in

© The Institute of Chartered Accountants of India


construction of farm houses. Hence it is not possible for a person resident outside India to
invest in the shares of a company engaged in construction of farm houses as such investment
is prohibited.
(iv) Acquisition of immovable property by person resident outside India: Reserve Bank
issued Foreign Exchange Management (Permissible Capital Account Transactions)
Regulations, 2000. The regulations specify the classes of capital account transactions of
persons resident outside India in Schedule II. Under this schedule, acquisition and transfer of
immovable property in India by a person resident outside India is permissible. Hence, the
person resident of Canada can acquire the immovable property in Mumbai.
3. (i) Adherence to sanctioned plans and project specifications by the promoter (Section 14)
The proposed project shall be developed and completed by the promoter in accordance with
the sanctioned plans, layout plans and specifications as approved by the competent
authorities.
Notwithstanding anything contained in any law, contract or agreement, after the sanctioned
plans, layout plans and specifications and the nature of the fixtures, fittings, amenities and
common areas, of the apartment, plot or building, as the case may be, as approved by the
competent authority, are disclosed or furnished to the person who agree to take one or more
of the said apartment, plot or building, as the case may be, the promoter shall not make —
(1) any additions and alterations in the sanctioned plans, layout plans and specifications
and the nature of fixtures, fittings and amenities described therein in respect of the
apartment, plot or building, as the case may be, which are agreed to be taken, without
the previous consent of that person.
Provided that the promoter may make such minor additions or alterations as may be
required by the allottee, or such minor changes or alterations as may be necessary due
to architectural and structural reasons duly recommended and verified by an authorised
Architect or Engineer after proper declaration and intimation to the allottee.
Explanation.—For the purpose of this clause, "minor additions or alterations" excludes
structural change including an addition to the area or change in height, or the removal
of part of a building, or any change to the structure, such as the construction or removal
or cutting into of any wall or a part of a wall, partition, column, beam, joist, floor including
a mezzanine floor or other support, or a change to or closing of any required means of
access ingress or egress or a change to the fixtures or equipment, etc.
(2) any other alterations or additions in the sanctioned plans, layout plans and specifications
of the buildings or the common areas within the project without the previous written
consent of at least two-thirds of the allottees, other than the promoter, who have agreed
to take apartments in such building.
Explanation.—For the purpose of this clause, the allottees, irrespective of the number of
apartments or plots, as the case may be, booked by him or booked in the name of his
family, or in the case of other persons such as companies or firms or any association of
individuals, etc., by whatever name called, booked in its name or booked in the name of
its associated entities or related enterprises, shall be considered as one allottee only.
In case any structural defect or any other defect in workmanship, quality or provision of
services or any other obligations of the promoter as per the agreement for sale relating to
such development is brought to the notice of the promoter within a period of five years by the
allottee from the date of handing over possession, it shall be the duty of the promoter to rectify
such defects without further charge, within thirty days, and in the event of promoter's failure
to rectify such defects within such time, the aggrieved allottees shall be entitled to re ceive
appropriate compensation in the manner as provided under this Act.

© The Institute of Chartered Accountants of India


Hence, in the instant case, reducing the number of rooms does not come under minor
additions or alterations. The promoter i.e. Mr. Pankaj Gupta shall not make any additions and
alterations in the sanctioned plans, layout plans and specifications within the project without
the previous written consent of at least two-thirds of the allottees, other than the promoter,
who have agreed to take apartments in such buildings.
(ii) No deposit or advance to be taken by promoter without first entering into agreement
for sale
According to section 13 of the said Act, a promoter shall not accept a sum more than ten per
cent of the cost of the apartment, plot, or building as the case may be, as an advance payment
or an application fee, from a person without first entering into a written agreement for sale
with such person and register the said agreement for sale, under any law for the time being
in force.
In the instant case, the cost of the flat is Rs. 2 crore and Mr. Shyam put a condition to pay
Rs. 50 Lakhs as an application fee before entering into a written agreement for sale with Mr.
Vivaan. This is invalid as a promoter can accept only Rs.20 Lakhs (10% of Rs. 2 Crore) as an
advance or an application fee without first entering into a written agreement for sale.

© The Institute of Chartered Accountants of India


ELECTIVE PAPER 6D: ECONOMIC LAWS

Case Study 6
Ronit Chawla was a Fellow Chartered Accountant (FCA) practicing in the field of corporate and economic laws. He
represented his clients before Company Law Board (CLB) and thereafter in National Company Law Tribunal
(NCLT). After coming into force of Insolvency and Bankruptcy Code, 2016 w.e.f. 28 May, 2016, he learnt about the
Limited Insolvency Examination (LIE) for becoming Insolvency Professional (IP). Since he had about eleven years
of experience as practicing CA, he attempted the very first examination of LIE conducted by Insolvency and
Bankruptcy Board of India (IBBI) in December 2016 and successfully cleared it. He then enrolled himself with a
reputed Insolvency Professional Agency (IPA) and got registered with IBBI by fulfilling the requisite formalities
including payment of non-refundable application fee of Rs. ten thousand.
His father Roopesh Chawla, a resident of Green Park, New Delhi, was recently posted as Chief Manager in Bank
of India, Delhi which was a full-fledged Foreign Exchange (FX) branch though Roopesh, being unable to get a
chance to work in a FX branch, had very little knowledge of rules relating to Foreign Exchange. Therefore, he used
to consult his son Ronit in the matters of foreign exchange from time to time. His mother Rukmani Chawla was a
senior teacher in Kendriya Vidyalaya, New Delhi, taking commerce classes.
Rajnish Sinha, a close friend of Roopesh, was heading a Delhi branch of Punjab National Bank (PNB) and knew
that Roopesh’s son Ronit besides being a Chartered Accountant was also an Insolvency Professional. Rajnish, on
behalf of PNB, wanted to initiate corporate insolvency resolution process (CIRP) before NCLT in the case of its
customer Manohar Masale Pvt. Ltd. (MMPL) of Delhi which had defaulted in repaying the dues of the bank totaling
approximately Rs. 23.00 lacs. Accordingly, PNB being financial creditor, while making an insolvency resolution
application to NCLT proposed the name of Ronit as Interim Resolution Professional (IRP). MMPL was sanctioned
cash credit limit of Rs. 10.00 lacs against hypothecation of stock of raw material and finished goods and another
bill discounting limit of Rs. 5.00 lacs against actionable claims. MMPL was registered with an authorised capital of
Rs. 25.00 lacs but its paid up capital was to the tune of Rs. 10.00 lacs.
Initially started as a registered partnership concern (Manohar Masale & Co.) by two brothers, namely, Ram
Manohar and Shyam Manohar, it did profitable business and keeping an eye on future business growth, it was
converted into a private limited company with Ram, Shyam and Shyam’s elder son Shivam as directors. Shyam’s
younger son Dwapam, an alumnus of IIFT, Delhi and also a law graduate, did not have any interest in the family
business and was more inclined to continue with his current employment in a German MNC having its office in
Gurugram.
MMPL’s factory in Okhla Industrial Area was located on the one-fourth portion of the plot which was co-owned by
the brothers. However, the bank had created an equitable mortgage on the plot as well as factory building while
sanctioning the working capital limits to the company. The elder brother Ram Manohar was the anchoring person
who steered the company to newer heights due to his sheer business acumen and inherent managerial skills but
one day, all of a sudden, he had a massive heart attack resulting in his untimely death. Since he was not married,
the business of ‘masale making’ was now run by Shyam and his son Shivam.
However, the father-son duo could not manage the business properly because of the lack of foresight, faulty inter-
personal relations and poor organisational skills. Their authoritative style of leadership resulted in demotivation of
workers which led to labour unrest and all sort of other conflicts. The paternalistic approach towards them which
Ram always displayed was missing altogether. Needless to say, the output started declining and wastage of raw
material turned north. Since there was no vigourous follow-up as well, the debtors to the tune of around Rs. 12.00
lacs were long overdue. Consequently, the company started suffering losses and also defaulted on dues from the
bank.

© The Institute of Chartered Accountants of India


When PNB, even after repeated reminders to MMPL, could not realise its dues and the liability touched the height
of around Rs. 23.00 lacs (including normal and overdue interest), Rajnish Sinha, on behalf of PNB, decided to file
corporate insolvency resolution application duly supported by ledger extracts and other specified evidences
(services of Information Utility could not be used as by the time application was filed there was no IU registered
with IBBI) with Adjudicating Authority i.e. NCLT, New Delhi for initiating CIRP against MMPL.
NCLT considered the corporate insolvency resolution application along with the proposed name of Ronit as Interim
Resolution Professional (IRP). Within next 10 days of receipt of application (which was lesser than the statutory
period of 14 days) NCLT ascertained that there existed default because the defaulted amount was much more
than the minimum required of Rs. one lac. Since the CIRP application was complete in all respects, NCLT admitted
it and within the statutory period of next seven days after admission, it conveyed its order of commencement of
CIRP to the financial creditor (i.e. PNB) and the corporate debtor (i.e. MMPL).
The order of NCLT confirmed the proposed appointment of Ronit as IRP for 30 days, for Ronit had a clean record
without any disciplinary proceedings pending against him. It was also stated in the order that a moratorium period
of 180 days had become applicable during which all suits and legal proceedings, etc. against MMPL (i.e. corporate
debtor) were to be held in abeyance so as to give time to the ailing company to resolve its status. MMPL was also
barred from transferring or disposing of any of its assets or any legal rights therein. However, the supply of specified
essential goods and services to the MMPL as mentioned in the order, were not to be interrupted during moratorium
period.
In the meantime, Ronit’s father Roopesh faced a peculiar problem related to the foreign exchange matter at his
branch. His FX officer brought to his knowledge that one of their exporter customers who had received an advance
of USD 75,000 from an importer based at California, USA against export of ready-made jeans had not shipped the
requisite items worth USD 2,00,000 by utilizing the advance so received. The exporter, not willing to ship the goods,
wanted to refund the advance to the importer along with interest for which permission of Roopesh was required.
Roopesh did not allow the refund immediately and in turn, advised the FX officer to gather more knowledge about
FX provisions whether refund along with interest was permissible. At the same time he also discussed the matter
with his son Ronit who advised him to refer FEM (Export of Goods and Services) Regulations, 2015. A scrutiny of
the relevant banking records revealed that 14 months had already expired since advance of USD 75,000 was
received. Further, he came to know that if goods were not shipped within one year of receipt of advance, such
advance could not be refunded without the permission of the RBI. Accordingly, he advised the customer to seek
permission of RBI through his branch.
After his appointment as IRP, Ronit assumed full control of the affairs of MMPL. Since powers of the board of
directors stood suspended he was empowered to exercise such powers. Accordingly, he took immediate custody
and control of all the assets of the MMPL including its business records.
Following the orders of NCLT, Ronit took steps to make a public announcement within three days from the date of
his appointment regarding the initiation of CIRP against MMPL.
Public announcement, included the following aspects:
• Name and address of the corporate debtor (i.e. MMPL) and its registration/incorporating authority.
• His details as IRP and the fact that he would be vested with the management of the corporate debtor and
be responsible for receiving claims.
• Penalties for false or misleading claims.
• The last date for the submission of the claims.
• The date on which the CIRP would end.

© The Institute of Chartered Accountants of India


After the expiry of last date for submission of claims, a Committee of Creditors was constituted which included
PNB and five trade creditors who had cumulative dues of Rs. 3.00 lacs. Within seven days of its constitution, the
first meeting of the committee was called. In the meantime, Ronit electronically submitted an Information
Memorandum to the creditors after they had given an undertaking regarding maintaining of confidentiality. This
Information Memorandum contained details of assets and liabilities of the MMPL with their estimated values,
audited financial statements for the last two financial years and provisional financial statements for the current
financial year made just eight days earlier from the date of the application, a list of creditors and the amounts
claimed by them which were duly admitted and other prescribed information.
In the meeting of the Committee of Creditors it was resolved to let Ronit continue as full-fledged Resolution
Professional (RP) since he was eligible to be appointed as an independent director and was not a related party of
the MMPL and such decision was conveyed to the NCLT as well as MMPL. As RP, Ronit assumed all those powers
which were conferred on him as IRP. He was required to manage the operations of the MMPL during the CIRP
period.
Based on the Information Memorandum, Rajnish on behalf of PNB as resolution applicant undertook to prepare a
resolution plan as per the provisions of the Code for onward submission to Ronit. Before finalizing the resolution
plan, he along with his two officers took up the matter with Shyam and his son Shivam regarding the revival of
MMPL and repayment of long outstanding dues or face liquidation if they were not inclined to revive the company.
The fear psychosis of liquidation made them think frantically to save their company from imminent death. Having
woken up from their slumber they started exploring ways to bring in short term finance and also to rope in some
professional who would help the company in its revival.
Shyam saw a ray of hope in his younger son Dwapam and persuaded him to participate in the management of the
affairs of the company at least for the first three months to which he ultimately agreed. In the meantime Shyam,
with a view to raise short term finance, consulted his elder sister Rama Devi to lend at least Rs. 5.00 lacs for a
short period of about one year and also convinced his daughter Ria, her husband Dushyant as well as Dwapam to
invest at least Rs. 3.00 lacs each in the share capital of the company. Shivam who had invested funds in the share
market agreed to sell his securities to raise Rs. 3.00 lacs against which he was to be allotted shares in the MMPL.
As per the advice of the bankers, Shyam also started inter-acting with long overdue debtors for recovery who
eventually agreed to pay 50% of Rs. 12.00 lacs in the current month and remaining amount in the next month. Out
of the raised amount, the operational creditors were to be paid fully while dues of PNB were to be satisfied to the
extent of Rs. 12.00 lacs. Further, Rs. 2.50 lacs were to be allocated towards insolvency resolution process costs
including fee of RP and remaining amount was to be utilized as working capital. Since both the directors of MMPL
had consented to repay Rs. 12.00 lacs in one lump sum, Rajnish on behalf of PNB assured them that he would
take up the matter of waiving of overdue interest up to Rs. 2.00 lacs with his Dy. General Manager and would also
seek permission to revive MMPL’s limits which were currently frozen.
Based on the experience he gathered while working with two MNCs, Dwapam assumed the role of a leader to set
the company on rails. He took note of the prevailing situation from which the ailing MMPL was passing through.
He observed that the current as well as liquid ratios were much far away from the standard norms of 2:1 and 1:1
respectively. The turnover ratios were also unhealthy and at the same time the operating ratio was very high - not
a good sign for any business. An investment of about Rs. 5.00 lacs was tied up in raw material like whole red
chillies, coriander seeds, turmeric, black pepper, dry mango, etc.
Since currently the business of spices was run in a traditional manner, Dwapam decided to take the following short,
medium and long term measures:
Short term measures:
• to understand the needs and wants of customers in the target market;

© The Institute of Chartered Accountants of India


• to apply the principles of scientific management;
• to set standards for raw material, wastage, working conditions, etc.;
• to conduct time and motion studies;
• to provide financial incentives and to adopt social security plans for the workers;
• to secure registration with FSSAI immediately;
• to appoint an Administrative Officer and, if need be, to appoint another one in future;
• to devise competitive pricing strategy;
• to create a corporate brand identity by assigning the product a brand name ‘Manohar Uttam Masale’ which
would help in building a brand image;
• to design an attractive package and label by using a graphic design of spices combining green, yellow
and red colours for different varieties of masale;
• to promote the masale by advertising initially in leading newspapers and depending upon income
generation in future, to advertise on FM radio, TV as well as cinema halls;
• to adopt sales promotion measures like free gift offers, contests, free sample distribution, etc.
• to select the similar channels of distribution as used by the competitors;
• to conduct SWOT analysis of MMPL and important competitors;
• to create an effective Website of the company;
• to take decisions regarding various activities under physical distribution of masale like order processing,
transportation, warehousing and inventory control;
• to adopt strict credit policy by reducing debtors’ days with a regular follow-up;
• to use an accounting software;
• to submit various Government Returns within the prescribed time limits so that avoidable hefty penalties
are not levied.
Medium and Long Term Measures:
• to stop heavy expenditure on repairs and maintenance by installing new machines and grinders;
• to establish direct contacts with the cultivators for obtaining raw material which would help in avoiding
middlemen and their high commissions;
• to develop the remaining three-fourth portion of the plot and rent out some of the developed portion to a
commercial establishment;
• to renovate the factory building.
• to manufacture more types of different spices like Rajma Masala, Pindi Chana Masala, Shahi Paneer
Masala, Dal Makhni Masala, Mushroom Matar Masala, etc;
• to diversify MMPL’s operations by manufacturing Jams and Ketchups;
• To explore offshore markets.
Rajnish prepared a resolution plan containing the above strategies and submitted it to Ronit for his consideration.
Later on, a meeting of committee of creditors was called by Ronit and the resolution plan was presented for its
approval. The plan was duly approved by full majority. Thereafter, Ronit submitted the approved resolution plan to
the NCLT for its approval.
Since the resolution plan was approved by the committee of creditors much before the statutory period of 180 days
and also met the prescribed requirements, NCLT approved it and passed an order to this effect. Now the plan was
binding on the MMPL and its employees, members, PNB and operational creditors as well as other stakeholders
involved in the resolution plan.

© The Institute of Chartered Accountants of India


I. Objective Type Questions (2 marks each)
Select the most appropriate answer from the options given for each question:
1. “Default” under the IBC is said to be occurred on the fulfillment of condition/s-
(a) Debts becoming due and payable
(b) Non- payment of the debt
(c) Liability /obligation in respect of a claim which is due
(d) Both (a) & (b)
2. In the case study PNB initiated Corporate Insolvency Resolution Process against MMPL for the default in the
capacity of-
(a) Corporate debtor
(b) Operational debtor
(c) Financial creditor
(d) Resolution applicant
3. If the goods against which an advance payment is received from a foreign buyer are not shipped within one
year and there exists no agreement regarding timing of shipment, the advance payment:
(a) shall be refunded within reasonable time without prior approval of Reserve Bank.
(b) Shall be refunded within one year from the date of receipt of advance payment without the prior approval
of Reserve Bank
(c) Shall be refunded within one year from the date of receipt of advance payment with the prior approval of
Reserve Bank
(d) Shall be refunded after one year from the date of receipt of advance payment on the basis of reasonable
cause.
4. PNB through an assignment agreement, assigned here the debt to the X trust. X trust filed the petition for
initiation of corporate Insolvency resolution process (CIRP) against MMPL. State the correct statement with
respect to the competency of the X trust in the filing of the petition in the above situation-
(a) X Trust is not a competent applicant as per section 6 of the IBC
(b) X Trust is being authorized by the PNB to file an application
(c) X Trust in the capacity of financial creditor can file a valid petition.
(d) None of the above
5. As per the Insolvency & Bankruptcy Code, 2016, resolution plan is prepared by ----------- is submitted to ------
----------- for examination and submission to ------------- for approval.
(a) Committee of Creditors, Adjudicating Authority, Resolution Professional
(b) Resolution applicant, committee of creditors, Adjudicating Authority
(c) Resolution applicant, Resolution Professional, Committee of Creditors
(d) Committee of Creditors, Resolution Professional, Adjudicating Authority
6. The maximum duration during which the appointment of Interim Resolution Professional (IRP) is valid shall
not exceed -------- days.
(a) 10
(b) 20
(c) 30
5

© The Institute of Chartered Accountants of India


(d) 40
7. In the case study, the expenses of public announcement shall be borne by the-
(a) MMPL
(b) Ronit
(c) Roopesh
(d) PNB
8. In the case study, committee of creditors of MMPL was constituted on 17.3.2018. Time limit, within which the
first meeting of committee of creditors should be held, is ----------------.
(a) 20.3.2018
(b) 22.3.2018
(c) 24.3.2018
(d) 31.3.2018
9. Ronit, being an Insolvency Professional can be appointed as Resolution Professional, if:
(a) he is eligible to be appointed as an independent director under section 149 of the Companies Act, 2013
(b) he is not a related party of the corporate debtor
(c) only (a)
(d) Both (a) and (b)
10. MMPL finds material irregularity in exercise of the powers of the Ronit during the corporate insolvency
resolution period. Remedy available to MMPL-
(a) File a complaint to the adjudicating authority
(b) Complain to the committee of creditor’s
(c) Complaint filed before the IBBI
(d) File an appeal against the order of adjudicating authority against the approval of resolution plan.
II. Descriptive Questions (10 marks each)
1. Suppose the resolution plan prepared by Rajneesh was delayed in approval by committee of creditors. Ronit,
further presented the said resolution plan, before NCLT after 180 days of insolvency commencement date.
Answer the following-
(i) What step shall be taken by NCLT on such presented resolution plan.
(ii) What, if MMPL contravened the resolution plan which effected its employees and stake holders.
(iii) What consequences be there where liquidator continued the business of MMPL during liquidation
process.
2. Ronit in an examination of sale of property of MMPL finds that a transaction was made by the MMPL to Rama
devi (the elder sister of Shyam) in 6 months preceding the Insolvency Commencement date, was undervalued.
Give the following answers in reference to the above situation-
(i) State the validity of the conduct of such transaction by MMPL to Ramadevi.
(ii) What will be the consequences when resolution professional determines such transactions undervalue
and fails to report that same to NCLT?
(iii) What order NCLT shall pass when MMPL entered into an undervalued transaction?

© The Institute of Chartered Accountants of India


3. (a) Discuss the legal position and liability of Mr. Shyam in the following given situations-
(i) Where Mr. Shyam fraudulently transferred his holding of shares in favour of his sister of Rs.1 lakh
within 1 year immediately preceding the insolvency commencement date.
(ii) Mr. Shyam makes false entry in the books of account of MMPL to defraud creditors on insolvency
commencement date.
(iii) Shyam permitted Shivam to provide information for initiation of CIRP which is false in material
particular and omits material fact related to a books of accounts of a specified period in the
application.
(b) What course of action can be taken by NCLT against the directors of the MMPL for transactions
defrauding creditors?

© The Institute of Chartered Accountants of India


ELECTIVE PAPER 6D: ECONOMIC LAWS

SOLUTION

Case Study 6

I. ANSWERS TO OBJECTIVE TYPE QUESTIONS


1. (d) [Hints: As per section 3(12), Default means non-payment of debt when whole or any part
or instalment of the amount of debt has become due and payable and is not repaid by the
debtor or the corporate debtor, as the case may be]
2. (c) [Hints: Financial creditor means any person to whom a financial debt is owed and includes
a person to whom such debt has been legally assigned or transferred to;{section 5(7)}]
3. (b) [Hint: Refer Regulation 15 of the FEM (Export of Goods and Services), Regulations, 2000]
4. (c) [Hint: Refer Section 5 (7) of the Code]
5. (c) [Hint: Refer Section 5 (25) read with section 28 of the Code]
6. (c) [Hint: Refer Section 16 of the Code]
7. (d) [Hint: Refer Section 15 of the Code]
8. (c) [Hint: Refer Section 22 (1)]
9. (d) [Hint: Refer Regulation 3 of the Insolvency and Bankruptcy (Insolvency Resolution
Process for Corporate Persons) Regulations, 2016]
10. (d) [Hint: section 61(3) of the IBC]

II ANSWERS TO OBJECTIVE TYPE QUESTIONS

1. (i) According to section 33 of the Insolvency and Bankruptcy Code, 2016, where the Adjudicating
Authority before the expiry of the insolvency resolution process period does not receive a
resolution plan as approved by the committee of creditors, it shall—
(a) pass an order requiring the corporate debtor to be liquidated as per the relevant
provisions
(b) issue a public announcement stating that the corporate debtor is in liquidation; and
(c ) require such order to be sent to the authority with which the corporate debtor is
registered.
According to section 12 of the Insolvency and Bankruptcy Code, 2016, the corporate
insolvency resolution process (CIRP) shall be completed within a period of one hundred and
eighty days from the date of admission of the application to initiate such process.
As per the facts, Ronit, presented the approved resolution plan, before NCLT after the
prescribed period for the completion of CIRP i..e, after 180 days of insolvency commence ment
date.
According to the above stated provisions, NCLT, shall pass an order requiring the corporate
debtor (MMPL) to be liquidated. It shall issue a public announcement of its liquidation and
send such order to the Registrar of companies.
(ii) As per Section 33(3) of the Insolvency and Bankruptcy Code, 2016, where the resolution plan
approved by the Adjudicating Authority is contravened by the concerned corporate debtor,
any person other than the corporate debtor, whose interests are prejudicially affec ted by such
contravention, may make an application to the Adjudicating Authority for a liquidation order

© The Institute of Chartered Accountants of India


as referred above. Accordingly, the employees and the stakeholders of MMPL, whose
interests are affected by contravention in compliances of the resolut ion plan, may make an
application to NCLT for initiation of liquidation. On receipt of an application, if the Adjudicating
Authority determines that the MMPL has contravened the provisions of the resolution plan, it
shall pass a liquidation order.
(iii) As per section 33(7) of the Insolvency and Bankruptcy Code, 2016, the order for liquidation
shall be deemed to be a notice of discharge to the officers, employees and workmen of the
corporate debtor.
However, where the business of the corporate debtor when continued during the liquidation
process by the liquidator, it shall not be deemed to be notice of discharge to the officers,
employees and workmen of the corporate debtor.
So the Conduct of business of MMPL during liquidation process by the liquidator i s tenable
and shall not be deemed to be notice of discharge to the officers, employees and workmen of
the MMPL.
2. (i) Validity of the conduct of undervalued transaction : As per the provisions given in section
45 of the Insolvency and Bankruptcy Code, 2016, Ronit, on an examination of the
transactions of the MMPL, determines that certain transactions were made by MMPL with a
related party (Rama devi) within the period of two years preceding the insolvency
commencement date (in 6 months preceding the Insolvency Commencement date), which
were undervalued. Ronit, shall make an application to the NCLT to declare such transactions
as void and reverse the effect of such undervalued transaction and requiring the person who
benefits from such transaction to pay back any gains he may have made as a result of such
transaction.
(ii) Failure to report to NCLT of undervalued transactions: As per the stated facts given in
the light of the provisions laid in Section 47 of the Insolvency and Bankruptcy Code, an
undervalued transaction has taken place and Ronit (Resolution Professional) has not reported
it to the NCLT, in such case, a creditor , member or a partner of a MMPL, as the case may
be, may make an application to the NCLT to declare such transactions void and reverse their
effect in accordance with the relevant provisions of this Code.
(iii) Order of NCLT: Where the NCLT, after examination of the application made above, is
satisfied that undervalued transactions had occurred; and Ronit (RP) after having sufficient
information or opportunity to avail information of such transactions did not report such
transaction, there it shall pass an order of —
(a) restoring the position as it existed before such transactions and reversing the effects
thereof in the manner as laid down in section 45 and section 48 of the Code. The order
of the Adjudicating Authority may provide for the following:—
(1) require any property transferred as part of the transaction, to be vested in the
corporate debtor(MMPL);
(2) release or discharge (in whole or in part) any security interest granted by the
corporate debtor (MMPL);
(3) require any person to pay such sums, in respect of benefits received by such
person, to the Ronit (RP), as the Adjudicating Authority may direct; or
(4) require the payment of such consideration for the transaction as may be determined
by an independent expert.
(b) requiring the Board(IBBI) to initiate disciplinary proceedings against Ronit.
3. (a) (i) As per the provisions given in section 68 of the Code, Mr. Shyam, Director (an officer in
default) has within the twelve months immediately preceding the insolvency
commencement date, fraudulently transferred his holding of shares in favour of his sister

© The Institute of Chartered Accountants of India


of Rs.1 lakh (which is more than value of ten thousand rupees ). So, Mr. shyam, shall
be punishable with imprisonment for a term which shall not be less than three years but
which may extend to five years, or with fine, which shall not be less than one lakh rupees,
but may extend to one crore rupees, or with both: However, he shall not be liable to any
punishment under this section if he proves that he had no intent to defraud or to conceal
the state of affairs of the corporate debtor.
(ii) According to section 71 of the Code, on and after the insolvency commencement date,
Mr. Shyam, makes a false entry in the books of account of MMPL with an intent to
defraud or deceive any person, he shall be punishable with imprisonment for a term
which shall not be less than three years, but which may extend to five years, or with fine
which shall not be less than one lakh rupees, but may extend to one crore rupees, or
with both.
(iii) As per Section 77 of the Code, as Shyam permitted Shivam to provide informations in
the application under section 10,which is false in material particular and omits material
fact related to a books of accounts of a specified period, so he shall be punishable
with imprisonment for a term which shall not be less than three years, but wh ich
may extend to five years or with fine which shall not be less than one lakh rupees,
but which may extend to one crore rupees, or with both.
(b) As per section 69 of the Code, on or after the insolvency commencement date, where the
directors of the MMPL—
(a) has made transfer of, or charge on, or has caused or connived in the execution of a
decree or order against, the property of the corporate debtor;
(b) has concealed or removed any part of the property of the corporate debtor within two
months before the date of any unsatisfied judgment, decree or order for payment of
money obtained against the corporate debtor,
such directors of MMPL, shall be punishable with imprisonment for a term which shall not be
less than one year, but which may extend to five years, or with fine, which shall not be less
than one lakh rupees, but may extend to one crore rupees, or with both.
However, directors of MMPL, shall not be punishable under this section if the acts mentioned
in clause (a) were committed more than five years before the insolvency commencement date;
or if he proves that, at the time of commission of those acts, he had no intent to defraud the
creditors of the corporate debtor.

© The Institute of Chartered Accountants of India


ELECTIVE PAPER 6D: ECONOMIC LAWS

Case Study 7

During March 2017, XMC Pvt. Ltd., a car manufacturing company, launched its TXI model of car with a lot of
advertisements and promotions in all types of media platforms, inter alia, highlighting the Ex-showroom price
of the said car model in Mumbai as Rs. 6.25 lacs.
Mr. Nazir, a prospective buyer of the said model, visited an authorised dealer of XMC Pvt. Ltd. i.e. M/s Ratan
Lal & Sons located at Bandra, Mumbai and after due consultation/ discussion with the representatives of M/s
Ratan Lal & Sons, booked a vehicle of the aforesaid model on 11 th May, 2017 on payment of Rs. 100,000/-.
M/s Ratan Lal & Sons in turn provided the money receipt for the aforesa id transaction with serial number
ABC/1010 as well as booking reference number 218/ 2017 to Mr. Nazir. He was assured by the
representatives of M/s Ratan Lal & Sons that the booked vehicle will be delivered within three months from
the date of booking i.e. by 10th August, 2017. However, the representative of M/s Ratan Lal & Sons have
stated to Mr. Nazir that as per XMC Pvt. Ltd.’s policy, five months’ time is given in writing so as to keep some
buffer for delays which may arise due to unforeseen exigencies or transportation of vehicle or other logistic
problems. Mr. Nazir, inter alia, noted the conditions in the booking document that “the vehicle would be
delivered within six months from the date of booking”. Believing the assurance given by the representative of
M/s Ratan Lal & Son, Mr. Nazir accepted the terms of the booking and thought that he will get the vehicle
within three months from the date of booking as assured by the representatives of M/s Ratan Lal & Son and
in worst scenario he will get delivery of the vehicle within six months from the date of booking as per the
terms and conditions of booking of the vehicle.
However, within three months of booking of the vehicle, M/s Ratan Lal & Son failed to deliver the vehicle to
Mr. Nazir despite repeated request and after 10 th August, 2017, Mr. Nazir contacted the representatives of
M/s Ratan Lal & Sons many times for delivery of the vehicle and they kept on giving assurances that the
delivery of the vehicle will be done within six months from the date of booking as per the conditions of booking.
After five months, on 15 th February 2018, Mr. Nazir written an e-mail to XMC Pvt. Ltd. highlighting the issue
of delay in delivery of the booked vehicle, but did not get any response. Then he wrote an email to the
President of XMC Pvt. Ltd. and got the reply that his grievances will be looked into by the sales team of the
Company and the concerned dealer.
Despite the assurance of the president of XMC Pvt. Ltd., the booked vehicle was not delivered to Mr. Nazir.
Rather, through M/s Ratan Lal & Sons, he was informed that due to delay in production of the said model,
the Company is not able to deliver the same and he was asked to wait for some more time. Subsequently, he
received a letter from XMC Pvt. Ltd wherein, inter alia, it was informed that due to unprecedented number of
bookings for the said model the delivery of the car will be delayed for two months. Through the said letter, it
was also informed that the price of the booked car will be revised and it will be eff ective from the date of
booking by dealer to the customer.
About the market and the state of competition
As per Mr. Nazir, XMC Pvt. Ltd. is a big player in the car manufacturing market. Its financial strength and
brand name is much more compared to other players in the market. Also, it commands largest market share
in terms of sales and revenue compared to its competitors and in the last financial year XMC Pvt. Ltd. acquired
a loss making car manufacturing company i.e. Trisha Ventures Pvt. Ltd. As per Mr. Nazir, XMC Pvt. Ltd. has
taken recourse to terms and conditions of the booking documents to enforce price hike and also not honouring
the commitment made for the delivery within the given time period despite repeated correspondence. XMC
Pvt. Ltd. and its dealer at Mumbai M/s Ratan Lal & Sons started the gimmick of non-delivery due to production
delay and started informing that there will be higher price of the vehicle. Mr. Nazir alleged that he and other
similarly situated consumers are being not given with delivery of the vehicle in due time and the delay tactics
done by XMC Pvt. Ltd. is to increase the price of the vehicle and to exploit the consumers by not giving the
benefit of initial launch price which is not fair in a competitive market.

© The Institute of Chartered Accountants of India


Concerns raised
As per Mr. Nazir, XMC Pvt. Ltd. has abused its powers to fix the price of the vehicle. It has initially priced
attractively and launched with heavy advertisements and promotions to lure the customers and take maximum
bookings by taking interest free amount of Rs. 100,000/- as booking amount. By doing this XMC Pvt. Ltd. has
been able to not only generate huge amounts of cash which is interest free but also create buzz in the market
because of publicity in the media regarding heavy bookings of the said vehicle. It is stated that XMC Pvt. Ltd.
has arbitrarily increased the price of the vehicle to encash on the market demand. Not only that, the Company
has also not passed on the benefit of recent GST reduction on the passenger cars by Government to the
consumers in the said car model. However, it has passed on the benefit of the GST reduction on its other car
models to the customers which are not in such demand. Most of the other car manufacturers have duly passed
on the GST reduction to the customers. As per Mr. Nazir, XMC Pvt. Ltd. has indulged in unfair practices in
connivance with its dealers by manipulating its delivery policy and price policy. After seeing huge response
because of attractive initial offer price, it not only delayed in giving delivery of t he booked car but also
increased the price which is nearly two times of the offer price at the time of booking. It has not honored the
commitment of delivery and price to the buyer who had booked on the very first day and first hour of the
launch. Mr. Nazir stated that it is not just an individual issue but it involves the larger interests of car buyers,
who do not have any recourse to effective mechanism against the abuse of dominant position by such auto
manufacturers for imposing anti-competitive terms on the buyers.
Based on the above submissions Mr. Nazir alleged that the aforesaid conduct of XMC Pvt. Ltd. is not in
tandem with the provisions of the Competition Act, 2002 and it has acted in a manner which can be termed
as anti-competitive.
I. Objective Type Questions (2 marks each)
Select the most appropriate answer from the options given for each question:
1. Which of the following is the appropriate authority for redressal of the grievances of Mr. Nazir?
(a) District Consumer Redressal Forum
(b) Competition Commission of India
(c) Car Manufacturers Association of India
(d) Both (a) and (b)
2. Under which provisions of the Competition Act, 2002, the grievances of Mr. Nazir can be examined?
(a) Prohibition of horizontal anti-competitive agreement under section 3(3) of the Competition Act, 2002
(b) Prohibition of abuse of dominant position under section 4 of the Competition Act, 2002
(c) Prohibition of vertical anti-competitive agreement under section 3(4) of the Competition Act, 2002
(d) Regulation of combination under section 6 of the Competition Act, 2002
3. Mr. Nazir stated that “it is not just an individual issue but it involves the larger interests of car buyers, who do
not have any recourse to effective mechanism against the abuse of dominant position by such auto
manufacturers for imposing anti-competitive terms on the buyers”. What would be his prime intention in stating
so?
(a) The car manufacturer’s conduct towards him is exploitative
(b) The car manufacturer is imposing anti-competitive terms on him.
(c) The conduct of car manufacturer is not conducive to the market as it affects larger consumers’ interest.
(d) All the above
2

© The Institute of Chartered Accountants of India


4. Let, Mr. Nazir approached the Competition Commission India for his grievances and you are the person in the
Commission to take a decision in the matter and according to you the matter pertains to abuse of dominance.
What would be your sequence of analysis of the matter?
(a) XMC Pvt. Ltd. is dominant or not
(b) Whether the alleged conduct is abusive under section 4 of the Competition Act, 2002
(c) Whether XMC Pvt. Ltd. falls under the definition of enterprise as defined under the Competition Act,
2002
(d) Define the relevant market where XMC Pvt. Ltd. is operating
5. Let Mr. Nazir approached the Competition Commission India for his grievances and you are the person in the
Commission to take a decision in the matter and according to you the matter pertains to vertical restraint under
section 3(4) of the Competition Act, 2002. What would be your sequence of analysis of the matter?
(a) Whether XMC Pvt. Ltd. and M/s Ratan Lal & Sons have entered into an agreement
(b) Whether XMC Pvt. Ltd. and M/s Ratan Lal & Sons are placed at vertical level.
(c) Whether there is any appreciable adverse effect on competition because of anti-competitive agreement
between XMC Pvt. Ltd. and M/s Ratan Lal & Sons.
(d) Whether XMC Pvt. Ltd. and M/s Ratan Lal & Sons have agreed on some issues which are anti-
competitive in terms of section 3(4) of the Competition Act, 2002.
6. If you think delineation of relevant market is necessary to examine the fact of the case, then what should be
the relevant product market in this case?
(a) Market for passenger car
(b) Market for dealership services for passenger car
(c) Market for motor vehicle
(d) Market for non-commercial passenger car
7. Mr. Nazir submitted that XMC Pvt. Ltd. is a dominant market player in the relevant market, if you agree with
his submission, what would be your reasoning?
(a) Market share of XMC Pvt. Ltd. is largest
(b) Competitors of XMC Pvt. Ltd. have lesser financial strength
(c) XMC Pvt. Ltd. is a known brand
(d) Consumers are dependent on XMC Pvt. Ltd.
8. Given the facts that XMC Pvt. Ltd. and M/s Ratan Lal & Sons, in connivance with each other, have delayed
the delivery of the booked passenger car to Mr. Nazir and revised the price of the said car, it cannot be a case
of cartelization. What would be the possible reason?
(a) The fact does not reveal any exclusive agreement between XMC Pvt. Ltd. and M/s Ratan Lal & Sons.
(b) The fact does not reveal any agreement of XMC Pvt. Ltd. with other car manufacture in fixing the price
(c) The fact does not reveal that M/s Ratan Lal & Sons is involved in price fixation of delay in giving delivery
of the car to Mr. Nazir
(d) None of the above

© The Institute of Chartered Accountants of India


9. In case the Competition Commission of India ordered that Mr. Nazir should approach in the appropriate forum,
what would be your reaction?
(a) The Competition Commission of India is rightly ordered so because the allegations of Mr. Nazir do not
raise any competition concerns in any market.
(b) The order of the Competition Commission of India should be challenged in National Company Law
Appellate Tribunal as it failed to address the concerns of Mr. Nazir in terms of the provisions of
Competition Act, 2002.
(c) Since it is grievance of an individual consumer, Consumer Redressal Forum is the appropriate authority
to deal this matter.
(d) None of the above
10. If you think that XMC Pvt. Ltd. has abused its dominant position, then which of the following conduct of XMC
Pvt. Ltd. is abusive in terms of Section 4 of the Competition Act, 2002?
(a) not giving delivery of the booked car within the assured time
(b) The President of XMC Pvt. Ltd. vide its mail to Mr. Nazir informed that the price of the booked vehicle
will revised and it will be applicable on the date of invoice by dealer to the customer
(c) XMC Pvt. Ltd has not passed the benefit of tax deduction to the consumers
(d) None of the above.
II. Descriptive Questions (10 marks each)
1. Do you think that the concerns raised by Mr. Nazir can be examined through the provisions of the
Competition Act, 2002? If yes, explain the steps through which the matter can be examined.
2. Do you think that not giving delivery of the booked car within the assured time without enabling
provisions in the booking form is tantamount to imposition of unfair conditions and revision of price of
the vehicle with effect from the date booking tantamount to imposition of unfair price on Mr. Nazir?
Examine the given situations in terms of the provisions of the Competition Act, 2002.
3. What is relevant market? State the provisions of the Competition Act, 2002 to delineate the relevant
market. Delineate the relevant market in the instant case.

© The Institute of Chartered Accountants of India


ELECTIVE PAPER 6D: ECONOMIC LAWS

SOLUTION

Case Study 7

I. ANSWERS TO OBJECTIVE TYPE QUESTIONS


1. (a) [Hint: The matter relates to concerns of an individual consumer regarding non-delivery of
booked vehicle in the given time]
2. (b) [Hint: The allegations essentially relate to abuse of dominance by a car manufacturing
company, directly or through its authorized dealer]
3. (c) [Hint: The tried to portray his issue as an issue of consumer exploitation to draw the attention
of the competition authority]
4. c, d, a, b [Hint: To examine a case under section 4 of the Competition Act, 2002, it is to be seen
first whether the alleged entity is an enterprise or not before defining the relevant market,
assessment of its position of dominance in the relevant market and examination of its conduct]
5. b, a, d, c [Hint: To examine a case under section 3 (4) of the Competition Act, 2002, first it is to be
seen whether the alleged two entities are in a vertical chain and whether they have entered into
any agreement as defined under the Competition Act, 2002. Then it is to seen whether such
agreement is anti-competitive and it has appreciable adverse effect on competition]
6. (a) [Hint: Essentially, the allegations relate to passenger car market]
7. (d) [Hint: All are the factors prescribed under section 19 (6) the Competition Act, 2002 to assess
dominance of an enterprise in a relevant market]
8. (b) [Hint: Cartelization requires agreement amongst players placed at horizontal level ]
9. (a) [Hint: No competition concerns raised in the matter as delay in giving delivery to a consumer
or not passing the benefit of tax reduction to consumer or increasing the price cannot said to
be anti-competitive]
10. (d) [ Hint: Refer section 4 of the Competition Act, 2002]
II. ANSWERS TO DESCRIPTIVE QUESTIONS
1. Even though the concerns raised by Mr. Nazir cannot be redressed by the competition authority
as it essentially relates to grievances of an individual consumer of a passenger car manufactured
by XMC Pvt. Ltd, however if the matter is placed before the competition authority it will be examined
in terms of section 4 of the Competition Act, 2002. It is so because the allegations of Mr. Nazir
essentially relate to abuse of dominance by XMC Pvt. Ltd, directly or through its authorized dealer
M/s Ratan Lal & Sons.
To examine the matter under section 4 of the Competition Act, 2002, it is to be seen first whet her
the alleged entity is an enterprise or not before defining the relevant market, assessment of its
position of dominance in the relevant market and examination of its conduct.
Enterprise: Yes, XMC Pvt. Ltd. is an enterprise in terms of Section 2 (h) of the Act.
Relevant Product Market: The market for passenger car [section 2 (t)]
Relevant Geographic Market: whole of India [see section 2 (s)]
Relevant Market: the market for passenger car in India [section 2 (r)]
Assessment of Dominance of XMC Pvt. Ltd.: Appear to be dominant in the market for
passenger car in India as it has highest market share and financial strength besides its brand name
and dependence of the consumer on it.

© The Institute of Chartered Accountants of India


Assessment of the alleged conduct of XMC Pvt. Ltd.: Not appear to be abusive. Delay in giving
delivery of a product to a consumer or not passing the benefit of tax reduction to consumer or
increasing the price cannot said to be anti-competitive in terms of section 4 of the Competition Act,
2002.
2. To examine the matter under section 4 of the Competition Act, 2002, it is to be seen first whether
the alleged entity is an enterprise or not before defining the relevant market, assessment of its
position of dominance in the relevant market and examination of its conduct. (all the steps of
answer no. 1 above to be followed)
Delay in giving delivery of a product to a consumer without enabling provisions in the booing
document may be an issue of breach of contract between two parties. It may not be a case of
imposition of unfair condition in term of the provisions of section 4 of the Competition Act, 2002.
Further, increasing price is a commercial decision of an enterprise which is taken considering the
market demand conditions of the product. If market is competitive then excess price, if any, can be
wiped out in the long run, no intervention of the competition authority is required. However, if the
company raised the price after negotiation with the consumer, it can be challenged in other
appropriate forum.
3. As per section 2(r) of the Act, ‘relevant market’ means the market which may be determined by the
Commission with reference to the relevant product market or the relevant geographic market or
with reference to both the markets. Further, the term ‘relevant product market’ has been defined in
section 2(t) of the Act as a market comprising all those products or services which are regarded as
interchangeable or substitutable by the consumer, by reason of characteristics of the products or
services, their prices and intended use. And, the term ‘relevant geographic market’ has been
defined in section 2(s) of the Act to mean a market comprising the area in which the conditions of
competition for supply of goods or provision of services or demand of goods or services are
distinctly homogenous and can be distinguished from the conditions prevailing in the neighbouring
areas.
In order to determine the ‘relevant product market’, the Commission, in terms of the factors
contained in section 19(7) of the Act, is required to have due regard to all or any of the following
factors viz. physical characteristics or end- use of goods, price of goods or service, consumer
preferences, exclusion of in-house production, existence of specialized producers and
classification of industrial products. Similarly in order to determine the ‘relevant geographic market’,
the Commission, in terms of the factors contained in section 19(6) of the Act, is required to have
due regard to all or any of the following factors viz., regulatory trade barriers, local speci fication
requirements, national procurement policies, adequate distribution facilities, transport costs,
language, consumer preferences and need for secure or regular supplies or rapid after - sales
services.
As stated above, as per the provisions of the Competition Act, 2002 the relevant market comprises
of the relevant product market and relevant geographic market. In the instant matter, the relevant
product market may be considered as the ‘market for passenger car’. It may be noted that the
allegations of Mr. Nazir pertains to purchase and after sale service of a passenger car which cannot
be substitutable with other type of vehicle in terms of price, end use, characteristics, etc. The
relevant geographic market in this matter may be considered as ‘India’ because the condition of
competition in passenger car market in India is homogeneous throughout India. A consumer can
buy a passenger car from any part of India with similar competitive condition. Thus, the market for
passenger car in India may be considered as the relevant market in this case.

© The Institute of Chartered Accountants of India


DISCLAIMER
This Suggested Answers do not constitute the basis for evaluation of the student’s answers in

the examination. The answers are prepared by the Faculty of the Board of Studies with a view

to assist the students in their education. While due care is taken in preparation of the answers,

if any errors or omissions are noticed, the same may be brought to the attention of the Director

of Studies. The Council of the Institute is not in anyway responsible for the correctness or

otherwise of the answers published herein.

Further, in the Elective Papers which are Case Study based, the solutions have been worked

out on the basis of certain assumptions/views derived from the facts given in the question or

language used in the question. It may be possible to work out the solution to the case studies

in a different manner based on the assumption made or view taken.

© The Institute of Chartered Accountants of India


2 FINAL (NEW) EXAMINATION: JANUARY 2021

PAPER 6D: ECONOMIC LAWS


NOTE: The question paper comprises of five case study questions. The candidates are
required to answer any four case study questions out of five.
CASE STUDY - 1
Part-A:
TJSB Sahakari Bank Ltd. (hereinafter called 'Petitioner') has sought the Corporate Insolvency
Resolution Process of Unimetal Castings Ltd. (hereinafter called the 'Corporate Debtor') on
the ground, that the Corporate Debtor committed default in repayment of loan facilities granted
to the Corporate Debtor to the extent of ` 6,38,78,417/- including interest of ` 2,07,95,568/-,
under Section 7 of the Insolvency and Bankruptcy Code, 2016 (hereafter called the 'Code')
read with Rule 4 of the Insolvency and Bankruptcy (Application to Adjudicating Authority)
Rules, 2016.
The Petition reveals that the following credit facilities were sanctioned on 25 -02-2013 to the
Corporate Debtor by SVC Bank consortium wherein the Petitioner Bank is the consortium
member:
Sr. No. Facility By TJSB (Petitioner Bank) By SVC Bank
1. CC Limit 60,00,000 3,90,00,000
2. OBD Limit - 1,00,00,000
3. Term Loan 1 45,50,000 1,63,80,000
4. Term Loan 2 90,00,000 75,55,000
5. Term Loan 3 1,11,50,000 36,90,000
6. Term Loan 4 50,00,000 1,77,81,000
7. Term Loan 5 1,50,00,000 -
TOTAL 5,10,00,000 9,44,06,000
The Petitioner on 04-08-20I5 issued recall notice to the Corporate Debtor under the provisions
of Multistate Co-Operative Societies Act, 2002 and further issued SARFAESI notice on 29-03-
2016.
The Corporate Debtor submitted that:
(a) It is a medium enterprise as defined under the Micro, Small and Medium Enterprises
Development Act, 2006 (MSMED Act').
(b) The declaration of the account of the Corporate Debtor as Non-Performing Asset ('NPA')
w.e.f. 30-06-2015 is illegal, void and non-est as the same is in contravention of
Regulations and Circulars issued by the Government, Reserve Bank of India, etc.

© The Institute of Chartered Accountants of India


PAPER – 6D: ECONOMIC LAWS 3

(c) The claim of ` 6,38,78,417 as claimed in the Petition is not due and payable by the
Corporate Debtor.
(d) The Corporate Debtor being a medium enterprise is statutorily recognised as extremely
important for the national economy and certain rights are provided under section 9 & 10
of MSMED Act.
(e) The Corporate Debtor is entitled to request the consortium members including the
Petitioner herein for restructuring the credit facilities as provided under RBl guidelines
such as "Prudential guideline on restructuring of advances by banks" and "Guidelines for
rehabilitation of sick, micro and small enterprises". The Central government has also
notified the "Framework for revival and rehabilitation- of micro, small and medium
enterprises". Despite the request of the Corporate Debtor in the year 2014 and 201 5 the
Petitioner or any other Financial Institution has not made any attempts to restructure the
facilities granted to the Corporate Debtor.
(f) Consequent to the meeting of the District Level Sick Unit Rehabilitation Committee held
on 15-03-2016 under the chairmanship of the District Collector of Kolhapur and the
meeting convened by the Joint Director of Industries, Pune, the petitioner by necessary
implications agreed to undertake the exercise of getting the requisite Eco- Techno
viability report of the Corporate Debtor in order to assist the eligibility/entitlement for the
purpose of availing the rehabilitation program but the petitioner failed to do that.
(g) The issue of SARFAESI notice dated 29-03-2016 by the petitioner under section 13(2) of
the SARFAESI Act, 2002 shows their high handedness in exploiting its dominant position
vis-a-vis the Corporate Debtor.
The Corporate Debtor further contended that, the claim of the Petitioner is barred under Article
137 of the Limitation Act, 1963 since whilst the date of alleged default was on 30-06-2015 i.e.
the date on which the account was declared as Non-Performing Asset (NPA). The cause of
action (i.e. the actual default) would have arisen much prior to the date of NPA. Hence, the
period of limitation would run starting even prior to 30-06-2015 and since this Petition was filed
on 23-08-2018, this Petition is barred by limitation.
For the above contention of the Corporate Debtor, the Petitioner submitted that the loan was
shown in the balance sheet of the Corporate Debtor which is an acknowledgement of liability
and hence the debt is not barred by limitation. The Corporate Debtor has not disputed the fact
that the loan was shown as a liability in the balance sheet of the Corporate Debtor.
The Adjudicating Authority having satisfied with the fact that the Corporate Debtor defaulted in
making payment towards the liability to the petitioner, ruled that the petition deserves to be
admitted under IBC, 2016.

© The Institute of Chartered Accountants of India


4 FINAL (NEW) EXAMINATION: JANUARY 2021

Another operational creditor, Wonder Bearings Limited, who underwent a corporate insolvency
resolution process which got completed on 15-03-2016, filed a petition under IBC 2016 on
10-05-2017 with regard to its dues from Unimetal Castings amounting to ` 1,50,80,000.
The Petitioner "TJSB Sahakari Bank Limited" seeks your view on the various provisions of the
Insolvency and Bankruptcy Code, 2016 with regard to above matter.
Part-B:
In another independent development various appeals were filed by the appellants/suppliers
against the orders passed by the Hon'ble Competition Appellate Tribunal (hereinafter referred
to as COMPAT) before the Hon'ble Supreme Court of India. The COMPAT, by the said
judgment, has upheld the findings of the Competition Commission of India (for short, CCI) that
the appellants/suppliers of Liquefied Petroleum Gas (LPG) Cylinders to the Indian Oil
Corporation Ltd. (for short, IOCL) had indulged in cartelization, thereby influencing and rigging
the prices, thus, violating the provisions of Section 3(3)(d) of the Competition Act, 2002 (for
short, the Act).
These suppliers have filed the instant appeals on the ground that there was no cartelization
and they have not contravened the provisions of the Act. For the sake of convenience these
suppliers will be referred to as the appellants hereinafter. We may point out at the outset that
all these appellants are manufacturing gas cylinders of a particular specification having
capacity of 14.2 kg which are needed for use by the three oil companies in India, namely,
IOCL, Bharat Petroleum Corporation Ltd. (BPCL) and Hindustan Petroleum Corporation Ltd.
(HPCL) [all are public sector companies]. It is also a matter of record that apart from the
aforesaid three companies there are no other buyers for these cylinders manufactured by the
appellants. Insofar as IOCL is concerned, it is a leading market player in LPG as its market
share is 48%. Thus, in case a particular manufacturer is not able to supply its cylinders to the
aforesaid three companies, there is no other market for these cylinders and it may force that
company to exit from its operations. The technical bid of the subject tender was opened on 3 -
3-2010 and the price bids of 50 qualified bidders were opened on 23-3-2010. According to the
Director General, there was a similar pattern in the bids by all the 50 bidders who submitted
price bids for various States. The bids of a large number of parties were exactly identical or
near to identical for different States. The Director General had observed that there were strong
indications of some sort of agreement and understanding amongst the bidders to manipulate
the process of bidding.
As per the Director Generals report, the process of bidding followed by the IOCL in the tender
was as under: -
• The bidders would submit their quotations with the bid documents.
• The existing bidders, who were existing suppliers, were required to submit the price bids
and technical bids.
• The bidders were to quote for supplies in different States of India in keeping with their
installed capacity.

© The Institute of Chartered Accountants of India


PAPER – 6D: ECONOMIC LAWS 5

• After price bids were opened the bidders were arranged according to the rates in the
categories of L-1, L-2 and L-3.
• The rates for the supplies in different States were approved after negotiations with L-I
bidder. In case the L-1 bidder could not supply a required number of cylinders in a
particular State, the orders of supplies went to L-2 and also L-3 bidder or likewise
depending upon the requirement in that State as per fixed formula provided in the bid
documents.
The Director General after analyzing the bids came to the conclusion that there was not only a
similarity of pattern in the price bids submitted by the 50 bidders for making supply to the
IOCL but the bids of large number of parties were exactly identical or ne ar to identical in
different States. It was also found that bidders, who belonged to same group, might have
submitted identical rates.
The similarity of the rates was found even in case of bidders whose factories and offices were
not located at one and the same place in the States and where they were required to supply
was far off from their factories located in different place.
The D.G. had found further that though the factors like market conditions and small number of
companies were different, there was large scale collusion amongst the bidding parties. He
also arrived at a finding to the effect that the LPG Cylinder Manufacturers had formed an
Association in the name of Indian LPG Cylinders Manufacturers Association and the members
were interacting through this Association and were using the same as a platform. The date for
submitting the bids in the case of the concerned tender was 3-3-2010 and just two days prior
to it, two meetings were held on 1st and 2nd March, 2010 in Hotel Sahara Star in Mumbai. As
many as 19 parties took part and discussed the tender and, in all probability, prices were fixed
there in collusion with each other. The D.G reported that the bidders had agreed for allocation
of territories, e.g., the bidders who quoted the bids for Western India had not generally quoted
for Eastern India and that largely the bidders who quoted the lowest in the group in Northern
India, had not quoted generally in Southern India. The D.G. also concluded that this behaviour
created entry barrier and that there was no accrual of benefits of consumers nor were there
any plus factors like improved production or distribution of the goods or the provision of
services.
Ultimately, the D.G. came to the conclusion that there was a cartel like behaviour on the pa rt
of the bidders and that the factors necessary for the formation of cartel existed in the instant
case. It was also found that, there was certainly a ground to hold concerted action on the part
of the bidders. The D.G. had also noted that the rates quoted for the year 2009-10 and in
years previous to that were also identical in some cases. Thus, he came to the conclusion that
the bids for the year 2010-11 had been manipulated by 50 participating bidders. It was
thereafter that the CCI decided to supply the D.G,s investigation report to the concerned
parties and invite their objections.
The Director General seeks your advice in light of Petition filed before the Hon'ble Supreme
Court of India against the order passed by the Hon'ble Competition Appellate Tribunal.

© The Institute of Chartered Accountants of India


6 FINAL (NEW) EXAMINATION: JANUARY 2021

Answer the following questions:


1.1 TJSB Sahakari Bank would like your views on, which of the following will not be
considered as insolvency resolution costs under the Code:
(A) The amount of any interim finance and the costs incurred in raising such finance;
(B) The fees payable to any person acting as a resolution professional;
(C) Any payment of fees for the services of an insolvency professional to any person
other than the insolvency professional;
(D) Any costs incurred at the expense of the Government to facilitate the insolvency
resolution process. (2 Marks)
1.2 When can the Committee of Creditors of Unimetal Castings Ltd. take the decision to
liquidate the Company?
(A) by simple majority any time during the resolution process but not before the
confirmation of the resolution plan and preparation of the informa tion memorandum;
(B) by 2/3rd majority any time during the resolution process but before the confirmation
of the resolution plan and preparation of the information memorandum;
(C) by 2/3rd majority any time during the resolution process but not before the
confirmation of the resolution plan and preparation of the information memorandum;
(D) by 3/4th majority any time during the resolution process but before the confirmation
of the resolution plan and preparation of the information memorandum. (2 Marks)
1.3 Is Wonder Bearings Ltd. eligible to initiate insolvency resolution process against Unimetal
Castings Ltd.?
(A) Not eligible, since requirement is to have completed the resolution process 24
months preceding the date of application;
(B) Eligible, there is no bar for a company who underwent insolvency resolution process
to initiate proceedings as long as the other requirements (existence of debt etc.)
under IBC 2016 is met;
(C) Eligible, since requirement is to have completed the resolution process 12 months
preceding the date of making of the application;
(D) Not eligible, prior consent of the Adjudicating Authority is required for filing an
application for insolvency process by Wonder Bearings Ltd., since it has itself
undergone an insolvency process. (2 Marks)
1.4 Which of the following is not the objective of the Competition Act, 2002?
(A) Promote practices having adverse effect on Competition;
(B) Sustain competition in market;
(C) Protect the interest of consumers;

© The Institute of Chartered Accountants of India


PAPER – 6D: ECONOMIC LAWS 7

(D) Ensure freedom of trade for Indian and foreign players in markets in India. (2 Marks)
1.5 An Association of manufacturers of die cast products will not be considered as a cartel if
the objective of the association is to :
(A) limit the distribution of die cast material only to petroleum industry in view of the
huge demand and higher realization;
(B) regulate the production of die cast products to ensure optimal sale prices;
(C) represent the industry issues on a collective basis to the Government;
(D) monitor and regulate the number of dealers in each state/city. (2 Marks)
1.6 Analyze and answer the following questions in the context of the case study:
(i) Evaluate the position taken by the Adjudicating Authority that the petition deserves
to be admitted having satisfied with the fact that Unimetal Castings Ltd. has
defaulted in making payment towards the liability to the petitioner. (4 Marks)
(ii) In light of the provisions of the Competition Act 2002, whether there was any
collusive agreement between the participating bidders which directly or indirectly
resulted in bid rigging of the tender floated by IOCL? (3 Marks)
(iii) Unimetal Castings Ltd. seeks your views regarding the impact of the clarifications
issued by Ministry of Corporate Affairs (MCA) regarding approval of resolution plans
under section 30 and 31 of Insolvency & Bankruptcy Code, 2016 vide general
circular (GC) dated 25th October, 2017. (4 Marks)
(iv) TJSB Sahakari Bank Ltd. seeks your advice on the time-limit for completion of the
Corporate Insolvency Resolution Process as per the lBC, 2016. (4 Marks)
ANSWER TO CASE STUDY 1
1.1 (C)
1.2 (B)
1.3 (C)
1.4 (A)
1.5 (C)
1.6 (i)
Given situation is based on the case law, B.K. Educational Services Pvt. Ltd. vs. Parag Gupta
and Associates, of Supreme Court, Civil Appeal No.23988 of 2017, dated 11.10.2018, in which
principle was laid down that the Limitation Act, 1963 is applicable to applications filed under
Sections 7 and 9 of the Insolvency and Bankruptcy Code, 2016(Code) from the inception of
the Code .i.e. from 28th May, 2016.

© The Institute of Chartered Accountants of India


8 FINAL (NEW) EXAMINATION: JANUARY 2021

In the given case study, the Corporate Debtor, Unimetal Castings Ltd. contended as under:
(i) Claim of the Petitioner, TJSB Sahakari Bank Ltd. is barred under Article 137 of the
Limitation Act, 1963.
(ii) Whilst the date of alleged default was 30.6.2015 (i.e. the date on which the account was
declared as Non Performing Assets (NPA)) and the cause of action (i.e. actual default)
arises much prior to the date of NPA.
(iii) The period of limitation would have started even prior to 30.06.2015 and as the petition
was filed on 23.08.2018, so it is barred by limitation.
As per the facts given in the case study, acknowledgement of liability in the Balance Sheet of
the Corporate Debtor reflects that default has already occurred i.e on 30.6.2015 and the
application for initiation of Corporate Insolvency Resolution process was filed on 23.08.2018.
Where the liability is shown in the balance sheet, it is a clear acknowledgement of debt by the
Corporate Debtor as was held in Bajan Singh Sharma Vs Wimpy International Limited,
185(2011) DLT 428 and in many other judgements.
In the light of the aforesaid ruling, the limitation period of 3 years will begin from the date of
coming of Code into enforcement i.e from 28th May 2016.
Therefore, the position taken by the Adjudicating Authority is correct and the petition deserves to
be admitted since the application filed under Section 7 of the Code is within the limitation period
and the Corporate Debtor has defaulted in making payment towards the liability of the petitioner.
1.6(ii)
As per Section 3(3) of the Competition Act, 2002 the identical bid price is not possible unless
there is some sort of prior and collective understanding. Further the contact and meeting
between the members of IOCL and Association, before submission of bids is also valid
evidence of the existence of an understanding among the parties.
In the case study as per given facts, it was found that there was large scale collusion amongst
the bidding parties. LPG Cylinder manufacturers formed an association, Indian LPG Cylinder
manufacturers Association and the members of IOCL were interacting through this
association. Two days before the date of bids i.e. on 1 st& 2nd March, 2010 two meetings were
held and 19 parties took part and discussed the tender and prices were fixed there in collu sion
with each other. This resulted in bid rigging of the tender floated by IOCL.
In view of the above, it can be concluded that there was collusive agreements between the
participating bidders which directly or indirectly resulted in bid rigging of the te nder floated by
IOCL.
1.6 (iii)
Impact of Clarification Issued by MCA:
Vide General Circular IBC/01/2017 dated 25th October, 2017, Ministry of Corporate
Affairs issued a clarification regarding approval of resolution plans under Section 30 and 31 of

© The Institute of Chartered Accountants of India


PAPER – 6D: ECONOMIC LAWS 9

Insolvency and Bankruptcy Code, 2016. The said clarification is sought in view of the
requirement under Section 30(2)(e) of the Code for the resolution professional to confirm that
each resolution plan received by him does not contravene any of the provisions of the law for
the time being in force.
The matter has been examined in the Ministry in the light of provisions of Sections 30 and 31
of the Code which provide a detailed procedure from the time of receipt of resolution plan by
the resolution professional to its approval by the Adjudicating Authority and there is no
requirement for obtaining approval of shareholders/members of the corporate debtor during
this process.
This clarification clears that the requirement of Section 30(2) (e) of the Code is to ensure that
the resolution plan(s) considered and approved by the Committee of Creditors and the
Adjudicating Authority is in compliant with the provisions of the applicable laws and therefore
is legally implementable.
Section 31(1) of the Code further provides that a resolution plan approved by the Adjudicating
Authority shall be binding on the Corporate Debtor and its employees, members, creditors,
guarantors and other stakeholders involved in the resolution plan.
1.6 (iv)
As per Section 12 of the Insolvency and Bankruptcy Code, 2016, the Corporate Insolvency
Resolution Process (CIRP) shall be completed within a period of one hundred and eighty days
from the date of admission of the application to initiate such process.
The Resolution Professional shall file an application to the Adjudicating Authority to extend the
period of the Corporate Insolvency Resolution Process beyond one hundred and eighty days,
if instructed to do so by a resolution passed at a meeting of the Committee of Creditors by a
vote of sixty- six per cent of the voting shares.
On receipt of an application, if the Adjudicating Authority is satisfied that the subject matter of
the case is such that CIRP cannot be completed within one hundred and eighty days, it may
by order extend the duration of such process beyond one hundred and eighty days by such
further period as it thinks fit, but not exceeding ninety days.
Provided that any extension of the period of CIRP shall not be granted more than once.
Accordingly, in the said case, time limit for competition of CIRP, will be:
01 Petition for initiation of corporate insolvency resolution was filed on 23.08.2018
02 Insolvency resolution process will be commenced within 14 days i.e, latest by
01.09.2018
03 Insolvency resolution process will be completed by 180th day from insolvency
commencement date (date of admission of the application) i.e., latest by
28.02.2019.
04 Further may extend till 29.05.2019.

© The Institute of Chartered Accountants of India


10 FINAL (NEW) EXAMINATION: JANUARY 2021

Amendment of Section 12(3) of IBC(Amendment) Act, 2019


Section 12(3) of the IBC was amended by way of the Insolvency and Bankruptcy (Amendment)
Act, 2019 and two provisos were added:
Proviso 1 states that a CIRP must mandatorily be completed within 330 days from the
insolvency commencement date, including any extension of the period of the CIRP granted
and the time taken in legal proceedings in relation to the resolution process.
Proviso 2 states that, when the CIRP of a Corporate Debtor (CD) has been pending for over
330 days, it must be completed within 90 days from the date of the amendment.
Thus, the overall timeline for completing a CIRP now stands at 330 days from the date of
insolvency commencement date.
CASE STUDY - 2
Visio India Private Limited (Visio) is an upcoming watch manufacturing company and is based
in Vishakapatnam. The Company was started during the year 2008 by Mr. Srinivas Kumar and
his wife, Ms. Kruthi who is a Chartered Accountant.
In order to meet their expanding operations in Delhi, Visio had in the month of January 2014
pre-booked a commercial office unit of approximately 1200 sq. ft. with JV Realty Limited, a
leading developer in that area in their "SAPPHIRE COURT" Greater Noida project launched
then by paying an amount of ` 25,00,000 as booking amount (50% of the total consideration)
but no Builder- Buyer agreement was entered into between the parties except that an
allotment letter was issued by the developer mentioning the unit details. This project was
being developed over an area of approximately 15,000 square meters and having over 100
office units in its plan outlay.
Visio had paid almost 90% of the entire cost of the property based upon percentage of
completion (progress) of the stage of construction as of April 2017 but the developer had
failed to provide, neither possession nor had completed the project and was also not
responding to their complaints on one pretext or the other.
The legal counsel of Visio, Mr. Aswin Nakshatra, in the month of May, 2017 informed Ms.
Kruthi about Real Estate (Regulation and Development) Act, 2016 (for short "the RERA"). He
further informed that RERA was enacted by the Parliament as Act 16 of 2016 in the year 2016
and by May 1, 2017, all 92 provisions of the Real Estate (Regulation and Development) Act,
2016 (RERA or the Act) were brought into force. The Act has introduced new obligations on
real estate developers and in cases of default, prescribes penal liabilities and Visio can
contemplate bringing a legal suit against the developers under RERA.
JV Realty on the other hand is of the view that RERA is not applicable to this project as the
same was launched and construction commenced much before the RERA came into force.
One of the group companies of JV Realty, Good Looking Homes Private Limited (GLHPL) was
into construction of high rise apartment complexes and commenced a large project "Kailash
Giri Views" in Vizag.

© The Institute of Chartered Accountants of India


PAPER – 6D: ECONOMIC LAWS 11

GLHPL took all the approvals under RERA and came up with the marketing strategy including
a brochure of 94 pages consisting of various pictures showing the following features included
in the project:
(a) Balcony at each floor
(b) Drawing room to be constructed with designed tiles at floor.
(c) Italic Marble at bedroom.
(d) Granite at kitchen.
(e) Swimming pool at the top floor.
(f) All rooms to be Centrally Air-conditioned.
(g) All floors and lifts will have CCTV camera.
(h) Open parking slot for one car.
(i) Ground covering Net for Cricket and Football.
(j) Handover of the apartments within 36 months from date of agreement.
It was also mentioned in the marketing brochure that the building will have 9 floors with
elevators and stair case and the total number of flats to be constructed would be 218 as
approved by RERA.
For the purpose of various projects, JV Realty had obtained several loans from banks and
financial institutions and there were certain allegations that some of the loan funds were
siphoned off by the promoters of JV Realty for other purposes. In five different cases, banks
and financial institutions had granted credit facilities against hypothecation/charge over certain
assets. In each of these cases, JV Realty was charged under certain provisions of the PMLA
(for offences under paragraph 2 of Part A of the Schedule) and orders were passed for
attachment of properties charged to banks and financial institutions affecting their vested
rights under other statutes such as Recovery of Debts and Bankruptcy Act (RDBA),
Securitization and Reconstruction of Financial Assets and Enforcement of Securities Interest
Act (SARFAESI) and Insolvency and Bankruptcy Code (lBC).
The Adjudicating Authority is of the view that:
• The provisions of PMLA prevail over RDBA, SARFAESI and IBC
• It is not only a "tainted property" that is to say a property acquired or obtained, directly or
indirectly, from proceeds of criminal, activity constituting a scheduled offence which can
be attached, but also any other asset or property of equivalent value of the offender of
money-laundering which has a link or nexus with the offence (or offender) of money -
laundering.

© The Institute of Chartered Accountants of India


12 FINAL (NEW) EXAMINATION: JANUARY 2021

• If the "tainted property" is not traceable, or cannot be reached, or to the extent found is
deficient, any other asset of the person accused or charged under PMLA can be attached
provided it is near or equivalent in value, the order of confiscation being restricted to take
over by the government of illicit gains of crime.
• An order of attachment under PMLA is not illegal only because a secured creditor has a
prior secured interest in the property, within the meaning of the expressions used in
RDBA and SARFAESI. Similarly, mere issuance of an order of attachment under PMLA
does not render illegal a prior charge of a secured creditor, the claim of the latter for
release from PM LA attachment being dependent on its bonafides.
• In case of secured creditor pursuing enforcement of "security interest" in the property
sought to be attached under PMLA, such secured creditor having initiated action for
enforcement prior to the order of attachment under PMLA, the directions of such
attachment under PMLA shall be valid and operative subject to satisfaction of the charge
of such third party and restricted to such part of the value of the property as is in ex cess
of the claim of the said third party.
• If the order confirming the attachment has attained finality or if the order of confiscation
has been passed or if the trial of a case under Section 4 of the PMLA has commenced,
the claim of a party asserting to have acted bonafide or having legitimate interest in the
nature mentioned above will be inquired into and adjudicated upon only by the special
court.
During the course of Kailash Giri View project, it was observed that whilst the construction was
for 9 floors, the total flats constructed were 225 due to efficient realignment of the blocks and
square feet area of the individual apartments. It was also observed that due to unavoidable
reasons, the swimming pool could only be made at the ground floor only and would be allowed
to those occupants only who will specifically pay for the swimming pool facility. On completion
of 34 months, GLHPL sent an email to all allottees that due to unforeseen circumstances the
project is getting delayed by 6 months as the structure is almost complete and the work
related to interior, plastering, plumbing etc., will be completed very soon.
Answer the following questions:
2.1 Which of the following is not a condition to be fulfilled for attachment of the property of JV
Realty Ltd. for alleged offences under the PMLA?
(A) Approval of the Special Court for the attachment;
(B) Submission of report to a Magistrate under Section 173 of the Code of Criminal
Procedures;
(C) Filing of complaint for taking cognizance of the scheduled offence;
(D) None of the options. (2 Marks)

© The Institute of Chartered Accountants of India


PAPER – 6D: ECONOMIC LAWS 13

2.2 What is the punishment which Mr. Bala Ganesh, the Managing Director of JV Realty Ltd.
is liable for under the PMLA ?
(A) No punishment, since the offence was not performed personally by him;
(B) Minimum of 3 years and maximum of 7 years, with fine;
(C) Minimum of 3 years and maximum of 10 years, without fine;
(D) Minimum of 3 years and maximum of 10 years, with fine. (2 Marks)
2.3 On receipt of a complaint under the PMLA, if the Adjudicating Authority has reasons to
believe that JV Realty Ltd. has committed an offence under section 3 or is in possession
of proceeds of crime, it may serve notice within not less than ---------------days calling
upon them to indicate the source of their income, earnings or assets etc.
(A) 15 days
(B) 60 days
(C) 30 days
(D) 7 days (2 Marks)
2.4 JV Realty Ltd. has decided to charge an amount of ` 5,00,000 on Visio for an open car
parking. Ms. Kruthi is of the view that JV Realty Ltd. cannot charge this amount since this
is not mentioned in the original agreement.
(A) Yes, this cannot be charged since this is not mentioned in the original agreement
between Visio and JV Realty Ltd.;
(B) Yes, this cannot be charged since JV Realty Ltd. cannot charge for open car
parking under RERA;
(C) No, this can be charged since the requirement for non-charging for open car park
under RERA is only for residential complexes and is not applicable for commercial
office space;
(D) This is purely based on mutual agreement between both parties. (2 Marks)
2.5 As per RERA, what is the maximum amount of advance or application fee which can be
collected by GLHPL from its customers?
(A) 15% of cost of apartment on entering into a Sale Deed;
(B) 10% of cost of apartment on entering into a written agreement to sell;
(C) 10% of cost of apartment on entering into a Sale Deed which is duly registered;
(D) 10% of cost of apartment on entering into a written agreement to sell which is duly
registered. (2 Marks)
2.6 Analyze and answer the following questions in the context of the case study:
(i) In the light of the given case study, evaluate if Visio can initiate legal proceedings

© The Institute of Chartered Accountants of India


14 FINAL (NEW) EXAMINATION: JANUARY 2021

against JV Realty Ltd. for their resultant rights towards delay in completion or
whether the contention of the developer that RERA is not applicable to the Project
is correct. (4 Marks)
(ii) Discuss the provisions of powers of Director to impose fine under the PMLA,
2002. (5 Marks)
(iii) Based on the provisions of the PMLA, analyse with reasons, the contentions of
the Adjudicating Authority with regard to the following:
(a) Whether the provisions of RDBA, SARFAESI and IBC prevail over PMLA ?
(b) Whether interest created in a property prior to event of money laundering
leading up to the attachment of property, takes priority over the attachment?
(c) Whether a mere nexus between the attached property where it did not qualify
as "proceeds of crime" under the PMLA and the party accused of money
laundering was sufficient for the attachment to take place? (6 Marks)
ANSWERS TO CASE STUDY 2
2.1 (A)
2.2 (B)
2.3 (C)
2.4 (B)
2.5 (D)
2.6(i)
Whether Visio can initiate proceedings against JV Reality ?
According to Section 18 of the Real Estate (Regulation and Development) Act, 2016 (the Act),
(1) If the promoter fails to complete or is unable to give possession of an apartment, plot
or building,—
(a) in accordance with the terms of the agreement for sale or, as the case may be, duly
completed by the date specified therein; or
(b) due to discontinuance of his business as a developer on account of suspension or
revocation of the registration under this Act or for any other reason,
he shall be liable on demand to the allottees, in case the allottee wishes to withdraw from
the project, without prejudice to any other remedy available, to return the amount
received by him in respect of that apartment, plot, building, as the case may be, with
interest at such rate as may be prescribed in this behalf including compensation in the
manner as provided under this Act.

© The Institute of Chartered Accountants of India


PAPER – 6D: ECONOMIC LAWS 15

However, where an allottee does not intend to withdraw from the project, he shall be
paid, by the promoter, interest for every month of delay, till the handing over of the
possession, at such rate as may be prescribed.
(2) If the promoter fails to discharge any other obligations imposed on him under this Act or
the rules or regulations made thereunder or in accordance with the terms and conditions
of the agreement for sale, he shall be liable to pay such compensation to the allottees, in
the manner as provided under this Act.
In the case study, Visio had paid almost 90% of the entire cost of the property. The developer
had failed to provide neither possession nor had completed the project.
Hence, Visio can initiate legal proceeding against JV Realty Ltd.
Whether the contention of the developer that RERA is not applicable to the project is
correct ?
As per Section 3 of the Act, RERA applies to projects that are ongoing on the date of
commencement of the Act and completion certificate has not been issued within a period of
three months from the date of commencement of the Act. In the given case study, completion
certificate of the project was not granted till April 2017 (even after RERA was formulated).
Hence, the contention of JV Realty Ltd. that RERA is not applicable to them, is incorrect.
2.6 (ii)
Powers of director to impose fine under the PMLA, 2002
Section 13 of the Prevention of Money Laundering Act, 2002, deals with the powers of the
Director to impose fine, which is as follows:
1. Inquiry from Director: The Director may, either of his own motion or on an
application made by any authority, officer or person, may make such inquiry or cause
such inquiry to be made, with regard to the obligations of the reporting entity.
2. Audit of records on direction of director: If at any stage of inquiry or any other
proceedings before him, the Director having regard to the nature and complexity of
the case, is of the opinion that it is necessary to do so, he may direct the concerned
reporting entity to get its records, audited by an accountant (i.e. Chartered
Accountant) from amongst a panel of accountants, maintained by the Central
Government for this purpose.
3. Bearing of expenses: The expenses of, and incidental to, any audit specified above
shall be borne by the Central Government.
4. Failure in compliance with the obligations: If the Director, in the course of any
inquiry, finds that a reporting entity or its designated director on the Board or any of
its employees has failed to comply with the obligations, then, he may-

© The Institute of Chartered Accountants of India


16 FINAL (NEW) EXAMINATION: JANUARY 2021

(a) issue a warning in writing; or


(b) direct such reporting entity or its designated director on the Board or any of its
employees, to comply with specific instructions; or
(c) direct such reporting entity or its designated director on the Board or any of its
employees, to send reports at such interval as may be prescribed on the measures
it is taking; or
(d) by an order, impose a monetary penalty on such reporting entity or its designated
director on the Board or any of its employees, which shall not be less than ten
thousand rupees but may extend to one lakh rupees for each failure.
5. Forwarding of copy of order: The Director shall forward a copy of the order passed
above to every banking company, financial institution or intermediary or person who
is a party to the proceedings.
2.6 (iii)
(a) Whether the provisions of RDBA, SARFAESI and IBC prevail over PMLA?
Section 71 of Prevention of Money Laundering Act, 2002, which deals with the overriding
effect of the act, provides that the provisions of this Act shall have effect notwithstanding
anything inconsistent therewith contained in any other law for the time being in force.
Thus, it can be construed that the provisions of Prevention of Money Laundering Act, 2002,
shall prevail over RBDA, SARFAESI and IBC, till the time nothing inconsistent therewith is
contained in any other law for the time being in force.
(b) Whether interest created in a property prior to attachment of property, takes
priority over attachment?
As per Section 5(4) of the Prevention of Money Laundering Act, 2002, nothing in this section
shall prevent the person interested in the enjoyment of the immovable property attached under
Section 5(1) from such enjoyment.
“Person interested”, in relation to any immovable property, includes all persons claiming or
entitled to claim any interest in the property.
Accordingly, an order of attachment under money laundering Act is not said to be illegal
merely because a person interested (i.e., third party) had a prior interest in such property and
further issuance of an order of attachment under PML Act cannot, by itself, render illegal the
prior statutory right of a person interested in attached property.
Therefore, interest created in a property prior to attachment of property, takes priority over
attachment.

© The Institute of Chartered Accountants of India


PAPER – 6D: ECONOMIC LAWS 17

(c) Whether mere nexus between the attached property whether it qualify as a
proceeds of crime and the party accused of money laundering, is sufficient for the
attachment of property?
According to Section 5 of the Prevention of Money Laundering Act, 2002, where the Director
or any other officer for the purposes of this section, has reason to believe, on the basis of
material in his possession, that—
(a) any person is in possession of any proceeds of crime; and
(b) such proceeds of crime are likely to be concealed, transferred or dealt with in any
manner which may result in frustrating any proceedings relating to confiscation of
such proceeds of crime under this Chapter,
he may, by order in writing, provisionally attach such property for a period not exceeding one
hundred and eighty days from the date of the order, in such manner as may be prescribed.
Hence, it is necessary that the attached property should qualify as ‘proceeds of crime’.
However, mere nexus between the attached property whether it qualify as a proceeds of
crime/not, the party accused of money laundering, is sufficient for the attachment of such
property to take place.
CASE STUDY - 3
The eastern part of India is very well known for the production of tea and it is exported world
over. However, the large amount of pendency of the payments by tea mills to tea produc ers
has been a cause of worry and it was decided that a common platform is an essential
requirement to provide solution to this problem. Accordingly, the Eastern Produce Co -
operative Society was formed to ensure the timely collection of sale proceeds from mills. The
Society developed a charter, in form of memorandum for its members, to regulate and control
supply, price, term of sales collection of sale proceeds and recovery if required. This
memorandum is binding on all the members of the Society.
The Society extends the support to growers, by giving them offer to sell their entire farm
produce to Society at mutually agreed price; which the Society will further sale to mills. But the
farmers who avail this facility have to necessarily sell the entire farm produce to the Society,
and the farmer cannot sell any portion of his farm produce directly in the open market.
Further, in order to trade with the mills, deal with regulatory authorities, and financial
institution, the Society decided to promote a Company named Eastern Limited. The extracts
from latest audited financial statements of Eastern Limited are as follows;
Sr. No. Particular Amount (in ` crore)
1. Proceed (Net of taxes) from sale 3,500
2. Operating assets 700
3. Paid-up share capital 490
4. Net profit 100

© The Institute of Chartered Accountants of India


18 FINAL (NEW) EXAMINATION: JANUARY 2021

With passage of time, Eastern Limited became the big hit, for the role it plays as an
intermediary and in incredible transformation in process of sale of tea by farmers.
Mr. Gaurav who is CEO of Eastern limited, heard about forward integration as method of
expansion and growth strategy. Mr. Gaurav prepared a proposal, which was duly approved by
Board of Directors and then by the members of Eastern Limited company to takeover Eastern
Tea Limited, by acquiring controlling stake from open market. Eastern Tea Limited is in the
business of running tea mills, with a global presence. Mr. Gaurav's wife, Ms. Sheetal, was
residing in Singapore and Mr. Gaurav wanted to send an amount of USD 20,000 per month to
her for her maintenance. However, the CFO of Eastern Limited mentioned to him that this is
not in accordance with FEMA.
Around 60% of sales by Eastern Tea Limited constitute exports of tea majorly to Iran. One
year back, Eastern Tea Limited opened one branch office in Iran, as Iran starts buying tea
from India, in order to settle trade balance; because Iran is blocked from the global financial
system; including using U.S. dollars to transact its oil sales. On such branch office, during last
financial year, an amount of ` 150 crore were incurred as expenditure for the Branch through
the EEFC account maintained by Eastern Tea Limited.
For last financial year, the turnover of Eastern Tea Limited was recorded at ` 1,200 crore,
which was ` 110 crore more than year earlier to last financial year; whereas operating assets
as on reporting date were ` 280 crore. The paid- up share capital was ` 130 crore. After
acquisition both the entities were not merged, and both kept their resp ective separate identity.
For the purpose of enhancing its global sales, Eastern Tea Limited decided to pay commission
for exports of tea under the Rupee State Credit Route at 6% of invoice value. Further, Eastern
Tea also decided to send a gift hamper to its 20 top distributors totaling to a value of USD
1,00,000 (INR 70 lakh).
Eastern Tea Limited has strong domestic Network or tie-up with retail shops and stores
through which they sale their tea under brand name 'leaf’ which constitute around 40% of sale.
Such retail shops and stores are provided with instruction not to charge the price more then
what is suggested by Eastern Tea Limited although lower prices can be charged and specific
jurisdiction is given to each retailer for resale.
According to Mr. Saurabh, who is head of marketing at Eastern Limited, also now look after
marketing at Eastern Tea Limited, in order to acquire substantial market share (in term of new
customers), Eastern Tea Limited has to sell tea at the prices lower than cost. Ignoring the
resistance from the governing body of Eastern Tea Limited, the new pricing policy
implemented. Resultantly price decreased from ` 150 per kilogram to ` 130 per kilogram. But
in order to restrict loss, on account of selling tea at price lower than cost, Eastern Tea Limited
asked all the shopkeepers and stores, not to sell more than 5 kg of leaf tea to a customer.
The Eastern Produce Co-operative Society promoted another company named South Limited,
whose object clause includes; provide weather research and forecast reports, other necessary

© The Institute of Chartered Accountants of India


PAPER – 6D: ECONOMIC LAWS 19

technical knowledge or guidance to members of parent's society apart from co nducting market
research for Eastern Limited.
In one market research conducted by South Limited, it was found that North Limited, which
holds major market share (around 30%) in retail packed tea under brand name 'Taste' (Price
of which is ` 150 per kilogram). For latest financial year, the turnover of North Limited is
recorded at 3,000 crore whereas operating assets are of ` 570 crore and paid-up share
capital is `365 crore. Since acquisition of Eastern Tea limited by Eastern Limited, remains
largely successful, hence showing trust in un-organic growth, a bear-hug letter was sent to
senior management of North Limited.
Since North Limited is already undisputed market leader, they refuse the bear hug offer.
Eastern Limited with help of South Limited performs a hostile acquisition and both the
companies acquire around 25.5% stake in voting rights each; by tender notice over the stock
exchange. Post acquisitions of North Limited, Eastern Limited got the dominance over the
market. Hence Eastern Limited decided to re-price their product which is renamed also 'Taste
leaf’ with a new price of ` 155 per Kilogram and to support the price rise, Eastern Limited also
started restricting supply in the end market.
Eastern Limited also entered in memorandum of understanding with West offshore Limited,
which is $ 21 million (assets base) company for transfer of technology.
Answer the following questions:
3.1 The CFO of Eastern Tea Limited seeks your views on whether the gifts sent to
distributors requires to be included in the export declaration
(A) Yes, since the aggregate value of the gifts to all distributors is more than the
prescribed limit;
(B) No, since no amounts are received from the distributors for the products;
(C) No, 'since the individual value of the gifts to each distributor is less than the
prescribed limit;
(D) Yes, any item exported should be included in the declaration, unless it will be
returned back to India. (2 Marks)
3.2 How much can Gaurav remit to his wife living in Singapore for her maintenance every
year?
(A) A maximum of USD 2,50,000 with the approval of the Reserve Bank of India;
(B) A maximum of USD 2,50,000 under the liberalized remittance scheme;
(C) Any amount subject to the approval of the Central Government;
(D) Any amount subject as long as he can prove that the amount has been earned by
him legitimately in India. (2 Marks)

© The Institute of Chartered Accountants of India


20 FINAL (NEW) EXAMINATION: JANUARY 2021

3.3. Evaluate if the commission paid by Eastern Tea Limited is in accordance with FEMA
(A) Yes, it is a current account transaction and can be freely remitted;
(B) Yes, it is below the limit of 10% of invoice of exports of tea under Rupee State
Credit Route;
(C) No, it cannot be remitted until and unless the export proceeds are received;
(D) No, payment of commission for such exports is prohibited. (2 Marks)
3.4 Takeover (acquisition) of Eastern Tea limited by Eastern limited, will b e considered as
combination if
(A) Assets of enterprise created after merger is equal to ` 2,000 crore;
(B) Turnover of enterprise created after merger is equal to ` 6,000 crore;
(C) Turnover of enterprise created after merger is more than ` 6,000 crore;
(D) Assets of enterprise created after merger is more than ` 6,000 crore. (2 Marks)
3.5 The decision of Eastern Tea Limited not to sell more than 5 kg of tea per customer
purchase can be categorized as
(A) Exclusive supply agreement;
(B) Exclusive distribution agreement;
(C) Refusal to deal;
(D) None of the options (2 Marks)
3.6 Analyze and answer the following questions in the context of the case study:
(i) In your view, explain with reasons if Eastern Produce Co-operative Society can be
considered as 'Cartel'? (3 Marks)
(ii) Does Eastern Limited hold dominance over the market, and if yes iden tify the
circumstances where it abuses its dominant position? (4 Marks)
(iii) Explain briefly the applicability of the Competition Act to the combinations
described in the case study and the regulatory aspects thereof. (5 Marks)
(iv) Evaluate if Eastern Limited is in compliance with the provisions of FEMA with
regard to the expenditure incurred for maintaining a branch a broad. (3 Marks)
ANSWERS TO CASE STUDY 3
3.1 (A)
3.2 (B)
3.3 (B)
3.4 (C)

© The Institute of Chartered Accountants of India


PAPER – 6D: ECONOMIC LAWS 21

3.5 (D)
3.6(I)
Whether Eastern Produce Co-operative Society can be considered as ‘Cartel’ ?
As per Section 2 (c) of the Competition Act 2002, the term “cartel” includes an association of
producers, sellers, distributors, traders or service providers who, by agreement amongst
themselves, limit, control or attempt to control the production, distribution, sale or price of, or,
trade in goods or provision of services.
From the above, it may be noted that the term ‘cartel’ has been given inclusive meaning.
Although, Eastern Produce Cooperative Society was formed to ensure the timely collection of
sale proceeds from mills, it also developed a charter, in the form of a memorandum for its
members, to regulate and control the supply, price, term of sale, collection of sale proceeds
and also recovery, if required. This charter, in the form of a memorandum, was binding on all
the members of the Society.
Hence, Eastern Produce Cooperative Society is a ‘Cartel’ within the meaning of Section 2 (c)
of the Competition Act, 2002.
3.6(ii)
Whether Eastern Limited holds dominance over the market?
Yes, Eastern Limited holds dominance over the market because as per Explanation (a) to
Section 4 of the Competition Act, 2002, “dominant position” means a position of strength,
enjoyed by an enterprise, in the relevant market, in India, which enables it to (i) operate
independently of competitive forces prevailing in the relevant market; or (ii) affect its
competitors or consumers or the relevant market in its favour.
Circumstances where dominant position is abused
(a) Predatory Pricing after the acquisition of Eastern Tea Limited – Eastern Limited
acquired a substantial network of the retailers after the takeover of Eastern Tea Limited and
due to such takeover, it tried to penetrate the market using predatory pricing [refer Section
4(2)(a)(ii) of the Competition Act, 2002]. Eastern Tea Limited reduced the price of the leaf tea
from ` 150 to ` 130 per kilogram which was lower than the cost incurred, whereas other
players in the market like North Limited were selling leaf tea at ` 150 per kilogram.
As per Explanation (b) to Section 4 of the Competition Act, 2002, the term “predatory price”
means the sale of goods or provision of services, at a price which is below the cost, as may be
determined by regulations, of production of the goods or provision of services, with a view to
reduce competition or eliminate the competitors.
(b) Increasing the price after the acquisition of North Limited – After the hostile
acquisition of North Limited by Eastern Limited with the help of another group company South
Limited, Eastern Limited raised the price of its tea leaf ‘Taste leaf’ from ` 130 to ` 155 per
kilogram, even though North Limited was originally selling its tea leaf ‘Taste’ at ` 150 per

© The Institute of Chartered Accountants of India


22 FINAL (NEW) EXAMINATION: JANUARY 2021

kilogram. According to Section 4 (2) (b) (i) of the Competition Act, 2002, there shall be an
abuse of dominant position under Section 4 (1), if an enterprise or a group limits or restricts
the production of goods or market therefor through unfair or discriminatory price.
(c) Cap on quantity
In order to restrict loss, on account of selling tea at price lower than cost, Eastern Tea Limited
asked all the shopkeepers and stores, not to sell more than 5 kg of leaf tea to a customer.
That would also be considered as abuse of dominance.
3.6(iii)
Provisions of the Competition Act, 2002 to “Combination”
In the context of Eastern Limited, the regulatory aspects of ‘combination’ as mentioned in
Section 5 of the Competition Act, 2002 are given as under:
Sr. Nature of Facts of the Criteria for Whether
No Combination case considering ‘Combination’ or not
‘Combination’
1 Acquisition by Eastern Limited Joint Asset over No. It is not a
single acquirer acquired ` 2,000 crores or combination.
but different Eastern Tea Turnover over Hint: Joint turnover is
goods [Section 5 Limited ` 6,000 crores ` 4,700crores
(a) (i) (A)] (3,500+1,200) which
is less than ` 6,000
crores. The joint
assets base of ` 980
crores (700+280)
which is less than
` 2,000 crores.
2 Acquisition by a Eastern Limited Group assets over No. It is not a
group with acquired North ` 8,000 crores or combination.
similar goods Limited with the turnover over Hint: Joint asset base
[Section 5 (b) (ii) help of another ` 24,000 crores of the ‘group’ is only
(A)] group company ` 1,550 crores
South Limited (980+570) and
aggregate turnover is
also ` 7,700 crores.
(4700+3000)
3 MOU for transfer Eastern Limited No criterion Not Applicable.
of technology enters into an prescribed for
MOU with West considering the
Offshore transfer of
Limited for technology as

© The Institute of Chartered Accountants of India


PAPER – 6D: ECONOMIC LAWS 23

transfer of ‘combination’.
technology.
Note – Limits are quoted in section 5 of the Competition Act 2002 and further modified
through notification number S.O. 675(E) dated 4th March 2016
Regulation of Combinations
According to Section 6 (1) of the Act, no person or enterprise shall enter into a combination
which causes or is likely to cause an appreciable adverse effect on competition within the
relevant market in India and such a combination shall be void.
Further Section 6 (2) of the act says, any person or enterprise, who or which proposes to enter
into a combination, shall give notice to the Commission in the specified form along with a
requisite fee, disclosing the details of the proposed combination, within thirty days of:
(a) Approval of the proposal relating to merger or amalgamation by the Board of
Directors of the enterprises concerned with such merger or amalgamation;
(b) Execution of any agreement or other document for acquisition or acquiring of
control.
Further Section 6 (2A) of the Act provides, no combination shall come into effect until two
hundred and ten days have passed from the day on which the notice has been given to the
Commission or the Commission has passed orders under Section 31, whichever is earlier.
3.6(iv)
Whether eastern limited is in compliance of FEMA, 1999 for expenditure incurred on
maintenance of its branch office abroad ?
Eastern Tea Limited opened one branch office in Iran, and on such branch office, during last
financial year, an amount of ` 150 crore were incurred as expenditure for the branch through
the EEFC account maintained by Eastern Tea Limited.
As per Foreign Exchange Management Act, 1999, no branch can be opened in Pakistan,
Bangladesh, Sri Lanka, Afghanistan, China, Iran, Nepal, Bhutan, Macau or Hong Kong without
prior permission of the Reserve Bank.
As the case study does not reflect anywhere about the prior permission of the RBI, the
expenditure by Eastern Tea Limited is not in compliance under FEMA, 1999.
CASE STUDY - 4
Mr. Zebra is a real estate mogul who has developed and constructed several apartment
complexes in Mumbai through his real estate company, Delight Homes Private Limited (DHPL)
which was into the business of construction of residential premises. In May 2019, Zebra
proposed to start a new residential project named "Delight Morning Dews". The project plan

© The Institute of Chartered Accountants of India


24 FINAL (NEW) EXAMINATION: JANUARY 2021

constituted 50 apartments with a mix of both 3 BHK and 2 BHK apartments. DHPL ensured
that the sanction plan etc. was approved appropriately under RERA.
DHPL devised the advertisement and marketing content for the project so it can be splashed
across the national and local newspapers and television channels. Mr. Zebra was of the view
that the project need not be highlighted in the website of the regulatory authority since the
same does not get a lot of views from the prospective customers and it is more efficient to
reach the customer directly through social media/television platforms. Further, to ensure more
customers are attracted, DHPL started the commercial marketing before applying for the
registration of the project under RERA.
DHPL also incorporated a subsidiary, Delight Interiors and Consultancy Private Limited
(DICPL) in India for engaging in the business of providing consultancy services on interior
designing etc. Mr. Zebra made his only daughter Ms. Rekha as the Managing Director of
DICPL. Rekha completed her masters in interior design in the London School of Design and
had her own design studio in London, which got her critical acclaim in the art and design
society. DICPL became a huge hit based on the proof of concepts it delivered in the London
School of Design and got many orders from customers located in the UK.
In the agreement to sale entered into with the aIIottees, DHPL did not specify the stage -wise
time schedule of the completion of the project, including the provisions for civic infrastructure
like water sanitation & electricity. Also, DHPL did not include any terms with regard to
cancellation of allotment etc. in the agreement. These clauses were not insisted by the
allottees since they were more than eager to buy their apartments and DHPL did not see any
reason to amend these agreements at a later date.
During the process of construction, DHPL intended to transfer the project to another real
estate construction company, Value Homes Private Limited (VHPL) through an assignment
agreement. No approval from the allottees or regulator was considered necessary since the
agreement made it clear that VHPL will take over all obligations of DHPL and there will not be
any difference for the regulator or the allottees in terms of the quality of constructions or timing
of delivery.
One of the customers of DICPL was interested in investing in the share capital of DHPL if the
same is allowed by the provisions of FEMA. However, Mr. Zebra indicated to him that FEMA
prohibits a person resident outside India to make investment in a company i nvolved in real
estate business. However, Ms. Rekha believes that the customer can invest as long as the
money is paid directly to the bank account of DICPL through the normal banking channel and
the FIRC clearly denotes that this is for the purpose of equity investment into DICPL.
DICPL entered into various contracts to provide consultancy services to real estate
companies. Due to a downturn in the demand for real estate in the U.K. due to Brexit, some of
its customers faced a lot of difficulty in making payments to their suppliers and DICPL had
invoices outstanding amounting to GBP 2 million for more than 2 years (which include GSP
5,00,000 outstanding for more than 3 years).

© The Institute of Chartered Accountants of India


PAPER – 6D: ECONOMIC LAWS 25

For the purpose of construction, VHPL decided to import certain raw materials such as PVC
boards and light fittings from China for a value of USD 10,00,000. VHPL paid an amount of
USD 80,000 on receipt of the products and decided to hold back the balance USD 20,000 for a
period of 1 year to ensure satisfactory performance of the products. The Authorised Dealer is
of the view that this is not in accordance with the requirements of FEMA.
Out of the 50 apartments, VHPL decided to retain the title to 10 of the apartments and entered
into a "lease" agreement with the allottees for a period of 99 years. The consideration for the
lease was 95% of the consideration for an outright sale (along with stamp duty to be paid by
the allottees) and the lease allottees were required to pay ` 1,000 per month as rent with
maintenance charges at actuals (in line with what was to be paid by the other apartment
owners). VHPL was of the view that these agreements are outside the purview of RERA since
there is no sale involved.
Answer the following questions:
4.1 A branch set and controlled by DICPL located in the U.K. will be considered as:
(A) Resident in India;
(B) Resident outside India;
(C) Resident in India, if funds are remitted back by the Branch to India on annual
basis;
(D) Resident outside India if significant portion of its funds are directly received from
its customers located outside India. (2 Marks)
4.2 One of DICPL's customers visits India and during the visit, pays an amount of USD 2,000
which was owed by the customer through cash. Is this a permissible transaction under
FEMA ?
(A) Yes, this is money rightfully owed to DICPL and money was received in foreign
exchange;
(B) No, unless DICPL had a money changer's license to accept foreign currency;
(C) No, unless DICPL obtain prior approval of the Reserve Bank of India for such
transaction;
(D) Yes, as long as DICPL deposits the money in its bank account. (2 Marks)
4.3 An advertisement in the Guardian Newspaper (London edition) by DICPL for an amount
of GBP 20,000 requires the permission of:
(A) the Central Government;
(B) the Board of Directors of DICPL;
(C) the Ministry of Finance, Department of Economic Affairs;
(D) the authorized dealer. (2 Marks)

© The Institute of Chartered Accountants of India


26 FINAL (NEW) EXAMINATION: JANUARY 2021

4.4 Evaluate Mr. Zebra's contention that the customer of DICPL cannot invest in DHPL.
(A) The customer can invest in DHPL since DHPL, although a real estate company, is
involved in the construction of residential premises only;
(B) The customer cannot invest - Mr. Zebra is right in pointing out that FEMA prohibits
investment by a person resident outside India in real estate activities;
(C) The customer can invest with the prior approval of the Reserve Bank of India;
(D) The customer can invest since the FIRC indicates that the money is for investment
in equity capital and has come through the normal banking channel. (2 Marks)
4.5 The allottees who obtained the apartments under Lease seek your guidance on whether
those apartments would fall within the purview of RERA ?
(A) No, since there is no sale transaction and transfer of title to the allottee and
therefore it will not fall under RERA;
(B) Yes, this would be covered under RERA since substantial portion of the
consideration is paid and the lessee is also responsible for paying stamp duty,
maintenance etc.;
(C) No, since the entire sale consideration is not paid upfront and the lea se is for 99
years only;
(D) Yes, since VHPL and DHPL are registered promoters under RERA. (2 Marks)
4.6 Analyze and answer the following questions in the context of the case study:
(i) Evaluate the marketing strategies adopted by DHPL with regard to the provisions
of RERA. (3 Marks)
(ii) Evaluate with reasons if DICPL is in compliance with the FEMA provi sions with
regard to collection of export proceeds. Explain the steps to be taken by DICPL to
ensure compliance with FEMA. (5 Marks)
(iii) What is your advice to VHPL with regard to the position taken by the authorized
dealer? (3 Marks)
(iv) Advise the allottees of Delight Morning Dew regarding the assignment agreement
entered into between DHPL and VHPL in the context of RERA provisions. What
are the obligations of DHPL and VHPL under RERA ? (4 Marks)
ANSWERS TO CASE STUDY 4
4.1 (A)
4.2 (B)
4.3 (C)
4.4 (A)

© The Institute of Chartered Accountants of India


PAPER – 6D: ECONOMIC LAWS 27

4.5 (C)
4.6(i)
According to Section 3 of the Real Estate (Regulation & Development) Act, 2016 (the Act), no
promoter shall advertise, market, book, sell or offer for sale, or invite persons to purchase in
any matter any plot, apartment or building, in any real estate project or part of it in any
planning area without registering the real estate project with the Real Estate Regulatory
Authority established under this Act.
Section 3 of the Act further provides that no registration of the real estate project shall be
required where the area of land proposed to be developed does not exceed 500 square
meters or the number of apartments proposed to be developed does not exceed eight
inclusive of all phases;
In the given case study, since the number of apartments are more than 8, hence registration
was required under RERA. As DHPL has not taken registration under RERA, hence it cannot
have opted for commercial marketing of the project.
4.6 (ii)
According to Foreign Exchange Management (Export of Goods and Services) Regulations,
2015:
1. The amount representing the full export value of goods / software/ services exported shall
be realized and repatriated to India within nine months or within such period as may be
specified by the Reserve Bank of India, in consultation with the Government, from time to
time, from the date of export, provided. Further the Reserve Bank of India, or subject to the
directions issued by that Bank in this behalf, the authorized dealer may, for a sufficient and
reasonable cause, extend the said period. In the present case study, an amount of GBP 2
million is outstanding for more than 2 years (which include GBP 5,00,000 outstanding for more
than 3 years) is not in compliance with the FEMA provisions.
Apart from the above compliance, the following steps must be taken by DICPL to ensure
compliance with FEMA:
1. DICPL is responsible for ensuring that the full export value of the goods exported are
realized through an authorized dealer in the manner specified in the Fore ign
Exchange Management (Manner of Receipt and Payment) Regulations, 2016.
2. In respect of export of services to which none of the Forms specified in these
Regulations apply, the exporter may export such services without furnishing any
declaration, but shall be liable to realize the amount of foreign exchange which
becomes due or accrues on account of such export, and to repatriate the same to
India in accordance with the provisions of the Act, and these Regulations, as also
other rules and regulations made under the Act.

© The Institute of Chartered Accountants of India


28 FINAL (NEW) EXAMINATION: JANUARY 2021

3. A person resident in India to whom any amount of foreign exchange is due or has
accrued shall, save as otherwise provided under the provisions of the Act, or the
rules and regulations made thereunder, or with the general or special pe rmission of
the Reserve Bank of India, take all reasonable steps to realize and repatriate to India
such foreign exchange, and shall in no case do or refrain from doing anything, or
take or refrain from taking any action, which has the effect of securing —
(a) that the receipt by him of the whole or part of that foreign exchange is delayed;
or
(b) that the foreign exchange ceases in whole or in part to be receivable by him.
4. DICPL is also required to apply to RBI for getting the required directions for th e
purpose of securing the payment for the services performed, since there is a delay in
receipt of payment in accordance with the provisions.
4.6(iii)
advise to VHPL regarding the position taken by the authorized dealer
In terms of the FEM Regulations, remittances against imports should be completed not later
than six months from the date of shipment, except in cases where amounts are withheld
towards guarantee of performance, etc.
The Authorized Dealer (AD) can consider granting extension of time for settlement of import
dues up to a period of six months at a time (maximum up to the period of three years)
irrespective of the invoice value for delays on account of disputes about quantity or quality or
non-fulfilment of terms of contract; financial difficulties and cases where importer has filed suit
against the seller.
While granting extension of time, AD must ensure that:
a. The import transactions covered by the invoices are not under investigation by
Directorate of Enforcement / Central Bureau of Investigation or other investigating
agencies;
b. While considering extension beyond one year from the date of remittance, the t otal
outstanding of the importer does not exceed USD one million or 10 per cent of the
average import remittances during the preceding two financial years, whichever is
lower; and
c. Where extension of time has been granted by the AD, the date up to whic h extension
has been granted may be indicated in the ‘Remarks’ column.

© The Institute of Chartered Accountants of India


PAPER – 6D: ECONOMIC LAWS 29

In the given case study, VHPL decides to pay USD 80,000 on receipt of products and hold
back USD 20,000 for a period of 1 year to ensure satisfactory performance (total bill amount of
raw materials was USD 1,00,000)*.
As per the above provisions, VHPL can do so. Hence, the contention of AD that VHPL cannot
withhold the amount for satisfactory performance is not correct.
*Note: In the given case study, the value of import material from China is given as USD
10,00,000. It is further given that VHPL paid an amount of USD 80,000 on receipt of the
products and decided to hold back the balance USD 20,000 for a period of 1 year. The total of
these two amounts (80,000 and 20,000) comes to 1,00,000. Hence, the total invoice amount of
USD 10,00,000 seems to be a typographical error.
4.6 (iv)
Advise to allottees:
As per the facts given in the case study, the position taken by DHPL and VHPL with regard to
transfer of project is incorrect. In terms of Section 15 of the RERA, DHPL shall not transfer or
assign his rights and liabilities in respect of the project to VHPL without obtaining the prior
written consent of two-thirds allottees of the (except the promoter). Further, prior written
approval of the RERA Authority is also mandatory. Hence, the transfer from DHPL to VHPL of
the project is not valid in accordance with the RERA.
Therefore, the allottees are advised that in order to enable the transfer effective, steps must
be taken by DHPL to get prior consent of two-thirds allottees and also the written approval of
the RERA Authority.
Obligations of the promoters: Section 15 of the RERA, 2016 provides for the obligations of
promoter in case of transfer of a real estate project to a third party under:
1. The promoter shall not transfer or assign his majority rights and liabilities in respect
of a real estate project to a third party without obtaining prior written consent from
two-third allottees, except the promoter and without the prior written ap proval of the
Authority,
However, such transfer or assignment shall not affect the allotment or sale of the
apartments, plots, or building as the case may be in the real estate project made by the
erstwhile promoter.`
2. On the transfer or assignment being permitted by the allottees and the Authority, the
intending promoter shall be required to independently comply with all the pending
obligations under the provisions of this Act or the rules and regulations made
thereunder, and the pending obligations as per the agreement for sale entered into
by the erstwhile promoter with the allottees.

© The Institute of Chartered Accountants of India


30 FINAL (NEW) EXAMINATION: JANUARY 2021

Any transfer or assignment permitted under provisions of this section shall not result in
extension of time to the intending promoter to complete the real estate project and he shall be
required to comply with all the pending obligations of the erstwhile promoter, and in case of
default, such intending promoter shall be liable to the consequences of breach or delay, as the
case may be, as provided under this Act or the rules and regulations made thereunder.
CASE STUDY - 5
Mr. Rohit Writer is a well-known industrialist based in Pune, India and is the founder director
of Good Phones Private Limited (Good Phones), a fixed line and mobile phone manufacturer.
Good Phone is one of the largest telephone companies in India and its products are much
sought after in India and abroad. Mr. Rohit visits various countries as part of his business
travels and during these visits he spends significant time in Philanthropic activities. and social
gatherings and because of this, he is quite well known in business circles globally. Mr. Rohit
has a penchant for investing his money in buying various real estate property all over India
and passed this trait on to his son, Mr. Rahul Writer as well. Mr. Rahul completed his MBA
from Stanford University and is assisting Mr. Rohit in his business. Mr. Rohit also has a
daughter, Ms. Sonali Writer, who studies Art in Italy and has opened her own Art Studio in
Milan. Mr. Rohit is very proud of Sonali and supports her financially for her stay in Italy as wel l
as expenses towards maintaining the studio.
The marketing department of Good Phones introduced various new models in the last couple
of months with new technology such as 2 selfie cameras, faster processor and sleeker look.
Good Phones expect these phones to be major attraction in the global markets due to
attractive price range and therefore want to promote these phones extensively on a global
basis. For the purpose of advertisements, Good Phones engaged the services of Mr. David
Smith, a prominent baseball player and Ms. Emma Drew, a Miss Universe winner and agreed
to pay a 'guaranteed' fee of USD 5,00,000 each plus 10% bonus based on the sales of the
new models in year 1.
Mr. Rohit sent 5 sample mobile phones and 5 fixed line phones to his dealers abr oad
(numbering 1000 dealers), clearly marked as not for sale and other promotional material such
as brochures, 3D moulds for display in dealer shops etc. The value of the items was
approximately INR 4 Crore. He also sent 1 mobile phone to each of his deale rs as token of gift
and appreciation (total value of INR 0.50 Crore). Mr. Srinivas Rajan, the CFO of Good Phones
indicated him that since these products have been sent free of cost and not for sale, these
need not be included in the export declaration to be filed by Good Phones.
On 15th February, 2018, Good Phones made a large sale to one of the dealers Delayed
Ringtone Enterprises, for USD 5 million and has received USD 2 million by 15th May, 2018
and did not receive the balance USD 3 million until 15th August, 2018, i.e. 6 months from the
date of sale. After several reminders and threatening calls to stop further shipment, another
USD 1 million was received on 10 th October, 2018 and the balance remained outstanding as at
31st December, 2018.

© The Institute of Chartered Accountants of India


PAPER – 6D: ECONOMIC LAWS 31

Based on the success of Good Phones, Mr. Rohit incorporated a new company, Stay
Connected Private Limited, (Stay Connected) as Internet service provider and purchased a
large consignment of networking equipment for providing internet operations through
dedicated broadband lines along with a landline facility. This would then provide Mr. Rohit
quite a few synergies with the existing Good Phone business and enable him to become an
end to end Telecom Czar. Mr. Rohit held 60% stake in Stay Connected and the balance 40%
was held by a foreign collaborator. Along with all the networking equipment, Stay Connected
hired transponders from a company in Australia and paid AUD 10 million through its
authorized dealer. Stay Connected also entered into an agreement with foreign collab orator
(holding 40% stake) to pay royalty and technical fees for the support provided by them.
During his visit to Milan to meet Ms. SonaIi, Mr. Rohit obtained EUR 10,000 from his Italian
dealer for his use during his stay in Italy and instructed the dealer to reduce the sum from the
payments to be made by the dealer for the supplies from Good Phones. Out of such funds, Mr.
Rohit used EUR 5,000 towards purchasing sweepstakes tickets in Milan, Italy, unfortunately,
he did not win any money in the sweepstakes event.
Mr. Rahul, after gaining experience in India, wanted the business in the U.A.E. (by
establishing a subsidiary of Good Phones in the U.A.E.) and therefore decided to move to the
U.A:E. along with his wife. For this purpose, he wanted to dispose off some of the properties
owned by him in India. Accordingly, Mr. Rahul sold an apartment in Mumbai owned by him to
Mr. Stuart Cooper, being an Overseas Citizen of India and a fellow student of his at Stanford
University. Mr. Stuart Cooper was planning to come to India in the next couple of months to
take up a job and therefore, wanted to secure a place for his stay. The remittance from
Mr. Stuart was received in India through banking channels.
Mr. Rahul also sold a villa and his agricultural land in Pondicherry to Mr. Rajesh
Subramanium, his professor at Stanford, who was a person of Indian origin. The payment for
the villa and agricultural land was paid by Mr. Rajesh (50%) from his FCNR account and the
balance in USD traveller cheques, which will be of use to Mr. Rahul when he visits U.A.E.
After obtaining his U.S. visa, Mr. Rahul purchased a ranch (farm house) in Texas for USD 2
million, using USD 1.50 million from RFC account and USD 500,000 sent from his INR
account through normal banking channels.
Mr. William Rutherford, one of Mr. Rohit's business acquaintances and a citizen of UAE, is
very much interested in Indian culture and practices and therefore stays in India for 8 months
(from April, 2018 to November, 2018) to attend an art of living course and to learn/practice
yoga. William believes that he has been resident in India for more than the prescribed 182
days and therefore, is a resident in India under FEMA.
Mr. Rohit, in his penchant for purchasing various properties, zeroed in on an exclusive
apartment complex in Bangalore having state-of-the art facilities. He purchased two 4-
bedroom apartments costing INR 2 crore each, one in the name of Ms. Sonali and one in the
name of Mr. Srinivas Rajan, since Mr. Rohit wanted Mr. Srinivas Rajan to feel ha ppy and
trusted. Both the apartments were given on rent to a large multinational bank and he received

© The Institute of Chartered Accountants of India


32 FINAL (NEW) EXAMINATION: JANUARY 2021

a rent of INR 0.20 Crore per year for each of the apartment in the bank accounts of Ms. Sonali
and Mr. Srinivas Rajan respectively, after 4 years, Mr. Srinivas Rajan transferred the property
back in the name of Mr. Rohit at zero consideration. Mr. Rohit also purchased a 3 -bedroom
apartment in the same complex in his name, jointly with his brother, Mr. Sunil Writer. The
property (along with the stamp duty) was paid for by Mr. Rohit and was being used by Mr.
Sunil for his stay though the property was pending registration due to Mr. Rohit's travel
abroad.
Once the property was transferred back by Mr. Srinivas Rajan, Mr. Rohit wanted to sell the
same to Mr. Arjun De Silva, a citizen of Sri Lanka. However, he was advised by Mr. Srinivas
Rajan that Mr. Arjun De Silva cannot acquire property in India and therefore Mr. Rohit
proposed to lease it to Mr. Arjun De Silva for a period of 20 years for an upfront consideration
of INR 1 Crore and an annual rent of INR 8 Lac payable in advance.
During the review of the bank reconciliation statements of Good Phones, Mr. Srinivas Rajan
noted that an amount of INR 2 Crore had been received in one of the bank accounts without
any details relating to the same. Mr. Srinivas Rajan informed this to Mr. Rohit and Mr. Srinivas
Rajan suggested to Mr. Rohit to immediately transfer that money out of the bank of Good
Phones to Mr. Rohit's personal bank account, so that the Company's bank account are cleared
and there are no reconciling items, which Mr. Rohit agreed to. Out of INR 2 Crore, Mr. Rohit
used INR 1.75 Crore for acquiring further 20% stake in Stay Connected from the foreign
collaborator and balance INR 0.25 Crore for purchasing a stunning diamond set for his wife,
Ms. Anjali Writer, as a gift for her 50 th birthday.
The extract of the last audited financial statements of Stay Connected was provided by
Mr. Srinivas Rajan to Mr. Rohit to evaluate (FMV) his acquisition as per the provisions of the
Prohibition of Benami Property Transactions Act, 1988:
Particulars Amount in INR
(Crore)
Immovable property (market value INR 8.00 Crore) 5.00
Other fixed assets (net of depreciation of INR 1.00 Crore) 4.00
Inventory 2.00
Receivables and Loans and Advances 1.50
Deferred Advertisement Costs 0.50
Advance tax paid 1.00
Total Assets 14.00
Shareholders' Funds (including 1,000,000 equity shares of INR 10 each, 4.00
fully paid- up)
Provision for taxation 0.50
Loans from Banks 3.00

© The Institute of Chartered Accountants of India


PAPER – 6D: ECONOMIC LAWS 33

Trade payables (including provision for unascertained liabilities-INR 1 6.50


Crore)
Total Liabilities 14.00
Other information:
(i) 'Contingent Liabilities - INR 2.00 crore (including INR 0.50 Crore relating to arrears' on
cumulative preference shares).
(ii) The Board of Directors has proposed a dividend payout of INR 1 crore to the equity
shareholders, which is pending approval of the shareholders.
The Bank, on noting the large transactions on Mr. Rohit's personal bank account, tipped the
Income Tax Authorities regarding the same and the Initiating Officer summoned information
from Mr. Rohit and Mr. Srinivas Rajan regarding the transactions to start proceedings under
the Prohibition of Benami Property Transactions Act, 1988 (PBPT Act, 1988) and inves tigate
the matter under the Foreign Exchange Management Act, 1999 (FEMA,1999).
Mr. Rohit and Mr. Srinivas Rajan reached out to you in order to understand the various
violations and implications during the course of various proceedings under the said Act's .
Answer the following questions:
5.1 Out of the below, what are the transactions that require prior approval of the Government
of India?
(A) Payment of "guaranteed" fee by Good Phones to Mr. David Smith and Ms. Emma
Drew;
(B) Payment of Royalty and Technical Fees by Stay Connected to the foreign
collaborator;
(C) Payment of hiring charges for the transponders by Stay Connected;
(D) Payment of INR 1.75 Crore by Mr. Rohit to acquire shares of Stay Connected from
the foreign collaborator. (2 Marks)
5.2 Is the use of EUR 5,000 towards purchasing sweepstakes by Mr. Rohit as per the
provisions of FEMA, 1999 ?
(A) No, drawl of foreign exchange for purchasing lottery tickets, sweepstakes etc. is
prohibited under FEMA, 1999;
(B) No, Mr. Rohit should have obtained the prior approval of the RBI before
purchasing the sweepstakes ticket;
(C) FEMA, 1999 will not be applicable, since the money was directly obtained by Mr.
Rohit from his Italian dealer outside the country;
(D) None of the options. (2 Marks)

© The Institute of Chartered Accountants of India


34 FINAL (NEW) EXAMINATION: JANUARY 2021

5.3 Is the purchase of Ranch in Texas by Mr. Rahul in accordance with the FEMA, 1999 ?
(A) No, Rahul as a citizen of India cannot purchase a Ranch outside India;
(B) Yes, there is no specific limit under FEMA, 1999 with regard to purchase of
immoveable property outside India;
(C) No, Rahul can purchase assets outside India only if the purchase is jointly with a
relative, who is resident outside India, and there is no outflow of funds;
(D) No, since Rahul has used funds from his INR account for making the payment to
the extent of USD 500,000. (2 Marks)
5.4 In case Mr. Rohit is proven guilty of violating the provisions of PBPT Act, 1988, what is
the maximum punishment that he is liable for under the PBPT Act, 1988 ?
(A) Rigorous imprisonment for a term of one to seven years, with a fine which may
extend to 25% of the fair market value of the property;
(B) Rigorous imprisonment for a term of three to seven years, without fine;
(C) Rigorous imprisonment for a term upto seven years, with fine which may extend to
50% of the fair market value of the property;
(D) Fine which may extend to 25% of the fair market value of the property. (2 Marks)
5.5 Assuming that the transactions relating to the receipt of INR 2 Crore in the bank account
of Good Phones and the subsequent transactions are considered as benami
transactions, can the Initiating Officer take action against Mr. Srinivas Rajan ?
(A) Yes he is the CFO of Good Phones and therefore responsible for ensuring
compliance with the Law;
(B) No, he has not received, held, or acquired the proceeds in his acc ount or
benefitted from the same;
(C) Yes, since he abets Mr. Rohit in transferring the money from the bank account of
Good Phones to Mr. Rohit's personal account;
(D) No, he is responsible only for Good Phones and he has ensured that the funds are not
retained in the books of Good Phones/used by Good Phones for its business.
(2 Marks)
5.6 Answer the following questions in the context of the provisions relating to Foreign
Exchange Management Act, 1999 :
(i) Mr. Srinivas Rajan reaches out to you to confirm his views regarding
inclusion/exclusion of the items sent free of cost to the dealers in the export
declaration. (1 Marks)
(ii) Examine the validity/appropriateness of the sale of immoveable property by Mr.
Rahul to Mr. Stuart Cooper and Mr. Rajesh Subramanium. (2 Marks)

© The Institute of Chartered Accountants of India


PAPER – 6D: ECONOMIC LAWS 35

5.7 Examine/advise regarding the below questions relating to the Prohibition of Benami
Property Transactions Act, 1988 :
(i) Examine the appropriateness/impact of the PBPT Act, 1988 on 3 apartments
purchased by Mr. Rohit in Bangalore. How does the transfer back of the
apartment by Mr. Srinivas Rajan to Mr. Rohit affect your conclusion? (2 Marks)
(ii) The Initiating Officer, who is probing the transactions relating to the INR 2 Crore
received and spent by Mr. Rohit, seeks your advice to identify the benami
properties/transaction, the benamidars, the beneficial owner. (2 Marks)
(iii) Explain the provisions of Fair Market Value (FMV) in relation to a property as per
section 2(16) of PBPT Act, 1988. (2 Marks)
(iv) What is the process to be followed by the Initiating Officer for attachment of the
property under PBPT Act, 1988? (2 Marks)
(v) Discuss the provisions with regards to issue of notice, attachment of property
involved in benami transactions and manner of service of notice under PBPT Act,
1988. (4 Marks)
ANSWERS TO CASE STUDY 5
5.1 (C)
5.2 (A)
5.3 (D)
5.4 (A)
5.5 (C)
5.6 (i)
In the light of Regulation 4 of the Foreign Exchange Management (Export of Goods and
Services) Regulations, 2015, trade samples of goods, may be exported without furnishing the
declaration on the items, sent free of cost. In the given case, sending 5 sample mobile phones
and fixed line phones to 1000 dealers is exempted and does not require Good Phones to give
declaration for export.
With regard to sending mobile phones to the dealers as gift for a total value of INR 0.50 crore
(i.e., 50 lakh), as per the above Regulation, the exemption for sending gifts by an export is
available only if the value of the goods is not more than ` 5 lakh in value. In the case study,
since the value of the goods is more than the exempted limit, they need to be included in the
export declaration.
5.6 (ii)
Validity of the Sale of immovable property by Mr. Rahul can be given in the light of Regulati on
3 of the Foreign Exchange Management (Acquisition and Transfer of Immovable Property in
India) Regulations, 2018.

© The Institute of Chartered Accountants of India


36 FINAL (NEW) EXAMINATION: JANUARY 2021

Sale of immovable property to Mr. Stuart Cooper: Mr. Staurt Cooper, being an Overseas
Citizen of India is entitled to acquire an apartment in Mumbai owned by Mr. Rahul and funds
were received in India through Banking Channels.
Sale of immovable property to Mr. Rajesh Subramanium: Whereas in case of Mr. Rajesh
Subramanium, being NRI, he may acquire immovable property in India other than agric ultural
land/farm house/plantation property. Therefore, Mr. Rajesh Subramanium can acquire Villa
but not an agricultural land from Mr. Rahul. Further, payment was made partly from FCNR
account and in USD Travellers cheques, which was against the mode of payment prescribed
in the said regulation.
Therefore, sale of immovable property by Mr. Rahul to Mr. Stuart Cooper is valid, whereas to
Mr, Rajesh Subramanium, the said transaction is invalid.
5.7(i)
In the given case study, Mr. Rohit purchased 3 flats in Bangalore in the name of Ms. Sonali,
Mr. Srinivas Rajan, and jointly with his brother Sunil.
Apartment purchased in the name of Ms. Sonali- The property has been purchased by
Rohit in the name of his daughter Ms. Sonali, is covered under the exemption give n in Section
2(9) of the Prohibition on Benami Property Transaction Act, 1988 (i.e., property purchased in
the name of his child). Thus, it is not a benami transaction.
Apartment purchased in the name of Mr. Srinivas Rajan- This is a case of benami
transaction as the property is in the name of Srinivas Rajan but the consideration is paid by
Rohit.
Apartment purchased jointly in the name of Rohit and his brother Sunil- A property jointly
held in the name of brother and they appear as joint owners. Hence, this is not a benami
transaction.
Prohibition on retransfer of property by benamidar: As per Section 6 of the Prohibition on
Benami Property Transaction Act, 1988, (PBPT, Act) in cases where benamidar re-transfers
any benami property held by him to the beneficial owner or any other person acting on his
behalf, then such a transaction of a property shall be deemed to be null and void.
In the said above case transaction of transfer back of the apartment by Mr. Srinivas Rajan to
Mr. Rohit is void.
5.7(ii)
Advise to the Initiating Officer:
Following are the benami transactions and benamidars:
Transaction Benami Property/ Benamidar/ beneficial
owner
Receipt of INR 2 crore in the bank account Good Phones is a Benamidar w.r.t said
of Good Phones benami transaction of INR 2 crore.

© The Institute of Chartered Accountants of India


PAPER – 6D: ECONOMIC LAWS 37

Transfer of INR 2 crore from the bank Mr. Rohit is the Beneficial owner
account of Good Phones to Mr. Rohit’s
personal bank account
Acquisition of shares of Stay Connected Shares of Stay Connected becomes
using the benami money benami property as per Section 2(8) of
PBPT Act.
Mr. Rohit is a beneficial owner.
Purchase of Jewellery as gift for Ms. Anjali The jewellery becomes benami property.
Writer Mr. Rohit is a Benficial owner as he
purchased jewellery by paying
consideration from unknown sources.
Ms. Anjali is a Benamidar, as jewellery has
been purchased in her name.
5.7 (iii)
According to Section 2(16) of the Prohibition of Benami Property Transaction Act, 1988, fair
market value, in relation to a property, means—
(1) the price that the property would ordinarily fetch on sale in the open market on the
date of the transaction; and
(2) where the price referred above is not ascertainable, such price as may be
determined in accordance with such manner as may be prescribed in Rule 3 of the
Prohibition of Benami Property Transaction Rules, 2016.
As per the said Rule, the price of unquoted equity shares shall be the higher of -
(a) its cost of acquisition;
(b) the fair market value of such equity shares determined, on the da te of transaction, by
a merchant banker or an accountant as per the Discounted Free Cash Flow method;
and
(c) the value, on the date of transaction, of such equity shares as determined by the
formula given in the Rules.
5.7 (iv)
As per Section 24 of the Benami Transactions (Prohibition) Act, 1988, where the Initiating
Officer on the basis of material in his possession has reason to believe that any person is a
benamidar in respect of a property, he may, after recording reasons in writing, issue a notice
to the person to show cause within such time as may be specified in the notice why the
property should not be treated as benami property.

© The Institute of Chartered Accountants of India


38 FINAL (NEW) EXAMINATION: JANUARY 2021

Where the Initiating Officer is of the opinion that the person in possession of the property held
benami, may alienate the property during the period specified in the notice, he may, with the
previous approval of the Approving Authority, by order in writing, attach provisionally the
property in the manner as prescribed in Rule 4 of the Benami Transactions Prohibition Rules,
2016, for a period not exceeding ninety days from the last day of the month in which the notice
is issued.
The Initiating Officer, after making inquiries, pass an order continuing the provisional
attachment of the property with the prior approval of the Approving Authority, till the passing of
the order by the Adjudicating Authority; or revoke the provisional attachment of the property
with the prior approval of the Approving Authority;
Where the Initiating Officer passes an order continuing the provisional attachment of the
property or passes an order provisionally attaching the property, he shall, within fifteen days
from the date of the attachment, draw up a statement of the case and refer it to the
Adjudicating Authority.
5.7 (v)
Notice and attachment of property involved in benami transaction [Section 24 of PBPT,
Act, 1988]
Issue of show cause notice: Section 24 (1) states that where the Initiating Officer, on the
basis of material in his possession, has reason to believe that any person is a benamid ar in
respect of a property, he may, after recording reasons in writing, issue a show cause notice to
the person.
A copy of the notice shall also be issued to the beneficial owner if his identity is known. s
possession of the property held benami, may alienate the property during the period specified
in the notice, may, with the previous approval of the Approving Authority, by order in writing,
attach provisionally the property for a period not exceeding ninety days from the last day of
the month in which the notice is issued.
After Inquiry: Initiating Officer, after making such inquires and calling for such reports or
evidence and taking into account all relevant materials, shall, within a period of ninety days
from the last day of the month in which the notice is issued —
(a) where the provisional attachment has been made
(i) pass an order continuing the provisional attachment of the property with the prior
approval of the Approving Authority, till the passing of the order by the Adjudicating
Authority or
(ii) revoke the provisional attachment of the property with the prior approval of the
Approving Authority;

© The Institute of Chartered Accountants of India


PAPER – 6D: ECONOMIC LAWS 39

(b) where provisional attachment has not been made


(i) pass an order provisionally attaching the property with the prior approval of the
Approving Authority, till the passing of the order by the Adjudicating Authority under;
or
(ii) decide not to attach the property as specified in the notice, with the prior approval of
the Approving Authority.
Section 24 (5) states that where the Initiating Officer passes an order continuing the
provisional attachment of the property under sub-clause (i) of clause (a) of sub-section (4) or
passes an order provisionally attaching the property under sub-clause (i) of clause (b) of that
sub-section as stated above, he shall, within fifteen days from the date of the attachment,
draw up a statement of the case and refer it to the Adjudicating Authority.
Manner of Service of Notice [Section 25]
A notice under Section 24 may be served on the person named therein either by post or
as if it were a summons issued by a Court under the Code of Civil Procedure, 1908.

© The Institute of Chartered Accountants of India


DISCLAIMER
This Suggested Answers do not constitute the basis for evaluation of the student’s answers in

the examination. The answers are prepared by the Faculty of the Board of Studies with a view

to assist the students in their education. While due care is taken in preparation of the answers,

if any errors or omissions are noticed, the same may be brought to the attention of the Director

of Studies. The Council of the Institute is not in any way responsible for the correctness or

otherwise of the answers published herein.

Further, in the Elective Papers which are Case Study based, the solutions have been worked

out on the basis of certain assumptions/views derived from the facts given in the question or

language used in the question. It may be possible to work out the solution to the case studies

in a different manner based on the assumption made or view taken.

© The Institute of Chartered Accountants of India


2 FINAL (NEW) EXAMINATION: NOVEMBER 2020

PAPER 6D: ECONOMIC LAWS


NOTE: The question paper comprises five case study questions. The candidates are required
to answer any four case study questions out of five.
Case Study 1
PART A
That one Mis Sun Energy (Pte.) Limited hereinafter addressed as the "petitioner" had invested
in an Indian Company 'Z', a company promoted by RR, by way of shares and debentures. The
petitioner held 51 per cent of the share capital of 'Z' respectively.
The petitioner filed writ petition with Hon'ble High Court seeking for issuance of writ of
prohibition, restraining the official respondents from in any manner proceeding with the show
cause notice dated 19-5-2017, issued by the Initiating Officer (Rank of Deputy Commissioner
Income Tax-Regular Company Circle) under section 24(1) under the Prohibition of Benami
Property Transactions Act,1988 (or in short PBPT Act), calling upon the petitioner to show
cause as to why 51 per cent shares and debentures were held by the petitioner in an Indian
Company 'Z' not be treated as a "benami property" and wanted to impose penalty under the
Prohibition of Benami Property Transactions Act, 1988. The petitioner were of the view that the
Adjudicating Authority is biased and may take adverse view on the case of the petitioner and
the petitioner even challenged the composition of the Adjudicating Authority on their
membership and qualification. The petitioner also sought for issuance of a writ of Certiorari, to
quash the impugned show cause notice dated 19-5-2017, issued under section 24(3) of the
Prohibition of Benami Property Transactions Act, 1988, intimating the petitioner that pursuant
to the provisional attachment of shares and debentures, enforced, the petitioner was
restricted/prohibited from dealing in any manner and from exercising any rights in relation to the
shares and debentures.
The petitioner stated that none of the transactions were benami transactions and the petitioner
was not a benamidar and the shares and debentures were not benami property. The
transactions done by the petitioner were completed well before the amendment to the Prohibition
of Benami Property Transactions Act, 1988. (The amendment received the assent of the
President of India on 11-8-2016 and the Act came into force with effect from 1-11-2016)
It was alleged by the petitioner that after receiving substantial investment from the petitioner,
RR was alleged to have siphoned money out of 'Z', refused to make necessary disclosures and
comply with the mandatory filings required under the Companies Act, 2013 and when the
petitioner sought for transparency of the transactions, RR and various companies controlled by
him initiated litigation against the petitioner with a view to prevent the petitioner from examining
the affairs of 'Z'. In the meanwhile, RR filed company petition before the National Company Law
Tribunal (NCLT) to restrain the petitioner from exercising its rights in relation to the shares and
debentures and also approached the High Court in this regard, where the Court initially granted
an ex parte interim injunction, which was vacated after the petitioner entered appearance and
contested the matter, by order dated 1-6-2017 and RR's plea was dismissed.

© The Institute of Chartered Accountants of India


PAPER – 6D: ECONOMIC LAWS 3

The petitioner explained about the shareholding pattern in 'Z' and the pattern of investment
made in the company and how the debentures and shares were allotted to the petitioner. It was
submitted that on the date of issuance of the impugned show cause notice, the Initiating Officer
had no jurisdiction to issue the same, as he was not the gazette initiating officer under the Act
and thus lacked statutory jurisdiction even to issue the impugned orders. The transactions done
by the petitioner with the Indian company were completed in all aspects long before the
Amendment Act came into force i.e., on 01-11-2016 based upon the provisions of section 18
read with section 24 of the Act.
It was further submitted by the petitioner that the case of the Initiating Officer was solely based
upon the date on which, the Gazette Notification was uploaded by the Directorate of Printing at
the Government of India press to justify the jurisdiction of the Initiating Officer to initiate
proceedings. It was submitted that the notification would come into operation as soon as it is
published in the Gazette of India, i.e., the date of publication of Gazette and this being the
correct legal position, the contention of the Initiating Officer referring to the date on which the
notification was uploaded in the official website, was not sustainable based upon the provi sions
of section 2(21) of the Act.
PART-B
Further to the above case scenario M/s Sun Energy (Pte.) Limited had in the month of January
2014 pre-booked a commercial office unit of approximately 1200 sq. ft. with M/s J V Realty
Limited, a leading developer in that area in their "S COURT" Greater Noida project developed
in phases launched then by paying an amount of ` 25,00,000/-as booking amount out of
` 1,00,00,000 the total cost of flat but no Builder-Buyer agreement was entered into between
the parties except that an allotment letter was issued by the developer mentioning the unit
details. This project being developed over an area of approximately 15000 sq mts and having
over I 00 office units in its plan outlay and the company had paid till April 2017 almost 90% of
the entire cost of the property based upon percentage of completion (progress) of the stage of
construction but the developer had failed to provide neither possession nor had completed the
project and was also not responding to their complaints on one pretext or the other. The legal
counsel of M/s Sun Energy (Pte.) Limited in the month of May, 2017 informed the Board of
Directors of the company about Real Estate (Regulation and Development) Act, 2016 (for s hort
"the RERA"). They further informed that RERA was enacted by the Parliament as Act 16 of 2016
in the year 2016. Some of the provisions of the RERA came into force on a date prescribed by
the Central Government under the notification published in the official gazette. Different dates
were appointed for different provisions of the RERA. By Notification No. S.0.1544 (E), dated 26 -
4-2016, the Central Government appointed 1st day of May 2016 as a date on which some of
provisions of the RERA came into force, namely, Sections 2, 20 to 39, 41 to 58, 17 to 78 and
81 to 92. By Notification No. S. 0.1216, dated 19-4-2017 some more provisions of the RERA
came into force, namely, Sections 3 to 19, 40, 59 to 70 and 79, 80 w.e.f 1st May, 2017. Meaning
thereby that on May 1, 2017, all 92 provisions of the Real Estate (Regulation and Development)
Act, 2016 (RERA or the Act) were brought into force. The Act has introduced new obligations
on real estate developers and in cases of default, prescribes penal liabilities and the company

© The Institute of Chartered Accountants of India


4 FINAL (NEW) EXAMINATION: NOVEMBER 2020

can contemplate bringing a legal suit against the developers under RERA. The developer on
the other hand is of the view that RERA is not applicable to this project as the same was
launched and construction commenced much before the RERA came into force.
Answer the following questions:
1.1 Which of the following is correct statement as per Prohibition of Benami Property
Transactions Act, 1988?
(A) Prohibition to hold benami property.
(B) Prohibition of benami transactions.
(C) Prohibition of right to recover property held benami.
(D) Prohibition on re-transfer of property by benamidar.
1.2. As per the provision of Prohibition of Benami Property Transactions Act, 1988 the appellate
tribunal or the adjudicating authority may in order to rectify any mistake apparent on face
of the record, amend any order made under section 26 and section 46 respectively within
a period
(A) of two years from the end of the quarter in which the order was passed.
(B) of three years from the end of the quarter in which the order was passed.
(C) of one year from the end of the month in which the order was passed.
(D) of one year from the date of passing of order.
1.3 Where a builder is planning to develop a particular project in different phases spread over
couple of years, then he is required to obtain registration under Real Estate (Regulation
and Development) Act, 2016.
(A) Only once for the entire project indicating all the phases.
(B) For each phase separately.
(C) As and when project commences registration will be required.
(D) As and when a particular phase is being developed registration of that phase will be
required.
1.4 A promoter shall not accept a sum of more than_________ percentage of the cost of the
apartment, plot or building, as an advance payment or an application fee from a person
without first entering________ a under the provisions of Real Estate (Regulation and
Development) Act, 2016.
(A) 15%, Sale Deed.

© The Institute of Chartered Accountants of India


PAPER – 6D: ECONOMIC LAWS 5

(B) 10%, written agreement for sale.


(C) 15%, Sale Deed which is duly registered.
(D) 10%, written agreement to sale which is duly registered.
1.5 Where a Real Estate Agent contravenes the provisions of section 9 or section 10 of the
Real Estate (Regulation and Development) Act, 2016 he shall be liable to penalty as
determined by the Authority of____________________
(A) `10,000.
(B) ` 10,000 for every day during which the default continues.
(C) `10,000 for every day during which the default continues upto 5% of the cost of the
plot, apartment or building of the project for which sale has been facilitated.
(D) `10,000 for every day during which the default continues upto 2% of the cost of the
plot, apartment or building of the project for which sale has been facilitated.
(5 x 2 = 10 Marks)
1.6 In the light of given case study state the quantum of penalty imposed whosoever enters
into any Benami Transaction on and after the date of commencement of the Benami
Transactions (Prohibition) Amendment Act, 2016. (3 Marks)
1.7 State the qualifications for appointment of Chairperson and Members of the Adjudicating
Authority under the Prohibition of Benami Property Transactions Act, 1988.
(2 Marks)
1.8 In the light of the given case study decide stating the provisions of the Real Estate
(Regulation and Development) Act, 2016, whether M/s Sun Energy (Pte.) Limited can
initiate legal proceedings against the developer M/s J V Realty Limited under the said Act
or the contention of the developer that the said Act is not applicable to the project is correct.
(5 Marks)
1.9 From the provisions of the Real Estate (Regulation and Development) Act, 2016, you are
of the view that the Act is applicable to the developer then decide as per the provisions of
the said Act, can the company seek refund of the entire amount paid to the developer till
date along with interest? Whether apart from principal and interest, can the company also
seek certain compensation from the developer? (5 Marks)
ANSWER TO CASE STUDY 1
1.1 Options B, C, & D
Note: In the light of the Preamble of the Prohibition of Benami Property Transaction
Act, 1988, Options B, C, & D, are correct Options

© The Institute of Chartered Accountants of India


6 FINAL (NEW) EXAMINATION: NOVEMBER 2020

1.2 Option C
1.3 Option B
1.4 Option D
1.5 Option C
Answer 1.6
Quantum of Penalty for Benami Transactions [Section 53]
As per Section 53 of the Prohibition of Benami Property Transactions Act, 1988: [substituted for
Benami Transactions (Prohibition) Act, 1988 by the Benami Transactions (Prohibition)
Amendment Act, 2016 w.e.f. 01.11.2016] :
Where any person enters into a benami transaction in order to defeat the provisions of any law
or to avoid payment of statutory dues or to avoid payment to creditors, the beneficial owner,
benamidar and any other person who abets or induces any person to enter into the benami
transaction, shall be guilty of the offence of benami transaction.
Whoever is found guilty of the offence of benami transaction referred to above shall be
punishable with rigorous imprisonment for a term which shall not be less than one year, but
which may extend to seven years and shall also be liable to fine which may extend to twenty-
five per cent of the fair market value of the property.
Answer 1.7
As per Section 9 of the Prohibition of Benami Property Transactions Act, 1988 as amended by
the Benami Transactions (Prohibition) Amendment Act, 2016 w.e.f. 01.11.2016:
(1) A person shall not be qualified for appointment as the chairperson or a Member of the
Adjudicating Authority unless he:
(a) Has been member of the Indian Revenue Service and has held the post of
Commissioner of Income tax or equivalent post in that service
(b) Has been a member of the Indian legal service and has held the post of joint Secretary
or equivalent post in that service.
(2) The Chairperson and other members of the Adjudicating Authority shall be appointed by
the Central Government in such manner as may be prescribed.
(3) The Central Government shall appoint the senior most member to be the chairperson of
the Adjudicating Authority.
Answer 1.8
As per Section 3(1) of The Real Estate (Regulation & Development) Act, 2016, (the Act) the
promoter shall make an application to the Authority for registration of the project that is ongoing

© The Institute of Chartered Accountants of India


PAPER – 6D: ECONOMIC LAWS 7

on the date of commencement of this Act and for which completion certificate has not been
issued within a period of three months from the date of commencement of this Act.
Further Section 3(2) of the Act says that no registration of the real estate project shall be
required :
(a) where the area of land proposed to be developed does not exceed 500 square meters or
the number of apartments proposed to be developed does not exceed eight inclusive of all
phases;
(b) where the promoter has received completion certificate for a real estate project prior to
commencement of this act;
As per the facts, “S COURT” greater Noida project was launched before the enforcement of the
Act. As M/s Sun Energy (Pte.) Limited had pre-booked in January 2014 with M/s J V Realty
Limited. So it was an ongoing project on the date of commencement of this Act and for which
completion certificate has also not been issued within a period of three months from the date of
commencement of this Act.
Further project was developed over an area of approximately 15000 sq. mts. and having over
100 office units in its plan outlay, which exceed 500 square meters and the number of
apartments exceeding eight inclusive of all phases.
Hence in the given case, M/s Sun Energy (Pte.) Ltd can initiate legal proceedings against
developer M/s J V Realty Limited under the Act stating the violation of the above mentioned
provisions under the Act and the contention of the developer that the said Act is not applicable
to the project, is incorrect.
Answer 1.9
Return of Amount and Compensation (Section 18)
Section 18 of the of The Real Estate (Regulation & Development) Act, 2016, (RERA) provides
for the return of amount and compensation.
(1) If the promoter fails to complete or is unable to give possession of an apartment, plot or
building,—
(a) in accordance with the terms of the agreement for sale or, as the case may be, duly
completed by the date specified therein; or
(b) due to discontinuance of his business as a developer on account of suspension or
revocation of the registration under this Act or for any other reason:
he shall be liable on demand to the allottees, in case the allottee wishes to withdraw from
the project, without prejudice to any other remedy available, to return the amount received
by him in respect of that apartment, plot, building, as the case may be, with interest at such

© The Institute of Chartered Accountants of India


8 FINAL (NEW) EXAMINATION: NOVEMBER 2020

rate as may be prescribed in this behalf including compensation in the manner as provided
under this Act.
However, where an allottee does not intend to withdraw from the project, he shall be paid,
by the promoter, interest for every month of delay, till the handing over of the possession,
at such rate as may be prescribed.
(2) The promoter shall compensate the allottees in case of any loss caused to him due to
defective title of the land, on which the project is being developed or has been developed,
in the manner as provided under this Act, and the claim for compensation under this sub -
section shall not be barred by limitation provided under any law for the time being in force.
(3) If the promoter fails to discharge any other obligations imposed on him under this Act or
the Rules or Regulations made thereunder or in accordance with the terms and conditions
of the agreement for sale, he shall be liable to pay such compensation to the allottees, in
the manner as provided under this Act.
Therefore in the given case study as per the provision of Section 18 reproduced herein above,
the Company can seek refund if they wish to withdraw and also claim interest apart from
compensation.
CASE STUDY 2
Mr. Kamal is engaged in the real estate business of development of townships through his
company- M/s P Homes Ltd. During the course of business, he has accumulated enormous
amount of wealth in the form of cash which was generated through illegal businesses. Police
cases under several sections of various Indian laws have also been registered against
Mr. Kamal.
Mr. Kamal has a son Mr. Vimal who was residing in India during F.Y. 2016 -17. He left for UAE
on 25th August 2017 to undergo training for a period of 4 years. Mr. Shyam, brother of Mr.
Kamal, has a daughter, Ms. Priyadarshini pursuing higher studies in UAE. Mr. Shyam intends
to:
(a) open a bank account in foreign currency in UAE.
(b) remit money from India to his daughter in her account for studies.
Separately, Ms. Priyadarshini has requested Mr. Shyam to sponsor a chess tournament in UAE
which will involve remittance amounting to USD 85,000 (after conversion). Mr. Shyam generally
remits money through TZB Bank Ltd. after complying necessary formalities.
On the other hand, since Mr. Vimal's interest lies in India, he intends to invest money in India in
the following manner:
(a) Incorporating a Company in India followed by infusion of capital in the said company.
(b) Buying an agricultural farm in his individual capacity.

© The Institute of Chartered Accountants of India


PAPER – 6D: ECONOMIC LAWS 9

Above investments require funding which will be sought from Mr. Kamal.
From the business of real estate, total wealth generated by Mr. Kamal amounts to approx.
` 775 Crore. The said amount was utilized by him in the following manner:
(a) Around `100 crore were used for meeting certain cash expenses and paying certain bribes.
(b) ` 325 crore were transferred through hawala transaction to Mr. Vimal.
Transferring money through hawala route was chosen by Mr. Kamal since the money available
with him in his bank account was not sufficient to remit legally under various provisions of
Foreign Exchange Management Act, 1999. Therefore, he decided to strike a deal with Mr. Bhola,
a hawala agent operating in India. Terms of the deal are as under:
• Mr. Kamal will pay ` 325 crore + commission in cash to Mr. Bhola.
• Mr. Bhola, through his counterparts in UAE, will pay equivalent USD (after conversion) to
Mr. Vimal against invoice for professional services dated 1st October 2018.
Further Mr. Kamal and Mr. Shyam are promoters and directors of M/s KS Cinemas Ltd., a
company engaged in the business of producing motion films in India.
For a very large upcoming film project, M/s KS Cinemas Ltd. has taken loan from TZB Bank Ltd.
amounting to ` 350 crore after mortgaging all the assets of the company including rights related
to the film. However, due to controversies surrounding the film, the Censor Board withheld the
certification of the film. Even the Honorable High Court turned down plea of the producers that
the film is not against the interest of the country or public at large. The Reserve Bank of India
during the course of annual audit sent a notice to TZB Bank Ltd on suspicion of non-compliance
of the provisions of the Foreign Exchange Management Act, 1999 by TZB Bank Ltd. In the said
notice, the Reserve Bank of India sought certain information on the transactions carried out by
Mr. Shyam. However, lawyer of TZB Bank Ltd. suggested not to provide any response to such
notice, since such notice is generally issued to every bank as a part of audit procedure and is
of routine in nature.
One of the disgruntled crew members filed a complaint against Mr. Kamal in police station under
Indian Penal Code (IPC) for investigation. The complaint was accompanied with the details of
how Mr. Kamal acquired massive amount of wealth and huge properties in his name and also
in joint names. The accused person accumulated movable and immovable properties and assets
not only in India but in abroad also. Those properties were acquired otherwise and were not
included in their disclosed assets. Their criminal acts indicated misappropriation of public
money. Accordingly, the complaint was registered under the Indian Penal Code, 1860 and the
Prevention of Corruption Act, 1988.
Later on, the investigation was taken over by the CBI., while the CBI. was proceeding with the
investigation, the Enforcement Directorate on the basis of allegation made, lodged Enforcement
Case Information Report (ECIR) against Mr. Kamal. Similarly, as per the said ECIR when
complaint was filed under Section 45 of the Prevention of Money Laundering Act, 2002,

© The Institute of Chartered Accountants of India


10 FINAL (NEW) EXAMINATION: NOVEMBER 2020

cognizance of the offence was taken against Mr. Kamal under section 3 of the Prevention of
Money Laundering Act, 2002, punishable under section 4 of the said Act. The Enforcement
Directorate issued a notice dated 27 th January 2018 to Mr. Kamal, which was received by him
on 31st January,2018 directing him to pay penalty.
Subsequently, an order was issued by the authorities to provisionally attach properties
belonging to Mr. Kamal. Mr. Kamal now intends not to challenge the action taken against him
under the Prevention of Money Laundering Act, 2002 before the Adjudicating Authorities. On
01st May,2018 a meeting was held with you in the said meeting Mr. Kamal informed that he
wanted to engage you to advise for understanding, powers and remedy for his matters under
the various provisions of theForeign Exchange Management Act, 1999 and the Prevention of
Money Laundering Act, 2002.
Answer the following questions:
2.1 Which of the following remittance will require prior approval of Government of India for
drawl of foreign exchange under the Foreign Exchange Management Act, 1999?
(A) Payment related to 'call back services' of telephones.
(B) Opening of foreign currency account abroad with a bank.
(C) Remittance of prize money / sponsorship of sports activity abroad by a person other
than International / National / State Level bodies, if the amount involved is USD
90,000.
(D) Remittance of freight of vessel charted by a Public Sector Undertaking.
2.2 As per the provisions of the Prevention of Money Laundering Act, 2002, person on whose
behalf a transaction is being conducted is known as:
(A) Client.
(B) Financial Institution
(C) Beneficial Owner.
(D) Authorized Dealer.
2.3 Under the Prevention of Money Laundering Act, 2002, Adjudicating Authority consists of
following:
(A) 3 persons including chairman.
(B) 4 persons including chairman.
(C) 2 persons one of whom can be appointed as a chairman
(D) 5 persons including a member from Ministry of Law and Justice.

© The Institute of Chartered Accountants of India


PAPER – 6D: ECONOMIC LAWS 11

2.4 Among other things, what is the qualification of a person to be appointed as a Public
Prosecutor before the Special Court under the provisions of the Prevention of Money
Laundering Act, 2002?
(A) Minimum 10 years of experience as an advocate.
(B) Minimum 5 years of experience as an advocate.
(C) Minimum 7 years of experience as an advocate.
(D) Minimum 15 years of experience as an advocate.
2.5. Under the Prevention of Money Laundering Act, 2002, property can be provisionally
attached for___________.
(A) Not exceeding 60 days.
(B) Not exceeding 90 days.
(C) Not exceeding 180 days.
(D) Not exceeding 300 days. (5 x 2 = 10 Marks)
2.6 Answer the following in light of the provisions of the Foreign Exchange Management
Act,1999:
Advise Mr. Kamal whether:
I. he can invest in M/s P Homes Ltd. engaged in the business of building low budget
homes.
II. he can buy agricultural farm in his individual capacity.
III. he can make payment through foreign currency notes. (3 Marks)
2.7 For investing activities in India by Mr. Kamal, he approached you on 1st May 2018 with a
notice dated 27 th January, 2018 received by him from the office of Enforcement Directorate
on 31st January 2018 directing him to pay penalty. Kindly advise Mr. Kamal on timelines to
pay the penalty and powers of the officers to recover the same. Mr. Kamal has informed
that he doesn't intend to file an appeal. (3 Marks)
2.8 On suspicion of non-compliance of the provisions of the Foreign Exchange Management
Act, 1999 by TZB Bank Ltd., the Reserve Bank of India had sent a notice to the bank
seeking certain information on the transactions carried out by Mr. Shyam. However, lawyer
of TZB Bank Ltd. had suggested not to provide any response to such notice since such
notice is generally issued to every bank as a part of audit procedure and is of routine in
nature. Explain the powers of the Reserve Bank of India in case of non -compliance to
notice. (3 Marks)

© The Institute of Chartered Accountants of India


12 FINAL (NEW) EXAMINATION: NOVEMBER 2020

2.9 Explain the following in light of the provisions of the Prevention of Money Laundering Act,
2002:
I. Money Laundering does not mean just siphoning of funds. In light of this statement,
explain the significance and aim of the Prevention of Money Laundering Act, 2002
and its three distinct stages. (2 Marks)
II. Mr. Kamal seeks your advice on the remedy available with him under the Act against
the said attachment order. (2 Marks)
III. Properties confiscated under the provisions of the Prevention of Money Laundering
Act, 2002 shall be available for disposal by Ministry of Finance as and when
necessary. Examine correctness of the statement. (2 Marks)
ANSWER TO CASE STUDY 2
2.1 Option D
2.2. Option C
2.3 Option A
2.4 Option C
2.5 Option C
Answer 2.6
(I) As per Foreign Exchange Management (Permissible Capital Account Transactions)
Regulations, 2000, no person resident outside India shall make an investment in India, in
any form, in any company or partnership firm or proprietary concern or any entity, wheth er
incorporated or not, which is engaged or proposes to engage in real estate business. Since
Mr. Kamal is a person resident in India, he can invest in M/s P Homes Ltd. engaged in the
business of building low budget homes.
(II) Yes, Mr. Kamal can buy agricultural farm in his individual capacity, since prohibitions as
regard the purchase of agricultural farm is exercised in favour of person resident outside
India. In other words there is no specific prohibition on person resident in India on buying
of agricultural farm in his individual capacity.
(III) A person resident in India can open, hold and maintain with an authorized dealer in India,
a Resident Foreign Currency (Domestic) Account, out of foreign exchange acquired in the
form of currency notes, Bank notes and travellers cheques from any of the sources like,
payment for services rendered abroad. Yes Mr. Kamal can make payment through foreign
currency notes through an authorized dealer.

© The Institute of Chartered Accountants of India


PAPER – 6D: ECONOMIC LAWS 13

Answer 2.7
Recovery of Fine or Penalty [Section 69 of the Prevention of Money Laundering Act, 2002]
Where any fine or penalty imposed on any person under Section 13 or Section 63 of Prevention
of Money Laundering Act, 2002 is not paid within 6 months from the day of imposition of fine or
penalty, the Director or any other officer authorized by him in this behalf may proceed to recover
the amount from the said person in the same manner as prescribed in Schedule II of the Income-
tax Act, 1961 for the recovery of arrears and he or any officer authorized by him in this behalf
shall have all the powers of the Tax Recovery Officer mentioned in the said Schedule for the
said purpose. Accordingly, Mr. Kamal must pay penalty latest by 31 st July, 2018.
Answer 2.8
Reserve Bank’s powers to issue directions to authorized person [Section 11]
(1) The Reserve Bank may, for the purpose of securing compliance with the provisions of this
Act and of any rules, regulations, notifications or directions made thereunder, give to the
authorized persons any direction in regard to making of payment or the doing or desist
from doing any act relating to foreign exchange or foreign security.
(2) The Reserve Bank may, for the purpose of ensuring the compliance with the provisions of
this Act or of any rule, regulation, notification direction or order made thereunder, direct
any authorized person to furnish such information, in such manner, as it deems fit.
(3) Where any authorized person contravenes any direction given by the Reserve Bank under
this Act or fails to file any return as directed by the Reserve Bank, the Reserve Bank may,
after giving reasonable opportunity of being heard, impose on the authorized person a
penalty which may extend to ten thousand rupees and in the case of continuing
contravention with an additional penalty which may extend to two thousand rupees for
every day during which such contravention continues.
As per above provisions, Reserve Bank of India may impose penalty on TZB Bank Ltd. for non -
compliance to notice.
Answer 2.9
(I) Money laundering does not mean siphoning of fund. It actually refers to a whole process
or an entire system by which money is actually generated from serious crimes but they are
given such shape (by disguising its origin into a series of transactions) that it looks like it
has originated from legitimate sources.
The Prevention of Money Laundering Act, 2002 As stated in the Preamble to the Act, it
is an Act to prevent money-laundering and to provide for confiscation of property derived
from, or involved in, money-laundering and to punish those who commit the offence of
money laundering.

© The Institute of Chartered Accountants of India


14 FINAL (NEW) EXAMINATION: NOVEMBER 2020

Money laundering is a single process however; its cycle can be broken down into three
distinct stages:
1. Placement: It is the first and the initial stage when the crime money is injected into
the formal financial System.
2. Layering: Then under the second stage, money injected into the system is layered
and moved or spread over various transactions in different accounts and different
countries. Thus, it will become difficult to detect the origin of the money.
3. Integration: Under the third and final stage, money enters the financial system in
such a way that original association with the crime is sought to be obliterated so that
the money can then be used by the offender or person receiving as clean money.
II. Section 25 of the Prevention of Money Laundering Act, 2002 (the Act) empowers the
Central Government to establish an Appellate Tribunal to hear appeals against the orders
of the Adjudicating Authority and the Authorities under the Act.
Section 26 of the Act deals with the rights and time frame to make an appeal to the
appellate Tribunal. Any person aggrieved by an order made by the Adjudicating Authority
may prefer an appeal to the Appellate Tribunal within a period of 45 days from the date on
which copy of the order is received by him. The appeal shall be in such form and be
accompanied by such fee as may be prescribed. The Appellate tribunal may extend the
period if it is satisfied that there was sufficient cause for not filing it within the period of 45
days.
The Appellate Tribunal may, after giving the parties to the appeal an opportunity of being
heard pass such orders thereon as it thinks fit, confirming, modifying or setting aside the
order appealed against.
According to Section 42 of the Act, any person aggrieved by any decision or order of the
Appellate Tribunal may file an appeal to the High Court within 60 days from the date of
communication of the order of the Appellate Tribunal.
In the light of the above provisions of the Act, Mr. Kamal is advised to prefer an appeal to
the Appellate Tribunal in the first instance.
(III) Management of Properties confiscated (Section 10)
Under Section 10 of the Prevention of Money laundering Act, 2002:
The Central Government may, by order published in the Official Gazette, appoint as many
of its officers (not below the rank of a Joint Secretary to the Government of India) as it
thinks fit, to perform the functions of an Administrator.

© The Institute of Chartered Accountants of India


PAPER – 6D: ECONOMIC LAWS 15

(1) The Administrator appointed under sub-section (1) shall receive and manage the
property in relation to which an order has been made under sub-section (5) or sub-
section (6) or sub-section (7) of Section 8 or Section 58B or sub-section (2A) of
Section 60 in such manner and subject to such conditions as may be prescribed.
(2) The Administrator shall also take such measures as the Central Government may
direct to dispose of the property which is vested in the Central Government under
Section 9.
In view of the above, the state that the properties under the Act shall be available for
disposal by the Ministry of Finance as and when necessary is correct.
CASE STUDY 3
SSTPL is one of India's leading television manufacturers and has its manufacturing plant in
Chennai, with more than 200 dealers across the country. SSTPL specializes in manufacturing
LED Smart televisions both for direct retail sales as well as contract manufacture for other
television manufacturers. SSTPL has a very robust Board of Directors who are highly involved
in the operations of the entity.
During one of the Board Meetings held in the month of July 2019, the Board of Directors
reviewed the amounts receivable from the dealers of SSTPL and noted the following:
Age Amount in Lakh ` Number of Dealers
0 to 180 days 1505 135
180 to 720 days 280 34
> 720 days 905 1
Total 2,690 170
The CFO went on to explain that the amount which is outstanding for more than 2 years is
receivable from DMPL and the Company has been following up with the dealer on a regular
basis. The independent director on the Board asked the CFO to explore the possibility of taking
action against DMPL under the Insolvency and Bankruptcy Code, 2016 (in short 'IBC 2016').
The CFO informed that the financial creditors of DMPL has already commenced the process
and the Interim Resolution Professional (IRP) reached out to the CFO last week to understand
the claims of SSTPL against DMPL.
The IRP identified the following assets and liabilities of DMPL:
• Bank loans taken by DMPL from Bank A amounting to `1500 lakh and Bank B amounting
to ` 1050 lakh.
• Loan taken from the son Mr. 'X' of the promoter of DMPL amounting to ` 75 lakh attended
Board Meetings to provide guidance/directions on policy making process.
• Payable to SSTPL ` 905 lakh.

© The Institute of Chartered Accountants of India


16 FINAL (NEW) EXAMINATION: NOVEMBER 2020

• Outstanding wages to workmen amounting to ` 75 lakh.


• Statutory employer contributions to the tune of ` 30 lakh.
• Realisable value of the fixed assets of DMPL, ` 2800 lakh.
• Receivables from various customers, ` 225 lakh, out of which 50% is not realisable.
• Bank balance off ` 22.5 lakh.
The IRP also received information that MCL, a Company registered in Germany, pursuant to an
agreement entered with DMPL and supplied spares to DMPL for an amount of EVR 500,000
(INR 400 lakh) (though this claim is not disputed by DMPL, the same was not recorded in the
books of accounts of DMPL inadvertently). Since this amount was not paid by DMPL even after
several reminders, MCL filed an application under the IBC 2016. However, this application was
rejected by the Adjudicating Authority since as per the agreement between MCL and DMPL, any
disputes between the parties are to be decided by the courts in Germany. DMPL, in its
agreement, with its distributors, specified that the distributors be necessarily required to
purchase spares for 2 models of cars on a bundled basis (the sale price fixed based on fair
market value/mutual discussion). On 14 th April 2020, ACL, another supplier of DMPL, to whom
DMPL owed INR 75 lakh, also wanted to initiate Corporate Insolvency Resolution Process
against DMPL for non-payment of undisputed dues.
During the aforesaid Board Meeting of SSTPL, the CFO also placed a revised draft agreement
to be entered into with all the dealers after introduction of GST and as part of the same, the
following clauses were proposed to be included:
• Dealers are required to obtain specific approval of SSTPL prior to making change in the
marketing model or technical developments to the prejudice of customers.
• Specify the geographical area where the dealers can market the cars.
• Limit the operation of service centres by specifying dealers who can operate service
centres.
• Bar transactions or transfer of cars and spares between dealers.
• Mandate the floor price at which services may be provided by the dealers.
• Higher pricing of substitutable products and services.
• Mandate the dealers to acquire certain number of cars of the base version, when ordering
high end variants.
The agreement envisaged that no sale would be made to dealers who do not comply with the
above conditions. The Directors of the Company felt that some of these clauses are not in
compliance with the provisions of the Competition Act 2002.

© The Institute of Chartered Accountants of India


PAPER – 6D: ECONOMIC LAWS 17

Answer the following questions


3.1 What is the percentage share of Bank A in the Committee of Creditors of DMPL under IBC,
2016 proceedings?
(A) 57.14%.
(B) 58.82%.
(C) 41.27%.
(D) 42.13%.
3.2 Out of the below, identify who is a related party of DMPL under the IBC, 2016?
(A) Mr. A, who holds 15% shares in DMPL.
(B) Indigenous Private Limited, who has one common independent director (with no
shareholding) with DMPL.
(C) Mr. X, who although not an employee or director of DMPL, is close to the promoter
and attends Board Meetings to provide guidance/directions on policy making process;
(D) Ms. Y, who controls the composition of Board of Directors of SSTPL.
3.3 Does the contract entered into by DMPL with its distributors cause an appreciable adverse
effect on competition under the Competition Act, 2002?
(A) Yes, since this is in the nature of a tie-in arrangement.
(B) No, this is a contract between a 'willing buyer' and 'willing seller' and they are free to
determine the contract terms;
(C) Yes, since transaction is in the nature of predatory pricing by DMPL to reduce
competition from other spares manufacturers.
(D) No, the contract actually promotes and sustains competition in the market.
3.4 The plan of SSTPL to consider a higher cost of substitutable goods and services for the
dealers is covered under which of the below factors under the Competition Act, 2002?
(A) Appreciable adverse effect on competition.
(B) Abuse of dominant position.
(C) Price rigging.
(D) Collusive pricing.

© The Institute of Chartered Accountants of India


18 FINAL (NEW) EXAMINATION: NOVEMBER 2020

3.5 Can ACL file Corporate Insolvency Resolution Process against DMPL under IBC, 2016?
(A) Yes, ACL is an operational creditor and all the conditions under IBC, 2016 have been
fulfilled.
(B) No, ACL is not a financial creditor.
(C) No, since the amount of default is less than the minimum amount of default (` 100
lakh) for being covered under Section 4 of IBC, 2016;
(D) Yes, since the amount of default is not, disputed by DMPL and there is no ongoing
dispute. (5 x 2 = 10 Marks)
3.6 Answer the following questions:
(i) Advice the IRP with regard to the appropriateness of the order of the Adjudicating
Authority regarding, the application made by MCL under the provisions of the
Insolvency and Bankruptcy Code, 2016. (4 Marks)
(ii) Calculate the amount receivable by SSTPL from DMPL based on the facts given in
the case study (assume-no liquidation costs) as per Section 53 of the Insolvency and
Bankruptcy Code, 2016. (5 Marks)
(iii) Evaluate the terms of the agreement proposed to be entered into by SSTPL with the
dealers based as per the provisions of the Competition Act, 2002. (6 Marks)
ANSWER TO CASE STUDY 3
3.1 Option (C)
3.2 Option (C)
3.3 Option (A)
3.4 Option (B)
3.5 Option (C)
Answer 3.6
Enabling provisions for cross border transactions: India is no more an isolated business
place. India is now part of global business hub. Indian businesses have investments outside
India while many businesses outside India have presence in India. India is now a global village.
Enabling provisions in the Code are Sections 234 and 235 for this purpose.
Agreements with Foreign Countries: The Central Government may enter into an agreement
with the Government of any country outside India for enforcing the provisions of this Code.
Letter of request to a country outside India in respect of assets: If, in the course of
insolvency resolution process, or liquidation or bankruptcy proceedings, as the case may be,

© The Institute of Chartered Accountants of India


PAPER – 6D: ECONOMIC LAWS 19

under this Code, the resolution professional, liquidator or bankruptcy trustee, is of the opinion
that assets of the corporate debtor or debtor, are situated in a country outside India with which
reciprocal arrangements have been made under section 234 of Insolvency and Bankruptcy
Code, 2016, he may make an application to the Adjudicating Authority that evidence or action
relating to such assets is required in connection with such process or proceeding.
The Adjudicating Authority on receipt of an application and, on being satisfied that evidence or
action relating to assets, is required in connection with insolvency resolution process or
liquidation or bankruptcy proceeding, may issue a letter of request to a Court or an authority of
such Country competent to deal with such request. [Section 235]
Accordingly, in the given case, order of the Adjudicating Authority of rejection of filing an
application under IBC, 2016 by MCL (a Company registered in Germany) is not in order because
as per Section 235, the Adjudicating Authority on receipt of an application on being satisfied
that evidence or action relating to assets, is required in connection with in solvency resolution
process or liquidation or bankruptcy proceeding, may issue a letter of request to a Court or an
authority of such Country competent to deal with such request.
Answer 3.6 (ii)
Section 53 of the Insolvency & Bankruptcy Code, 2016 lays the provisions related to distribution
of assets or the proceeds from the sale of the liquidation assets.
Distribution of proceeds from the sale of the liquidation assets: The proceeds from the sale
of the liquidation assets shall be distributed in the following order of priority —
(a) the insolvency resolution process costs and the liquidation costs paid in full;
(b) the following debts which shall rank equally between and among the following :—
(i) workmen's dues for the period of twenty-four months preceding the liquidation
commencement date; and
(ii) debts owed to a secured creditor in the event such secured creditor has relinquished
security in the manner set out in section 52;
(c) wages and any unpaid dues owed to employees other than workmen for the period of
twelve months preceding the liquidation commencement date;
(d) financial debts owed to unsecured creditors;
(e) the following dues shall rank equally between and among the following: —
(i) any amount due to the Central Government and the State Government including the
amount to be received on account of the Consolidated Fund of India and the
Consolidated Fund of a State, if any, in respect of the whole or any part of the period
of two years preceding the liquidation commencement date;

© The Institute of Chartered Accountants of India


20 FINAL (NEW) EXAMINATION: NOVEMBER 2020

(ii) debts owed to a secured creditor for any amount unpaid following the enforcement of
security interest;
(f) any remaining debts and dues;
(g) preference shareholders, if any; and
(h) equity shareholders or partners, as the case may be.
Realisable value of the fixed assets + realisable value of receivables (50% of Rs. 225 lakhs) +
Bank Balance Amount=
(Rs 2800 + 112.5 + 22.5) lakhs = Rs. 2935 lakhs
Less outstanding wages to workmen = Rs.75 lakhs*
Less unpaid dues on account of statutory employer’s contribution treating them as workmen’s
dues = 30 lakhs
Less amount debts owed to a secured creditor** = (1500 + 1050) = Rs. 2550 lakhs
Less Loan taken from Mr. X = 75 lakhs
Balance amount available = 2935 – (75+30+2550+75) lakhs = 205 lakhs (which to be shared
between SSTPL and ACL***)
Therefore, amount receivable by SSTPL (205 / 980****x905) = Approx. Rs. 189.31 lakhs.
* It is assumed that outstanding wages of Rs. 75 Lakhs due to the workmen relate to the period
of 24 months preceding to the date of commencement of liquidation. [The question does not
mention the date of commencement of liquidation. Moreover, the term IRP needs to be replaced
by the term Liquidator since it is a case of Liquidation of a Corporate Person].
** It is assumed that the both the banks have relinquished their security inter est and their
securities have been realized by the Liquidator for inclusion in the Liquidation estate. [In fact,
consolidated amount of Rs. 2800 lakhs being the realizable value of fixed assets validates this
assumption.]
***In respect of MCL, a Company registered in Germany, the Adjudicating Authority (AA) has
rejected its application filed under IBC, 2016. Further, no direction has been issued by the AA
regarding the outstanding amount of Rs. 400 lakhs. MCL has also not approached the Appellate
Authority for revival of rejected application. No stay order has been issued favouring MCL. In
addition, MCL has not filed any suit in Germany against DMPL for recovery of dues till the date
of commencement of liquidation. In such a case, from the facts of the questions which are not
elaborated in nature and from the limited information available, MCL cannot be treated as an
operational creditor at par with SSTPL or ACL.
**** After considering SSTPL (Rs. 905 lakhs) and ACL (Rs. 75 lakhs) as operational creditors.

© The Institute of Chartered Accountants of India


PAPER – 6D: ECONOMIC LAWS 21

Answer 3.6 (iii)


Any agreement entered into between enterprises or associations of enterprises or persons or
associations of persons or between any person and enterprise or practice carried on or decision
taken by any association of enterprises or association of persons, including cartels’ engaged in
identical or similar trade of goods or provision of services shall be presumed to have an adverse
effect on competition which:
(a) directly or indirectly determines purchase or sale prices
(b) limits or contracts production, supply, markets, technical development, investment or
provision of services
(c) shares the market or source of production or provision of services by way of allocation of
geographical area of market or type of goods or services or number of customers in the
market or any other similar way.
(d) directly or indirectly results in bid rigging or collusive bidding
Any agreement entered into between enterprises or persons at different levels of the production
chain in different markets in respect of production, supply, distribution, storage, sale or price of
trade in goods or provision of services shall be a void agreement if it causes or is likely to cause
an appreciable adverse effect on competition in India including:
(a) Tie-in agreement
(b) Exclusive supply agreement
(c) Exclusive distribution agreement
(d) Refusal to deal
(e) Resale price maintenance
Accordingly the clauses proposed in the revise draft agreement by SSTPL, is limiting and
restricting to the production of goods or provision of services or market therefore specifying
geographical areas where dealers can market the cars , restricting technical or scientific
development relating to goods or services to the prejudice of consumers; resulting in denial of
market access by limiting the operations of service centers, bar on the transactions of car and
spares between dealers themselves, imposes unfair price in purchase or sale by mandating
floor price, higher pricing of substitutable products and services and imposing the dealers to
acquire certain numbers of cars while ordering high end variants
These all terms of agreement entered by SSTPL with dealers shows the abuse of dominant
position as per section 4 of the Competition Act, 2002.

© The Institute of Chartered Accountants of India


22 FINAL (NEW) EXAMINATION: NOVEMBER 2020

CASE STUDY 4
The decade of 1960 was known as the golden period for goldsmiths in India and there was
tremendous interest in the minds of the people to buy and wear gold jewelry. Hard work and
expertise in making this jewelry made many goldsmiths millionaires in a very short period. Two
such goldsmiths were Mr. Selva Chetty and Mr. Thiagu Chetty, brothers who lived in Sivaganga
district, Tamil Nadu. Using the boom period, the Selva ventured to start several new business,
one of which was a small real estate company called Gangaikondan Holiday Properties Limited
(GHPL).
In the year 1970, Mr. Thiagu migrated to the United Kingdom and started his jewelry business
there. He used to visit India every year and give substantial sums to Mr. Selva to invest in India
on behalf of Mr. Thiagu and for his benefit to use once he comes back to India Mr. Selva
mentioned to him that it may be worthwhile to invest the money in buying large tracts of land
near Sivaganga and the same is expected to appreciate significantly in the next 10 years.
Mr. Thiagu was very much interested in this and therefore, in the year 1989, Mr. Selva
purchased 10 acres of land from the Government in his name, in the capacity as fiduciary
relationship/trustee of Mr. Thiagu and hold the property on behalf of and for the benefit of
Mr. Thiagu. Mr. Selva used the land for cultivation of crops and was using the crops for his
consumption and for sale. The proceeds from the sale was deposited by Mr. Selva in his bank
account.
In the meantime, Mr. Selva got married and was blessed with a son Mr. Venkat. In the year
1971, when Mr. Venkat was 6 years old, Mr. Selva acquired a new residential house comprising
of 4 individual units in the name of Mr. Venkat since he felt that buying the new home in his
son's name will be auspicious for Mr. Selva and the new home. For this purpose, Mr. Selva took
a 5 year loan from Bank of Sivaganga and was repaying the loans promptly on the due dates
and got back the title deeds from the Bank once the loan was repaid. The new home was
occupied by Mr. Selva and his family and Mr. Selva rented out 2 portions on rent to tenants. Mr.
Selva paid the property taxes for the property and maintained the property on his own account.
In 1980, Mr. Selva was blessed with another child who was named Ms. Bhagyalakshmi. In 1984,
Mr. Selva prepared his will as per which he considered that the residential house will belong to
Mr. Venkat and Ms. Bhagyalakshmi in equal measure, which was not disclosed to anyone.
GHPL commenced construction of a large apartment complex in an upcoming industrial belt of
Sivaganga. There was tremendous expectation that several large companies were going to set
up factories in the location and therefore, the demand for housing expanded significantly. A lot
of housing companies commenced projects in the location.
In one of the discussions between the real estate companies, GHPL was approached by other
leading real estate developers who were constructing high rise apartments in the vicinity to have
a tacit (unwritten) understanding for jacking up the prices of the apartments and also in
unbundling of the open car parking given to the allottees from the total price and charging
separately for the same. This would help the companies in providing the best-in-class facilities
to the apartment buyers at the same time ensure good profitability for the companies. GHPL did

© The Institute of Chartered Accountants of India


PAPER – 6D: ECONOMIC LAWS 23

not immediately agree to the same but wanted to evaluate the implications of such an
agreement. One of the real estate developers wanted to extend the understanding to the
infrastructure projects by these companies in UAE also (since many of them are constructing
homes in UAE as well).
In the year 1986, Mr. Venkat got married and declared that he is the absolute owner of the
residential house since the house is in his name and was purchased by his father in his name
purely for his benefit when he was a minor and to help him settle down in his life. He then asked
for vacation of the property by Mr. Selva and his family as well as the tenants. Mr. Selva was
enraged by this act of Mr. Venkat and filed a suit for declaring the property as a benami property
where Mr. Venkat was a benamidar and he was the rightful owner of the same. They discussed
the matter with various consultants for determination of a benami transaction as decided by
Hon'ble Supreme Court of India.
In May 2017, GHPL is evaluating the acquisition of another large real estate company in
Sivaganga and is contemplating the implications of the Competition Act, 2002 in this regard.
Answer the following questions:
4.1 The CFO of GHPL seeks your views to understand which of the following would not be a
violation of the provisions of the Competition Act, 2002?
(A) Predatory Pricing.
(B) Limiting production of goods.
(C) Agreement for Protection of rights under the Designs Act, 2000.
(D) Denial of market access.
4.2 What is the term of the members of the Competition Commission under the Competition
Act, 2002 which is reviewing the agreement / tacit understanding between the real estate
companies in the case study?
(A) 5 years, eligible for re-appointment for one more term.
(B) 5 years, eligible for re-appointment.
(C) 5 years, not eligible for re-appointment.
(D) Upto the discretion of the Central government.
4.3 Assuming that the acquisition of another real estate company by GHPL happened in the
year 2019, what is the maximum amount of assets and revenue that can be acquired by
GHPL for being accepted from the provisions of Section 5 of the Competition Act, 2002
(A) Post-acquisition (incl. GHPL) asset value off ` 350 crore and `1000 crore
respectively.

© The Institute of Chartered Accountants of India


24 FINAL (NEW) EXAMINATION: NOVEMBER 2020

(B) Asset value off ` 350 crore and turnover off ` 1000 crore of the target entity being
acquired.
(C) Post-acquisition (incl. GHPL) value off ` 1000 crore or turnover of ` 3000 crore of the
target entity.
(D) Asset value off ` 350 crore or turnover of ` 1000 crore of the target entity being
acquired.
4.4 Assuming that the proposed combination is covered under Section 5 of the Competition
Act, 2002, and GHPL gave notice to the Commission on 15 th May, 2018, what is the latest
date by when the combination will come into effect (no orders have been passed by the
Commission)?
(A) 13th August 2018.
(C) 15th May 2019.
(B) 11th December 2018.
(D) 11th November 2018.
4.5 Under the Prohibition of Benami Property Transactions Act, 1988, who is responsible for
issuing notice for furnishing evidence to Selva and Venkat?
(A) Approving Authority
(C) Adjudicating Authority.
(B) Initiating Officer
(D) Administrator. (5 x 2 = 10 Marks)
4.6 Answer the following questions:
(I) Discuss the judicial pronouncements on tests for determination of a benami
transaction as decided by Hon'ble Supreme Court of India under Prohibition of
Benami Property Transactions Act, 1988. (6 Marks)
(II) Analyse the case with regard to Mr. Selva's contention regarding the house
purchased by him in the name of Mr. Venkat and Mr. Selva's rights under the
Prohibition of Benami Property Transactions Act, 1988 to recover the property.
(4 Marks)
(III) GHPL reaches out to you for your advice regarding the proposal from the other real
estate developers under the Competition Act, 2002. (5 Marks)

© The Institute of Chartered Accountants of India


PAPER – 6D: ECONOMIC LAWS 25

ANSWER TO CASE STUDY 4


4.1 Option (C)
4.2 Option (B)
4.3 Option (C)
4.4 Option (B)
4.5 Option (A)
Answer 4.6
(I) Judicial pronouncements on tests for determination of a benami transaction:
In the matter of Bhim Singh & Anr vs Kan Singh (And Vice Versa) 1980 AIR 727, 1980 SCR
(2) 628, the Hon’ble Supreme Court of India, observed –
The principle governing the determination of the question whether a transfer is a benami
transaction or not may be summed up thus:
(a) The burden of showing that a transfer is a benami transaction lies on the person who
asserts that it is such a transaction;
(b) if it is proved that the purchase money came from a person other than the person in
whose favour the property is transferred, the purchase is prima facie assumed to be
for the benefit of the person who supplied the purchase money, unless there is
evidence to the contrary;
(c) the true character of the transaction is governed by the intention of the person who
has contributed the purchase money and
(d) the question as to what his intention was has to be decided on
(i) the basis of the surrounding circumstances,
(ii) the relationship of the parties,
(iii) the motives governing their action in bringing about the transaction and
(iv) their subsequent conduct etc.
All the four factors stated above may have to be considered cumulatively [O P Sharma
vs. Rajendra Prasad Shewda & Ors. (CA 8609-8610 of 2009) (SC)].
In the matter of Valliammai(D) by LRS.V.Subramaniam and Others (2004) 7 SCC
2330 the Honorable Supreme Court observed that the essence of a benami
transaction is the intention of the party or parties concern and often, s uch intention is
shrouded in a thick veil which cannot be easily pierced through. But such difficulties

© The Institute of Chartered Accountants of India


26 FINAL (NEW) EXAMINATION: NOVEMBER 2020

do not relieve the person asserting the transaction to be benami of any part of the
serious onus that rests on him nor justify the acceptance of mere co njectures or
surmises as a substitute for proof.
Answer 4.6 (II)
"Benami transaction" as per Section 2(9) of the Prohibition of Benami Transaction Act, 1988
means, a transaction or an arrangement where the property is held for the immediate or future
benefit, direct or indirect, of the person who has provided the consideration, except when the
property is held by any person being an individual in the name of his spouse or in the name of
any child of such individual and the consideration for such property has been provided or paid
out of the known sources of the individual.
In the instant case, Mr. Selva purchased the house in the name of his son Mr. Venkat through
a 5 year bank loan and used 2 units for his family and rented out 2 portions on rent.
In the light of the above provisions, the said transaction is not a benami transaction and Mr.
Venkat is not a benamidar and is a real owner.
Right of Mr. Selva under Section 4 of the PBPTA, 1988
No suit, claim or action to enforce any right in respect of any property held benami against the
person in whose name the property is held or against any other person shal l lie by or on behalf
of a person claiming to be the real owner of such property.
Moreover, the transaction in question was registered in the year 1978. The suit was filed in the
year 1986, which was before coming into force of the PBTP Act in 1988. Since, the PBTP Act
cannot have any retrospective applicability.
Accordingly, Mr. Selva’s right is prohibited to recover the property.
Answer 4.6 (III)
As per Section 3 of the Competition Act, 2002, any agreement entered into between
enterprises or associations of enterprises or persons or associations of persons or between any
person and enterprise or practice carried on, or decision taken by, any association of enterprises
or association of persons, including cartels, engaged in identical or similar trade of goods or
provision of services, shall be presumed to have an appreciable adverse effect on competition,
which—
(a) directly or indirectly determines purchase or sale prices;
(b) limits or controls production, supply, markets, technical development, investment or
provision of services;
(c) shares the market or source of production or provision of services by way of allocation of
geographical area of market, or type of goods or services, or number of customers in the
market or any other similar way;

© The Institute of Chartered Accountants of India


PAPER – 6D: ECONOMIC LAWS 27

(d) directly or indirectly results in bid rigging or collusive bidding.


However, any agreement entered into by way of joint ventures, if such agreement increases
efficiency in production, supply, distribution, storage, acquisition or control of goods or provision
of services, shall not be considered to be an anti-competitive agreement.
Therefore the proposal of other leading real estate to have understanding with GHPL, in the
light of facts, will increase efficiency in providing best class facilities to the apartment buyers
and at the same time ensure good profitability for the companies. This proposal shall not be an
anti-competitive agreement.
CASE STUDY 5
An Investigation was carried out at the office of WWL Mumbai by the Assistant Director under
the Prevention of Money Laundering Act, 2002, in the process they came across violation of
the Foreign Exchange Management Act,1999. The Assistant Director discussed the case with
you and apprised the matter as under:-
WWL is based in Mumbai and is India's premier watch manufacturing company and specializes
in designing and manufacturing high-end watches. Its products are sold across premier stores
in India and abroad. WWL was established by Mr. Virender Kohli, a first-time entrepreneur. The
marketing department of WWL introduced new models in the past 4 months and expects these
watches to be a major attraction in the global markets especially UK, France and US markets.
For the purpose of advertisements, WWL engaged the services of Mr. George Mckenzie, a
prominent NBA player and Ms. Rudy Hobbs, a Miss Universe winner and agreed to pay a
"guaranteed" fee of USD 1,000,000 each plus 5% bonus based on the sales of the new models
in 1st year. The marketing strategy was highly successful and Virender earned a significant
amount through the sale of 10% stake in WWL to a private equity investor.
This was invested in his various businesses to acquire agricultural farm land (to grow and export
opium), acquiring and selling (export) of antiquities etc. A Marks majority of his dealings on the
farm and antiquities businesses were done through cash transactions or through a specific bank
account maintained with ABC Bank Limited. Amounts were received in cash from his
international customers through a hawala agent known to Mr. Virender. He also purchased villas
in India and in Spain using the money earned through his farm and antiquities businesses. Mr.
Virender also established Sure Returns Private Limited, a small non-banking finance company
for securing the lives of his employees and their families. Virender invested an amount of `5
crore in Sure Returns out of the funds received from his antiquities business.
WWL sent 10 watches to his 500 dealers abroad, clearly marked as riot for sale and other
promotional material, for display in dealer shops etc. The value of the items were approximately
INR 6 crore. He also sent 1 watch for each of his dealers as a token of gift and appreciation
(total value of INR 40 lakh). The CFO of WWL is of the view that since these products have
been sent free of cost and not for sale, these need not be included in the export declaration to
be filed by WWL.

© The Institute of Chartered Accountants of India


28 FINAL (NEW) EXAMINATION: NOVEMBER 2020

Mr. Virender attended one of the manufacturing conferences held in Mumbai, in which he met
one Mr. Alex Smith, who runs a watch designing studio in Italy and showed quite a few exhibits
to Mr. Virender. Mr. Virender was impressed by the designs and the prices quoted by Alex. Alex
was also amenable to receive funds in cash in India through an intermediary and then provide
the material to Virender from Italy. Based on the same, Mr. Virender arranged for making cash
payment to the extent of INR 3 crore to an intermediary in Delhi and the material was received
from Alex in a month. During his visit to India, Alex noted that his Euro passport got expired and
he did not realise the same. Since he did wanted to leave India immediately, he got in touch
with a travel agent, who helped him get a forged passport, for which Mr. Alex paid INR 3 lakh in
cash.
In order to clear the imported material critical for its manufacturing process, WWL used cash
amounting to INR 30 lakhs to pay amounts to various intermediaries to facilitate timely and
smooth import process and the amounts were paid by the intermediaries to Mr. Raghav Kapoor.
Using this money, Mr. Raghav purchased a 1 acre farm house in Munnar in the name of his
spouse, Ms. Anu Kapoor, who was not aware of the source of the funds and was residing in the
farm house along with her parents. The Enforcement Directorate, as p art of the proceedings
against Mr. Raghav Kapoor sought to attach and confiscate the farm house owned / purchased
in the name of Ms. Anu. This was challenged by Mr. Raghav on the basis that this property was
owned and possessed by Anu who is not charged under a scheduled offence under the
Prevention of Money Laundering Act, 2002. With Mr. Alex's help, Mr. Virender transferred an
amount of INR 260 lakh to an intermediary in Delhi and invested the amount to incorporate a
shell company in the Isle of Mann. The funds were then transferred back by the Shell Compan y
to the bank account of WWL. For this purpose, WWL raised export invoices in its books on the
Shell Company for providing professional services relating to watch designing. Based on these
invoices, WWL claimed export incentives under the relevant laws in India and received INR 15
lakh as export incentive.
On 30th March 2018, WWL made a large sale to one of the dealers in Switzerland for EURO 8
million and had received EURO 3 million by 15 th May 2018 and did not receive the balance
EURO 5 million until 30 th October 2018, i.e. 7 months from the date of sale. After several
reminders and threating calls to stop further shipment, another EURO 1 million was received on
10th October 2018 and the balance remained outstanding as at 31 st December 2018. The CFO
of WWL reaches out to Mr. Z and seek Mr. Z support to evaluate the level of compliances as
stipulated under the Foreign Exchange Management Act, 1999.
Based on investigation carried out, the Assistant Director sought to arrest Viren der and also
wanted to attach the property for contravention of provision of Prevention of Money laundering
Act, 2002 (in short 'PMLA, 2002')
After the discussions the Assistant Director sought your views on powers for attachment of
property involved in money-laundering and on punishment for the offence of money laundering
under the provisions of Prevention of Money Laundering Act, 2002.

© The Institute of Chartered Accountants of India


PAPER – 6D: ECONOMIC LAWS 29

Answer the following questions:


5.1 Out of the below, which are the items that require inclusion in the export declara tion by
WWL under the, Foreign Exchange Management Act, 1999?
(A) Goods imported free of cost for re-export.
(B) Publicity materiality supplied free of cost; ·
(C) Gift of goods for a value of INR 10 lakh.
(D) Unaccompanied personal effects of travellers.
5.2 Out of the below, what is not part of the responsibility of ABC Bank Limited under the
Prevention of Money Laundering Act, 2002?
(A) Report suspicious transactions undertaken by Mr. Virender and the Group;
(B) Furnish all-information requested by the Director;
(C) Verify the identity of the clients and beneficial owners;
(D) Maintain records of transaction for a period of 5 years;
5.3 A friend of Mr. Virender is an Indian citizen resident outside India, is seeking to transfer his
agricultural property held by him in India. Who can he transfer the property to?
(A) Any person resident in India.
(B) Any person resident outside India if he is a citizen of India or a person of Indian origin.
(C) Any person resident in India and any person resident outside India if he is a citizen
of India or a person of Indian origin.
(D) Neither any person resident in India nor any person resident outside India if he is a
citizen of India or a person of Indian origin.
5.4 Mr. Virender bought gold watches worth INR 25 lakh from Italy through the green channel
which he asked his Italian dealer to pay and deduct from their monthly payments to WWL.
Is this an offence under the Prevention of Money Laundering Act, 2002?
(A) Yes, because he came through the green channel and evaded duty of customs.
(B) No, whilst it is an offence, it is not actionable under the Prevention of Money
Laundering Act, 2002.
(C) No, since he did not pay any cash for the purchase.
(D) Yes, since import of gold items from European countries requires specific consent as
per the agreement entered with foreign countries as per Section 56 of Prevention of
Money Laundering Act, 2002.

© The Institute of Chartered Accountants of India


30 FINAL (NEW) EXAMINATION: NOVEMBER 2020

5.5 Does the Assistant Director have powers to arrest a person under the Prevention of Money
Laundering Act, 2002?
(A) Director or Deputy Director or Assistant Director have the powers to arrest an offender
without prior approval of Central Government
(B) Any arrest under the Prevention of Money Laundering Act, 2002 requires the prior
approval of the Central Government
(C) Only a Director or Deputy Director have the powers to arrest without prior approval of
the Central Government
(D) Any arrest under the Prevention of Money Laundering Act, 2002 requires the prior
approval of the special court. (5 x 2 = 10 Marks)
5.6 Answer the following questions:
(I) The Enforcement Directorate wanted to take your view on powers for attachment of
property involved in money-laundering and your views on punishment for the offence
of money laundering under the provisions of the Prevention of Money Laundering Act,
2002. Express your views on the same. (7 Marks)
(II) The Enforcement Directorate, as part of the proceedings against Mr. Raghav Kapoor
sought to attach and confiscate the farm house owned /purchased by Anu, This was
challenged by Mr. Raghav on the basis that this property was owned and possessed
by Anu who is not charged under a scheduled offence under the Prevention of Money
Laundering Act, 2002. Advice Mr. Raghav on the validity or otherwise of his
contention. (4 Marks)
(III) The CFO of WWL reaches out to Mr. Z and seek Mr. Z support to evaluate if there is
a non-compliance under the Foreign Exchange Management Act, 1999 regarding the
sale made to the dealer in Switzerland and the receipt of the proceeds and if so, the
quantum, the consequences and the future course of action that needs to be taken
by WWL relating to the same. (4 Marks)
ANSWER TO CASE STUDY 5
5.1 Option (C)
5.2 Option (A)
5.3 Option (A)
5.4 Option (A)
5.5 Option (B)

© The Institute of Chartered Accountants of India


PAPER – 6D: ECONOMIC LAWS 31

Answer 5.6 (I)


Attachment of property involved in money-laundering [Section 5 of the Prevention of
Money Laundering Act, 2002]
1. Where the Director or any other officer (not below the rank of Deputy Director authorized
by the Director) for the purposes of this section, has reason to believe on the basis of
material in his possession, that—
(a) any person is in possession of any proceeds of crime; and
(b) such proceeds of crime are likely to be concealed, transferred or dealt with in any
manner which may result in frustrating any proceedings relating to confiscation of
such proceeds of crime under this Chapter,
he may, by order in writing, provisionally attach such property for a period not exceeding
one hundred and eighty days from the date of the order, in such manner as may be
prescribed.
Conditions for Attachment: Provided that no such order of attachment shall be made
unless, in relation to the scheduled offence:
- a report has been forwarded to a Magistrate under Section 173 of the Code of
Criminal Procedure, 1973, or
- a complaint has been filed by a person authorized to investigate the offence
mentioned in that Schedule, before a Magistrate or Court for taking cognizance of the
scheduled offence, as the case may be, or
- a similar report or complaint has been made or filed under the corresponding law of
any other Country.
2. The Director, or any other officer not below the rank of Deputy Director, shall, immediately
after attachment forward a copy of the order, along with the material in his possession, to
the Adjudicating Authority, and such Adjudicating Authority shall keep such order and
material for such period as may be prescribed.
3. Every order of attachment made shall cease to have effect after the expiry of the period
specified in that sub-section or on the date of an order made under sub-section (3) of
Section 8, whichever is earlier.
4. The Director or any other officer who provisionally attaches any property under sub-section
(1) shall, within a period of thirty days from such attachment, file a complaint st ating the
facts of such attachment before the Adjudicating Authority.

© The Institute of Chartered Accountants of India


32 FINAL (NEW) EXAMINATION: NOVEMBER 2020

Section 4 provides for the Punishment for Money-Laundering - Whoever commits the offence of
money-laundering shall be punishable with rigorous imprisonment for a term which shall not be
less than three years but which may extend to seven years and shall also be liable to fine.
But where the proceeds of crime involved in money-laundering relate to any offence under the
Narcotic Drugs and Psychotropic Substances Act, 1985, the maximum punishment may extend
to ten years instead of seven years.
Answer 5.6 (II)
Section 2(1)(u) of the Prevention of Money Laundering Act, 2002 , "proceeds of crime" can
be understood as any property derived or obtained, directly or indirectly, by any person as a
result of criminal activity relating to a scheduled offence or the value of any such property or
where such property is taken/held outside the country, then the property equivalent in value held
within the country or abroad.
As per the stated facts, farm house was purchased by Mr. Raghav on the name of his spouse
Ms. Anu who was not aware of sources of the funds. ED sought to attach the farm house and
confiscate as a part of proceeding against Mr. Raghav. Here the contention of Mr. Raghav is
not valid because the said property was derived from the proceeds of crime.
Answer 5.6(III)
Period within which export value of goods/software/ services to be realized: -
(1) The amount representing the full export value of goods / software/ services exported shall
be realized and repatriated to India within nine months or within such period as may be
specified by the Reserve Bank, in consultation with the Government, from time to time.
(a) that where the goods are exported to a warehouse established outside India with the
permission of the Reserve Bank, the amount representing the full export value of
goods exported shall be paid to the authorised dealer as soon as it is realis ed and in
any case within fifteen months or within such period as may be specified by the
Reserve Bank, in consultation with the Government, from time to time;
(b) further that the Reserve Bank, or subject to the directions issued by that Bank in this
behalf, the authorised dealer may, for a sufficient and reasonable cause shown,
extend the said period, as the case may be.
Delay in Receipt of Payment:
Where in relation to goods or software export of which is required to be declared on the
specified form and export of services, in respect of which no declaration forms has been
made applicable, the specified period has expired and the payment therefor has not been
made as aforesaid, the Reserve Bank may give to any person who has sold the goods or
software or who is entitled to sell the goods or software or procure the sale thereof, such
directions as appear to it to be expedient, for the purpose of securing,

© The Institute of Chartered Accountants of India


PAPER – 6D: ECONOMIC LAWS 33

(a) the payment therefor if the goods or software has been sold and
(b) the sale of goods and payment thereof, if goods or software has not been sold or re-
import thereof into India as the circumstances permit, within such period as the
Reserve Bank may specify in this behalf;
Provided that omission of the Reserve Bank to give directions shall not have the effect of
absolving the person committing the contravention from the consequences thereof.
Quantum: In the given case, out of total sale of EUR 8 million, an amount of EUR 4 million
was received within the stipulated time period of 9 months and the balance EUR 4 million
is outstanding for a period of more than 9 months. Accordingly, WWL is required to apply
for an extension of time with the Authorized Dealer giving sufficient and reasonable
reasons for the delay in receipt.
As per Section 8, where any amount of foreign exchange is due or has accrued to any
person resident in India, such person shall take all reasonable steps to realize and
repatriate to India such foreign exchange within such period and in such manner as may
be specified by the Reserve Bank. WWL will act in compliance with the above provisions.

© The Institute of Chartered Accountants of India


DISCLAIMER

This Suggested Answers do not constitute the basis for evaluation of the student’s answers in

the examination. The answers are prepared by the Faculty of the Board of Studies with a view

to assist the students in their education. While due care is taken in preparation of the answers,

if any errors or omissions are noticed, the same may be brought to the attention of the Director

of Studies. The Council of the Institute is not in anyway responsible for the correctness or

otherwise of the answers published herein.

Further, in the Elective Papers which are Case Study based, the solutions have been worked

out on the basis of certain assumptions/views derived from the facts given in the question or

language used in the question. It may be possible to work out the solution to the case studies

in a different manner based on the assumption made or view taken.

© The Institute of Chartered Accountants of India


2 FINAL (NEW) EXAMINATION: NOVEMBER, 2019

PAPER 6D: ECONOMIC LAWS

The solutions to case studies have been worked out on the basis of certain assumptions/views
derived from the facts given in the question or language used in the question. It may be
possible to work out different solutions based on the assumption made or view taken. Further,
there should be no negative marking for wrong answers in MCQ based questions.

NOTE: There are five case study questions in the question paper. Candidates are
required to answer all the questions of any four case study questions.
Citation of case laws, sections, subsections, rules may not form part of the answer.

CASE STUDY 1
1. Delta Corporation, a government corporation purchases Aluminium Phosphide Tablets
(APT) on bulk basis through a formal tender process for the past several years. The main
market of APT in India was that of the institutional sales and a majority of buyers were
Government agencies. The number of private buyers was insignificant.
APT is manufactured only by 4 companies in the country, namely M/s. Easy,
M/s. Samurai, M/s. Multicrop and, M/s. Agro Chemicals. Sometime during the year 2018,
Mr. Rohit the Chairman and Managing Director of Delta Corporation, as part of his review
of the operations, analysed the purchase of APT over the last several years, and noted a
trend that the four manufacturers of APT had formed a cartel by entering into an
anticompetitive agreement amongst themselves and on that basis they had been
submitting their bids for last eight years by quoting identical rates in the tenders invited
by the Delta Corporation for the purchase of APT. Based on the above, Mr. Rohit wrote a
complaint to the Competition Commission of India (CCI) on February 4, 2018 and the CCI
assigned the complaint to the Director General (DG) for investigation.
Based on the investigation carried out, the DG noted the following:
• Right from the year 2009, upto the year 2016, all the four parties used to quote
identical rates, excepting for the year 2014. In 2009, ` 245 was the rate quoted by
these four parties and in the year 2012 it was ` 310 (though the tender was
scrapped in this year). In November, 2012, though the tenders were invited, all the
parties had abstained from quoting. In 2014, M/s. Samurai had quoted the price
which was much below the price of other competitors. In 2015, all the parties
abstained from quoting, while in 2016 only the three appellants, barring Agro
Chemicals, participated and quoted uniform rate of 388, which was ultimately
brought down to ` 386 after negotiations.

© The Institute of Chartered Accountants of India


PAPER – 6D: ECONOMIC LAWS 3

• It was also found that the tender documents were usually submitted in-person and
the rates were normally filled with hand;
• In respect of the tender floated in March, 2016, the three appellants had quoted
identical rates of ` 388.
• The DG also analysed the bidding pattern for tenders issued by other corporations
during the period from 2014 to 2018 and concluded that the pricing pattern was
similar between the parties in such tenders as well, as indicated below :
Corporations Year Price Quoted
Easy Samurai Multicrop Agro Chemical
A 2014 225 225 - -
B 2015 260 260 - -
C 2015 450 - 450 -
C 2016 414 414 - -
Delta 2016 388 388 388 -
B 2016 399 - - 399
D 2016 - - 399 399
B 2017 419 - - 410
C 2017 421 421 421 -
B 2018 - 415 - 415

Based on the investigation carried out above, the DG concluded that:


• The pricing pattern definitely showed the practice of quoting identical pricing by all
the parties.
• The explanation given by the parties (rise in price was mostly attributed to increase
in price by China) for the common pricing was unconvincing since it was noticed
that even during the period when the Phosphorours prices had fallen, no reflection
thereof was seen in the high prices quoted by the parties.
• Examination of the cost structure of each company reflected that there was nothing
common between the parties as far as the said cost structure was concerned and,
therefore, quoting of identical prices by all the parties was unnatural.
• Joint boycotting by the parties, at times, showed their concerted action, which
happened again in March, 2018 when the Delta Corporation had issued e-tender,
which was closed on July 25, 2018.

© The Institute of Chartered Accountants of India


4 FINAL (NEW) EXAMINATION: NOVEMBER, 2019

On the basis of the aforesaid findings, the DG framed an opinion that the appellants had
contravened the provisions of Sections 3(3)(a), 3(3)(b) and 3(3)(d) read with Section 3(1) of
the Competition Act, 2002.
The CCI called for the responses of the parties for the above observations of the DG and the
responses of the parties are as under:
• In so far as tender of 2018 is concerned, it was contended that inquiry in respect of
boycotting the said tender by the appellants was without jurisdiction in as much as the
Delta Corporation in its complaint dated February 04, 2018 did not mention about the
said tender.
• On the merits, increase in the price over a period of time, particularly between years
2016 and 2018, was sought to be justified on the ground that the "price of yellow
phosphorous, which was to be procured from China, had increased". It was further
submitted that merely because there was identical prices quoted by the parties, it would
not mean that there was any bid rigging or formation of cartel by the parties. Submission
in this behalf was that· the market forces brought the situation where the prices became
so competitive and it had led to the aforesaid trend.
• It was further submitted that, notwithstanding the same price quoted by the parties, each
time the tender was evaluated by a Committee of Officers of the Delta Corporation and
no such suspicion was raised by the Committee. On the contrary, this aspect was
specifically gone into and the Committee was satisfied that quoting of identical price was
not due to any cartelisation.
The CCI rejected each of the responses provided by the parties and concluded 'that the
parties had entered into an agreement or understanding, and indulged in anti-competitive
activities while submitting their bids in response to the tenders issued by the Delta
Corporation.
Prosper Extractors Limited (PEL) is one of the key operational creditors of Multicrop and was
the sole supplier of Phosphorous to Multicrop for the manufacture of the APT. The
arrangement between PEL and Multicrop was formally documented through a blanket
Purchase Order on an annual basis with weekly supply schedule and a 30 days credit period.
Due to the financial issues including losses of Multicrop, there was a significant backlog in the
payment by Multicrop and in line with the terms of the purchase order, the matter was referred
to an Arbitral Tribunal with claims and counter claims by both parties. The Arbitral Tribunal
delivered its award in favour PEL for the entire balance payable (including receivables
assigned to the bank without recourse basis) by Multicrop and rejected the cross claims of
Multicrop. Multicrop proceeded to file a petition under the Arbitration and Conciliation Act,
1996 challenging the award of the Arbitral Tribunal. Based on the opinion of CFO that the
object of IBC, 2016 is also to hold promoters personally financially liable for default of the
firms they control, an application was then filed by PEL under Section 9 of the IBC, 2016 as
the sole operational creditor of Multicrop. The NCLT, based on the application; admitted the

© The Institute of Chartered Accountants of India


PAPER – 6D: ECONOMIC LAWS 5

same since there is a clear evidence of a demand and the appropriate notice has been
submitted by PEL as per the IBC, 2016.
Answer the following questions :
1.1 Which of the following is not part of the objectives for introduction of the IBC, 2016?
(A) Avoiding destruction of value.
(B) Hold Promoters personally financially liable for default of the firms that they control
as opined by CFO in the case study.
(C) Improve handling of conflicts between creditors and debtor through process of
negotiation.
(D) Clear allocation of losses during downturn. (2 Marks)
1.2 Which of the following is not covered under the definition of a financial debt under IBC,
2016?
(A) Interest on Unsecured debentures issued by a corporate debtor.
(B) Market value of a derivative taken to hedge foreign currency fluctuations of an ECB
loan.
(C) Amount raised from an allottee of an apartment under a real estate project.
(D) Receivables assigned to a Bank on without recourse basis. (2 Marks)
1.3 The IRP appointed for Multicrop is seeking your views on the constitution of the
Committee of Creditors of Multicrop. Multicrop does not have any financial debt other
than a loan obtained from Mr. Ajay Jhawar, son of the Mr. Vijay Jhawar, the Managing
Director of Multicrop. Considering the above, identify the appropriate constitution of the
Committee of Creditors out of the following :
(A) Mr. Ajay Jhawar, 18 largest operational creditors, and 1 representative of all
workmen.
(B) 18 largest operational creditors, 1 representative of workmen and 1 representative
of employees.
(C) Only Mr. Ajay Jhawar since he is the only financial creditor.
(D) 18 largest operational creditors, 1 representative of workmen and 1 representative
of employees and the resolution professional. (2 Marks)
1.4 Which of the following are not factors which need to be considered for determining the
relevant product market under the Competition Act, 2002 ?
(A) Existence of specialised producers

© The Institute of Chartered Accountants of India


6 FINAL (NEW) EXAMINATION: NOVEMBER, 2019

(B) Market structure and size of market


(C) Consumer preferences
(D) Actual end use of the products (2 Marks)
1.5 When evaluating whether the arrangement between the parties involved shall be
presumed to be anti-competitive and likely to have an appreciable adverse effect on
competition, which of the following are not factors to be considered by the Director
General ?
(A) Limit and control the use of technology used by all parties in manufacturing APT.
(B) Allocate the supply of APT in India between the parties and limit new entrants.
(C) Collectively determine the purchase price of the key raw material (phosphorous)
from the vendors.
(D) Joint venture between the parties to share distribution channels and logistics
services to reduce cost. (2 Marks)
1.6 Answer the following questions in the context of the provisions relating to Competition
Act, 2002.
(i) Analyse whether the CCI can consider the tender called for in March, 2009 and
negotiations finalised in July, 2009 for examination under Section 3, which became
operational only on 20th May, 2009. (3 Marks)
(ii) Whether CCI was barred from investigating the matter pertaining to the tender
floated by Delta Corporation in March, 2018 on the basis that this was not a subject
matter contained in the complaint submitted by Delta Corporation on 4 February,
2018. (3 Marks)
(iii) Analyse based on the facts of the case, regarding the conclusion of CCI that the
appellants had entered into an agreement to indulge in collusive bidding by forming
a cartel, resulting into contravention of Section 3 of the Act. (5 Marks)
1.7 Examine/advise regarding the below questions relating to the Insolvency and Bankruptcy
Code, 2016:
What is your view with regard to the stand taken by NCLT in admitting the application of
PEL for initiating insolvency proceedings against Multicorp? (4 Marks)
ANSWER TO CASE STUDY 1
1.1 (B)
1.2 (D)

© The Institute of Chartered Accountants of India


PAPER – 6D: ECONOMIC LAWS 7

1.3 (B)
1.4 (B)
1.5 (D)
Descriptive Answers
1.6 (i) According to Section 36 of the Competition Act, 2002, the Commission shall have,
for the purposes of discharging its functions under this Act, the same powers as are
vested in a Civil Court under the Code of Civil Procedure, 1908, while trying a suit,
in respect of the following matters, namely:
(a) summoning and enforcing the attendance of any person and examining him on
oath;
(b) requiring the discovery and production of documents;
(c) receiving evidence on affidavit;
(d) issuing commissions for the examination of witnesses or documents;
(e) requisitioning, subject to the provisions of Sections 123 and 124 of the Indian
Evidence Act, 1872, any public record or document or copy of such of record
or document from any office.
The Commission may also direct any person:
(a) to produce before the Director General or the Secretary or an Officer
authorized by it, such books, or other documents in the custody or under the
control of such person so directed as may be specified or described in the
direction, being documents relating to any trade, the examination of which may
be required for the purposes of this Act;
(b) to furnish to the Director General or the Secretary or any other Officer
authorized by it, as respects the trade or such other information as may be in
his possession in relation to the trade carried on by such person, as may be
required for the purposes of this Act.
Hence, CCI can also consider the tender called for in March, 2009.
Alternate Solution
The bid was called in March 2009 and negotiations finalized in July, 2009 by which
date, Section 3 of the Competition Act, 2002 had already been activated. Therefore,
the principle of retro-activity shall become applicable as the process of finalization
of the tender was still on. Therefore, the inquiry into the tender of March, 2009 by
the CCI is covered by Section 3 of the Act in as much as the tender process, though
initiated prior to the date when Section 3 became operational, continued much

© The Institute of Chartered Accountants of India


8 FINAL (NEW) EXAMINATION: NOVEMBER, 2019

beyond May 20, 2009, the date on which the provisions of Section 3 of the Act were
enforced.
In the light of the above, it can be concluded that CCI can consider the tender called
for.
(ii) According to Section 19 of the Competition Act, 2002, the Commission is
empowered to inquire into any alleged contravention of the provisions contained in
Section 3(1) or Section 4(1) either on its own motion or on:—
(a) receipt of any information in such manner and accompanied by such fee as
may be determined by regulations, from any person, consumer or their
association or trade association; or
(b) a reference made to it by the Central Government or a State Government or a
Statutory Authority.
As per the situation given and provisions of the Act, CCI is empowered to inquire
into any alleged contravention of the provisions contained in Section 3(1) or Section
4(1) on its own motion also. Hence, CCI can also investigate the matter
pertaining to the tender floated by Delta Corporation in March, 2018 (though it
was not the subject matter contained in the complaint submitted by Delta
Corporation on 4th February, 2018).
(iii) "Bid Rigging" means any agreement, between enterprises or persons engaged in
identical or similar production or trading of goods or provision of services, which has
the effect of eliminating or reducing competition for bids or adversely affecting or
manipulating the process for bidding.
As per the facts of the case study, there seems to be collusive bid rigging by
forming cartel due to the following reasons:
1. All the parties (namely M/s Easy, M/s Samurai, M/s Multicrop and M/s Agro
Chemicals) quoted identical rates from 2009 to 2014.
2. In tender floated in March 2016, the three applicants quoted identical prices.
3. If we see the bidding patterns for other corporations also (i.e. A, B, C and D)
we see that participating applicants quoted identical prices always.
Further, the response given by the parties (namely M/s Easy, M/s Samurai, M/s
Multicrop and M/s Agro Chemicals) did not support that there was no cartelization,
on the following grounds:
1. CCI is empowered to inquire into any alleged contravention of the provisions
contained in Section 3(1) or Section 4(1) on its own motion also. Hence, CCI
can also investigate the matter pertaining to the tender floated by Delta

© The Institute of Chartered Accountants of India


PAPER – 6D: ECONOMIC LAWS 9

Corporation in March, 2018 (though it was not the subject matter contained in
the complaint submitted by Delta Corporation on 4th February, 2018).
2. The said parties pleaded that the price rise of APT was due to increase of
price of yellow phosphorous, which was to be procured from China, had
increased. However, all the parties quoted identical prices which has resulted
in adversely affecting/ manipulating the process of bidding.
1.7 Initiation of Insolvency resolution by PEL (operational creditor) against Multicrop.
According to Section 8 of the IBC, 2016, following requirements are to be met for
initiation of corporate insolvency resolution process by operational creditor, i.e. by PEL
against the corporate debtor, Multicrop:
(1) On the occurrence of default, an operational creditor shall first send a demand
notice and a copy of invoice to the corporate debtor.
(2) The corporate debtor shall, within a period of ten days of the receipt of the
demand notice or copy of the invoice bring to the notice of the operational
creditor about existence of a dispute about debt, if any, record of the pendency of
the suit or arbitration proceedings filed before the receipt of such notice or invoice in
relation to such dispute; Where corporate debtor might have already paid the
unpaid operational debt, there in such situation, corporate debtor will inform within
10 days send an attested copy of the record of electronic transfer of the unpaid
amount from the bank account of the corporate debtor; or sends an attested copy
of record that the operational creditor has encashed a cheque issued by the
corporate debtor. [Section 8]
According to Section 9 of the IBC, an application for initiation of corporate
insolvency resolution process by operational creditor may be filed, if no reply is
received or payment or notice of the dispute under Section 8(2) from the corporate
debtor within ten days from the date of delivery of the notice or invoice demanding
payment, operational creditor can file application before Adjudicating Authority
(NCLT) for initiating a corporate insolvency resolution process.
As per the facts stated in the case study, PEL had not served demand notice and a
copy of invoice to the Multicrop. In fact it directly went to the Arbitral tribunal, for
settlement of the claim as per the term of agreement. Award was passed in the
favour of PEL. However, the award of the Arbitral Tribunal was challenged by the
Multicrop. Whereas PEL also filed an application before the NCLT for initiation of
CIRP against Multicrop.
According to the above provision, due to prima facie non-compliance of serving of
demand notice and a copy of invoice to the Multicrop by the operational creditor

© The Institute of Chartered Accountants of India


10 FINAL (NEW) EXAMINATION: NOVEMBER, 2019

(PEL) and of further no notice of dispute about debt regarding the pendency of the
suit in appeal before Appellate Arbitration by the Corporate Debtor (Multicrop).
Therefore NCLT stand as regard the admission of application of PEL on
initiation of CIRP against Multicrop, is not appropriate.

CASE STUDY - 2
Teddy Bear Technology Private Limited (TBTPL), is one of India's fastest growing start-up
companies. TBTPL was incorporated in the year 2015 by two promoters Mr. Sudhir Shankar
and Mr. Ajay Vinod, who were college mates at IIT Bombay and completed their masters in the
United States of America (USA). Both Mr. Sudhir Shankar and Mr. Ajay Vinod worked in the
USA for more than 10 years.
Post that they came back to India in 2015 (and continue to stay in India) to serve the country
and established TBTPL to develop technology and software relating to aviation technology
and machine learning. TBTPL has around 300 employees in India and has several clientele in
US and the company is also looking at rapid expansion over the next 3 years. The Company is
registered with the Software Technology Parks but is not a status holder exporter.
The details of export sales and realization of export proceeds by TBTPL during the last 3
financial years is as under :
Particulars 2015-16 2016-17 2017-18 Average
Export Turnover (USO) 500,000 2,500,000 4,500,000 2,500,000
Realisation of Export Proceeds (USD) 300,000 2,000,000 3,000,000 1,600,000
One of the export invoices amounting to USD 200,000 raised by TBTPL in the financial year
2016-17 was outstanding for more than one year as of 31st March, 2018 and the Company's
auditors insisted on the Company taking action for recovery. However, even after the best
efforts, no amounts could be recovered and therefore, during the financial year 2018-19, the
Company wrote off the entire amount of USD 200,000 without obtaining the approval from the
Authorised Dealer (AD). Out of the export proceeds received by TBTPL, the Company lent an
amount of USD 500,000 in foreign currency to one of its key Indian vendors to enable them to
create / maintain core working capital. The Management convinced the Board of Directors to
approve the loan since the vendor was providing, critical services for business continuity of
TBTPL. Further, this loan has been guaranteed by the holding company of the vendor, which
is located in Mauritius.
In order to expand its operations, TBTPL was intending to lease a commercial property in
India in Mumbai for a period of 5 years at an upfront lease premium of ` 5 crores, TBTPL was
in great urgency to complete the transaction soonest in view of the great demand for the
property and therefore, M/s. DoCorrect Consultants, the agency assisting TBTPL used a
counterfeit government stamp paper for the purpose of registering the lease deed and this was
informed by the agency to Mr. Ajay Vinod at the time of transaction to minimise the cost of

© The Institute of Chartered Accountants of India


PAPER – 6D: ECONOMIC LAWS 11

stamp duty. The funds for acquiring the stamp papers was paid by the agency and was in-turn
billed by the agency on TBTPL as part of its invoice for agency fee / commission. The invoice
was settled by TBTPL to the agency in cash without deduction of tax, even though the CFO of
TBTPL was of the view that the same is not in accordance with the applicable statutory
requirements.
For the purpose of enhancing its capabilities, TBTPL engaged the services of two reputed
organizations to train the employees of TBTPL. For this purpose, TBTPL paid an amount of
USD 500,000 to one company and USD 1,500,000 to the second company. For the purpose of
investing money into the business, TBTPL sold a commercial plot owned by it in India to a
friend of Mr. Ajay Vinod who was a Non-resident Indian in the USA, through an agent based in
Chicago, USA for an amount of USD 500,000. In accordance with the terms of the agreement
with the agent, TBTPL paid an amount of USD 30,000 as commission to the agent. TBTPL
also published an advertisement costing USD 100,000 in the New York Times weekend
edition calling for employees to join its proposed office in New York.
Mr. Siddarth Shankar, brother of Sudhir Shankar who works as a CFO in a listed entity in
India, provided certain price sensitive information to Mr. Sudhir Shankar about his employer
based on which Mr. Sudhir Shankar purchased equity shares of the entity and made a profit of
` 2 crores. With these proceeds, he sent ` 1 crore to his wife Ms. Anne Shankar (as part of
the liberalised remittance scheme) to purchase a small apartment in the USA. He also
purchased a very old statue of an Indian king in an amount of ` 0.20 crores and sent it to his
wife for display in his home in USA. He invested the balance amount of ` 0.80 crores in
TBTPL as an equity investment.
During one of the discussions with the customers in USA, Mr. Ajay Vinod indicated to the
customer that TBTPL has capabilities to develop new robotic technology on aviation and
accordingly, entered into a contract for an amount of USD 2,000,000. TBTPL developed the
robotic platform in 2 months and delivered to the customer, although the patent and copyright
was owned by another competitor of TBTPL. TBTPL is of the view that the company rightfully
owns the patent for the same, although it has not applied / registered for the same.
The Enforcement Directorate (ED) got wind of the transactions carried out by TBTPL and the
Directors, through one of the employees of the Company and have issued a notice to the
Company and the Directors.
Answer the following questions:
2.1 Which of the following are not actions that could be taken by the ED on TBTPL or its
employees, for not complying with its orders under PMLA, 2002?
(A) Issue a warning in writing.
(B) Direct the entity or its employees to directly send reports.
(C) Direct the relevant courts to take civil or criminal proceedings against TBTPL or its
employees.

© The Institute of Chartered Accountants of India


12 FINAL (NEW) EXAMINATION: NOVEMBER, 2019

(D) Impose a monetary penalty on TBTPL or its employees. (2 Marks)


2.2 In order to obtain more information from Mr. Sudhir Shankar, the ED wanted to detain Mr.
Sudhir Shankar for a period of 3 days to make enquiries and get the relevant information
from him. Evaluate if this is appropriate under PMLA, 2002.
(A) Yes, the Director is well within his powers to detain Sudhir until all informations are
collected.
(B) No, maximum period of detention under PMLA is 24 hours before which Sudhir
should be presented before the superior ranking office or the magistrate.
(C) Yes, however, the Director is required to take the prior approval of his superior
ranking officer.
(D) No, the Director is not within his rights to detain Sudhir. (2 Marks)
2.3 The Appellate Tribunal has concluded that the Director who searched Mr. Sudhir Shankar
and his property indulged in a vexatious search without recording proper reasons in
writing and has sought your views on the next course of action :
(A) Suspension / Dismissal from service, as may be decided by the central government.
(B) Fine which may extend to ` 2 lakhs.
(C) Imprisonment for a term which may extend to four years and fine which may extend
to ` 2 lakhs.
(D) Imprisonment for a term which may extend to two years or fine which may extend to
` 50,000 or both. (2 Marks)
2.4 What is the maximum amount of export receivables which can be written off by TBTPL
during the financial year 2018-19?
(A) With approval of AD - USD 450,000; Without approval of AD – USD 225,000
(B) With approval of AD - USD 250,000; Without approval of AD – USD 125,000
(C) With approval of AD - USD 300,000; Without approval of AD – USD 150,000
(D) With approval of AD - USD 160,000; Without approval of AD – USD 80,000
(2 Marks)
2.5 Under FEMA, 1999, what is the amount that can be paid by TBTPL for publishing an
advertisement in New York Times ?
(A) USD 10,000
(B) USD 100,000

© The Institute of Chartered Accountants of India


PAPER – 6D: ECONOMIC LAWS 13

(C) USD 250,000, subject to the approval of the Reserve Bank of India.
(D) None, all such transactions require approval of the government of India. (2 Marks)
2.6 Advise the Board of Directors of TBTPL on the compliance with FEMA, 1999 with regard
to the below transactions :
a. Payments made by TBTPL for consultancy services
b. Payment of commission
c. Loan provided in foreign currency to vendor in India and the validity of the
guarantee provided by the vendor's holding company. (7 Marks)
2. 7 Examine / advise regarding the below questions relating to the Prevention of Money
Laundering Act, 2002 :
(i) The Enforcement Directorate has sought your advice on identifying all the offences
committed by the parties under the PMLA, 2002 described in the case study.
Identify :
(a) the offences along with explanations,
(b) the parties involved, and
(c) the proceeds of crime. (5 Marks)
(ii) The Enforcement Directorate is proposing to perform a search of M/s. DoCorrect
Consultants premises in connection with the investigation of TBTPL's transactions.
This has been challenged by M/s DoCorrect consultants. Evaluate the
appropriateness of the position taken by M/s. DoCorrect Consultants. (3 Marks)
ANSWER TO CASE STUDY 2
2.1 (C)
2.2 (B)
2.3 (D)
2.4 (C)
2.5 (D)
Descriptive Answers
2.6 As per Schedule III, the following remittances by persons other than individuals shall
require prior approval of the Reserve Bank of India:

© The Institute of Chartered Accountants of India


14 FINAL (NEW) EXAMINATION: NOVEMBER, 2019

(i) Commission, per transaction, to agents abroad for sale of residential flats or
commercial plots in India exceeding USD 25,000 or five percent of the inward
remittance whichever is more.
(ii) Remittances exceeding USD 1,00,00,000 per project for any consultancy services in
respect of infrastructure projects and USD 10,00,000 per project, for other
consultancy services procured from outside India.
a. TBTPL made a payment of USD 500,000 to one Company and USD 1,500,000
to another Company for training the employees of TBTPL. Thus, in total, made
a payment of USD 2,000,000.
As per the provision of law and facts of case study, TBTPL require prior
approval of the Reserve Bank of India to make a payment of USD 200,000 as
it exceeds the limit of USD 1,000,000 given under law.
Alternate Solution
a. As per Schedule III, the following remittances by persons other than
individuals shall require prior approval of the Reserve Bank of India:
(i) Commission, per transaction, to agents abroad for sale of residential flats
or commercial plots in India exceeding USD 25,000 or five percent of the
inward remittance whichever is more.
(ii) Remittances exceeding USD 10,000,000 per project for any consultancy
services in respect of infrastructure projects and USD 1,000,000 per
project, for other consultancy services procured from outside India.
TBTPL made a payment of USD 500,000 to one Company and USD 1,500,000
to another Company for training the employees of TBTPL.
Therefore, the prior approval of the RBI is required for the payment of USD
1,500,000 to the second Company. No specific approval of the RBI is required
for the payment of USD 500,000 to the first Company.
b. TBTPL made a payment of USD 30,000 as commission to agent abroad for
selling a commercial plot owned by it in India to a Non- resident Indian in USA.
As per facts of case and provision of law, TBTPL can make a remittance of
USD 25,000 or five percent of the inward remittance from sale of commercial
plot, without RBI approval.
Thus, TBTPL have to take prior approval of RBI to make a payment of USD
30,000 as commission to agent abroad (as it exceeds the limit of USD 25,000
or 5% of USD 500,000, whichever is higher).
c. As per FEMA provisions, a resident cannot lend to another resident in foreign
currency. However, Loan and guarantee can be extended to an overseas

© The Institute of Chartered Accountants of India


PAPER – 6D: ECONOMIC LAWS 15

entity only if there is already an existing equity / CCPS ( Compulsorily


Convertible Preference Shares) participation by way of direct investment.
In the given case study, TBTPL lent an amount of USD 500,000 in foreign
currency to one of its vendor. This loan was guaranteed by the holding
Company of the vendor, which is located in Mauritius.
As per the facts of the case study and the provision enumerated above,
TBTPL cannot give loan to its vendor.
2.7 (i) In the given case study, Enforcement Directorate identified following offences
committed by the parties under the PMLA, 2002-
(a) Offences with Explanation: (1) Use of counterfeit government stamp paper
for the purpose of registering the lease deed to minimise the cost of stamp
duty- offence under Part A of the Schedule , (2) invoice for agency fees
/commission for acquiring the stamp papers, settled in cash without deduction
of tax by TBTPL- Offence under Part C of the Schedule (3) Use of patent and
copyright owned by another competitor of TBTPL -- offence under Part A of the
Schedule (4) Providing of price sensitive information to Mr. Sudhir Shankar of
an employee on the basis of which he purchased equity shares of the entity-
offence under Part A of the Schedule (5) sending to ` 1 Crore out of proceeds
from purchase of an equity shares to Ms Anne Shankar- offence under Part A
of the Schedule.
(b) Parties Involved: Offence pertaining to use of counterfeit government stamp
paper- TBTPL, Mr. Ajay Vinod, Agency M/s DoCorrect Consultants, CFO of
TBTPL.
Invoice for agency fees /commission for acquiring the stamp papers- Agency
M/s DoCorrect Consultants, TBTPL, CFO of TBTPL.
Use of patent and copyright owned by another competitor by TBTPL: TBTPL,
Mr. Ajay Vinod.
Purchase of equity shares of an entity on the price sensitive information: Mr.
Siddarth Shanker & Mr. Sudhir Shankar
Out of proceeds obtained above, sent certain amount to Ms. Anne- Mr.
Siddarth Shanker, Mr. Sudhir Shankar, Ms. Anne Shanker.
(c) Proceeds of Crime: ` 5 Crore (Lease premium), USD 2,000,000 (for
development of Robotic Platform under the patent & copyright owned by
another), & ` 2 Crore (obtained by the purchase of equity shares).
(ii) According to Section 17 of the Prevention of Money Laundering Act, 2002, where
the Director or any other officer not below the rank of Deputy Director authorised by
him for the purposes of this Section, on the basis of information in his possession,

© The Institute of Chartered Accountants of India


16 FINAL (NEW) EXAMINATION: NOVEMBER, 2019

has reason to believe (the reason for such belief to be recorded in writing) that any
person—
(i) has committed any act which constitutes money-laundering, or
(ii) is in possession of any proceeds of crime involved in money-laundering, or
(iii) is in possession of any records relating to money-laundering, or
(iv) is in possession of any property related to crime,
then, subject to the rules made in this behalf, he may authorise any officer
subordinate to him to—
enter and search any building, place, vessel, vehicle or aircraft where he has
reason to suspect that such records or proceeds of crime are kept;
Thus, the Enforcement Directorate can perform a search of M/s DoCorrect
Consultants’ premises in connection with the investigation of TBTPL’s connection.
Hence, the position taken by M/s DoCorrect Consultants is not appropriate
based on the above legal provisions.

CASE STUDY: 3
The Indian pharmaceutical manufacturing industry comprises of 3 large companies, LPPL,
SMCL and HLL. The above 3 companies, in total supply more than 90% of the across the
counter medicine market in India and their products were available across India through the
sale of medicines to registered agencies / stockists, who in turn supplied to the local chemists
and drugstores. In addition to the business of manufacturing across the counter medicines, all
the 3 entities were also engaged in the manufacture of 'Active Product Ingredients' (API),
which were supplied to global pharmaceutical companies for production of medicines. The
entire API manufacturing in India is performed only by the 3 companies.
During one of the discussions between LPPL and its overseas customer based in Canada, the
overseas customer requested LPPL to supply API for manufacturing diabetes medicines and
also stated that as per the latest research carried out by them, coca leaves have a lot of
medicinal properties and have tremendous potential to supress diabetes and other ailments.
LPPL stated that they could supply coca leaves from India and pursuant to a purchase order
from the customer, LPPL sold coca leaves for an amount of Rs. 5 crores and the CFO of LPPL
ensured that the proceeds was received from the customer into LPPL’s EEFC account in
compliance with FEMA, 1999. For the purpose of increasing their operations in Canada, LPPL
wanted to set up its branch office in Canada and accordingly, used the consideration received
for acquiring Land and Building in Toronto, Canada for an amount of ` 4 crores. The CFO of
LPPL was informed by the internal auditor that the above acquisition of immovable property in
Canada was in accordance with the provisions of FEMA, 1999.
During the year 2017, the Pharmaceutical Agents Association of Uttar Pradesh filed a
complaint against the 3 companies with the Director General that the companies were

© The Institute of Chartered Accountants of India


PAPER – 6D: ECONOMIC LAWS 17

engaging in anti-competitive market activities by forcing stockists to obtain a Non-Objection


Certificate from the local chemists and druggists association and the companies were denying
the supply of medicines to the stockists solely because they were not able to obtain the NOC.
LPPL, SMCL and HLL responded to the DG that sub-clause (a) of Clause 28 of the Drugs
(Price Control) Order, 2013 creates an obligation on a pharmaceutical company/distributor to
sell drugs/medicines unless there is a ‘good and sufficient reason' to refuse sale. Based on
their evaluation of the facts and circumstances, the non-availability of NOC from the local
chemists association tantamount to 'good and sufficient reason’.
Based on the investigation carried out by the DG and analysis of all the documents and
information provided by the Pharmaceutical manufacturing companies, the stockists etc. and
notwithstanding the above views of the pharmaceutical manufacturers, the DG concluded that
the 3 companies, LPPL. SMCL and HLL contravened the provisions of Section 3(3)(b) read
with Section 3(1) of the Competition Act, 2002. For indulging in anti-competitive practices in
violation of the provisions of Section 3 of the Act, the CCI imposed penalties upon all the three
appellants at 9% of average 3 years’ total turnover of these appellants under the Act.
LPPL, SMCL and HLL accepted the order of the DG in principle and accepted to remove, the
condition of obtaining NOC for supply to the stockists. However, they contested the manner in
which the DG had computed the penalty by considering the total turnover of the entities (as
per the Statement of Profit and Loss) without considering that the turnover includes incomes
from the API business, which is not forming part of the investigation of the DG. They filed an
appeal before the Appellate Tribunal that the penalty could be calculated only based on the
turnover relating to the "Across the Counter" operations of the pharmaceutical companies.
In the meantime during the year 2018, LPPL entered into an agreement with HLL to acquire
the API business of HLL for a consideration of ` 200 Crores. The latest available financial
information relating to the entities are as under:
` in Crores
Particulars LPPL HLL
Total entity API business Total Entity API business
Assets 900 800 500 300
Turnover 2800 2400 1000 800
Note: The entities do not have any business / operations outside India.
SMCL is of the view that the above arrangement will cause an appreciable adverse effect on
competition in the API manufacturing market in India and requires the approval of the
Competition Commission.
The Authorised dealer, when reviewing the export invoices raised by LPPL noted the sale of
coca leaves and informed the income tax authorities regarding the same. The authorities, after
review of the documents and other information, concluded that the transactions was in

© The Institute of Chartered Accountants of India


18 FINAL (NEW) EXAMINATION: NOVEMBER, 2019

violation of the Prevention of Money Laundering Act, 2002 and have sent a notice to LPPL,
who is not a willful defaulter.
Answer the following questions:
3.1 Which of the following terms and conditions as per the agreement between LPPL and
HLL is not likely to cause an appreciable adverse effect on competition under the
Competition Act, 2002?
(A) All purchase of raw materials by HLL should be made from SMCL or from LPPL
only.
(B) The API manufactured by HLL should be sold to the customers as mandated by
LPPL.
(C) Any purchase. of API by HLL should be along with purchase of the packing material
and preservatives.
(D) A maximum price ceiling on the resale price that may be charged by HLL for
ultimate sale of the goods purchased by it from LPPL. (2 Marks)
3.2 Considering the nature of the operations of LPPL and HLL, what is the requirement of
giving notice regarding the· proposed combination as per Form Il as specified in the
Schedule II to the Competition Commission of India (Procedure in regard to the
transaction of business relating to combinations) Regulations, 2011 ?
(A) Mandatory, if the combined market share after such combination is more than 15 %
of the market.
(B) Optional, 'if the combined market share after such combination is more than 25% of
the market.
(C) Optional, if the combined market share after such combination is more than 15% of
the market.
(D) Mandatory, if the combined market share after such combination is more than 25%
of the market. (2 Marks)
3.3 Which of the following are not included within arrangements entered into by central
government with another country, in relation to reciprocal arrangements under PMLA,
2002?
(A) Enforcement of the provisions of PMLA, 2002.
(B) Prevention of offence in India under the corresponding PMLA law in force in the
other country.
(C) Exchange the history of LPPL if it is wilful offenders under the PMLA on annual
basis.
(D) Exchange information to prevent any offence under PMLA, 2002. (2 Marks)

© The Institute of Chartered Accountants of India


PAPER – 6D: ECONOMIC LAWS 19

3.4 The composition of an Adjudicating Authority (AO) under the PMLA, 2002 referred in the
case study is :
(A) One Chairperson, appointed by central government and two other members.
(B) Three members, one of whom will be a Chairperson, as per seniority.
(C) Four members, each of whom will be a Chairperson on rotation.
(D) Five members, appointed by central government and four other members. (2 Marks)
3.5 On the basis that the transactions entered into by LPPL is considered to be in
contravention of the PMLA, 2002, what is the punishment that the CFO of LPPL would be
liable under the PMLA, 2002 ?
(A) Minimum 3 years and maximum 10 years with fine.
(B) No punishment since he is not a director of LPPL and therefore cannot be held
liable under PMLA, 2002.
(C) Minimum 3 years and maximum 7 years with fine.
(D) No punishment since he was not aware that the transaction was indeed a non-
compliance under PMLA, 2002. (2 Marks)
3.6 Answer the following questions in the context of the provision relating to Competition Act,
2002 with reasons and explanations:
(i) SMCL has reached out to you to seek your advice on their views regarding the
impact of the provisions of the Competition Act on the proposed combination
between LPPL and HLL. (4 Marks)
(ii) Whether penalty under Section 27(b) of the Act has to be on total/entire turnover of
the offending company or it can be only on "relevant turnover", i.e., relating to the
product in question. (3 Marks)
3. 7 Answer the following questions in the context of the provisions relating to PMLA, 2002
with reasons and explanations:
(i) LPPL has challenged the notice and without admitting to any of the offences, is of
the view that only immovable property held within India is to be considered for
identifying proceeds of crime under PMLA. Evaluate. (4 Marks)
(ii) In the above case study, what is the mechanism to be followed by the Enforcement
Directorate for attachment of property situated in Canada? (4 Marks)
ANSWER TO CASE STUDY 3
3.1 (D)
3.2 (C)
3.3 (C)

© The Institute of Chartered Accountants of India


20 FINAL (NEW) EXAMINATION: NOVEMBER, 2019

3.4 (A)
3.5 (A)
Descriptive answers
3.6 (i) The given proposed combination between LPPL & HLL in terms of Section 5 of the
Competition Act, 2002, is a combination of the enterprises by acquisition where the
parties to the acquisition, being the acquirer and the enterprise, whose control,
shares, voting rights or assets have been acquired or are being acquired jointly
have in India, the assets of the value of more than rupees one thousand crores or
turnover more than rupees three thousand crores.
Pursuant to Notification No. S.O. 675 (E) dated March 4, 2016 the value of
assets and the value of turnover has been enhanced by the Central Government by
100% for the purposes of Section 5 of the Act.
So, the revised value of assets and turnover is presently more than ` 2000 crore
and ` 6000 Crore.
Since, here the proposed combination between LPPL and HLL was to acquire the
API business of HLL only, therefore, it will not be valid as they have not met with
the requirement of assets of the value of more than ` 2000 crore [i.e., total value of
asset of LPPL (900+800) + value of asset of API business of HLL (300)] and
turnover of ` 6000 crore [i.e., total turnover of LPPL (2800+2400) + turnover of API
business of HLL (800)]
(ii) As per Section 27 of the Competition Act, 2002, where after inquiry the Commission
finds that any agreement referred to in Section 3 or action of an enterprise in a
dominant position, is in contravention of Section 3 or Section 4, as the case may be,
it may impose such penalty, as it may deem fit, which shall be not more than ten per
cent of the average of the turnover for the last three preceding financial years, upon
each of such person or enterprises which are parties to such agreements or abuse.
In case any agreement referred to in Section 3 has been entered into by a cartel,
the Commission may impose upon each producer, seller, distributor, trader or
service provider included in that cartel, a penalty of up to three times of its profit for
each year of the continuance of such agreement or ten per cent of its turnover for
each year of the continuance of such agreement, whichever is higher.
Accordingly, the penalty under Section 27(b) of the Act has to be on total/
entire turnover of the offending Company.
3.7 (i) In the light of Section 2(1)(u) of the Prevention of Money Laundering Act, 2002,
"proceeds of crime" means as any property derived or obtained, directly or

© The Institute of Chartered Accountants of India


PAPER – 6D: ECONOMIC LAWS 21

indirectly, by any person as a result of criminal activity relating to a scheduled


offence or the value of any such property or where such property is taken/held
outside the country, then the property equivalent in value held within the country or
abroad.
In the said case, LPPL challenged the notice and not admitting to any of the
offences on the ground that only immovable property held within India is to consider
for identifying proceeds of Crime under PMLA. According to the above stated
provision, LPPL challenge to the notice and not admitting to any of the offences
pertaining to the immovable property held outside India, is not valid and therefore
the notice served on LPPL cannot be challenged.
(ii) Following are the ways for attachment of property situated in Canada in the given
case study in the light of Section 60 of the PMLA, 2002 -
1. Issue of letter of request: Where the Director has made an order for
attachment of any property under Section 5 or for freezing under sub-Section
(1A) of Section 17 or where an Adjudicating Authority has made an order
relating to a property under Section 8 or where a Special Court has made an
order of confiscation relating to a property under sub-Section (5) or sub
Section (6) of Section 8, and such property is suspected to be in a
contracting State,
- the Special Court, on an application by the Director or the Administrator
appointed under sub- Section 10(1), as the case may be, may issue a letter of
request to a court or an authority in the contracting State for execution of such
order.
2. Forwarding of letter of request for execution on its receipt by CG : Where
a letter of request is received by the Central Government from a Court or an
Authority in a Contracting State requesting attachment, seizure, freezing or
confiscation of the property in India, derived or obtained, directly or indirectly,
by any person from the commission of an offence under a corresponding law
committed in that Contracting State, the Central Government may forward
such letter of request to the Director, as it thinks fit, for execution in
accordance with the provisions of this Act.
3. Issue of Order of confiscation: Where on closure of the criminal case or
conclusion of trial in a criminal court outside India under the corresponding law
of any other country, such court finds that the offence of money-laundering
under the corresponding law of that country has been committed, the Special
Court shall, on receipt of an application from the Director for execution of
confiscation under sub-Section (2), order, after giving notice to the affected
persons, that such property involved in money-laundering or which has been
used for commission of the offence of money-laundering stand confiscated to

© The Institute of Chartered Accountants of India


22 FINAL (NEW) EXAMINATION: NOVEMBER, 2019

the Central Government.


4. The provisions of this Act relating to attachment, adjudication, confiscation and
vesting of property in the Central Government shall apply to the property in
respect of which letter of request is received from a court or contracting State
for attachment or confiscation of property.
CASE STUDY - 4
Highcity Partners LLP (Highcity), is a recently established limited liability partnership between
Seaview Constructions Private Limited, a real 'estate development company owned by Mr.
Vyas Chakraborty (Seaview constructions) and Mr. Ved Chakraborty. Highcity was established
for the purpose of acquiring an existing apartment complex "Riverview Bliss" (comprising of 12
luxury apartments) in Kolkata and redevelopment of the same. Seaview Constructions is a
very successful real estate company and has completed more than 20 apartment complexes
and is known for quality constructions, adherence to timelines and profitable growth.
6 of the 12 apartments in Riverview Bliss is currently owned by SPZ Private Limited (SPZ) and
the balance 6 are owned by the senior employees of SPZ. Due to the strategic location of the
property and the quality of construction, Highcity and the current owners have agreed for a
price of ` 3 crores for each of the 12 apartments and therefore the total consideration to be
paid by Highcity is ` 36 crores.
SPZ is an associate Company of True & Fair Finance Company Limited (TFFC), a listed
company in the business of providing loans for large corporate projects. Both SPZ Private
Limited and TFFC have common promoters and senior employees and operate out of the
same registered office.
In the past, Seaview Constructions has obtained loans from TFFC for many of their projects
and has established a strong professional relationship with them on account of the mutual
benefit realised by both the entities from the transactions between them. Therefore,
considering the size of the transaction to be entered into by Highcity, Mr. Vyas Chakraborty
had discussions with TFFC and based on the business case submitted by Highcity, TFFC
approved a secured loan of ` 30 crores to Highcity to enable purchase of the apartments in
Riverview Bliss from its owners. The loan was fully utilised by Highcity to acquire the
apartments and a charge was created against the property for the secured loan obtained from
TFFC. Highcity obtained further loans amounting to ` 10 crores from SPZ for the purpose of
the redevelopment of the property.
During the scrutiny assessment of Highcity, the Income tax authorities noted the details of the
transactions and concluded that the entire transaction is a benami transaction where Highcity
is the Benamidar and SPZ / TFFC are the beneficial owners. The Initiating Officer sent a show
cause notice under Section 24(1) of the Prohibition of Benami Property Transactions Act, 1988
(PBPT Act, 1988) and on the same day, an order was passed by the Dy. Commissioner of
Income Tax for provisional attachment of the Riverview Bliss property based on the following
averments :

© The Institute of Chartered Accountants of India


PAPER – 6D: ECONOMIC LAWS 23

• Highcity did not have any business or operations prior to the acquisition of the benami
property.
• Mere approvals in the name of benamidar do not prove in any way that the benefits from
the property are actually enjoyed by it and not by the beneficial owner.
• Highcity received huge amounts of money from SPZ which it used for the development of
property, thereby establishing that SPZ is directly involved in the development of project
in order to derive future benefits arising out of the same.
• The entire transaction is only for the benefit of TFFC and SPZ, who are owned by
common promoters since the person providing the consideration i.e. TFFC and person
reaping the benefits of such transaction i.e. SPZ are same as they are linked to each by
means of common directors and promoters.
• The benefits to the beneficial owner arising out of property held in the name of the
benamidar need not be direct and immediate and· that indirect and future benefits are
also covered under the definition of a benami transaction under section 2(9)(A) of the
PBPT Act, 1988.
The Initiating Officer further stated in his show cause notice seeking response and proof from
Highcity and SPZ that the above transactions are not benami transactions. Highcity is of the
strong view that the above averments are incorrect and that the entire transaction is a genuine
business transaction and the loan from TFFC was obtained in the ordinary course of business
(similar to the other loans taken by Seaview Constructions).
Seaview Constructions was operating as a profit making company until 2016 and whilst it was
having debt, the entity was able to service the debt promptly from its business cash flows.
However, due to the downturn of the real estate industry and commencement of additional
businesses, Seaview Construction's profits and operations started to deteriorate and it had to
obtain significant borrowings during 2017 from a consortium of banks for working capital
purposes. However, due to the difficulties in the business operations and the economic
slowdown, Seaview Constructions could not repay its borrowings and the entire net worth got
eroded due to significant operating losses. This led to Seaview Constructions filing a petition
under the Insolvency and Bankruptcy Code, 2016. The petition was accepted by the National
Company Law Tribunal (NCLT) and an Interim Resolution Professional (IRP) was appointed,
who was later approved as the Resolution Professional (RP). The Committee of Creditors,
comprising of the financial creditors was formed with the following vote share:
Particulars Voting Share (%)
A Bank 22.33%
B Bank 14.39%
C Bank 15.15%.

© The Institute of Chartered Accountants of India


24 FINAL (NEW) EXAMINATION: NOVEMBER, 2019

D Bank 26.36%
E Bank 10.94%
F Bank 10.83%
The resolution plan submitted by the RP was placed before the Committee of Creditors at its
meeting held on 4th December, 2018 wherein, the resolution plan was approved by A Bank, B
Bank and C Bank. D Bank rejected the resolution plan and provided its reasons in writing to
the RP. E Bank and F Bank did not approve or reject the proposal and abstained from voting
at the meeting. Seaview Constructions (the Corporate Debtor) is of the view that the resolution
plan has been approved by the Committee of Creditors since the resolution plan has been
approved by more than the prescribed percentage of creditors who actually voted in the
meeting (i.e. after excluding the percentage relating to the creditors who abstained). The RP
did not agree to this view since more than 25% of the creditors present in the meeting had out
rightly rejected the resolution plan and therefore, proceeded for liquidation under the IBC
since no resolution plan was approved within the prescribed time limit under the Code.
M/s. Sunflower Estates Private Limited (Sunflower Estates), a Company under the common
control of the promoter of Seaview Constructions had also subscribed to the secured
debentures of Seaview Constructions to the extent of ` 50 Crores (representing 15% of the
total financial debts of Seaview Constructions). The IRP rejected the request received from
Sunflower Estates for inclusion into the Committee of Creditors.
Answer the following questions:
4.1 The owner (one of the employees of SPZ) of one of the apartments in Riverview Bliss is
not aware of his ownership of the apartment. He is seeking your advice on the impact on
the same under PBPT Act, 1988.
(A) No impact, since the property has already been sold off to Highcity.
(B) The property is not a benami property since the employee had continuous
possession of the property through the period he was the owner.
(C) The property is not a benami property since the sale agreement was registered
appropriately and stamp duty was also paid.
(D) The property is a benami property since the owner of the property is not aware of
such ownership. (2 Marks)
4.2 Mr. Vyas Chakraborty is of the view that the Initiating Authority does not have the right to
send the notice for attachment of the property and those powers are vested with the
adjudicating authority, as per PBPT, 1988 and seeks your advice :
(A) Yes. Initiating Authority has only powers to summon and conduct inquiries.

© The Institute of Chartered Accountants of India


PAPER – 6D: ECONOMIC LAWS 25

(B) No. The adjudicating authority's function is to confiscate and vest the property. The.
Initiating Officer has powers to send the notice for attachment of property.
(C) No. The approving authority has to send the notice for attachment of property and
the adjudicating authority is required to confiscate and vest the property.
(D) Yes. The initiating authority can provisionally attach properties only with the prior
approval of the adjudicating authority. (2 Marks)
4.3 Assuming that the Riverview Bliss property is considered as a benami property, the
Initiating Officer seeks your views on whether the rental income earned by Highcity from
the lease of the apartment (pending commencement of redevelopment) is also a benami
transaction.
(A) No, the rental income is an independent transaction between a landlord and a
tenant for legitimate use of the property.
(B) No, as long as Highcity remits Income tax on the rental income earned.
(C) Yes, benami transaction includes any income or proceeds received or earned out of
a benami property,
(D) Yes, if the proceeds from the rental income are used by Highcity for making interest
payment or loan repayment to TFFC or SPZ. (2 Marks)
4.4 How should the voting share of each of the Banks who have lent to Seaview
Constructions be determined under IBC, 2016 ?
(A) Based on the financial debt owed by Seaview Constructions to each bank as a
proportion to the total debt (financial + operational) owed by Seaview Constructions.
(B) Based on the financial debt owed by Seaview Constructions to each bank as a
proportion to the total financial debt owed by Seaview Constructions to third parties
(i.e. other than related parties).
(C) Based on· the financial debt owed by Seaview Constructions to each bank as a
proportion to the total financial debt owed by Seaview Constructions.
(D) Based on the financial debt owed by Seaview Constructions to each bank as a
proportion to the total financial debt and statutory dues owed by Seaview
Constructions. (2 Marks)
4.5 Which of the following operational creditors of Seaview Constructions are eligible to
initiate corporate insolvency process against Seaview Constructions ?
(A) G Limited, completed a corporate insolvency resolution process 15 months prior to
the date of making the application.

© The Institute of Chartered Accountants of India


26 FINAL (NEW) EXAMINATION: NOVEMBER, 2019

(B) H Limited, who is currently undergoing a insolvency resolution process.


(C) I Limited, who could not meet its resolution plan under a insolvency resolution
process.
(D) J Limited, who supplied goods to ACL one month prior to the date of making the
application and the invoice demanding payment is in transit. (2 Marks)
4.6 Answer the following questions in the context of the provision relating to PBPT Act, 1988
with reasons and explanations :
(i) Analyse based on the facts of the case provided above, whether the Initiating
Officer's actions were appropriate in concluding that the transaction was a benami
transaction. (3 Marks)
(ii) What are the factors that will need to be considered for the purpose of determining
whether a. transaction is a benami transaction ? (4 Marks)
4.7 Answer the following questions in the context of the provision relating to IBC, 2016 with
reasons and explanations :
(i) Examine the appropriateness of the approval or otherwise of the resolution plan of
Seaview Constructions and whether the view taken by the RP is appropriate.
(5 Marks)
(ii) Advise Sunflower Estates with regard to the rejection of the request for inclusion
into the Committee of Creditors of Seaview Constructions. (3 Marks)
ANSWER TO CASE STUDY 4
4.1 (D)
4.2 (B)
4.3 (C)
4.4 (B)
4.5 (A) [In order to drive the answer for initiation of CIRP against Seaview
constructions, word “Operational Creditor” is to be assumed as corporate
person].
Descriptive Answers
4.6 (i) Course of action taken by Initiating Officer under Section 24 of the PBPT Act,
2016: Where the Initiating Officer, on the basis of material in his possession, has
reason to believe that any person is a benamidar in respect of a property, he may,
after recording reasons in writing, issue a notice to the person to show cause within

© The Institute of Chartered Accountants of India


PAPER – 6D: ECONOMIC LAWS 27

such time as may be specified in the notice why the property should not be treated
as benami property.
Where the notice specifies any property as being held by a benamidar, a copy of the
notice shall also be issued to the beneficial owner if his identity is known. Where the
Initiating Officer is of the opinion that the person in possession of the property held
benami may alienate the property during the period specified in the notice, he may,
with the previous approval of the Approving Authority, by order in writing, attach
provisionally the property in the manner as prescribed in Rule 4 of the Benami
Transactions Prohibition Rules, 2016, for a period not exceeding ninety days
from the date of issue of notice.
The Initiating Officer, after making such inquires and calling for such reports or
evidence as he deems fit and taking into account all relevant materials, shall, within
a period of ninety days from the date of issue of notice —
(a) where the provisional attachment has been made —
(i) pass an order continuing the provisional attachment of the property with
the prior approval of the Approving Authority, till the passing of the order
by the Adjudicating Authority; or
(ii) revoke the provisional attachment of the property with the prior approval
of the Approving Authority;
(b) where provisional attachment has not been made—
(i) pass an order provisionally attaching the property with the prior approval
of the Approving Authority, till the passing of the order by the Adjudicating
Authority; or
(ii) decide not to attach the property as specified in the notice, with the prior
approval of the Approving Authority.
Where the Initiating Officer passes an order continuing the provisional attachment of
the property or passes an order provisionally attaching the property, he shall, within
fifteen days from the date of the attachment, draw up a statement of the case and
refer it to the Adjudicating Authority under Section 26 of the PBPT Act, 2016.
Yes, the actions taken by the initiating officer, were appropriate in the compliance of
the above stated provisions.
Alternate Solution
For a transaction to be covered under Section 2 (9)(A) of PBPT Act, 1988 the
following two conditions are to be met.

© The Institute of Chartered Accountants of India


28 FINAL (NEW) EXAMINATION: NOVEMBER, 2019

(i) The consideration for the property has been provided or paid by another
person and
(ii) The property is held for immediate or future benefit direct or indirect of the
person who provided the consideration
In order to ascertain whether a particular sale is Benami and the apparent
purchaser is not the real owner, the burden lies on the person who sets up the case
and such burden has to strictly discharged based on legal evidence of definite
nature. Therefore, the Initiating Officer (IO) cannot show cause High City and seek
proof as to why the transaction cannot be treated as a Benami transaction.
Therefore, the onus is on the IO to prove, if at all the transaction is a Benami
transaction. Moreover, if it is shown by the parties to the alleged Benami transaction
that such transaction is done through a registered sale deed and valid loan
agreements, the burden of proof would be shifted upon the IO to prove the
transaction as a Benami transaction.
Merely because the source of consideration paid by the alleged benamidar is
funded by way of loan received from a party related with the alleged beneficial
owner, it cannot be ipso facto held that the consideration has been provided by the
alleged beneficial owner and more so when evidences has been brought on record
to show that the aforesaid loan was a genuine transaction and was done at arms
length in the normal course of business. Therefore the IO’s action is not
appropriate in concluding that the transaction was a Benami transaction.
(ii) Factors that will need to be considered for the purpose of determining of a
benami transaction:
As per Section 2(9) of the PBPT Act, 2016 in order to be a "Benami Transaction"
such transaction /arrangement w.r.t a property is to be considered as a benami—
(i) where a property is transferred to, or is held by, a person, and the
consideration for such property has been provided, or paid by, another person;
and
(ii) the property is held for the immediate or future benefit, direct or indirect, of the
person who has provided the consideration,
(iii) a transaction or an arrangement in respect of a property carried out or made in
a fictitious name; or
(iv) a transaction or an arrangement in respect of a property where the owner of
the property is not aware of, or, denies knowledge of, such ownership;
(v) a transaction or an arrangement in respect of a property where the person
providing the consideration is not traceable or is fictitious;

© The Institute of Chartered Accountants of India


PAPER – 6D: ECONOMIC LAWS 29

4.7 (i) Procedure of seeking approval of the Resolution plan in the light of Section 30
of the Insolvency and Bankruptcy Code:
(i) Seeking approval of CoC: The resolution professional shall present such
resolution plans to the committee of creditors for its approval by a vote of not
less than sixty-six per cent of voting share of the financial creditors.
(ii) Submission of the Resolution Plan: The resolution professional shall submit
the resolution plan as approved by the committee of creditors to the
Adjudicating Authority. [Section 30]
(iii) Approval of Resolution Plan: If the Adjudicating Authority is satisfied that
the resolution plan as approved by the committee of creditors meets the
requirements as per Section 30(2), it shall by order approve the resolution
plan.
(iv) Rejection of the Resolution Plan: Where the Adjudicating Authority is
satisfied that the resolution plan does not confirm to the above requirements, it
may, by an order, reject the resolution plan.
The resolution applicant shall obtain the necessary approval pursuant to the
resolution plan approved, within a period of one year from the date of approval of
the resolution plan by the Adjudicating Authority or within such period as provided
for in such law, whichever is later.
In the given instance, the resolution plan of Seaview Constructions will not be
passed as it was not approved by a vote of sixty-six per cent of voting share of the
financial creditors. Out of Total six financial creditors, four financial creditors voted
on the resolution plan and two abstained from voting. Further out of 4 Financial
creditors, 3 Financial creditors with the voting share (22.33% + 14.39%+15.15% =
51.87%) approved the Resolution plan. However,1 Financial creditor with voting
share 26.36% voted against the resolution plan. Resolution professional stand
was correct as regarding the filing of liquidation as the resolution plan was not
approved by the CoC with the requisite majority of 66% of voting share.
(ii) When Financial Creditor /authorized representative is not entitled to
participate in the CoC:
As per Section 21 of the IBC, for the Financial Creditor or the authorised
representative of the financial creditor referred to in Section 24(6), 24(6A), or 24(5),
related to the conduct of meetings of creditors, if it is a related party of the
corporate debtor, shall not have any right of representation, participation or voting
in a meeting of the committee of creditors.
Here the rejection of the request to Sunflower Estates, for inclusion into the
committee of creditors of Seaview constructions, is valid.

© The Institute of Chartered Accountants of India


30 FINAL (NEW) EXAMINATION: NOVEMBER, 2019

CASE STUDY - 5
Decor Design Constructions Private Limited (Decor Constructions) is a reputed construction
company based in Pune, India and specialises in construction mid-sized apartments
(approximately 20 apartments in each project). Decor Constructions was founded by 2
brothers, Mr. Ravi Rao and Mr. Giri Rao, and are the Directors of Decor Constructions. Mr.
Ravi Rao studied civil engineering in the UK and worked extensively in the UK in various
infrastructure and construction companies before moving back to India to establish Decor
Constructions. During the year 2014, Decor Constructions commenced a new project called as
Decor Dream Home, which comprises of 30 apartments, each having a super built-up area of
1,800 square feet and carpet area of 1,500 square feet. All the 30 apartments were sold by
Decor Constructions within a period of 3 months and they entered into a sale agreement with
the allottees in the month of November, 2014. The following were the key features of the sale
agreement:
• The apartments were sold to the allottees at a square feet rate of ` 5,000 per square feet
and the total consideration for each of the apartments were calculated based on the
super built-up area.
• The application fee to be paid prior to entering into the sale agreement was fixed as 8%
of the total consideration.
• The entire amount of consideration should be paid by the allottee within 6 months from
the sale agreement, irrespective of the date / stage of completion of the construction.
This is to facilitate the speedy completion of construction. Decor Constructions has
already factored in a discount in the per square feet rate to compensate the allottees for
the upfront payment.
• Free open car parking to the allottees who pay the entire consideration at the time of sale
agreement. For other allottees, the open car parking will be allotted on payment of
` 200,000.
• The apartment will be handed over to the allottees within 30 months from the date of the
agreement i.e. by 31st May, 2017.
All the 30 allottees made the payment to Decor Constructions in accordance with the
agreement (10 of the allottees paid the full amount on the date of the sale agreement thereby
getting a free open car park) and an amount of ` 2,700 lakhs was received by Decor
Constructions. During the month of August 2016, Decor Constructions sent an e-mail to all the
30 allottees that the Promoter has filed the required forms for approval from the Municipal
Corporation for water, sewerage and electricity connections and this is taking substantial time
to complete, which is not in the control of the Promoter and therefore, the date of handing over
will get slightly delayed to 31st December, 2017. None of the allottees responded to the
communication. In the meanwhile, with the introduction of Maharashtra Real, Estate
(Regulation and Development) Act with effect from 1st May,2017, Decor Constructions

© The Institute of Chartered Accountants of India


PAPER – 6D: ECONOMIC LAWS 31

registered the project under the RERA and as part of the registration stated the expected date
of completion as 30th June, 2018.
Although Mr. Ravi Rao has been in India for more than three years, his ultimate aim is to settle
down in Switzerland, which is the home country of his spouse, Ms. Anne Rao. Therefore, Ravi
wanted to buy a colonial villa in Switzerland for an amount of EUR 2 million. Mr. Giri Rao is of
the view that the FEMA rules does not allow Mr. Ravi Rao to invest in immovable property
outside India when he is resident in India.
Ms. Anne Rao (spouse of Mr. Ravi Rao) who is a citizen of USA, wants to purchase an
immovable property (apartment) in India jointly along with Mr. Ravi Rao. For this purpose, Ms.
Anne Rao is proposing to take a housing loan in her personal name from Bank of Bengaluru, a
bank operating in India. However, considering the fact that she is a citizen of USA, the Bank
has included a pre-condition that the loan be guaranteed by Decor Constructions. Based on
such request, Decor Constructions has provided the required guarantee in favour of Bank of
Bengaluru. Ms. Anne Rao is also interested in investing USD 200,000 in a Special Purpose
Vehicle (in the form of an unincorporated joint venture) which is engaging in the business of
providing managed farm to its investees and provide the land after a period of 20 years. Ms.
Anne Rao before attempting further transactions approached the consultant to advise on the
transactions which are not capital account transactions.
In the month of June 2017, Decor Constructions sent another e-mail to the 30 allottees that
the construction of the super structure of Decor Dream Home is almost complete and what is
left is only to complete the interior plastering, flooring, plumbing etc. and this will get
completed by 31st March, 2018 and the slight extension of the timeline is only on account of
labour shortage at Pune due to the extensive construction spree happening in the city. Decor
Dream Home also suggested to the allottees that they were ready to handover the apartment
in the month of December, 2017 (before receiving the occupancy certificate) to the allottees
for them to get the interior/furnishing work done so that the allottees can occupy the
apartments in March/April, 2018 as soon as occupancy certificate is received.
All the 30 allottees were not happy on account of the further delay in completion and filed a
complaint against Decor Constructions under the Maharashtra RERA provisions. Out of the 30
allottees, 25 allottees sought cancellation of the sale agreement and refund of the amounts
paid by the allottees along with interest at 21% p.a. The balance 5 allottees wanted to be
compensated by Decor Constructions for the delay in completion-but do not want to cancel the
sale agreement.
Decor Constructions has submitted before the RERA authorities the following:
• Notwithstanding the registration of the project under RERA as per the requirements of
Section 3 of the RERA, the sections relating to compensation for delay etc. do not apply
to the project since the date of commencement of project / date of sale agreement is
prior to the date when RERA came into effect.

© The Institute of Chartered Accountants of India


32 FINAL (NEW) EXAMINATION: NOVEMBER, 2019

• Even otherwise, the date of completion stated in the RERA registration is 30th June 2018
and therefore, the date of handover finally indicated allottees is 31st March 2018, which is
well within the timelines and therefore, there is no non-compliance with the RERA
requirements.
• The Company had already informed the reasons for the delay of the project upto
31st December, 2017 in August, 2016 itself and there was no response / issue raised by
the allottees at that time. Further, Decor Constructions has also agreed to provide the
apartments for interior work during December, 2017 and therefore, it is effectively agreed
to handover the apartment as per the revised timelines communicated in August, 2016.
• Even presuming the applicability of the RERA provisions, there is no unanimity in the
decisions of the allottees on the way forward (since 25 have opted for cancellation and 5
have opted for compensation) and therefore, this cannot be anyway given effect to under
RERA.
Accordingly, Decor Constructions has submitted that they are not liable for any compensation
to be paid under RERA and have re-iterated that they will handover the apartments to the
allottees by the revised timelines indicated in the e-mail sent in June, 2017.
Answer the following questions:
5.1 What is your view regarding the terms of the agreement relating to the open car
parking arrangement with the allottees ?
(A) Decor Constructions is free to stipulate any terms and conditions in this regard,
since this is a transaction between a willing buyer and a willing seller.
(B) Decor Constructions is required to provide open car parking for all allottees on
equitable terms and there cannot be a discrimination based on payment schedule.
(C) Open parking areas cannot be sold for consideration since they are to be
considered as common area of the Project.
(D) Open parking is part of internal development works and is part of overall project
costs which can be charged by the Promoter equally to all allottees. (2 Marks)
5.2 One of the allottees of Decor Dream Home have-reached out to you for your advice on
whether the 'collection of the entire consideration by Decor Constructions without regard
to the stage of constructions is appropriate.
(A) Appropriate. The terms/timing of payment are governed by the sale agreement
between the promoter and allottee.
(B) Not appropriate. The timing of payment should be in line with the stage wise
completion / construction schedule.

© The Institute of Chartered Accountants of India


PAPER – 6D: ECONOMIC LAWS 33

(C) Appropriate, since the necessary discount has already been factored into the
consideration by Decor Constructions.
(D) Appropriate, provided Decor Constructions has obtained the approval of the terms
at the time of registration of the Project under RERA. (2 Marks)
5.3 Advice of the consultant to Ms. Anne Rao for the transaction which do not fall under the
definition of a capital account transaction under FEMA, 2002 will be:
(A) Transactions which alter the assets and liabilities of non-residents in India.
(B) Transactions which alter the assets and liabilities (including contingent liabilities) of
residents outside India.
(C) Transactions relating to transfer of a security by a branch in India of a company
resident outside India.
(D) Transactions which alter the assets and liabilities (including contingent liabilities) of
non-residents in India. (2 Marks)
5.4 Mr. Vishy Rao, brother of Mr. Ravi Rao, is a resident of Singapore and he owns an
immovable property in Chennai which he inherited from his father, who was a resident of
India. Can Mr. Vishy Rao continue to hold the property?
(A) No, he cannot hold transfer or invest in India, since he is resident outside India.
(B) Yes, he can continue to hold in India, since he is a person of Indian Origin and the
property is located in India.
(C) Yes, he can continue to hold the property, since this was inherited from a person
who was resident in India.
(D) Yes, he can continue to hold the property, since his brother (Mr. Ravi Rao) uses the
property whenever he travels to Chennai. (2 Marks)
5.5 Decor Constructions is in the process of entering into certain business transactions with
international agencies and in this context Mr. Girl Rao seeks your views on the maximum
amount that can be paid by Decor Constructions under the Liberalised Remittance
Scheme and how much he can pay in his own individual capacity under the Scheme, per
year ?
(A) Decor Constructions - USD 250,000; Individually - USD 250,000.
(B) Decor Constructions - USD Nil; Individually - USD 250,000.
(C) Decor Constructions - No limit for specified objects; Individually - USD 200,000.

© The Institute of Chartered Accountants of India


34 FINAL (NEW) EXAMINATION: NOVEMBER, 2019

(D) Decor Constructions - USD 500,000 (USD 250,000 for each director); Individually -
USD Nil, since the same is considered under Decor Constructions' limit. (2 Marks)
5.6 Answer the following questions in the context of the provisions relating to Real Estate
Regulation Act, 2016 (RERA):
(i) Analyse whether the provisions of RERA (which came into effect from 1st May, 2017)
are applicable to the Decor Dream Home project and if Decor Constructions is liable
for obligations under RERA. (3 Marks)
(ii) Analyse based on the facts of the case, regarding each of the averments of Decor
Constructions with regard to its obligations under RERA for the alleged delay in
handover of the apartments to the allottees and whether it is liable for payment of
compensation under RERA. (6 Marks)
5.7 Examine / advise regarding the below questions relating to the Foreign Exchange
Management Act, 1999 :
(i) How would you advise Mr. Ravi Rao with regard to his aim of acquiring a colonial
villa in Switzerland when he is a resident in India. (2 Marks)
(ii) Evaluate the implications of the transactions proposed to be entered into by
Ms. Anne Rao, including the consequential / related transactions. (4 Marks)
ANSWER TO CASE STUDY 5
5.1 (C)
5.2 (A)
5.3 (D)
5.4 (C)
5.5 (B)
Descriptive Answers
5.6 (i) The project was commenced in November, 2014 and was in progress on the
effective date of coming into force of RERA, 2016 i.e. on 1st May, 2017. As per
Section 3(1) of RERA, 2016, the promoter shall make an application to the Authority
for registration of the project that is ongoing on the date of commencement of this
Act and for which completion certificate has not been issued within a period of three
months from the date of commencement of this Act.
Accordingly, the provisions of RERA are said to be applicable to the Décor
Dream Home Project as no completion certificate has been issued within a period of
three months from the date of commencement of this Act i.e., uptill July end 2017.

© The Institute of Chartered Accountants of India


PAPER – 6D: ECONOMIC LAWS 35

(ii) Return of amount and compensation (Section 18)


This Section provides for the return of amount and compensation.
If the promoter fails to complete or is unable to give possession of an apartment,
plot or building, in accordance with the terms of the agreement for sale or, as the
case may be, duly completed by the date specified therein; he shall be liable on
demand to the allottees, in case the allottee wishes to withdraw from the project,
without prejudice to any other remedy available, to return the amount received by
him in respect of that apartment, plot, building, as the case may be, with interest at
such rate as may be prescribed in this behalf including compensation in the manner
as provided under this Act.
However, where an allottee does not intend to withdraw from the project, he shall be
paid, by the promoter, interest for every month of delay, till the handing over of the
possession, at such rate as may be prescribed.
If the Promoter fails to discharge any other obligations imposed on him under this
Act or the rules or regulations made thereunder or in accordance with the terms and
conditions of the agreement for sale, he shall be liable to pay such compensation to
the allottees, in the manner as provided under this Act.
According to the relevant provisions, Décor Constructions will not be liable under
RERA for handover of the apartments to the allottees as it was within the expected
date of completion i.e., 30th June, 2018. Therefore, Decor Constructions shall not
be liable for payment of compensation.
Alternate Solution
Analysis of each of Averments of Décor Constructions with regard to its
obligations under RERA for the alleged delay in handing over the apartments to the
allottees:
AVERTMENT (1): Even though, the date of completion stated in RERA registration is
30th June, 2018 and therefore, the date of handover finally indicated to the allottees is
31st March, 2018 which is well within the timelines and therefore, there is no non-
compliance with the RERA requirements
As per Section 18, if the promoter fails to complete or is unable to give possession of an
apartment, plot or building, in accordance with the terms of the agreement for sale or, as
the case may be, duly completed by the date specified therein; he shall be liable on
demand to the allottees, in case the allottee wishes to withdraw from the project, without
prejudice to any other remedy available, to return the amount received by him in respect
of that apartment, plot, building, as the case may be, with interest at such rate as may be
prescribed in this behalf including compensation in the manner as provided under this
Act.

© The Institute of Chartered Accountants of India


36 FINAL (NEW) EXAMINATION: NOVEMBER, 2019

On a plain reading of this provision, it becomes clear that date of completion referred to
in this provision is the date specified in the agreement. The word “therein” refers to the
“agreement” and not the date of completion revised by the Promoters unilaterally while
registering the project. Hence, the submission of Décor Constructions that as till the date
of completion mentioned in the registration certificate is not crossed, there is no delay
in not valid.
AVERTMENT – 2: The Company had already informed the reasons for the delay of the
project upto 31st December, 2017 in August, 2016 itself and there was no response /
issues raised by the Allottees at that time. Further, Décor Constructions has also agreed
to provide the apartments for interior work during December,2017 and therefore, it is
effectively agreed to handover the apartment as per the revised timelines communicated
in August, 2016.
From the facts of the case, it appears that Décor Constructions is of the view that since
the complainants did not object to the extended time, hence, the complainants by their
conduct agreed to extend the period of delivery of the possession of the flats. This is not
acceptable because a party cannot take unilateral decision and impose it upon the other
party. The parties have decided to withdraw from the project since the flats were not
delivered on time and no where have they agreed to the new dates as unilaterally
declared by the Company. The handover of the apartments prior to obtaining the
occupancy certificate is mere paper possession and possession without such certificate
is illegal and cannot be permitted in law. Therefore, this offer has been rejected by the
complainants and have exercised their right to claim back their money.
AVERTMENT – 3: Even presuming the applicability of the RERA provisions, there is no
unanimity in the decisions of the allottees on the way forward (since 25 have opted for
cancellation and 5 have opted for compensation) and therefore, this cannot be anyway
given effect to under RERA.
Section 18 offers two options to the allottees – one is for return of the amounts, or
compensation, if the allottees decide not to withdraw from the project. It is not
necessary for unanimity in the decision of the allottees and the promoter is liable
to refund / compensate the allottees based on the option that they choose.
5.7 (i) As per FEMA, 1999 under Section 6(4), a person resident in India may hold, own,
transfer or invest in foreign currency, foreign security or any immovable property
situated outside India if such currency, security or property was acquired, held or
owned by such person when he was resident outside India or inherited from a
person who was resident outside India.
The RBI vide A.P. (DIR Series) Circular No. 90 dated 9th January, 2014 has
issued a clarification on Section 6(4) of the Act. According to which a person
resident in India may freely utilize all their eligible assets abroad as well as income
on such assets or sale proceeds thereof received after their return to India for

© The Institute of Chartered Accountants of India


PAPER – 6D: ECONOMIC LAWS 37

making any payments or to make any fresh investments abroad without approval of
Reserve Bank, provided the cost of such investments and/or any subsequent
payments received therefor are met exclusively out of funds forming part of eligible
assets held by them and the transactions is not in contravention to extant FEMA
provisions.
Accordingly, Mr. Ravi Rao aim of acquiring a colonial villa in Switzerland when
he is resident in India is possible and in compliance with the above provision.
(ii) In the given case, Ms. Anne Rao proposed for two types of investments in India:
(i) Purchase of immovable property in India Jointly with Mr. Ravi Rao
(ii) Investing USD 2,00,000 in special purpose vehicle
W.r.t. part (i) of the transaction proposed by Ms Anne Rao, according to Section
6(3), the Reserve Bank may, by regulations, prohibit, restrict or regulate the giving
of a guarantee or surety in respect of any debt, obligation or other liability incurred
by a person resident outside India.
Therefore, proposed transaction as to purchase of immovable property to be
entered by Ms. Anne Rao, is valid on the guarantee of Décor Construction.
W.r.t. part (ii) of the transaction proposed, investments (or financial commitment) in
JV/WOS abroad by Indian parties through the medium of a Special Purpose Vehicle
(SPV) are also permitted under the Automatic Route if the Indian party is not
appearing in the Reserve Bank's caution list or is under investigation by the
Directorate of Enforcement or included in the list of defaulters to the banking system
circulated by the Reserve Bank/any other Credit Information Company as approved
by the Reserve Bank.
As in the given case, investment in a Special Purpose Vehicle in the form of an
unincorporated joint venture, is invalid in line with the above provision.
Alternate Solution to Part (ii)
As per the Foreign Exchange Management (Permissible Capital Account
Transactions) Regulations, 2000, the person resident outside India is prohibited
from making investments in India in any form, in any Company, or partnership firm
or proprietary concern or any entity whether incorporated or not which is engaged or
proposes to engage in agricultural or plantation activities.
Accordingly, Ms. Anne Rao cannot invest in the aforesaid business since
managed farm business is included under prescribed business of agricultural
and plantation activities.

© The Institute of Chartered Accountants of India


DISCLAIMER

This Suggested Answers do not constitute the basis for evaluation of the

student’s answers in the examination. The answers are prepared by the

Faculty of the Board of Studies with a view to assist the students in their

education. While due care is taken in preparation of the answers, if any

errors or omissions are noticed, the same may be brought to the attention

of the Director of Studies. The Council of the Institute is not in anyway

responsible for the correctness or otherwise of the answers published

herein.

Further, in the Elective Papers which are Case Study based, the solutions

have been worked out on the basis of certain assumptions/views derived

from the facts given in the question or language used in the question. It may

be possible to work out the solution to the case studies in a different manner

based on the assumption made or view taken. Further, there should be no

negative marking for wrong answers in MCQ based questions.

© The Institute of Chartered Accountants of India


2 FINAL (NEW) EXAMINATION: MAY 2019

PAPER-6D – ECONOMIC LAWS


There are three case study questions in the question paper. Candidates are required to
answer all the questions of any two case study questions.

CASE STUDY 1
Mr. Rohit writer is a well known industrialist based in Pune, India and is the founder director of
M/s. Good Phones Private Limited (Good Phones), a fixed line and mobile phone manufacturer.
Good Phones is one of the largest telephone companies in India and its products are much
sought after in India and abroad. Mr. Rohit visits various countries as part of his business travels
and during these visits he spend significant amount of time in Philanthropic activities and social
gatherings and because of this, he is quite well known in the business circles globally. Mr. Rohit
has a penchant for investing his money in buying various real estate property all over India and
has passed this trait on to his son, Mr. Rahul Writer as well. Mr. Rahul completed his MBA from
Stanford University and is assisting Mr. Rohit in his business. Mr. Rohit also has a daughter,
Mr. Sonali Writer, who studies Art in Italy and has opened her own Art Studio in Milan. Mr. Rohit
is very proud of Sonali and supports her financially for her stay in Italy as well as expenses
towards maintaining the studio.
The marketing department of Good Phones introduced various new models in the last couple of
months with new technology such as 2 selfie cameras, faster processor and sleeker look. Good
Phones expect these phones to be a major attraction in the global markets due to the attractive
price range and therefore wanted to promote these phones extensively on a global basis. For
the purpose of advertisements, Good Phones engaged the services of Mr. David Smith, a
prominent baseball player and Ms. Emma Drew, a Miss Universe winner and agreed to pay a
“guaranteed” fee of USD 500,000 each plus 10% bonus based on the sales of the new models
in year 1.
Mr. Rohit sent 5 sample mobile phones and 5 fixed line phones to his dealers abroad (numbering
1000 dealers), clearly marked as not for sale and other promotional material such a brouchers,
3D moulds for display in dealer shops etc. The value of the items were approximately INR 4
crore. He also sent 1 mobile phone to each of his dealers as a token of gift and appreciation
(total value of INR 0.50 Crores). Mr. Srinivas Rajan, the CFO of Good Phones indicated him
that since these products have been sent free of cost and not for sale, these need not be
included in the export declaration to be filed by Good Phones.
On 15 February 2018, Good Phones made a large sale to one of the dealers M/s. Delayed
Ringtone Enterprise, Germany, for USD 5 million and had received USD 2 million by 15 May
2018 and did not receive the balance USD 3 million until 15 August 2018, i.e. 6 months from the
date of sale. After several reminders and threatening calls to stop further shipment, another
USD 1 million was received on 10 October 2018 and the balance remained outstanding as at
31 December 2018.

© The Institute of Chartered Accountants of India


PAPER – 6D: ECONOMIC LAWS 3

Based on the success of Good Phones, Mr. Rohit incorporated a new company, M/s. Stay
Connected Private Limited, (Stay Connected) an Internet service provider and purchased a large
consignment of networking equipment for providing internet operations through dedicated
broadband lines along with a landline facility. This would then provide Mr. Rohit quite a few
synergies with the existing Good Phones business and enable him to become an end to end
Telecom Czar. Mr. Rohit held 60% stake in Stay Connected and the balance 40% was held by
a foreign collaborator. Along with all the networking equipment, Stay Connected hired
transponders from a company in Australia and paid AUD 10 million through its authorized healer.
Stay Connected also entered into an agreement with foreign collaborator (holding 40% stake)
to pay royalty and technical fees for the support provided by them.
During his visit to Milan to meet Ms. Sonali, Mr. Rohit obtained EUR 10,000 from his Italian
dealer for his use during his stay in Italy and instructed the dealer to reduce the sum from the
payments to be made by the dealer for the supplies from Good Phones. Out of such funds,
Mr. Rohit used EUR 5,000 towards purchasing sweepstakes tickets in Mila, Italy, unfortunately,
he did not win any money in the sweepstakes event.
Mr. Rahul, after gaining experience in India, wanted to expand the business in the USA (by
establishing a subsidiary of Good Phones in the USA) and therefore decided to move to the
USA along with his wife. For this purpose, he wanted to dispose off some of the properties
owned by him in India. Accordingly, Mr. Rahul sold an apartment in Mumbai owned by him to
Mr. Stuart Cooper, a citizen of USA, and a fellow student of his at Stanford University. Mr. Stuart
was planning to come to India for the first time in the next couple of months to take up a job and
therefore, wanted to secure a place for his stay. Mr. Rohit also sold a villa and his agricultural
land in Pondicherry to Mr. Rajesh Subramanian, his professor at Stanford, who was a person of
India origin. The payment for the villa and agricultural land was paid by Mr. Rajesh partly (50%)
from his FCNR account and the balance in USD traveller cheques, which will be of use to Mr.
Rahul when he visits USA.
After obtaining his US visa, Mr. Rahul purchased a ranch (farm house) in Texas for USD 2
million, using USD 1.50 million from his RFC account and USD 500,000 sent from his INR
account through normal banking channels.
Mr. William Rutherford, one of Mr. Rohit’s business acquaintances and a citizen of the USA, is
very much interested in Indian culture and practices and therefore stays in India for 8 months
(from April 2018 to November 2018) to attend an art of living course and to learn/practice yoga.
William believes that he has been resident in India for more than the prescribed 182 days and
therefore, is a resident in India under FEMA.
Mr. Rohit, in his penchant for purchasing various properties, zeroed in on an exclusive
apartment complex in Bangalore having state-of-the art facilities. He purchased two 4 bedroom
apartments costing INR 2 crore each, one in the name of Ms. Sonali and one in the name of Mr.
Srinivas Rajan, since Mr. Rohit wanted Mr. Srinivas Rajan to feel happy and trusted. Both the
apartments were given on rent to a large multinational bank and he received a rent of INR 0.20
Crores per year for each of the apartments in the bank accounts of Ms. Sonali and Mr. Srinivas

© The Institute of Chartered Accountants of India


4 FINAL (NEW) EXAMINATION: MAY 2019

Rajan, respectively. After 4 years, Mr. Srinivas Rajan transferred the property back in the name
of Mr. Rohit at zero consideration. Mr. Rohit also purchased a 3 bedroom apartment in the same
complex in his name, jointly with his brother, Mr. Sunil Writer. The property (along with the stamp
duty) was paid for by Mr. Rohit and was being used by Mr. Sunil for his stay though the property
was pending registration due to Rohit’s travel abroad.
Once the property was transferred back by Mr. Srinivas Rajan, Mr. Rohit wanted to sell the same
to Mr. Arjun De Silva, a citizen of Shi Lanka. However, he was advised by Mr. Srinivas Rajan
that Mr. Arjun De Silva cannot acquire property in India and therefore, Mr. Rohit proposed to
lease it to Mr. Arjun De Silva for a period of 20 years for an upfront consideration of INR 1 crore
and an annual rent of INR 8 lakhs payable in advance.
During the review of the bank reconciliation statements of Good Phones, Mr. Srinivasan Rajan
noted that an amount of INR 2 Crore had been received in one of the bank accounts without
any details relating to the same. Mr. Srinivas Rajan informed this to Mr. Rohit and Mr. Srinivas
Rajan suggested to Mr. Rohit to immediately transfer that money out of the bank of Good Phones
to Mr. Rohit’s personal bank account, so that the Company’s bank accounts are cleared and
there are no reconciling items, which Mr. Rohit agreed to. Out of the INR 2 Crore, Mr. Rohit
used INR 1.75 Crores for acquiring further 20% stake in Stay Connected from the foreign
collaborator and the balance INR 0.25 Crores for purchasing a stunning diamond set for his
wife, Ms. Anjali Writer, as a gift for her 50 th birthday.
The extract of the last audited financial statements of Stay Connected was provided by
Mr. Srinivas Rajan to Mr. Rohit to evaluate his acquisition.
Particulars Amount in
INR (Crores)
Immovable property (market value INR 8.00 Crores) 5.00
Other fixed assets (net of depreciation of INR 1.00 Crores) 4.00
Inventory 2.00
Receivables and Loans and Advances 1.50
Deferred Advertisement Costs 0.50
Advance tax paid 1.00
Total Assets 14.00
Shareholders’ Funds (including 1,000,000 equity shares of INR 10 each, 4.00
fully paid up)
Provisions for taxation 0.50
Loans from Banks 3.00
Trade Payables (including provision for unascertained liabilities - INR 6.50
1 crore)
Total Liabilities 14.00

© The Institute of Chartered Accountants of India


PAPER – 6D: ECONOMIC LAWS 5

Other information:
(i) Contingent liabilities-INR 2.00 Crores (including INR 0.50 cores relating to arrears on
cumulative preference shares).
(ii) The Board of Directors has proposed a dividend payout of INR 1 crore to the equity
shareholders, which is pending approval of the shareholders.
The Bank, on noting the large transactions on Mr. Rohit's personal bank account, tipped
the Income tax authorities regarding the same and the Initiating Officer summoned
information from Mr. Rohit and Mr. Srinivas Rajan regarding the transactions to start
proceedings under the Prohibition of Benami Property Transactions Act, 1988 (PBPT Act,
1988).
Answer the following questions:
1.1 An Indian citizen resident outside India is permitted to transfer his agricultural property in
India to:
(A) any person resident in India
(B) a person resident in India if he is a citizen of India or a person of Indian origin
(C) a person resident in or outside India if he is a citizen of India or a person of Indian
origin
(D) any person who is resident in the country where the Indian citizen currently resides
(2 Marks)
1.2 Out of the below, what are the transactions that requires prior approval of the Government
of India ?
(A) Payment of "guaranteed" fee by Good Phones to Mr. David Smith and Ms. Emma
Drew.
(B) Payment of Royalty and Technical fees by Stay Connected to the foreign collaborator.
(C) Payment of hiring charges for the transponders by Stay Connected.
(D) Payment of INR 1.75 Crores by Mr. Rohit to acquire shares of Stay Connected from
the foreign collaborator (2 Marks)
1.3 Is the use of EUR 5,000 towards purchasing sweepstakes by Mr. Rohit as per the
provisions of FEMA, 2002 ?
(A) No, drawal of foreign exchange for purchasing lottery tickets, sweepstakes, etc. is
prohibited under the FEMA 2002

© The Institute of Chartered Accountants of India


6 FINAL (NEW) EXAMINATION: MAY 2019

(B) No, Mr. Rohit should have obtained the prior approval of the RBI before purchasing
the sweepstakes ticket
(C) FEMA 2002 will not be applicable, since the money was directly obtained by
Mr. Rohit from his Italian dealer outside the country
(D) None of the above (2 Marks)
1.4 As per the provisions of FEMA, 2002, Mr. William Rutherford is :
(A) a person resident in India in the financial year 2018-19 as per FEMA, "since he has
resided for more than 182 days during the year
(B) a person resident in India in the financial year 2019-20 as per FEMA, since he has
resided for more than 182 days during the previous financial year
(C) not a person resident in India since he is a foreign citizen
(D) not a person resident in India, since he is on a short term visit to India and is not on
a long term visa (2 Marks)
1.5 Is the purchase of Ranch in Texas by Mr. Rahul in accordance with FEMA, 2002?
(A) No, Rahul, as a citizen of India cannot purchase a Ranch outside India.
(B) Yes, there is no specific limit under FEMA 2002 with regard to purchase of
immoveable property outside India.
(C) No, Rahul can purchase assets outside India only if the purchase is jointly with a
relative, who is a resident outside India, and there is no outflow of funds.
(D) No, since Rahul has used funds from his INR account for making the payment to the
extent of USD 500,000. (2 Marks)
1.6 In case Mr. Rohit is proven guilty of violating the provisions of PBPT Act, 1988, what is the
maximum punishment that he is liable for under the PBPT Act, 1988 ?
(A) Rigorous imprisonment for a term of one to seven years, with fine which may extend
to 25 of the fair market value of the property.
(B) Rigorous imprisonment for a term of three to seven years, without fine.
(C) Rigorous imprisonment for a term upto seven years, with fine which may extend to
50% of the fair market value of the property.
(D) Fine which may extend to 25% of the fair market value of the property. (2 Marks)
1.7 Once the benami property acquired by Mr. Rohit and his family have been identified, which
authority has the power to confiscate and vest the property?

© The Institute of Chartered Accountants of India


PAPER – 6D: ECONOMIC LAWS 7

(A) Initiating Officer


(B) Administrator
(C) Approving Authority
(D) Adjudicating Authority (2 Marks)
1.8 Which of the following is a benami property under the PBPT Act, 1988 ?
(A) Property held by a member of a Hindu Undivided Family (not the Karta) which is held
for the benefit of the members of the family
(B) Property purchased by a person in the name of his spouse paid out of money received
from his father in law as a gift.
(C) Property purchased by a person in the name of his brother out of his own funds.
(D) Property purchased in the name of an individual, for which consideration was paid by
another person and such another person enjoys the possession of the property.
(2 Marks)
1.9 Assuming that the transactions relating to the receipt of INR 2 crores in the bank account
of Good Phones and the subsequent transactions are considered as benami transactions,
can the Initiating Officer take action against Mr. Srinivas Rajan ?
(A) Yes, he is the CFO of Good Phones and therefore, responsible for ensuring
compliance with the law.
(B) No, he has not received, held, or acquired the proceeds in his account or benefitted
from the same.
(C) Yes, since he abets Mr. Rohit in transferring the money from the bank account of
Good Phones to Mr. Rohit's personal account.
(D) No, he is responsible only for Good Phones and he has ensured that the funds are
not retained in the books of Good Phones / used by Good Phones for its business.
(2 Marks)
1.10 The Initiating Officer believes that Ms. Anjali is not a benamidar under the PBPT Act,
1988. What is your view?
(A) No, she is not a benamidar, since she has not purchased the diamond set, but
received as a Gift.
(B) Yes, she is a benamidar as she is in possession of a property acquired out of benami
funds.

© The Institute of Chartered Accountants of India


8 FINAL (NEW) EXAMINATION: MAY 2019

(C) No, she is not a benamidar, since the transaction is not a benami transaction.
(D) Yes, she is a benamidar, but will not be liable for any prosecution under the PBPT
Act, since she is not a party to any of the transactions but only a beneficiary.
(2 Marks)
1.11 Answer the following questions in the context of the provisions relating to the Foreign
Exchange Management Act, 2002 :
(i) Srinivas Rajan reaches out to you to confirm his views regarding inclusion / exclusion
of the items sent free of cost to the dealers in the export declaration.
(4 Marks)
(ii) Examine the validity / appropriateness of the sale of immoveable property by Mr.
Rahul to Mr. Stuart Cooper and Mr. Rajesh Subramanian. (4 Marks)
(iii) Srinivas Rajan reaches out to you and seek your support to evaluate if there is a non-
compliance with the FEMA regulations regarding the sale made to M/s Delayed
Ringtone and the receipt of the proceeds and if so, the quantum, the consequences
and the future course of action that needs to be taken by Good Phones relating to the
same. (4 Marks)
(iv) Arjun disagrees with the advice received from Srinivas Rajan and asks your views on
why he cannot purchase the home from Mr. Rohit and if not, whether the terms and
conditions of the lease are acceptable. (2 Marks)
1.12 Examine/advice regarding the below questions relating to the Prohibition of Benami
Property Transaction Act, 1988:
(i) Examine the appropriateness/impact of the PBPT Act 1988 on the 3 apartments
purchased by Mr. Rohit in Bangalore. How does the transfer back of the apartment
by Mr. Srinivas Rajan to Mr. Rohit affect your conclusion? (4 Marks)
(ii) The Initiating Officer, who is probing the transactions relating to the INR 2 crores
received and spent by Mr. Rohit, seeks your advice on identify the benami
properties/transaction, the benamidars, the beneficial owner. (4 Marks)
(iii) Assuming that the cost of acquisition and the market value based on discounted cash
flow method is INR 2 crores, calculated the fair market value of M/s. Stay Connected
in accordance with Rule 3 of the Prohibition of Benami Transactions Rules, 2016.
(4 Marks)
(iv) What is the process to be followed by the Initiating Officer for attachment of the
property under a benami transaction? (3 Marks)

© The Institute of Chartered Accountants of India


PAPER – 6D: ECONOMIC LAWS 9

Answer Case study 1


1.1 (A)
1.2 (C).
1.3 (A)
1.4 (D)
1.5 (D)
1.6 (A)
1.7 (D)
1.8 (D)
1.9 (C)
1.10 (B)
1.11 (i) As per the Regulation 3 of the Foreign Exchange Management (Export of Goods and
Services) Regulations, 2015, in case of exports taking place through Customs manual
ports, every exporter of goods or software to any place outside India, shall furnish to
the specified authority, a declaration as regards the export value of the Goods. In
respect of export of services to which none of the Forms specified in these
Regulations apply, the exporter may export such services without furnishing any
declaration, but shall be liable to realise the amount of foreign exchange whic h
becomes due or accrues on account of such export, and repatriate the same to India.
However, Regulation 4 of the said Regulation states of exemptions w.r.t. export of
goods / software may be made without furnishing the declaration on the following
items which are sent free of cost:
• trade samples of goods and publicity material supplied free of payment;
• by way of gift of goods accompanied by a declaration by the exporter that they
are not more than five lakh rupees in value.
Therefore, sending 5 sample mobile phones and fixed line phones to 1000 dealers is
exempted and does not require Good Phones to include in the export declaration.
With regard to sending mobile phones to the dealers as gift for a total value of
INR 50 lakhs, as per the above Regulation, the exemption for sending gifts by an
export is available only if the value of the goods are not more than ` 5 lakhs in value.
In the case study, since the value of the goods is more than the exemption limit, they
need to be included in the export declaration.
(ii) As per regulation 3 of the Foreign Exchange Management (Acquisition and Transfer
of Immovable Property in India) Regulations, 2018, an NRI or OCI, may acquire

© The Institute of Chartered Accountants of India


10 FINAL (NEW) EXAMINATION: MAY 2019

immovable property in India other than agricultural land/farm house/plantation


property. Provided that the consideration, if any, for transfer, shall be made out of (i)
funds received in India through banking channels by way of inward remittance from
any place outside India or (ii) funds held in any non resident account maintained in
accordance with the provisions of the Act, rules or regulations framed thereunder.
Mr. Staurt Cooper, being an Overseas Citizen of India is entitled to acquire an
apartment in Mumbai owned by Mr. Rahul.
Whereas in case of Mr, Rajesh Subramanian, being NRI, he may acquire immovable
property in India other than agricultural land/farm house/plantation property.
Provided further that no payment for any transfer of immovable property shall be
made either by traveler's cheque or by foreign currency notes or by any other mode
other than those specifically permitted under this regulation. Since with related
transactions, payment was made partly from FCNR account and in USD Traveler
cheques, which was against the mode of payment prescribed in the said regulation.
Therefore, sale of immovable property by Mr. Rahul to Mr. Stuart Cooper is valid,
whereas to Mr. Rajesh Subramanian, the said transaction is invalid.
(iii) Regulation 9 of the Foreign Exchange Management (Export of Goods and Services)
Regulations, 2015 specifies the Period within which export value of
goods/software/services to be realised.
According to it the amount representing the full export value of
goods/software/services exported shall be realised and repatriated to India within
nine months from the date of export.
Further that the Reserve Bank, or subject to the directions issued by that Bank in this
behalf, the authorised dealer may, for a sufficient and reasonable cause shown,
extend the period of nine months.
Due to delay in of the proceeds of sale, M/s Delayed Ringtone contravened provision
of this Act & Regulations.
According to Regulation 14 of the Foreign Exchange Management (Export of Goods
and Services) Regulations, 2015, where in relation to goods or software export of
which is required to be declared on the specified form and export of services, in
respect of which no declaration forms has been made applicable, the specified period
has expired and the payment therefor has not been made as aforesaid, the Reserve
Bank may give to any person who has sold the goods or software or who is entitled
to sell the goods or software or procure the sale thereof, such directions as appear
to it to be expedient, for the purpose of securing,
(a) the payment therefor if the goods or software has been sold and
(b) the sale of goods and payment thereof, if goods or software has not been sold
or reimport thereof into India as the circumstances permit, within such period as

© The Institute of Chartered Accountants of India


PAPER – 6D: ECONOMIC LAWS 11

the Reserve Bank may specify in this behalf;


Provided that omission of the Reserve Bank to give directions shall not have the effect
of absolving the person committing the contravention from the consequences thereof.
Therefore, in such situation, M/s Delayed Ringtone shall, upon adjudication under
Section 13 of the FEMA, be liable to a penalty up to thrice the sum involved in such
contravention where such amount is quantifiable, or up to two lakh rupees where the
amount is not quantifiable, and where such contravention is a continuing one, further
penalty which may extend to five thousand rupees for every day after the first day
during which the contravention continues.
Hence, Mr. Delayed will be liable to USD 6 million [2 million (sum involved in such
contravention) x 3)] and further penalty up to five thousand rupees for every day after
the first day during which the contravention continues.
(iv) As per Regulation 9 of the Foreign Exchange Management (Acquisition and Transfer
of Immovable Property in India) Regulations, 2018, prohibition on acquisition or
transfer of immovable property in India is cast on the citizens of certain countries.
Accordingly, no person being a citizen of Sri Lanka, without prior permission of the
Reserve Bank shall acquire or transfer immovable property in India, other than lease,
not exceeding five years.
Since in the given case, Arjun, being a citizen of Sri Lanka, is prohibited to acquire
property in India, except on lease for 5 years. If Mr. Rohit proposes to lease Mr. Arjun
for a period of 20 years, can do so only on the prior permission of the RBI.
1.12 (i) In the given case study, Mr. Rohit purchased 3 flats in Bangalore in the name of Ms.
Sonali, Mr. Srinivas Rajan, and jointly with his brother Sunil.
Apartment purchased in the name of Ms. Sonali- The property has been purchased
by Rohit in the name of his daughter Ms. Sonali. Therefore, although the
consideration for the purchase has been made by Mr. Rohit, since this is covered by
the exemtion provided (since the property has been purchased in the name of his
child), it is not covered as a benami transaction.
Apartment purchased in the name of Mr. Srinivas Rajan- A benami transaction is
defined as a transaction where a property is transferred to or held by one person and
consideration is paid by some other person. In this case, the property is in the name
of Srinivas Rajan although the consideration is paid by Rohit. Therefore, this is a
benami transaction.
Apartment purchased jointly in the name of Rohit and his brother Sunil- A property
jointly held in the name of brother and they appear as joint owners. Hence, this is not
a benami transaction.

© The Institute of Chartered Accountants of India


12 FINAL (NEW) EXAMINATION: MAY 2019

First two flats were rented to a large multi-national bank. After 4 years, Mr, Srinivas
Rajan transferred the property back in the name of Mr. Rohit without any
consideration.
As per section 6 of the Prohibition of Benami Property Transactions Act, 1988, no
person, being a benamidar shall re-transfer the benami property held by him to the
beneficial owner or any other person acting on his behalf. In cases where any property
is re-transferred, then such a transaction of a property shall be deemed to be null and
void.
In the said above case transaction of transfer back of the apartment by Mr. Srinivas
Rajan to Mr. Rohit is void.
(ii) The Initiating Officer, who is probing the transactions relating to the INR 2 crores
received and spent by Mr. Rohit, seeks your advice on identifying the benami
properties/ transactions, the benamidars, the beneficial owner.
The following are the benami transactions and benamidars:
Transaction Benamidar
Receipt of INR 2 crores in the bank Good Phones
account of Good Phones
Transfer of INR 2 crores from the Mr. Rohit (person abetting the
bank account of Good Phones to Mr. transaction, Mr. Srinivas Rajan)
Rohit’s personal bank account
Acquisition of shares of Stay Shares of Stay Connected becomes
Connected using the benami money benemi property
Purchase of Jewellery as gift for Ms. The jewellery becomes benami property.
Anjali Writer Mrs. Anjali also becomes Benamidar.
The original beneficial owner is not identified- the person who should have been the
original recipient of the funds, may be the original beneficial owner, although these
funds are not being held by Rohit/ Anjali for their benefit.
(iii) According to section 2(16) of the Prohibition of Benami Property Transaction Act,
1988, fair market value", in relation to a property, means—
(1) the price that the property would ordinarily fetch on sale in the open market on
the date of the transaction; and
(2) where the price referred to in sub-clause (i) is not ascertainable, such price as
may be determined in accordance with such manner as may be prescribed in
Rule 3 of the Prohibition of Benami Property Transaction Rules, 2016.
As per the said Rule, the price of unquoted equity shares shall be the higher of-
(i) its cost of acquisition;
(ii) the fair market value of such equity shares determined, on the date of

© The Institute of Chartered Accountants of India


PAPER – 6D: ECONOMIC LAWS 13

transaction, by a merchant banker or an accountant as per the Discounted Cash


Flow method; and
(iii) the value, on the date of transaction, of such equity shares as determined by
the formula given in the Rules.
The value of (iii) above is determined as below:
Particulars Amount Value to be Remarks
(` In considered for
Crores) calculation
(` In Crores)
Immovable Property 5.00 8.00 Market value to be
considered
Other fixed assets 4.00 4.00 Book value net of
depreciation
Inventories 2.00 2.00 Book value
Receivables and Loans and 1.50 1.50 Book value
Advances
Deferred Advertisement Costs 0.50 0.00 Not to be considered
Advance tax paid 1.00 0.00 Not to be considered
Total Value of Assets 15.50
Shareholders’ funds -4.00 0.00 Share capital and
Reserves not to be
considered
Provision for taxation -0.50 0.00 Not to be considered
Loans from Banks -3.00 -3.00
Trade Payables -6.50 -5.50 Provision for
unascertained
liabilities not to be
considered
Contingent Liabilities -2.00 -0.50 Arrears of divided on
cumulative preference
shares to be
considered
Proposed dividend on equity -1.00 -0.00 Not to be considered
shares
Total Value of Liabilities -9.00
Fair Market Value (Asset- 6.50
Liabilities) *Paid up Equity

© The Institute of Chartered Accountants of India


14 FINAL (NEW) EXAMINATION: MAY 2019

Capital/ Paid up value of equity


shares
Value of equity shares 1.30
acquired i.e. 20% of total

In the said question, the cost of acquisition is assumed at ` 2.00 crores, the value,
on the date of transaction, of such equity shares as determined by the formula given
in the Rules is ` 1.30 crores and the market value based on discounted cash flow
method is given as ` 2 crores. Thus, the fair market value of the acquisition in M/s.
Stay Connected will be ` 2 crores being highest of above and ` 10 crores for the
company as a whole (i.e. 2.00/0.20).
(iv) As per section 24 of the Prohibition of Benami Property Transactions Act, 1988, where
the Initiating Officer, on the basis of material in his possession, has reason to believe
that any person is a benamidar in respect of a property, he may, issue a show cause
notice to the person.
Where the notice specifies any property as being held by a benamidar, a copy of the
notice shall also be issued to the beneficial owner if his identity is known.
Where the Initiating Officer is of the opinion that the person in possession of the
property held benami may alienate the property during the period specified in the
notice, he may, with the previous approval of the Approving Authority, by order in
writing, attach provisionally the property in the manner as prescribed in Rule 4 of the
Benami Transactions Prohibition Rules, 2016, for a period not exceeding ninety days
from the date of issue of notice.
The Initiating Officer, after making such inquires and calling for such reports or
evidence as he deems fit and taking into account all relevant materials, shall, within
a period of ninety days from the date of issue of notice -
(i) pass an order continuing the provisional attachment of the property with the prior
approval of the Approving Authority, till the passing of the order by the
Adjudicating Authority; or
(ii) revoke the provisional attachment of the property with the prior approval of the
Approving Authority;
where provisional attachment has not been made, pass an order provisionally
attaching the property with the prior approval of the Approving Authority, till the
passing of the order by the Adjudicating Authority; or decide not to attach the property
as specified in the notice, with the prior approval of the Approving Authority.
Where the Initiating Officer passes an order continuing the provisional attachment of
the property or passes an order provisionally attaching the property, he shall, within
fifteen days from the date of the attachment, draw up a statement of the case and
refer it to the Adjudicating Authority.

© The Institute of Chartered Accountants of India


PAPER – 6D: ECONOMIC LAWS 15

CASE STUDY 2
Winner Builders Private Limited ("Winner") is a premium real estate builder who specializes in
constructing mid-sized apartment complexes (20 - 40 apartments) in South India. Winner was
started in the year 2004 by Mr. Vijay Nair, Managing Director and has its head office in Kochi,
Kerala with branches in Trivandrum, Bengaluru, Chennai and Tirupati. Mr. Vijay Nair has been
in the real estate business for more than 30 years and comes from a family of civil engineers
who are highly respected by their customers. Mr. Arun Nair, son of Mr. Vijay Nair, is a Chartered
Accountant and is the Chief Financial Officer of Winner. Mr. Vijay and Mr. Arun together own 60
of the share capital of Winner and the balance is held by a large private equity investor.
Although the company is a private limited company, the affairs of the company are handled in
the most professional manner akin to a listed company and Mr. Arun ensures that the financial
statements are properly prepared and presented to the Board of Directors (Mr. Vijay, Mr. Arun
and a representative of the PE investor) on a quarterly basis. The financial performance of
Winner has been reasonable and being a conservative person, Mr. Vijay was never in the mind-
set of taking aggressive positions with regard to business. Over the last few months, the PE
investor has been pushing the company in making changes in the operational mechanism, sale
prices etc. to increase the profits of the company and ensure decent return on their investment.
Due to this, Mr. Vijay and Mr. Arun are under tremendous pressure to complete the ongoing
projects fast and start new projects immediately and increase the revenues / profits of the
Company.
In June 2018, Winner announced a new 80 apartment project in Kochi named as "Winner
Shikaram", an ultra-modem luxury apartment complex with a variety of amenities including
swimming pool, skating rink, basketball court, fully equipped club house with all amenities, etc.
As per RERA regulations the Company applied for registration of the project on 15 th June 2018.
On 20th June 2018, the Company announced the launch of the project and commenced a big
advertisement campaign in the TV media and also through release of promotion material
through social media. It also collaborated with a regional TV Channel and announced a free
home in "Winner Shikaram" for the first prize winner of a popular reality show. The property was
registered by RERA on 10th July 2018 after scrutiny of the information provided by the promoter.
Based on the past performance of the Winner group and the general image of Mr. Vijay Nair,
there was tremendous demand for the apartments in the project and all the apartments were
booked within 1 month from the date of launch (20 th June, 2018). The following were some of
the conditions mentioned in the agreement to sale entered into by Winner with its allottees:
1. Expected date of completion of construction -31st March, 2020.
2. Expected date of handover-31s t May, 2020, subject to a grace period of 4 months.
3. Booking Advance amount to be paid prior to entering into agreement to sale – 20% of
total cost of apartment
4. Open car parking cost-INR 200,000

© The Institute of Chartered Accountants of India


16 FINAL (NEW) EXAMINATION: MAY 2019

5. Any delay in payment of dues by the allottees will liable for interest on such delayed
payments.
6. Return of booking amount shall not be entertained for any reason whatsoever.
7. Winner Group shall be liable for any deficiency in quality of construction for a period
of 3 years from the date of handing over the apartments.
Winner Group collected a total amount of INR 80 crores from the allottees and deposited an
amount of INR 60 crores in an escrow account for exclusive use for construction of the complex.
Separately, an amount of INR 5 lakhs each was collected from the 80 allottees in cash,
aggregating to INR 400 lakhs towards interior work, modular kitchen, supplying fans and lights,
etc. This money was accounted as receipt in a separate company, M/s. Wonderful Interiors,
which was owned by Ms. Anusha Nair, daughter of Mr. Vijay Nair and Mr. Arun Nair.
Although the construction was proceeding apace, the Company encountered severe rock
formations under the ground in one section of the land area which was previously not known
and due to the same, the Company concluded that the swimming pool could not be constructed
as designed and the size of the same had to be reduced. Winner got in' touch with the allottees
and proposed that the reduction of the size of swimming pool will be compensated suitably by
Winner by providing a Jacuzzi and Spa inside the club house. This was accepted by majority
(45 of the 80) of the allottees and, accordingly, Winner proceeded with the construction based
on the amended plan.
Few of the allottees reached out to Mr. Vijay Nair and stated that the carpet area for their
apartments was lesser than the size stipulated in the sale agreement and therefore, wanted to
be compensated. Mr. Vijay Nair mentioned to them that the reduction in the area was on account
of the exterior walls appurtenant to their apartments and this is the case with all the apartments
and not specific to their homes alone.
Mr. Arun Nair attended one of the real estate conclaves held in Bangalore, in which he met one
Mr. Henry Stewart, who runs an interior designing warehouse in Dubai UAE and showed quite
a few exhibits to Arun. Arun was impressed by the designs and the prices quoted by Mr. Henry.
Mr. Henry was also amenable to receive funds in cash in India through an intermediary and then
provide the material to Arun from UAE. Based on the same, Arun arranged for making cash
payment to the extent of INR 200 lakhs (Out of the INR 400 lakhs received by M/s Wonderful
Interiors) to an intermediary in Delhi, and the material was received from Henry in a month.
During his visit to India, Henry noted that his UAE passport got expired and he did not realise
the same. Since he did wanted to leave India immediately, he got in touch with a travel agent,
Mr. Anil Kumar, who helped him get a forged passport, for which Mr. Henry paid INR 2 lakhs in
cash.
Out of the balance INR 200 lakhs cash available with Wonderful Interiors, Arun used cash
amounting to INR 25 lakhs to pay amounts to various intermediaries to facilitate timely and
smooth registration process of thie apartments of Winner Shikharam, which was paid by the

© The Institute of Chartered Accountants of India


PAPER – 6D: ECONOMIC LAWS 17

intermediaries to the officials of the Sub- Registrar. With Henry's help, Arun transferred the
balance amount of INR 175 lakhs to an intermediary in Delhi and invested the amount to
incorporate a shell company in the Cayman Islands. The funds were then transferred back by
the Shell Company to the bank account of Winner. For this purpose, Mr. Arun raised export
invoices in the books of Winner on the Shell Company for providing professional services
relating to real estate business. Based on these invoices, Winner claimed export incentives
under the relevant laws in India and received INR 20 lakhs as export incentive.
On 30th March 2019, a meeting was organised by the Company and all the allottees during which
Mr. Vijay Nair provided a status update on the project and stated that bulk of the construction
activities will be completed by the timeline mentioned in the sale agreement (31 st May, 2020)
and the apartments will be handed over by 31 st July, 2020 (i.e. within the grace period). The
common areas will be completed in parallel and handed over by 30 th September, 2020. The
slight delay in completion was on account of non-availability of quality labour and he wanted
only the best labour to work on the project to ensure that the home owners have a happy life
after hand over. He also mentioned that the labour rates increased by 15% after the sale
agreements were entered and the Company did not ask for increase in prices from the allottees
only for good will reasons. The allottees were unhappy with the delay but, accepted the same,
since there was no other choice.
As one of the shareholders of Wonderful Interiors, Ms. Anusha Nair decided to visit Dubai to
see the interior designs and then place an order for the upcoming projects. During her visit, she
purchased 500 grams worth of gold (costing INR 15 lakhs) and since, she did not have enough
money, she asked Mr. Arun Nair to make the payment through the intermediary in Delhi. Based
on the discussion with the intermediary, Mr. Arun Nair provided an antique painting which he
got from one of his social friends to the intermediary as consideration for the gold purchased by
Ms. Anusha Nair in Dubai. Based on the same, Ms. Anusha brought the gold with her through
the green channel.
One of the employees of Wonderful Interiors, noting the substantial amount of cash transactions,
informs the Bank regarding the same, which in turn informs the enforcement directorate. The
ED has issued a show cause notice to all the parties regarding the above transactions.
Answer the following questions:
2.1 RERA authorities sent a notice to Winner that their advertisement campaign was not
in accordance with the RERA 2016. Evaluate.
(A) Valid, Since Winner decided to use Social media platform for promotion, without
obtaining specific approval from RERA.
(B) Valid, Since Winner collaborated with a TV channel to give a free home in Winner
Shikaram when the construction itself was not complete.
(C) Valid, Since Winner launched the project and commenced marketing even before the
project received registration from RERA.

© The Institute of Chartered Accountants of India


18 FINAL (NEW) EXAMINATION: MAY 2019

(D) Not Valid, Since Winner applied for the registration prior to the launch of campaign
and the registration was ultimately received within the stipulated period. (2 Marks)
2.2 As per RERA, Winner is required to enable the formation of the association of allottees
of Winner Shikaram within-months.
(A) 3 months of the majority of the allottees having booked their apartment.
(B) 3 months of the receipt of occupancy certificate.
(C) 3 months of the majority of the allottees registering their apartments with the sub-
registrar.
(D) 3 months of all the allottees making the full payment for the apartments. (2 Marks)
2.3 After registering the apartments in the name of the allottees, Winner informed the
allottees that they need to pay the water and electricity charges to the concerned
departments for their apartments. Evaluate.
(A) The registration of the apartments denote that the allottees are now the legal owners
of the apartments and hence, need to bear the water and electricity charges.
(B) The promoter is liable for making payment for the water and electricity charges until
the physical possession is transferred to the allottees.
(C) This is dependent on the terms of the agreement of sale between Winner and the
allottees.
(D) This amount need to be paid equally by Winner and the allottees, since the
registration is completed and only transfer of physical possession is pending.
(2 Marks)
2.4 The time limit within which the allottees of winner Shikaram are required to take
physical possession of the apartment after issuance of occupancy certificate is:
(A) Three months
(B) One month
(C) Five months
(D) Two months (2 Marks)
2.5 As per provisions of RERA, collection of cash by Wonderful Interiors for interior work,
modular kitchen, supplying fans and lights, etc. :
(A) May be appropriate, since RERA does not specify the mode of collection.

© The Institute of Chartered Accountants of India


PAPER – 6D: ECONOMIC LAWS 19

(B) May not be appropriate, since collection should be done as per the stipulations of
RERA.
(C) May be appropriate, since provisions of RERA are not applicable.
(D) May not be appropriate, since Wonderful Interiors are not registered with RERA.
(2 Marks)
2.6 What are the three distinct stages of Money Laundering?
(A) Information, Interrogation, Indictment
(B) Placement, Layering, Integration
(C) Planning, Comingling, Profiting
(D) Monitoring, Adjudicating, Punishing (2 Marks)
2.7 Which of the following are not circumstances which need to be considered by the
Director of Enforcement for performing search of the offices of Winner and other parties
mentioned in the case study?
(A) Possession of any property related to crime
(B) Possession of any records relating to money laundering
(C) Possession of records relating to RERA compliance by Winner
(D) Possession of any proceeds of crime involved in money laundering (2 Marks)
2.8 Ms. Anusha Nair brought gold jewellery worth INR 15 lakhs from D ubai through the
green channel. Is this an offence under the PMLA 2002 ?
(A) Yes, because she came through the green channel and evaded duty of customs.
(B) No, whilst it is an offence, it is not actionable under the PMLA 2002.
(C) No, she did not pay any cash for the purchase.
(D) Yes, since purchase of gold from gulf countries requires specific consent as per the
agreement entered with foreign countries as per Section 56 of PMLA 2002. (2 Marks)
2.9 As per RERA 2016, what is the minimum amount that Winner was required to deposit
in the escrow account ?
(A) INR 50 crores
(B) INR 56 crores
(C) INR 54 crores

© The Institute of Chartered Accountants of India


20 FINAL (NEW) EXAMINATION: MAY 2019

(D) INR 58.8 crores (2 Marks)


2.10 Of the below, which of the practices are not common schemes of money laundering?
(A) Bribery and Corruption
(B) False declarations under Customs act
(C) Usage of false trade Marks/copyrights
(D) Possession of foreign currency over and above permitted limit (2 Marks)
2.11 Answer the following questions in the context of the prov isions relating to the Real
Estate (Regulation & Development) Act, 2016 (RERA 2016).
(i) Examine the appropriateness of the conditions mentioned in the agreement to sale,
in the context of the provisions of RERA 2016. (4 Marks)
(ii) What are the provisions in RERA 2016 relating to the changes in design of the
construction from the sanctioned plans ? Analyse if the changes made by Winner are
appropriate in this context. (4 Marks)
(iii) What would be your advice if the customers of Winner reach out to you for your views
with regard to the validity of the explanations provided by Mr. Vijay Nair on the
reduction of carpet area? (3 Marks)
(iv) Evaluate the statements made by Mr. Vijay Nair in the meeting with the allottees on
30th March 2019 regarding the delay and the increase in labour costs in the context
of provisions of RERA 2016. (4 Marks)
2.12 Examine / advice regarding the below questions relating to the Prevention of Money
Laundering Act, 2002 (PMLA 2002).
(i) As a leading consultant on PMLA matters, the enforcement directorate has sought
your advice on identifying :
(a) the offences (b) the parties involved and (c) the punishment for the offence of
money laundering. (8 Marks)
(ii) The Bank, in which Winner holds its bank account, has reached out to you to
understand their obligations for maintaining and reporting of transactions under the
PMLA 2002. Advise. (4 Marks)
(iii) When the Enforcement Directorate proposed to take action against Mr. Vijay Nair
under the PMLA 2002, Mr. Vijay Nair contended that he was not a party to any of the
alleged offences and he was managing the real estate business of Winner only.

© The Institute of Chartered Accountants of India


PAPER – 6D: ECONOMIC LAWS 21

Examine whether his statement is valid. What would be the position of t he nominee
director of the PE investor? (3 Marks)

Answer Case study 2


2.1 (C)
2.2 (A)
2.3 (B)
2.4 (D)
2.5 (C)
2.6 (B)
2.7 (C)
2.8 (B)
2.9 (B)
2.10 (C)
2.11 (i) 1. Expected date of completion of construction- 31st March, 2020 -This
condition is valid.
2. Expected date of handover- 31st May 2020, subject to a grace period of 4
months. -This condition is valid.
3. According to Section 13, a promoter shall not accept a sum more than ten
per cent of the cost of the apartment, plot, or building as the case may be,
as an advance payment or an application fee, from a person without first
entering into a written agreement for sale with such person and register the
said agreement for sale, under any law for the time being in force.
Hence, the condition in the agreement for sale for booking advance amount
to be paid prior to entering into agreement to sale @20% of total cost of
apartment is not valid.
4. Section 2(n) of RERA, 2016 defines ‘common areas’ to include ‘open
parking areas’, thus open parking areas cannot be sold to the allottees.
Hence, the condition in the agreement for sale for open car parking cost
` 2,00,000 is not valid.
5. As per section 19(7) of RERA, 2016, the allottee shall be liable to pay
interest, at such rate as may be prescribed, for any delay in payment
towards any amount or charges to be paid.
Hence, the condition about any delay in payment of dues by the allottees
will be liable for interest on such delayed payments, is valid.

© The Institute of Chartered Accountants of India


22 FINAL (NEW) EXAMINATION: MAY 2019

6. The allottee shall be entitled to claim the refund of amount paid along with
interest at such rate as may be prescribed and compensation in the manner
as provided under this Act, from the promoter, if the promoter fails to
comply or is unable to give possession of the apartment, plot or building,
as the case may be, in accordance with the terms of agreement for sale or
due to discontinuance of his business as a developer on account of
suspension or revocation of his registration under the provisions of this Act
or the rules or regulations made thereunder.
Hence, the condition for return of booking amount shall not be entertained
for any reason whatsoever is not valid.
7. The builder has to provide five-year warranty for any structural defects in
the building. They are liable to pay equal rate of interest in case of default
or delays as home buyers.
Hence, the condition that Winner Group shall be liable for any deficiency
in quality of construction for a period of 3 years from the date handing over
the apartments is not valid.
(ii) Adherence to sanctioned plans and project specifications by the promoter
(Section 14)
(1) The proposed project shall be developed and completed by the promoter
in accordance with the sanctioned plans, layout plans and specifications
as approved by the competent authorities.
(2) Notwithstanding anything contained in any law, contract or agreement,
after the sanctioned plans, layout plans and specifications and the nature
of the fixtures, fittings, amenities and common areas, of the apartment, plot
or building, as the case may be, as approved by the competent authority,
are disclosed or furnished to the person who agree to take one or more of
the said apartment, plot or building, as the case may be, the promoter shall
not make—
(i) any additions and alterations in the sanctioned plans, layout plans
and specifications and the nature of fixtures, fittings and amenities
described therein in respect of the apartment, plot or building, as the
case may be, which are agreed to be taken, without the previous
consent of that person.
Provided that the promoter may make such minor additions or
alterations as may be required by the allottee, or such minor changes
or alterations as may be necessary due to architectural and structural
reasons duly recommended and verified by an authorised Architect or
Engineer after proper declaration and intimation to the allottee.
Explanation.—For the purpose of this clause, "minor additions or

© The Institute of Chartered Accountants of India


PAPER – 6D: ECONOMIC LAWS 23

alterations" excludes structural change including an addition to the


area or change in height, or the removal of part of a building, or any
change to the structure, such as the construction or removal or cutting
into of any wall or a part of a wall, partition, column, beam, joist, floor
including a mezzanine floor or other support, or a change to or closing
of any required means of access ingress or egress or a change to the
fixtures or equipment, etc.
(ii) any other alterations or additions in the sanctioned plans, layout plans
and specifications of the buildings or the c ommon areas within the
project without the previous written consent of at least two-thirds of
the allottees, other than the promoter, who have agreed to take
apartments in such building.
Explanation.—For the purpose of this clause, the allottees,
irrespective of the number of apartments or plots, as the case may
be, booked by him or booked in the name of his family, or in the case
of other persons such as companies or firms or any association of
individuals, etc., by whatever name called, booked in its name or
booked in the name of its associated entities or related enterprises,
shall be considered as one allottee only.
(3) In case any structural defect or any other defect in workmanship, quality or
provision of services or any other obligations of the promoter as per the
agreement for sale relating to such development is brought to the notice of
the promoter within a period of five years by the allottee from the date of
handing over possession, it shall be the duty of the promoter to rectify such
defects without further charge, within thirty days, and in the event of
promoter's failure to rectify such defects within such time, the aggrieved
allottees shall be entitled to receive appropriate compensation in the
manner as provided under this Act.
In the instant case, the proposal of Winner for reduction of the size of
swimming pool and the same to be compensated by providing a Jacuzzi
and spa inside the club house was accepted by majority (45 of the 80) of
the allottees and accordingly, Winner proceeded with the construction
based on the amended plan.
According to the above provisions, the promoter shall not make any other
alterations or additions in the sanctioned plans, layout plans and
specifications of the buildings or the common areas within the project
without the previous written consent of at least two-thirds of the allottees,
other than the promoter, who have agreed to take apartments in such
building.
Hence, approval by majority (45 of 80) is not valid.

© The Institute of Chartered Accountants of India


24 FINAL (NEW) EXAMINATION: MAY 2019

(iii) As per section 2(k) of the Real Estate (Regulation & Development)Act, 2016
"carpet area" means the net usable floor area of an apartment, excluding the
area covered by the external walls, areas under services shafts, exclusive
balcony or verandah area and exclusive open terrace area, but includes the area
covered by the internal partition walls of the apartment.
Accordingly, Sale of property will be on carpet area, not super built area.
Therefore, the homebuyer will have to pay only for the carpet area, that is the
area within walls, and the builder cannot charge for the super built-up area.
Therefore, the explanations provided by Mr. Vijay Nair on the reduction of the
carpet area was invalid. So, home buyers/ customers are liable to pay only for
the carpet area, that is the area within walls.
(iv) As given in the question that on 30th March, 2019, meeting was organized by
the company with all the allottees. During the meeting, Mr. Vijay Nair provided
a status update on the project and of the construction activities to be completed
and the other information mentioned in the sale agreement. As per the Section
11 of Real Estate (Regulation & Development) Act, 2016, it is the duty of the
promoter, to alter a project plan, structural design and specifications of the plot,
apartment or a building, the promoter has to get the consent of minimum two-
third allottees (buyers) after the necessary disclosures.
Since in the given case no approval of 2/3rd of the allottees was taken w.r.t. to
delay and the increase in labour costs i.e., as to the updation of the status of the
said project. This act of Mr. Vijay Nair is not in compliance with the Law.
2.12 (i) (a) Offences: The term “scheduled offence" has been defined in clause (y) of sub-
section (1) of section 2. It means –
(a) the offences specified under Part A of the Schedule; or
(b) the offences specified under Part B of the Schedule if the total value
involved in such offences is one crore rupees or more; or
(c) The offences specified under Part C of the Schedule.
The Schedule to the Act gives a list of all the above offences. The Schedule is
divided into three parts- Part A, Part B and Part C, which are given in Annexure
to the Chapter.
(b) The parties involved: Clause (p) of sub section (1) of section 2 provides that
"money-laundering" has the meaning assigned to it in section 3. Moving to
section 3, it is observed that whosoever directly or indirectly attempts to indulge
or knowingly assists or knowingly is a party or is actually involved in any process
or activity connected with the proceeds of crime including its concealment,
possession, acquisition or use and projecting or claiming it as untainted property
shall be guilty of offence of money laundering.

© The Institute of Chartered Accountants of India


PAPER – 6D: ECONOMIC LAWS 25

(c) The punishment for the offence of money laundering: Section 4 provides for the
Punishment for Money-Laundering - Whoever commits the offence of money-
laundering shall be punishable with rigorous imprisonment for a term which shall
not be less than three years but which may extend to seven years and shall also
be liable to fine.
But where the proceeds of crime involved in money-laundering relate to any
offence specified under paragraph 2 of Part A of the Schedule (i.e. Offences
under the Narcotic Drugs and Psychotropic Substances Act, 1985), the
maximum punishment may extend to ten years instead of seven years.
(ii) Obligation of Banking Companies, Financial Institutions and Intermediaries
Reporting entity to maintain records
Section 12 provides for the obligation of Banking Companies, Financial Institutions
and Intermediaries.
1. Maintenance of records: According to sub-section (1), every reporting entity
shall –
(a) maintain a record of all transactions, including information relating to
transactions covered under clause (b), in such manner as to enable it to
reconstruct individual transactions;
(b) furnish to the Director within such time as may be prescribed, information
relating to such transactions, whether attempted or executed, the nature
and value of which may be prescribed;
(c) verify the identity of its clients in such manner and subject to such
conditions, as may be prescribed;
(d) identify the beneficial owner, if any, of such of its clients, as may be
prescribed;
(e) maintain record of documents evidencing identity of its clients and
beneficial owners as well as account files and business correspondence
relating to its clients.
2. Confidentiality: Every information maintained, furnished or verified, save as
otherwise provided under any law for the time being in force shall be kept
confidential.
3. Maintenance of records (for clause a): The records referred to in clause (a)
of sub-section (1) shall be maintained for a period of five years from the date of
transaction between a client and the reporting entity.
4. Maintenance of records (for clause e): The records referred to in clause (e)
of sub-section (1) shall be maintained for a period of five years after the business
relationship between a client and the reporting entity has ended or the account

© The Institute of Chartered Accountants of India


26 FINAL (NEW) EXAMINATION: MAY 2019

has been closed, whichever is later.


5. Exemption by the Central Government: The Central Government may, by
notification, exempt any reporting entity or class of reporting entities from any
obligation under this chapter.
Access to information [Section 12A]
1. The Director may call for from any reporting entity any of the records referred to
in sub-section (1) of section 12 and any additional information as he considers
necessary for the purposes of this Act.
2. Every reporting entity shall furnish to the Director such information as may be
required by him under sub-section (1) within such time and in such manner as
he may specify.
3. Save as otherwise provided under any law for the time being in force, every
information sought by the Director under sub-section (1), shall be kept
confidential.
(iii) Offences by companies [Section 70]
1. Where a person committing a contravention of any of the provisions of this Ac t
or of any rule, direction or order made thereunder is a company, every person
who, at the time the contravention was committed, was in charge of and was
responsible to the company, for the conduct of the business of the company as
well as the company, shall be deemed to be guilty of the contravention and shall
be liable to be proceeded against and punished accordingly.
Nothing contained in this sub-section shall render any such person liable to
punishment if he proves that the contravention took place without his knowledge
or that he exercised all due diligence to prevent such contravention.
2. Notwithstanding anything contained in sub-section (1), where a contravention of
any of the provisions of this Act or of any rule, direction or order made
thereunder has been committed by a company and it is proved that the
contravention has taken place with the consent or connivance of, or is
attributable to any neglect on the part of any director, manager, secretary or
other officer of any company, such director, manager, secretary or other officer
shall also be deemed to be guilty of the contravention and shall be liable to be
proceeded against and punished accordingly.
In the instant case, Mr. Vijay Nair contended that he was not a party to any of
the alleged offences and he was managing the real estate business of Winner
only.
His statement is not valid on the grounds of section 70 of the PMLA 2002.
The position of the nominee director of the PE investor would be same as of

© The Institute of Chartered Accountants of India


PAPER – 6D: ECONOMIC LAWS 27

Mr. Vijay Nair and he shall be deemed to be guilty of the contravention and shall
be liable to be proceeded against and punished accordingly.
CASE STUDY 3
(Part-A)
The appellant "M/s Transmission Corporation of Andhra Pradesh Limited" is a successor of
Andhra Pradesh State Electricity Board (for short, 'APSEB') and is in the activities relating to
transmission of electricity. It had awarded certain contracts to the respondent "M/s Equipment
Conductors & Cables Limited" herein for supply of goods and services. Some disputes arose
and the respondent initiated arbitration proceedings. As many as 82 claims were filed by the
respondent before Haryana Micro and Small Enterprises Facilitation Council (hereinafter
referred to as 'Arbitral Council'). These proceedings culminated into Award dated June 21 st,
2010. The Arbitral Council came to the conclusion that the claims made on the basis of Invoice
Nos. 1 - 57 were barred by law of limitation and, therefore, no amount could be awarded against
the said claims. In respect of Invoice Nos. 58-82, the award was passed in favour of the
respondent. Against the aforesaid award rejecting claims in respect of Invoice Nos. 1 - 57 as
time barred, the respondent herein filed an application under Section 34 of the Arbitration and
Conciliation Act before the Additional District Judge, Chandigarh. The Additional District Judge
passed the order dated August 28, 2014 in the said application thereby remanding the case
back to the Arbitral Council for fresh decision. Against this order, the appellant filed the appeal
before the High Court of Punjab and Haryana at Chandigarh. This appeal was allowed by the
High Court by its order dated January 29, 2016 thereby setting aside the direction of the
Additional District Judge remanding the matter to Arbitral Council for fresh consideration.
The respondent herein filed execution petition for execution of judgment dated January 29, 2016
passed by the High Court of Punjab and Haryana as well as the award dated June 21, 2010
passed by the Arbitral Council. In so far as award of Arbitral Council is concerned, as noted
above the respondent's claim pertaining to Invoice Nos. 58 - 82 was allowed and the execution
thereof was sought. The respondent, however, filed another execution petition seeking
execution of amount in respect of Invoice Nos. 1 - 57 also. This application was entertained and
both the petitions were directed to be dealt with simultaneously vide orders dated August 17,
2016. The High Court vide its order dated November 08,2016 allowed the said Revision Petition
holding that there was no award in respect of claim towards Invoice Nos. 1 - 57 and, therefore,
it was not permissible for the respondent to seek the execution. When the things rested at that,
the respondent approached the NCLT by means of a Company Petition under Section 9 of IBC,
2016 read with Rule 6 of Insolvency and Bankruptcy (AAA) Rules, 2016. In this petition, the
respondent stated that it had served demand notice dated October 14, 2017 upon the appellant
under the provisions of the IBC, thereby claiming the amount of ` 45,69,31,233/- which was not
paid by the appellant. As mentioned above, this petition was dismissed by the NCLT filed under
section 9 of IBC vide its order dated April 09, 2018 being non maintainable on account of
existence of a dispute between the parties and this assertion of the NCLT is based on the fact
that these very claims of the respondent were subject matter of arbitration and the award was
passed rejecting these claims as time barred. Against this order, the respondent has filed appeal

© The Institute of Chartered Accountants of India


28 FINAL (NEW) EXAMINATION: MAY 2019

before the NCLAT in which impugned orders dated September 04,2018 have been passed. The
Honourable NCLAT passed an order stating "Prima facie case has been made out by the
Appellant in view of the part decree awarded by the competent court under Section 34 of the
Arbitration and 'Conciliation Act, 1996. However, taking into consideration the fact that if appeal
is allowed and Corporate Insolvency Resolution Process is initiated against the Respondent -
"Transmission Corporation of Andhra Pradesh Ltd. ", the government undertaking may face
trouble. Therefore, by way of last chance we grant one opportunity to respondents to settle the
claim with the appellant, failing which this Appellate Tribunal may pass appropriate order on
merit."
This very order of the National Company Law Appellate Tribunal, (for short, 'NCLAT) dated
September 04, 2018 is the subject matter of challenge before the Honourable Supreme Court
by the appellant M/s Transmission Corporation of Andhra Pradesh Limited and prays that the
same be reversed as there exists a Dispute and the application under IBC cannot be accepted.
(Part-B)
One Shri Rajendra Singh (Informant') filed an information under Section 19(1)(a) of the
Competition Act, 2002 (the 'Act') against Ghaziabad Development Authority (‘OP' /'GDA')
alleging contravention of the provisions of Section 4 of the Act. As per the information, the
Informant is an allottee of a flat under the Adarsh Vihar residential housing scheme for the
Economically Weaker Sections (EWS) ('Scheme') being developed by the OP in Ghaziabad, U.
P. in 2008. It is informed that OP is constituted under Section 4 of the Urban Planning and
Development Act, 1973 of Uttar Pradesh and is, inter alia, engaged in the activity of
development and sale of real estate in Ghaziabad, U. P.
It is further stated that the OP had conducted a lottery draw for allotment of EWS flats under the
aforesaid scheme. On being successful in the said lottery draw, the Informant was allotted a flat
bearing no. A-1/222 and accordingly, an allotment letter dated 04.05.2009 was issued in favour
of the Informant mentioning the final price of the flat as ` 2,00,000 and other conditions relating
to payment plan, date of giving possession, penal interest in case of delay in the payment of the
balance amount etc. As per the condition of the scheme, the Informant paid
` 20,000 to the OP as registration amount constituting 10% of the total price of the said house.
It is averred by the Informant that on 27.11.2015, the OP issued a letter to all the allottees of
the aforesaid scheme asking them to pay ` 7,00,000 as sale price of the flats allotted to them
failing which their allotment would stand cancelled. It is alleged that the OP has arbitrarily
increased the sale price of the said flat to ` 7,00,000 from ` 2,00,000 which was mentioned in
the allotment letter dated 04.05.2009. As per the Informant, the OP has indulged in unfair and
arbitrary practices and has misused its dominant position in the market. It is further averred that
the OP has indulged in the said practice even after knowing that the allottees of the scheme
belong to the weaker sections of the society and they are not in a position to challenge the OP
for its unfair and arbitrary conduct. Further, it is stated that the allottees of the said scheme are
dependent upon the OP for the residential flats under the said scheme. The Informant has

© The Institute of Chartered Accountants of India


PAPER – 6D: ECONOMIC LAWS 29

averred that the alleged conduct of OP is in contravention of the provisions of Section 4(2)(a)
of the Act.
Based on the above submissions, the Informant has prayed before the Commission to initiate
an inquiry against the OP under the provisions of the Act, set aside the impugned letter dated
27.11.2015 of the OP demanding ` 7,00,000 for the aforementioned flat, and direct the OP to
deliver possession of the flat to the informant under the said scheme at the price of ` 2,00,000
per flat. Besides hearing the parties on 27.12.2016, the Commission has perused the
information available on record and the documents submitted by the OP. From the facts of the
case, it appears that the grievance of the Informant relates to the letter dated 27.11.2015 of the
OP demanding a higher price of ` 7,00,000 for a EWS flat allotted to the Informant under the
aforesaid scheme as compared with the price of ` 2,00,000 as declared in the scheme's initial
brochure and intimated to the Informant vide allotment letter dated 04.05.2009. It is the case of
the Informant that the OP has abused its dominant position by arbitrarily increasing the price of
the said flat in contravention of the provisions of Section 4 of the Act.
The Commission observes that GDA is established under Section 4 of the Urban Planning and
Development Act, 1973 of the State of Uttar Pradesh. It has a common seal with power to
acquire, hold or dispose of both movable and immovable properties. The Urban Planning and
Development Act, 1973 of Uttar Pradesh empower GDA to pursue activities for promoting and
securing development of Ghaziabad in a planned manner. GDA has the power to acquire, hold,
manage and dispose of land and other properties in Ghaziabad and to carry out building,
engineering, mining and other operations, etc. Further, GDA is, inter alia, engaged in the
activities of development and sale of buildings, flats, complexes etc. for residential, commercial,
institutional and other purposes and with regard to the relevant geographic market. The
Commission is of the view that the geographic area of Ghaziabad district of the State of Uttar
Pradesh exhibits homogeneous and distinct market conditions for the development and sale of
low cost residential flats under affordable housing schemes for EWS and can be distinguished
from the conditions of competition prevailing in other adjacent areas of Ghaziabad such as Delhi,
Noida, etc. It may be noted that a consumer intending to buy a low cost residential flat under
affordable housing scheme for EWS in Ghaziabad may not prefer to purchase the same in other
adjacent areas of Ghaziabad because of factors such as difference in regulatory authorities (and
hence, different rules, regulation and taxes), distance to locations frequently commuted,
regional or personal preferences, transport connectivity etc.
Simultaneously, the Competition Commission of India (CCI) received a complaint from the Tamil
Nadu State Government alleging that two companies, M/s Sun Limited, a company engaged in
the business of manufacturing solar panels, and M/s Shine Limited, a company engaged in the
sale, installation and maintenance of solar energy generation plants, have entered into an
informal agreement to limit or control the production, supply and marketing of the products to
ensure maximum price realisation. M/s Sun Limited sells its manufactured panels on an
exclusive basis to M/s Shine Limited, which is India's largest solar power generation company
supplying solar plants to more than 60% of the current market.

© The Institute of Chartered Accountants of India


30 FINAL (NEW) EXAMINATION: MAY 2019

Therefore, it is the case of the Tamil Nadu State Government that the agreement between M/s
Sun Limited and M/s Shine Limited is anti-competitive and has an adverse effect on competition
since the entities have abused their dominance in the market.
Answer the following questions:
3.1 Which of the following are not duties of the Competition Commission of India?
(A) To promote and sustain competition in markets in India.
(B) To protect the interests of consumers.
(C) To ensure freedom of trade carried on by Indian suppliers in global market.
(D) To eliminate practices having adverse effect on competition. (2 Marks)
3.2 Notwithstanding anything contained in sub-regulation (2), the Commission may, after
recording reasons, invalidate a notice filed under regulation 5 or regulation 8 of The
Competition Commission of India (Procedure in regard to the transaction of business
relating to combinations) Regulations, 2011 as amended when it comes to the knowledge
of the Commission that such notice is not valid as per sub-regulation (1) and, in that case,
the Secretary shall convey the decision of the Commission to the parties to the combination
within _____________.
(A) Seven days of such decision of the Commission.
(B) Fourteen days of such decision of the Commission.
(C) Seven working days of such decision of the Commission.
(D) Fourteen working days of such decision of the Commission. (2 Marks)
3.3 Operate independently of competitive forces prevailing in the relevant market is-------------
--- component.
(A) Abuse of Dominance
(B) Anti-Competition agreements
(C) Combinations Regulation
(D) Competition Advocacy (2 Marks)
3.4 Which of the following are not functions of Insolvency Professional Agencies (IPAs)?
(A) Monitoring, Inspecting and Investigating members.
(B) Recommending Insolvency Professionals to Committee of Creditors.
(C) Drafting detailed standards and code of conduct for insolvency professionals.

© The Institute of Chartered Accountants of India


PAPER – 6D: ECONOMIC LAWS 31

(D) Addressing grievances, hearing complaints and taking suitable action. (2 Marks)
3.5 Following are the liabilities of M/s A Limited, which is under insolvency process under IBC
2016.
(i) Loan from Bank - INR 100 crores.
(ii) Secured Debentures issued to M/s B Limited - INR 20 crores.
(iii) Trade Payable (10 creditors, including B Ltd., whose outstanding is ` 2 crores) - INR
14 crores.
(iv) Amounts payable to workmen - INR 4 crores. (2 Marks)
Calculate the voting share of M/s B Limited in the Committee of Creditors.
(A) 15.9420%
(B) 16.6667%
(C) 16.4179%
(D) 16.1290% (2 Marks)
3.6 The liquidation process relating to corporate debtors under IBC 2016 will not be initiated
under which of the following circumstances ?
(A) The Committee of Creditors do not approve a resolution plan within 180 days.
(B) The NCLT rejects the resolution plan submitted to it on technical grounds.
(C) The Committee of Creditors resolve to liquidate the debtor with a majority (> 50%).
(D) The debtor contravenes the agreed resolution plan and an affected person makes an
application to the NCLT to liquidate the debtor. (2 Marks)
3.7 The liquidator of M/s Wrongway has sought your help in prioritizing the claims against M/s
Wrongway, as per IBC 2016:
(1) Costs payable to liquidator and resolution professional.
(2) Property tax payable to Government of Goa.
(3) Salary payable to the Finance team for past 6 months.
(4) Amounts payable to M/s Dhara Bank towards secured loans, where security
was relinquished.
(5) Amounts payable to Holding company of M/s Wrong way for Royalty fees.
(A) (1), (2), (4), (3), (5)

© The Institute of Chartered Accountants of India


32 FINAL (NEW) EXAMINATION: MAY 2019

(B) (2), (1), (4), (5), (3)


(C) (1), (4), (2), (3), (5)
(D) None of the above (2 Marks)
3.8 Whether an operational creditor can assign or legally transfer any operational debt to a
financial creditor ?
(A) Yes. However, the transferee shall be considered as an operational creditor to such
extent of transfer.
(B) Yes but the transferee shall be considered as a financial creditor in relation to such
transfer.
(C) No. An operational creditor cannot assign or legally transfer any operational debt to
a financial creditor.
(D) No. An operational creditor can assign or legally transfer an operational debt only to
an operational creditor. (2 Marks)
3.9 Which of the following agenda items should be taken up in the first meeting of committee
of creditors (COC) ___________.
(A) Appointment of interim resolution professional as insolvency professional or
replacement of the interim resolution professional by another resolution professional
(B) Preparation of draft resolution plan.
(C) Discussion of the status of the corporate debtor as on the present date and the road
map ahead.
(D) Collection of information on corporate debtor from independent sources (2 Marks)
3.10 What is quorum in case of meeting of committee of creditors (CoC)?
(A) Members of the Committee representing at least 33% of the voting rights present
either in person or video conference or other audio visual means.
(B) Members of the Committee representing at least 50% of the voting rights are present
either in person or proxy.
(C) Members of the Committee representing at least 50% of the voting rights are present
either in person or video conference or other audio visual means or proxy.
(D) Members of the Committee representing at least 66% of the committee present in
person or proxy. (2 Marks)

© The Institute of Chartered Accountants of India


PAPER – 6D: ECONOMIC LAWS 33

3.11 What is to be construed as a "Dispute" under the Insolvency & Bankruptcy Code, 2016 ?
State its significance for the maintainability of an application filed under section 9 of the
Code.
In the given case study whether the appellant M/s Transmission Corporation of Andhra
Pradesh Limited will succeed in its appeal ? Decide. (11 Marks)
3.12 (i) Examine the provisions of the Competition Act, 2002:
(a) Decide whether the agreement between Sun Limited and Shine Limited is
covered under the scope of the Act with reasons. Also, clarify the nature of the
agreement based on facts provided. (4 Marks)
(b) What factors shall the CCI consider while evaluating the views of the
Government of Tamil Nadu? (6 Marks)
(c) What orders can the CCI pass on completion of the inquiry? (4 Marks)
(ii) In the given case study, decide with reasons whether Rajendra Singh (Informant) will
succeed against the Opposite Party (OP) for alleged violation of Section 4(2)(a) of
the Competition Act, 2002 ? (6 Marks)
Answer Case Study 3
3.1 (C)
3.2 (C) Note: Prior to Amendment in the combination Regulation, it was Option A.
3.3 (A)
3.4 (B)
3.5 (B)
3.6 (C)
3.7 (D)
3.8 (A)
3.9 (A)
3.10 (A)
3.11 Meaning of dispute: As per section 5(6) of the Insolvency and Bankruptcy Code, 2016 the
word “Dispute” includes a suit or arbitration proceedings relating to—
(a) the existence of the amount of debt;
(b) the quality of goods or service; or
(c) the breach of a representation or warranty;
Significance of “dispute” for filing of an application by operational creditor under
section 9 of the IBC: If there is any dispute about debt, the corporate debtor is required to

© The Institute of Chartered Accountants of India


34 FINAL (NEW) EXAMINATION: MAY 2019

reply within ten days of receipt of copy of invoice, existence of a dispute, or record of the
pendency of the suit or arbitration proceedings filed before the receipt of such notice or invoice
in relation to such dispute [section 8(2)(a) of Insolvency & Bankruptcy Code, 2016].
After the expiry of the period of ten days from the date of delivery of the notice or invoice
demanding payment, if the operational creditor does not receive payment from the
corporate debtor or notice of the dispute, the operational creditor may file an application
before the Adjudicating Authority for initiating a corporate insolvency resolution process.
The operational creditor shall, along with the application furnish the relevant documents,
containing an affidavit to the effect that there is no notice given by the c orporate debtor
relating to a dispute of the unpaid operational debt. The Adjudicating Authority shall, within
fourteen days of the receipt of the application, by an order admit the application and
communicate such decision to the operational creditor and the corporate debtor if no notice
of dispute has been received by the operational creditor or there is no record of dispute in
the information utility.
In the given case study, appellant M/s Transmission Corporation, filed an appeal against
the order of NCLAT before the Supreme court on the ground of existence of a dispute, so
the application under IBC cannot be accepted and so order passed in the favour of
respondent (M/s Equipment Conductors & Cables Ltd.) to be revered.
As per the facts given in the case study, respondent filed petition seeking execution of
amount in respect of Invoice Nos. 1-57. Vide order dated Nov, 8, 2016 high court held that
this revision petition holding that there was no award in respect of claim towards Invoice
Nos. 1- 57 and therefore, it was not permissible for the respondent to seek the execution.
Against this order, respondent approached NCLT on the ground that it has served demand
notice dated October 2014, 2017. This application was dismissed by the NCLT Vide Order
April 9, 2018. Against this order of NCLT, respondent filed appeal before NCLAT. NCLAT
challenged the orders and passed an order dated 4 th September 2018.
Cause of action arised when, high court rejected on the execution of the petition which was
holding that there was no award in respect of claim towards Invoice Nos. 1- 57, was
passed, which means that still the dispute is pending. Against this order, respondent,
served demand notice dated October 14, 2017.
As per the Code, if there is dispute about claim of debt between parties prior to issue of
demand notice by operational creditor, application cannot be admitted. On the basis of this
ground, Appellant challenged the subject matter of the order passed by NCLAT dated 4th
September 2018 before Supreme Court.
In the judicial pronouncement, it was held that, application by operational creditor to initiate
insolvency process was accepted when it was found that there was no existing dispute
prior to date of demand notice and dispute raised after receipt of demand notice was not
genuine [Badjate Stock v. Snowblue Trexim (2018) 145 SCL 441 = 89 taxmann.com 64
(NCLT)].

© The Institute of Chartered Accountants of India


PAPER – 6D: ECONOMIC LAWS 35

Also, If appeal has been filed under section 34 of Arbitration Act, the proceedings are
pending as appeal is continuation of the adjudication proceedings. Hence, application for
insolvency resolution is not maintainable. [CG Power & Industrial Solutions Ltd. v. ACC
Ltd. (2018) 91 taxmann.com 363 (NCLT)].
Therefore, in the light of the given facts and circumstances, Appellant M/s Transmission
Corporation of Andhra Pradesh Limited will succeed in its appeal.
3.12 (i) (a) Anti-Competitive Agreements [Section 3]:
It shall not be lawful for any enterprise or association of enterprises or person
or association of persons to 'enter' into an agreement in respect of production,
supply, storage, distribution, acquisition or control of goods or provision of
services, which causes or is likely to cause an appreciable adverse effect on
competition within India. All such agreements entered into in contravention of
the aforesaid prohibition shall be void.
Any agreement entered into between enterprises or associations of enterprises
or persons or associations of persons or between any person and enterprise or
practice carried on, or decision taken by, any association of enterprises or
association of persons, including cartels, engaged in identical or similar trade of
goods or provision of services, shall be presumed to have an appreciable
adverse effect on competition, which—
(a) directly or indirectly determines purchase or sale prices;
(b) limits or controls production, supply, markets, technical development,
investment or provision of services;
(c) shares the market or source of production or provision of services by way
of allocation of geographical area of market, or type of goods or services,
or number of customers in the market or any other similar way;
(d) directly or indirectly results in bid rigging or collusive bidding.
In the instant case, M/s Sun Limited and M/s Shine Limited have entered into an
informal agreement to limit or control the production, supply and marketing of
the products to ensure maximum price realization. M/s Sun Ltd. sells its
manufactured panels on an exclusive basis to M/s Shine Ltd. which is India’s
largest solar power generation company supplying solar plants to more than
60% of the current market.
The above agreement is covered under the scope of the Competition Act, 2002
as it is an Anti-competitive agreement under section 3.
(b) Dominant position of enterprise: The Commission shall, while inquiring
whether an enterprise enjoys a dominant position or not, have due regard to all
or any of the following factors, namely:—
(a) market share of the enterprise;

© The Institute of Chartered Accountants of India


36 FINAL (NEW) EXAMINATION: MAY 2019

(b) size and resources of the enterprise;


(c) size and importance of the competitors;
(d) economic power of the enterprise including commercial advantages over
competitors;
(e) vertical integration of the enterprises or sale or service network of such
enterprises;
(f) dependence of consumers on the enterprise;
(g) monopoly or dominant position whether acquired as a result of any statute
or by virtue of being a Government company or a public sector undertaking
or otherwise;
(h) entry barriers including barriers such as regulatory barriers, financial risk, high
capital cost of entry, marketing entry barriers, technical entry barriers,
economies of scale, high cost of substitutable goods or service for consumers;
(i) countervailing buying power;
(j) market structure and size of market;
(k) social obligations and social costs;
(l) relative advantage, by way of the contribution to the economic
development, by the enterprise enjoying a dominant position having or
likely to have an appreciable adverse effect on competition;
(m) any other factor which the Commission may consider relevant for the inquiry.
(c) Orders by Commission after inquiry into agreements or abuse of domina nt
position [Section 27]
Where after inquiry the Commission finds that any agreement referred to in
section 3 or action of an enterprise in a dominant position, is in contravention of
section 3 or section 4, as the case may be, it may pass all or any of the following
orders, namely:—
(a) direct any enterprise or association of enterprises or person or association
of persons, as the case may be, involved in such agreement, or abuse of
dominant position, to discontinue and not to re-enter such agreement or
discontinue such abuse of dominant position, as the case may be;
(b) impose such penalty, as it may deem fit which shall be not more than ten
per cent. of the average of the turnover for the last three preceding financial
years, upon each of such person or enterprises which are parties to such
agreements or abuse.
In case any agreement referred to in section 3 has been entered into by a
cartel, the Commission may impose upon each producer, seller, distributor,
trader or service provider included in that cartel, a penalty of up to three

© The Institute of Chartered Accountants of India


PAPER – 6D: ECONOMIC LAWS 37

times of its profit for each year of the continuance of such agreement or
ten per cent. of its turnover for each year of the continuance of such
agreement, whichever is higher.
(c) Omitted
(d) direct that the agreements shall stand modified to the extent and in the
manner as may be specified in the order by the Commission;
(e) direct the enterprises concerned to abide by such other orders as the
Commission may pass and comply with the directions, including payment
of costs, if any:
(f) Omitted
(g) pass such other order or issue such directions as it may deem fit.
While passing orders under this section, if the Commission comes to a finding,
that an enterprise in contravention to section 3 or section 4 of the Act is a
member of a group as defined in clause (b) of the Explanation to section 5 of
the Act, and other members of such a group are also responsible for, or have
contributed to, such a contravention, then it may pass orders, under this section,
against such members of the group.
(ii) Section 4(2)(a) of the Competition Act, 2002 says there shall be an abuse of
dominant position under sub-section (1), if an enterprise or a group,—
(a) directly or indirectly, imposes unfair or discriminatory—
(i) condition in purchase or sale of goods or services; or
(ii) price in purchase or sale (including predatory price) of goods or service; or
Explanation.—For the purposes of this clause, the unfair or discriminatory
condition in purchase or sale of goods or services referred to in sub-clause (i)
and unfair or discriminatory price in purchase or sale of goods (including
predatory price) or service referred to in sub-clause (ii) shall not include such
discriminatory conditions or prices which may be adopted to meet the
competition.
As Opposite Party (OP) has increased the price of the flat allotted to Rajendra
Singh from ` 2,00,000 to ` 7,00,000. OP has been indulged in unfair and
arbitrary practices and has misused its dominant position in the market.
Hence, OP has violated section 4(2)(a) of the Competition Act, 2002.

© The Institute of Chartered Accountants of India


DISCLAIMER

This Suggested Answers do not constitute the basis for evaluation of the student’s answers in

the examination. The answers are prepared by the Faculty of the Board of Studies with a view

to assist the students in their education. While due care is taken in preparation of the answers,

if any errors or omissions are noticed, the same may be brought to the attention of the Director

of Studies. The Council of the Institute is not in anyway responsible for the correctness or

otherwise of the answers published herein.

Further, in the Elective Papers which are Case Study based, the solutions have been worked

out on the basis of certain assumptions/views derived from the facts given in the question or

language used in the question. It may be possible to work out the solution to the case studies

in a different manner based on the assumption made or view taken.

© The Institute of Chartered Accountants of India


2 FINAL (NEW) EXAMINATION: NOVEMBER, 2018

Case Study 1
Mr. Sharp was appointed as whole time member of the Competition Commission of India in
2015 and is presently a whole time member. Before joining Competition Commission of India,
during 2000-2014, he was acting as a trustee of several charitable trusts. For his contribution
towards the society, he was awarded several times by the State Government. Prior to that
during 1995-1999, he was acting as a managing director of 'Poor' Ltd., a Public Limited
Company engaged in the commercial real estate and was overall in-charge of finance and
sales function. During 2005, based on complaints filed by foreign investors, investigation into
the affairs of the company was initiated by the SFIO and CBI. The Report of CBI and SFIO
issued in October 2017 has revealed that affairs of the company were not managed in the
interest of the company during 1998-2005 and has resulted into financial loss of ` 400 Cr to
the shareholders and the Government. Considering the outcome of the report, Central
Government has issued an order of expulsion of Mr. Sharp from the post of whole time
member of the Competition Commission of India with immediate effect. He is in double mind to
challenge the order keeping in view the grounds for disqualification and the fact that
enactment of the Competition Act, 2002 is to provide an establishment of a Commission with
certain objectives or join back 'Poor' Ltd. 'Poor' Ltd. has been a party to a proceeding before
the Commission on the following issues:
(a) Whether a person who is purchasing goods for resale can also be considered as a
consumer?
(b) Whether all agreements which causes or is likely to cause an appreciable adverse effect
on competition in India, entered into in contravention shall be void?
(c) Whether the Commission also has powers to enquire into the acts taking place outside
India?
Poor', Ltd., now is in default in repayment of mainly on account of the General slowdown in
construction activities resulting in low capacity utilization and inadequate cash generation for
timely repayment of dues to all concerned. Repeated follow-up by the Financial Institutions
with the corporate debtor, 'Poor' Ltd., for submitting its specific plan of action for repayment of
dues did not evoke any meaningful response. Therefore, after a joint lenders' meeting, all the
financial creditors unanimously decided to apply under the provisions of the Insolvency and
Bankruptcy Code, 2016 to the National Company Law Tribunal (NCLT) for starting the process
of insolvency resolution in respect of corporate debtor, 'Poor' Ltd'. Financial Creditors filed an
application before NCLT which was admitted by NCLT on 20th May, 2018 and orders issued
for commencement of a moratorium period of 180 days, appointment of Mr. Ram, an Interim
Resolution Professional and for his making a public announcement inviting claims from all
concerned. With the advent of the public announcement the following creditors were identified:
(1) Financial debts owed to unsecured creditors (D1)-` 10 crores.

© The Institute of Chartered Accountants of India


PAPER – 6D: ECONOMIC LAWS 3

(2) Workmen's dues for the period of 24 months preceding the liquidation commencement
date (D2)-` 30 crores.
(3) Debts owed to a secured creditor who has relinquished his security (D3)-` 60 crores.
(4) Debts owed to the Central Government (D4)-34 Crores.
(5) Debts owed to a secured creditor for an amount unpaid following the enforcement of
security interest (D5)-` 52 Crores.
Mr. Ram who has been appointed as Interim Resolution Professional wants to know the
functional responsibilities of Insolvency Professional Agency (IPA).
Mr. Ram, in the last financial year, has given some legal opinions on financial matters to 'Poor'
Ltd. and has charged fees.
Smart was the statutory auditor of the corporate debtor. Mr. Sharp is the whole time member
of the Competition Commission of India and has been identified as a relative of Mr. Dull,
present Managing Director of 'Poor' Ltd. Mr. Dull is not dear on the provisions of Insolvency
and Bankruptcy Code, 2016 (IBC) and requested Company Secretary to advise him on the
vital objectives which are intended to be achieved with the Code and also whether the
initiation of insolvency resolution process can be done by creditors only or by debtor also.
Mr. Dull also wants to know the specified procedure and term of appointment of an IRP. In
case, 'Poor' Ltd. approach NCLT before the financial creditors and decide to appoint Mr. Ram
as Interim Resolution Professional, advise Mr. Ram on the consent to be provided by him as
required by regulations.
Answer the following questions:
(1.1) Mr. Ram who has been appointed as the resolution professional can take the following
actions without the approval of the Committee of Creditors:
(A) Undertake transactions with Mr. Sharp.
(B) Make changes in the appointment of Smart, the statutory auditor.
(C) File applications for avoidance of preferential or undervalued transactions.
(D) Record any change in the ownership interest of 'Poor' Ltd. (2 Marks)
(1.2) The Adjudicating Authority has by an order declared moratorium period on the 'Poor' Ltd.
Vide the moratorium order, the following shall not be prohibited:
(A) the action to foreclose security interest created by the corporate debtor in respect of
its property.
(B) the institution of arbitration proceedings.

© The Institute of Chartered Accountants of India


4 FINAL (NEW) EXAMINATION: NOVEMBER, 2018

(C) the recovery of any property by an owner or lessor where such property is occupied
by or in the possession of the corporate debtor.
(D) The supply of raw material essential for construction of commercial real estate from
its suppliers. (2 Marks)
(1.3) The NCLT rejected the resolution plan for want of compliance with the Insolvency and
Bankruptcy Code, accordingly the proceeds from the sale of liquidation shall be
distributed in the following order of priority :
(A) D2- D1-D3-D4 & D5 (ranked equally).
(B) D2 & D5 (ranked equally)-D3-D1-D4.
(C) D2 & D5 (ranked equally)-D1-D3-D4.
(D) D3 & D2 (ranked equally)-D1-D5 & D4 (ranked equally). (2 Marks)
(1.4) The NCLT rejected the resolution plan for want of compliance with the Insolvency and
Bankruptcy Code and proceeded to initiate liquidation proceedings. During the course of
liquidation, it was found that 'Poor' Ltd. had gifted some valuable assets of the Company
to another friendly company Soft Ltd. on 20th April, 2016 and D1 (unsecured financial
creditors) reported the transaction to the National Company Law Tribunal by way of an
application. The National Company Law Tribunal may pass an order:
(A) Rejecting the application of D1.
(B) Requiring the Insolvency and Bankruptcy Board to initiate disciplinary proceedings
against the liquidator.
(C) Require any person to pay sums in respect of benefits received by such person to
the liquidator.
(D) Require any person to submit relevant documents of transaction to Insolvency and
Bankruptcy board. (2 Marks)
(1.5) The management of the affairs of 'Poor' Ltd., the corporate debtor undergoing corporate
insolvency resolution process vests in the _______.
(A) Mr. Ram, Interim Resolution Professional
(B) Board of Directors
(C) Committee of Creditors
(D) Insolvency and Bankruptcy Board of India (2 Marks)
(1.6) Mr. Sharp cannot be removed from the Competition Commission of India by the Central
Government, if he:

© The Institute of Chartered Accountants of India


PAPER – 6D: ECONOMIC LAWS 5

(A) has engaged at any time, in any paid employment.


(B) has become physically or mentally incapable of acting as a member.
(C) has been convicted of an offence which, in the opinion of the Central Government,
involves moral turpitude.
(D) is, or at any time has been, adjudged as an insolvent. (2 Marks)
(1.7) Mr. Sharp shall not for a period ________of years from the date on which cease to hold
office in the Competition Commission of India, accept any employment in, or be
connected with the management of administration of, any enterprise which has been a
party to a proceeding before the Competition Commission of India.
(A) 1
(B) 2
(C) 1
(D) None of the above (2 Marks)
(1.8) ‘Agreement’ under the Competition Act, 2002 includes any arrangement or understanding
or action in concert:
(A) if it is in writing only.
(B) if it is enforceable by legal proceedings only.
(C) if it is in writing and enforceable by legal proceedings only.
(D) If it is whether or not, in formal or writing or whether or not enforceable by legal
proceedings. (2 Marks)
(1.9) Any agreement under the Competition Act, 2002 shall be presumed to have an
appreciable adverse effect on competition, which:
(A) directly or indirectly determines purchase or sale prices.
(B) limits or controls production, supply, markets, technical development, investment or
provision of services.
(C) directly or indirectly results in bid rigging or collusive bidding.
(D) All of the above (2 Marks)
(1.10) If Central Government issues expulsion order to Mr. Sharp, the order:
(A) is valid from the date of his joining the Commission.
(B) is not valid but cannot be challenged.

© The Institute of Chartered Accountants of India


6 FINAL (NEW) EXAMINATION: NOVEMBER, 2018

(C) is not valid and can be challenged.


(D) is valid on the basis of outcome of the report from the date of receipt of the order. (2 Marks)
(1.11) Answer the following based on the facts given in the question with reference to the
provisions of the Insolvency and Bankruptcy Code, 2016 (Code):
(A) Advise Mr. Dull on vital objectives which are intended to be achieved with the Code.
(2 Marks)
(B) Advise Mr. Ram on the functional responsibilities of Insolvency Professional
Agencies (IPA). (2 Marks)
(C) Advise Mr. Ram on the independence with the Corporate Debtor. (2 Marks)
(D) Advise 'Poor' Ltd. whether the initiation of insolvency resolution process can be
done by creditor only or by corporate debtor also. (2 Marks)
(E) Interim Resolution Professional is to be appointed by following the specified
procedure and for a specific term. Examine and advise Mr. Ram on the consent to
be provided by him in Form 2 as required by the relevant rules. (7 Marks)
(1.12) Answer the following under the provisions of the Competition Act, 2002:
(A) Enactment of the Competition Act, 2002 is to provide for an establishment of a
Commission with objectives. Advise Mr. Sharp for taking decision whether to
challenge the order of the Supreme Court? (3 Marks)
(B) A person who is purchasing goods for resale can also be considered as a
'Consumer'. Examine and advise 'Poor' Ltd. (3 Marks)
(C) All agreements which causes or is likely to cause an appreciable adverse effect on
competition in India, entered into in contravention shall be void. Examine and advise
'Poor' Ltd. (5 Marks)
(D) Advise 'Poor' Ltd. whether the Commission also has power to enquire into the acts
taking place outside India. (4 Marks)
Answer to Case Study 1
(1.1) Option (C): File applications for avoidance of preferential or undervalued transactions
(1.2) Option (D): The supply of raw material essential for construction of commercial real
estate from its suppliers.
(1.3) Option (D): D3 & D2 (ranked equally)-D1-D5 & D4 (ranked equally)
(1.4) Option (A): Rejecting the application of D1.
(1.5) Option (A): Mr. Ram, Interim Resolution Professional

© The Institute of Chartered Accountants of India


PAPER – 6D: ECONOMIC LAWS 7

(1.6) Option (A) : has engaged at any time, in any paid employment.
(1.7) Option (B) : 2
(1.8) Option (D) : if it is whether or not, in formal or writing or whether or not enforceable by
legal proceedings
(1.9) Option (D) : All of the above
(1.10) Option (C): Is not valid and can be challenged.
(1.11) (A) Vital objectives which are intended to be achieved with the IBC: The Insolvency
and Bankruptcy Code, 2016 is intended to strike the right balance of interests of all
stakeholders of the business enterprise so that the corporates and other business
entities enjoy availability of credit and at the same time the creditor do not have to
bear the losses on account of default. The purpose of enactment of the Insolvency
and Bankruptcy Code, 2016 is as follows:
(a) To consolidate and amend the laws relating to re-organization and insolvency
resolution of corporate persons, partnership firms and individuals.
(b) To fix time periods for execution of the law in a time bound manner.
(c) To maximize the value of assets of interested persons.
(d) To promote entrepreneurship
(e) To increase availability of credit.
(f) To balance the interests of all the stakeholders including alteration in the order
of priority of payment of Government dues.
(g) To establish an Insolvency and Bankruptcy Board of India as a regulatory body
for insolvency and bankruptcy law.
(B) Functional responsibilities of Insolvency Professional Agencies (IPA):
It will perform three key functions:
(i) Regulatory functions
• drafting detailed standards and codes of conduct through bye-laws, that
are made public and are binding on all members
(ii) Executive functions
• monitoring, inspecting and investigating members on a regular basis
• gathering information on their performance, with the over-arching
objective of preventing frivolous behaviour, and
• malfeasance in the conduct of IP duties

© The Institute of Chartered Accountants of India


8 FINAL (NEW) EXAMINATION: NOVEMBER, 2018

(iii) Quasi-judicial functions


• addressing grievances of aggrieved parties, hearing complaints against
members and taking suitable actions
(C) Eligibility of an insolvency Professional to be appointed as a Resolution
Professional: As per Regulation 3 of the Insolvency and Bankruptcy (Insolvency
Resolution process for Corporate Persons) Regulation, 2016, an insolvency
professional shall be eligible for appointment as a resolution professional for a
corporate insolvency resolution process if he and all partners and directors of the
insolvency professional entity of which he is partner or director are independent of
the corporate debtor:-
(a) He is eligible to be appointed as an independent director on the board of the
corporate debtor under section 149 of the Companies Act, 2013, where the
corporate debtor is a company.
(b) He is not a related party of the corporate debtor.
(c) He is not an employee or proprietor or a partner of a firm of auditors or
company secretaries in practice or cost auditors of the corporate debtor in the
last three financial years.
(d) He is not an employee or proprietor or a partner of a legal or consulting firm
that has or had any transaction with the corporate debtor amounting to ten per
cent or more of the gross turnover of such firm in the last three financial years.
(D) As per Section 6 of the IBC, 2016, where any corporate debtor commits a default, a
financial creditor, an operational creditor or the corporate debtor itself may initiate
corporate insolvency resolution process in respect of such corporate debtor in the
manner as provided under this Chapter (Chapter II of part II). Therefore, Insolvency
resolution process can be initiated by creditor as well as by the corporate debtor.
(E) Appointment of IRP: As per the Code, Adjudicating authority shall appoint an
Interim Resolution Professional within 14 days from the commencement date of the
Insolvency process. Section16 of the Code lays down the procedure for
appointment of an Interim Resolution Professional.
Where the application for corporate insolvency resolution process is made by a
financial creditor or the corporate debtor, the resolution professional as proposed in
the application shall be appointed as the interim resolution professional, if no
disciplinary proceedings are pending against him.
Where the application for corporate insolvency resolution process is made by an
operational creditor and
(a) No proposal for an interim resolution professional is made. The
Adjudicating Authority shall make a reference to the Board for the
recommendation of an insolvency professional who may act as an interim

© The Institute of Chartered Accountants of India


PAPER – 6D: ECONOMIC LAWS 9

resolution professional.
(b) A proposal for an interim resolution professional is made the proposed
resolution professional shall be appointed as the interim resolution
professional, if no disciplinary proceedings are pending against him.
The Board shall recommend the name of an insolvency professional to the
Adjudicating Authority against whom no disciplinary proceedings are pending, within
ten days of the receipt of a reference from the Adjudicating Authority.
Period of appointment of IRP: The term of Interim Resolution Professional shall
not exceed 30 days from the date of appointment. [Section 16]
As per Form 2, written consent by proposed IRP is given to the Adjudicating
authority under the relevant rule of the Insolvency and Bankruptcy (Application to
Adjudicating Authority) Rules, 2016.
(1.12)(A) The preamble of the Competition Act, 2002 provides that it is an Act to establish a
Commission to prevent anti-competitive practices, promote and sustain competition,
protect the interests of the consumers and ensure freedom of trade in markets in
India.
However Section 53T of the Competition Act, 2002, provides that the Central
Government or any State Government or the Commission or any statutory authority
or any local authority or any enterprise or any person aggrieved by any decision or
order of the Appellate Tribunal may file an appeal to the Supreme Court.
Since the Supreme Court(SC) as per the Indian constitution is the apex body, so the
decision of SC is binding on the Central Government / any State Government / the
Commission / any statutory authority / any local authority /any enterprise / any
person. So Mr. Sharp ‘s decision to challenge the order of the Supreme court is not
possible.
(B) The term ‘consumer’ is defined in section 2(f) of Competition Act, 2002. Accordingly,
‘consumer’ means any person who buys any goods for a consideration, which has
been paid or promised or partly paid and partly promised, whether such purchase of
goods is for resale or for any commercial purpose or for personal use.
Hence, it is not necessary that a person must purchase the goods for personal use
in order to be considered as a ‘consumer’ under Competition Act, 2002. Even a
person purchasing goods for resale or for any commercial purpose will also be
considered as a ‘consumer’ within the meaning of Section 2(f) of the Competition
Act, 2002.
(C) All agreements which causes or is likely to cause an appreciable adverse effect on
competition in India, entered into in contravention shall be void

© The Institute of Chartered Accountants of India


10 FINAL (NEW) EXAMINATION: NOVEMBER, 2018

It shall not be lawful for any enterprise or association of enterprises or person or


association of persons to 'enter' into an agreement in respect of production, supply,
storage, distribution, acquisition or control of goods or provision of services, which
causes or is likely to cause an appreciable adverse effect on competition within
India. All such agreements entered into in contravention of the aforesaid prohibition
shall be void.
Any agreement entered into between enterprises or associations of enterprises or
persons or associations of persons or between any person and enterprise or
practice carried on, or decision taken by, any association of enterprises or
association of persons, including cartels, engaged in identical or similar trade of
goods or provision of services, shall be presumed to have an appreciable adverse
effect on competition, which—
(a) directly or indirectly determines purchase or sale prices;
(b) limits or controls production, supply, markets, technical development,
investment or provision of services;
(c) shares the market or source of production or provision of services by way of
allocation of geographical area of market, or type of goods or services, or
number of customers in the market or any other similar way;
(d) directly or indirectly results in bid rigging or collusive bidding.
However, any agreement entered into by way of joint ventures, if such agreement
increases efficiency in production, supply, distribution, storage, acquisition or control
of goods or provision of services, shall not be considered to be an anti-competitive.
(D) Acts taking place outside India but having an effect on competition in In dia
(Section 32)
The Commission shall, notwithstanding that,—
(a) an agreement referred to in section 3 has been entered into outside India; or
(b) any party to such agreement is outside India; or
(c) any enterprise abusing the dominant position is outside India; or
(d) a combination has taken place outside India; or
(e) any party to combination is outside India; or
(f) any other matter or practice or action arising out of such agreement or
dominant position or combination is outside India;
have power to inquire in accordance with the provisions contained in sections 19,
20, 26, 29 and 30 the Act into such agreement or abuse of dominant position or

© The Institute of Chartered Accountants of India


PAPER – 6D: ECONOMIC LAWS 11

combination if such agreement or dominant position or combination has, or is likely


to have, an appreciable adverse effect on competition in the relevant market in India
and pass such orders as it may deem fit in accordance with the provisions of this
Act.
Case Study 2
Mr. Cute had given an application to the state authorities for purchase of land for farmin g and
agricultural use. This application was made by him through his company M/s Hip Hop Farms
Ltd. (HHFL). HHFL was initially incorporated in 2003 with two shareholders Mr. A and Mr. B.
Through an executed share transfer deed, shares of this company were transferred to
Mr. Cute and his wife Mrs. Pretty. Consequent to transfer of shares, first directors were also
replaced with new directors i.e. Mr. D, Mrs. E and Mr. Sharp. Mr. D and Mrs. E are parents of
Mrs. Pretty.
To enable HHFL to purchase the said piece of land, Mr. Cute, had given unsecured loan
amounting to ` 11 Crore to HHFL. Since the subject piece of land was an agricultural land,
during the time of representation, Mr. Sharp declared himself an agriculturist. Accordingly, the
additional collector allowed the purchase of the land on condition that it would b e used for
farming within two years.
During the year 2010, Ms. F (sister of Mrs. Pretty) was appointed as director of HHFL in place
of Mr. Sharp.
HHFL was preparing financial statements on a regular basis and was compliant in filing
various documents with the Registrar of Companies. Financial Statements for the year ended
on 31st March, 2018 and previous years did not show any income from farm activities or
agricultural activities. Instead, the said piece of land was developed by HHFL and constructed
a palatial bungalow with swimming pool and a dedicated space to facilitate landing and
parking fixed wing aircrafts.
Mr. Cute had separately obtained a loan for his personal use from a Non- Banking Finance
Company amounting to ` 65.Crore. The said loan was secured by the mortgage created on
the property owned by HHFL. Mr. Cute defaulted on payment of last few installments and tried
persuading bank to restructure the covenants of the loan agreement. Bank officials did not
agree to his request and decided to take action against him and the said mortgaged property.
This particular case came under the scanner of the authorities when the Collector of the
region claimed that this particular property along with other 110 properties have allegedly
flouted other applicable regulations prevailing in the State. Due to this matter, the case was
forwarded to the Income Tax department. Acting proactively on the matter, the Income Tax
department had issued an attachment notice under the Prohibition of Benami Property
Transaction Act, 1988 to HHFL for provisionally attaching the property and filed a report
before an adjudication authority to confirm the attachment.

© The Institute of Chartered Accountants of India


12 FINAL (NEW) EXAMINATION: NOVEMBER, 2018

Mr. Cute had engaged a lawyer to prepare a reply in response to the notice received. His
lawyer had advised that maximum penalty for contravention, if any would be 10% of the cost
of the property. Further, he has stated that in the worst case situation, attachment in no case
under the provisions of the Prevention of Benami Property Transactions Act, 1988 exceed 3
months.
Mr. Cute after knowing the provisions, had instructed the lawyer to furnish a fabricated reply in
response to the notice and include a point as to why notice has been issued to him. The said
notice should have been issued to HHFL only.
After the legal proceedings were completed, the order was passed by the adjudicating
authorities. After, perusing the order, Mr. Cute identified certain errors and misplaced facts,
and asked his lawyer to discuss the same with the authorities. However, his lawyer forgot the
matter due to other cases in hand. When he was reminded again after almost 11 months, he
responded that the matter is time barred.
Besides the said piece of land on which bungalow and swimming pool were constructed, there
were other seven pieces of non-agricultural land just adjacent to the land. Survey numbers of
the same were 112/1, 112/2, 112/3, 112/4, 112/5, 112/6 and 112/7.
Mr. D and his family were quite affluent and generally they were seen in lavish social
gatherings apart from managing their real estate development business. During a family
function in 2012, they made a fixed deposit amounting to ` 10 crore in the name of Mrs. Pretty
which was a gift for her.
Mrs. Pretty on maturity of the said deposit, transferred the amount in the name of Mr. Cute for
his personal use. During 2015, the said amount was used by Mr. Cute to buy a piece of land
bearing survey number 112/1 in the name of Mrs. Pretty. Owner as per the land records was
Mrs. Pretty and payment for the said land bearing survey number 112/1 was made by Mr.
Cute.
Mr. D was the owner of land bearing survey numbers 112/2, 112/3, 112/4, 112/5, 112/6 and
112/7. During the third quarter of financial year 2017-18, he developed and launched a new
residential-cum-commercial project on the said pieces of land after seeking registration under
the Real Estate (Regulation and Development) Act, 2016 (RERA). For the said project Ms. F
was acting as an authorized agent for marketing. When the commercial launch was organized,
it was announced by Ms. F that the project is available at an attractive rate of ` 8,800 per
square feet and the units are very spacious since they admeasure 1500 square feet built-up
with total 100 units.
Also, marketing brochure contains following features included in the project:
(1) Italian marble in the kitchen
(2) 5 Star rated Air Conditioners
(3) 3 Star rated Geysers

© The Institute of Chartered Accountants of India


PAPER – 6D: ECONOMIC LAWS 13

(4) French Windows of reputed brand


(5) Elevators of top brands
(6) Open parking slot at a nominal price of ` 11,000
(7) Massive multi-level kids play area
(8) Ducts attached to each flat
(9) Comprehensive insurance for the project
Marketing brochure mentioned that builder provides warranty of 5 years of the products with
additional free 1 year warranty.
It was informed in the marketing material that the project would be completed in a time frame
of 5 years. One of the allottee complained about Ms. F for project's registration to which she
replied that project is already registered and since she is daughter of the promoter, she is not
required to take the separate registration, only outsiders are required to take registration
under RERA. Ms. F receives facilitation fees from the company owning the project.
Mr. Bhakt was one of the allottee who bought flat number 205 in Tower 1 of the project after
several rounds of meeting with Ms. F. It was told to him that a Ganesh Temple would be
constructed as a part of the project in the eastern side of Podium 2.
During the course of the project, an intimation along with a certificate from engineer was sent
to all the allotees that due to a technical objection received from fire department, temple will
have to be shifted from Podium 2 to Podium 3.
When this fact came in the knowledge of Mr. Bhakt, he consulted his lawyer who advised to
file a complaint against the builder with the authorities. Also, he mentioned in the complaint
that he bought flat through Ms. F who was not registered under RERA and reported several
defects in the features contained in the marketing brochure.
Just before the completion of the project, the promoter got an offer to sell the entire project to
an American builder at an attractive price. The acquirer informed the promoter that since it is
the deal between us and I have never defaulted on the delivery in projects in last 50 years
there is practically no use of seeking approval of allottees. There were several rounds of
discussions between the promoter and the acquirer; however, the deal did not go through due
to difference in valuation.
The project was completed on time and the invitation was sent to all the allottees to take
physical possession of their respective units. After staying for about 8 months in the flat
number 406 in Tower 4, Mr. Sultan informed builder that he is facing serious issues wi th the
quality of MCB provided and there is a potential risk of short circuit which could lead to
massive losses to the building as a whole. On investigation by an independent electrician
appointed by Mr. Sultan, it was found that lining of electricity wire was done along with water
pipe lines and due to internal damage, problem is arising. However, the promoter was harping

© The Institute of Chartered Accountants of India


14 FINAL (NEW) EXAMINATION: NOVEMBER, 2018

on the fact that the issue is in the MCB and not in the wirings. The investigation done by
electrician was confirmed by other electricians who surveyed a few other flats.
Further, Mr. Sultan complained that the grass given by the builder in the flower bed area was
of sub-standard quality and needs replacement.
Answer the following Questions:
(2.1) Who is Benamidar in the above case as per Prevention of Benami Property Transactions
Act, 1988?
(A) HHFL.
(B) Mr. D.
(C) Mr. E.
(D) All of the above (2 Marks)
(2.2) Whether is it a requirement under Prevention of Benami Property Transactions Act, 1988
that Benamidar shall be aware that property is registered in his / her name to categorize
a transaction as Benami ?
(A) Yes, it is necessary.
(B) No, it is not necessary
(C) Can't say
(D) None of the above (2 Marks)
(2.3) Under Prevention of Benami Property Transactions Act, 1988, property which has been
declared as Benami can be confiscated by which authority?
(A) The President of India.
(B) State Government.
(C) Central Government.
(D) None of the above (2 Marks)
(2.4) In a scenario where authorities conclude that the subject property is hit by the provisions
of the Prevention of Benami Property Transactions Act, 1988, what could be the quantum
of penalty?
(A) 25% of the cost of the property.
(B) 10% of the fair market value of the property.
(C) 10% of the cost of the property.

© The Institute of Chartered Accountants of India


PAPER – 6D: ECONOMIC LAWS 15

(D) 25% of the fair market value of the property. (2 Marks)


(2.5) Under Prevention of Benami Property Transactions Act, 1988, notice for initiating action
shall be submitted by following means?
(A) By Post.
(B) By way of summons.
(C) By e-mail.
(D) Either (A) or (B) (2 Marks)
(2.6) As per the provisions of RERA, which of following are treated as part of common area?
(A) Kids play area.
(B) Duct attached to the units.
(C) Balcony attached to the living room.
(D) All of the above (2 Marks)
(2.7) Under RERA, 20% of the flat cost cannot be accepted unless:
(A) Property is registered.
(B) Marketing brochure mentioned terms of payment.
(C) 20% project is completed.
(D) All of the above (2 Marks)
(2.8) Under RERA, provision related to 5 years warranty is applicable to following:
(A) Chipped beam in the kitchen.
(B) Loose tiles in the washrooms.
(C) Leakage in the internal pipe lines.
(D) All of the above (2 Marks)
(2.9) Under RERA, when all documents in connection with insurance shall be handed over by
the promoter to the allotees?
(A) On receipt of final payment / installment.
(B) On receipt of occupancy certificate.
(C) On receipt of NOC from fire department.
(D) On formation of society. (2 Marks)

© The Institute of Chartered Accountants of India


16 FINAL (NEW) EXAMINATION: NOVEMBER, 2018

(2.10) On completion of the project and after receipt of occupancy certificate, when can an
allottee take physical possession of the flat?
(A) Within two months.
(B) Within three months.
(C) Within six months.
(D) None of the above (2 Marks)
(2.11)Explain the following in light of the provisions of the Prevention of Benami Property
Transactions Act, 1988:
(A) Owner of the land as per land records shall make payment for the land standing in
his/her name. Examine the correctness of the statement. (3 Marks)
(B) Whether action proposed by the officials of the bank is defensible? Advise officials
of the bank. (3 Marks)
(C) Examine legal ramifications of the instructions made by Mr. Cute to his lawyer and
advice by his lawyer in the matter. (6 Marks)
(D) Mr. Cute has approached you after hearing response from his lawyer after 1 month.
Please advise him. (3 Marks)
(2.12)Explain the following in light of the Provisions of the Real Estate (Regulation and
Development) Act, 2016 (RERA):
(A) Mr. Bhakt has approached you to confirm advice given by his lawyer. Kindly assist
Mr. Bhakt on the points mentioned by the lawyer. (9 Marks)
(B) Promoter of the project has appointed you to advise on the issue raised by
Mr. Sultan. (4 Marks)
(C) Examine legal validity of the proposal given by the American builder. (3 Marks)

Answer to Case Study 2


(2.1) Option (A): HHFL
(2.2) Option (B): No, it i s not necessary.
(2.3) Option (C): Central Government
(2.4) Option (D): 25% of the fair market value of the property
(2.5) Option (D): Either (A) or (B)
(2.6) Option (D): All of the above

© The Institute of Chartered Accountants of India


PAPER – 6D: ECONOMIC LAWS 17

(2.7) Option (A): Property is registered


(2.8) Option (D): All of the above
(2.9) Option (D): On formation of society
(2.10) Option (A): Within two months
(2.11)(A) As per section 2(9) of the Prohibition of Benami Property Transactions Act, 1988, all
such type of transaction or an arrangement made in respect to a property, where -
• suc h a property is transferred to or held by one person and consideration is
paid by some other person,
• suc h a property c arried out or m ade in a fictitious name,
• owner of a property is not aware of, or, denies knowledge of, suc h
ownership;
• where the person providing the c onsideration is not trac eable or is
fic titious.
Such a transaction is said to be a benami transaction.
Accordingly, in the light of the above provisions, the owner of the land as per land
records shall make payment for the land standing in his/her name in order to be
valid transaction and not to be considered as benami transaction in the terms of
section 2(9) of the said Act.
(B) As per the facts given in the case study, Mr. Cute defaulted in the payment of few
installments on the loan secured on the property owned by the HHFL. He tried
persuading bank to restructure the covenants of loan agreement. Bank Officials did
not agree to his request and decided to take action against him and the said
mortgaged property.
As per Section 18 of the Prohibition of Benami Property Transactions Act, 1988, the
following Authorities shall be there for the purposes of this Act, namely:—
(a) the Initiating Officer;
(b) the Approving Authority;
(c) the Administrator; and
(d) the Adjudicating Authority.
The authorities shall exercise all or any of the powers and perform all or any of the
functions conferred on, or, assigned, as the case may be, to it under this Act or in
accordance with such rules as may be prescribed.

© The Institute of Chartered Accountants of India


18 FINAL (NEW) EXAMINATION: NOVEMBER, 2018

The authorities under this Act shall have the same powers as are vested in a civil
court (Under section 19).
Accordingly, denial to agree to the request of Mr. Cute to restructure the covenants
of loan agreement by bank officials, is right.However, the decision to take action
against him and the said mortgaged property is not available with the Bank officials
under the Prohibition of Benami Property Transactions Act, 1988. Any property,
which is subject matter of benami transaction, shall be liable to be confiscated by
the Central Government under section 5 of the Prohibition of Benami Property
Transactions Act, 1988.
(C) Following will be the legal ramifications of the instructions m ade by Mr. Cute to his
Lawyer and advice by his lawyer with respect to furnishing of a fabricated reply in
response to the notice and to include a point as to why notice has been issued to
him. The said notice shall be issued to HHFL only-
As per section 24 of the Prohibition of Benami Property Transactions Act, 1988,
where the Initiating Officer, on the basis of material in his possession, has reason to
believe that any person is a benamidar in respect of a property, he may, after
recording reasons in writing, issue a notice to the person to show cause within such
time as may be specified in the notice why the property should not be treated as
benami property.
Where the notice specifies any property as being held by a benamidar, a copy of the
notice shall also be issued to the beneficial owner if his identity is known. Where the
Initiating Officer is of the opinion that the person in possession of the property held
benami may alienate the property during the period specified in the notice, he may,
with the previous approval of the Approving Authority, by order in writing, attach
provisionally the property in the manner as prescribed in Rule 4 of the Benami
Transactions Prohibition Rules, 2016, for a period not exceeding ninety days from
the date of issue of notice.
The Initiating Officer, after making such inquires and calling for such reports or
evidence as he deems fit and taking into account all relevant materials, shall, within
a period of ninety days from the date of issue of notice —
(a) where the provisional attachment has been made —
(i) pass an order continuing the provisional attachment of the property with
the prior approval of the Approving Authority, till the passing of the order
by the Adjudicating Authority; or
(ii) revoke the provisional attachment of the property with the prior approval
of the Approving Authority;

© The Institute of Chartered Accountants of India


PAPER – 6D: ECONOMIC LAWS 19

(b) where provisional attachment has not been made—


(i) pass an order provisionally attaching the property with the prior approval
of the Approving Authority, till the passing of the order by the Adjudicating
Authority; or
(ii) decide not to attach the property as specified in the notice, with the prior
approval of the Approving Authority.
Where the Initiating Officer passes an order continuing the provisional
attachment of the property or passes an order provisionally attaching the
property, he shall, within fifteen days from the date of the attachment, draw up
a statement of the case and refer it to the Adjudicating Authority.
Parties to be issued notice: On receipt of a reference under Section 24, the
Adjudicating Authority shall issue notice, to furnish such documents,
particulars or evidence as is considered necessary on a date to be specified
therein, on the following persons, namely:—
(a) the person specified as a benamidar therein;
(b) any person referred to as the beneficial owner therein or identified as
such;
(c) any interested party, including a banking company;
(d) any person who has made a claim in respect of the property.
Therefore, as per the above given provisions, the Adjudicating Authority shall
issue notice, to furnish such documents, particulars or evidence as is
considered necessary on a date to be specified therein in the notice to the
person specified as a benamidar therein; and to as the beneficial owner
therein or identified as such.
(D) After persuing the order passed by Adjudicating authorities, Mr. Cute identified
certain errors and misplaced facts. He asked his lawyer to discuss the same with
the authorities. However, his lawyer forgot the same. Reminding after 11 months, he
responded that the said matter is time-barred.
According to section 47 of the Prohibition of Benami Property Transactions Act,
1988, the Appellate Tribunal or the Adjudicating Authority may, in order to rec tify any
mistake apparent on the face of the record, amend any order made by it under
section 26 and section 46 respectively, within a period of one year from the end of
the month in which the order was passed.
No amendment shall be made, if the amendment is likely to affect any person
prejudicially, unless he has been given notice of intention to do so and has been
given an opportunity of being heard.

© The Institute of Chartered Accountants of India


20 FINAL (NEW) EXAMINATION: NOVEMBER, 2018

Accordingly, above stated course of action may be available to the Mr. Cute in
compliance with the said provision.
2.12 (A) Mr. Bhakt, an allottee was told that Ganesh Temple would be constructed as a part
of the project in podium 2. Due to technical objection, an intimation along with the
certificate from engineer was sent to allottees stating that as of consequences
temple will have to be shifted from podium 2 to podium 3. Also filed a complained
against Ms. F and towards builders for several defects in the features contained in
the marketing brochure.
According to section 18 of the Real Estate (Regulation & Development) Act, 2016, if
the promoter fails to complete or is unable to give possession of an apartment, plot
or building, in accordance with the terms of the agreement for sale or, as the case
may be, duly completed by the date specified therein;
He shall be liable on demand to the allottees, in case the allottee wishes to
withdraw from the project, without prejudice to any other remedy available, to return
the amount received by him in respect of that apartment, plot, building, as the case
may be, with interest at such rate as may be prescribed in this behalf including
compensation in the manner as provided under this Act.
However, where an allottee does not intend to withdraw from the project, he shall be
paid, by the promoter, interest for every month of delay, till the handing over of the
possession, at such rate as may be prescribed.
If the promoter fails to discharge any other obligations imposed on him under this
Act or the rules or regulations made thereunder or in accordance with the terms and
conditions of the agreement for sale, he shall be liable to pay such compensation to
the allottees, in the manner as provided under this Act.
Further Section 9 of the Act specifies that no real estate agent shall facilitate the
sale or purchase of or act on behalf of any person to facilitate the sale or purchase
of any plot, apartment or building, as the case may be, in a real estate project or
part of it, being the part of the real estate project registered under section 3, being
sold by the promoter in any planning area, without obtaining registration under this
section. In case of contravention, Ms. F will be liable under section 62 of the said
Act.
Accordingly, Mr. Bhakt will have above remedies under the RERA against the Ms. F
and against builder with the authorities.
(B) According to section 14 of the RERA, the proposed project shall be developed and
completed by the promoter in accordance with the sanctioned plans, layout plans
and specifications as approved by the competent authorities.

© The Institute of Chartered Accountants of India


PAPER – 6D: ECONOMIC LAWS 21

As per section 14(3) of the Act, in case any structural defect or any other defect in
workmanship, quality or provision of services or any other obligations of the
promoter as per the agreement for sale relating to such development is brought to
the notice of the promoter within a period of five years by the allottee from the date
of handing over possession, it shall be the duty of the promoter to rectify such
defects without further charge, within thirty days, and in the event of promoter's
failure to rectify such defects within such time, the aggrieved allottees shall be
entitled to receive appropriate compensation in the manner as provided under this
Act.
In the given case, Mr. Sultan after 8 months of his staying, informed the builder of
the quality of MCB with a potential risk of short circuit.Considering it a structural
defect Mr. Sultan intimated within time frame. So promoter of the project will be
liable here.
Whereas complain of grass given by builder in flower bed area for replacement is in
the nature of "minor additions or alterations”, so promoter will be discharged of his
liabilities.
(C) As per section 15(1) of the Real Estate (Regulation & Development) Act, 2016, the
promoter shall not transfer or assign his majority rights and liabilities in respect of a
real estate project to a third party without obtaining prior written consent from two-
third allottees, except the promoter, and without the prior written approval of the
Authority.
However, such transfer or assignment shall not affect the allotment or sale of the
apartments, plots or buildings as the case may be, in the real estate projec t made
by the erstwhile promoter.
In the case study, the American Builder’s, proposal was not valid as no prior written
consent from two-third allottees and the authority were obtained.
Case Study 3
Question 3
Mr. Inder and Mr. Sunder are promoter directors of India Exports Limited having registered
office in Jammu, is engaged in the export of software products to various countries in the
world. One of the customer in U.S. to whom the company exported certain products failed to
pay the amount due for these exports resulting into non-repatriation of amount to India. The
Adjudicating Authority on coming to know about this, levied a penalty on the company under
the provisions of the Foreign Exchange Management Act, 1999. The Company has sought
advice on the followings:
(a) Relevant provisions for realization of export amount and its timeline.

© The Institute of Chartered Accountants of India


22 FINAL (NEW) EXAMINATION: NOVEMBER, 2018

(b) Timeline to surrender the realized foreign exchange under the Act.
(c) Cases where realization and repatriation enjoy exemption.
Later, the company settled the amount for 50% with the customer and the amount was
transferred through Hawala to India. The money so received was partly used by the company
to part finance it's office building in Mumbai. During search in the premises of Hawala
businessman, some documentary evidence was captured by the search officer and based on
which, the Adjudicating Authority appointed under the Prevention of Money Laundering Act,
2002 issued an order attaching the office of the company alleged to be involved in scheduled
offence of money laundering Mr. Prabhat, one of the employee was sent to Japan to develop a
software program on deputation for 2 years. He earned a sum of US$ 3000 as a honorarium.
Ms. Lilly, the daughter of Mr. Inder is an air hostess with the British Airways and flies for 12
days in a month and thereafter takes a break for 18 days. During the break, she
accommodated of 'base', which is normally the city, outside India where the airways are
headquartered. However, for security considerations, she was based on Mumbai, during the
current financial year and was accommodated at Mumbai for more than 182 days.
Mr. Victor, son of Mr. Sunder, having Indian origin and resident of USA desires to acquire two
immovable properties in India comprising a residential flat in Noida and a farm house on the
outskirts of Delhi. Further, Mr. Sunder has won lottery and want to remit the amount to his son
Mr. Victor in USA for buying-immovable property in USA under joint ownership of 50% with Mr.
Sunder. Mr. Sunder also wants to remit money to meet his obligation of 50% in the above
immovable property.
The balance of the money received through Hawala was used by the company to part finance
the residential flat in Noida purchased by Mr. Victor.
The Adjudicating Authority appointed under the Prevention of Money Laundering Act, 2002
issued an order attaching the flat alleged to be involved in scheduled offence of money
laundering. The company decides to challenge the action of the Adjudicating Authority.
In the meantime, Mr. Sunder requested the Chief Financial Officer to examine the following
issues under the Prevention of Money Laundering Act, 2002:
(a) Process of money laundering
(b) Multiple method of money laundering
(c) The connection between 'proceeds of crime' and 'criminal activity'
(d) The request from a contracting state for investigation.
(e) The powers of the authority under the Act to survey
Answer the following questions:
(3.1) Which of these is not a permissible capital account transactions?

© The Institute of Chartered Accountants of India


PAPER – 6D: ECONOMIC LAWS 23

(A) Investment by person resident in India in Foreign Securities.


(B) Foreign currency loans raised in India and abroad by a person resident in India.
(C) Export, Import and holding of currency/currency notes.
(D) Trading in transferable development rights. (2 Marks)
(3.2) Mr. Prabhat can retain the honorarium earned by him on deputation to the extent of US $:
(A) 3000
(B) 2000
(C) 1000
(D) Nil (2 Marks)
(3.3) The residential status of Ms. Lilly for the current financial year under FEMA would be:
(A) Non-Resident irrespective of her citizenship.
(B) Resident irrespective of her citizenship.
(C) Non-Resident since she is British citizen.
(D) Resident though she is British citizen. (2 Marks)
(3.4) The time limit within which the appeal can be lodged against the decision of the
Adjudicating Authority by India Export Limited:
(A) Within 30 days from receipt of order.
(B) Within 60 days from receipt of order.
(C) Within 45 days from receipt of order.
(D) Within 90 days from receipt of order. (2 Marks)
(3.5) Mr. Victor can acquire the following properties by following the steps as mentioned in the
provisions of the Foreign Exchange Management Act, 1999:
(A) a farm house in outskirt of Delhi, only.
(B) both farm house in the outskirt of Delhi and a flat in Noida.
(C) a flat in Noida, only.
(D) None of above (2 Marks)
(3.6) Section 2 of the Prevention of Money Laundering Act, 2002 defines the term 'scheduled
offence', which accordingly means:

© The Institute of Chartered Accountants of India


24 FINAL (NEW) EXAMINATION: NOVEMBER, 2018

(A) the offences specified under Part A of the Schedule.


(B) the offences specified under Part B of the Schedule if the total value involved in
such offences is thirty lakh rupees or more.
(C) the offences specified under Part C of the Schedule.
(D) All of the above (2 Marks)
(3.7) Whoever commits offence of Money Laundering shall be punishable with:
(A) imprisonment only.
(b) fine only
(C) imprisonment or fine.
(D) Imprisonment and fine. (2 Marks)
(3.8) Money Laundering is a single process however, its cycle can be broken down into
following three distinct stages:
(A) Integration, Layering and Placement.
(B) Layering, Placement and Integration.
(C) Placement, Layering and Integration.
(D) Placement, Integration and Layering. (2 Marks)
(3.9) Where an order of confiscation has been made under the provisions of section 58B of the
Prevention of Money Laundering Act, 2002, in respect of any property of a person, all
rights and title in such property shall vest absolutely in the _______free from all
encumbrances.
(A) Central Government
(B) Supreme Court
(C) President of India
(D) None of the above (2 Marks)
(3.10)The offences under the Prevention of Money Laundering Act, 2002 shall be:
(A) cognizable and bailable.
(B) non-cognizable and non-bailable.
(C) cognizable and non-bailable.
(D) non-cognizable and bailable. (2 Marks)

© The Institute of Chartered Accountants of India


PAPER – 6D: ECONOMIC LAWS 25

(3.11)Answer the following with reference to the provisions of the Foreign Exchange
Management Act, 1999 (FEMA):
(A) The FEMA extends to the whole of India. Examine and advise India Export Ltd. as
they have registered office in Jammu. (2 Marks)
(B) The drawal of foreign exchange is prohibited for certain current account
transactions. Examine and advise Mr. Sunder whether he can remit the amount of
lottery won by him to Mr. Victor in USA. (2 Marks)
(C) The Act restricts acquisition or transfer of immovable property outside India by a
person resident in India. Examine and advise Mr. Sunder whether he can remit
amount to buy immovable property in USA. (2 Marks)
(D) The amount representing full export value shall be realized within time limit
permitted under the Act. Explain and advise India Export Ltd. the relevant provisions
for realization of export value and its timeline. (4 Marks)
(E) The realized foreign exchange is to be surrendered within the period specified
under the Act. Examine and advise India Export Ltd. (2 Marks)
(F) The realization and repatriation in certain cases enjoy exemption. Examine and
advise India Export Ltd. (3 Marks)
(3.12) Examine and advise Mr. Sunder on the following with reference to the provisions of the
Prevention of Money Laundering Act, 2002 (PMLA):
(A) Money Laundering is a process. (3 Marks)
(B) There are multiple methods of money laundering. (3 Marks)
(C) The 'proceeds of crime' and 'criminal activity' have connection. (2 Marks)
(D) The request from a contracting state can be accepted for investigation. (3 Marks)
(E) The Authority under the Act can make survey only based on the material in his
possession. (4 Marks)

Answer to Case Study 3


(3.1) Option (D) :Trading in transferable development rights
(3.2) Option (B) : 2000
(3.3) Option (A) : Non-Resident irrespective of her citizenship
(3.4) Option (C) :within 45 days from receipt of order

© The Institute of Chartered Accountants of India


26 FINAL (NEW) EXAMINATION: NOVEMBER, 2018

(3.5) Option (C):a flat in Noida, only.


(3.6) Option (D) :All of the above
(3.7) Option (D) :imprisonment and fine
(3.8) Option (C) :Placement, Layering and Integration
(3.9) Option (A) :Central Government
(3.10) Option (C) :Cognizable and non-bailable
(3.11)(A) Extent and Application [Sections 1 of FEMA, 1999]
FEMA, 1999 extends to the whole of India. In addition, it shall also apply to all
branches, offices and agencies outside India owned or controlled by a person
resident in India and also to any contravention thereunder committed outside India
by any person to whom this Act applies.
Accordingly, FEMA does not apply to citizens of India who are outside India unless
they are resident of India. The scope of the Act has been further extended to
include branches, offices and agencies outside India. The scope is thus wide
enough because the emphasis is on the words “Owned or Controlled”. Even
contravention of the FEMA committed outside India by a person to whom this Act
applies will also be covered by FEMA.
(B) According to Section 5 of the FEMA, 1999 and rules/regulations made thereunder,
the drawal of foreign exchange for certain current account transactions is
prohibited, a few need permission of appropriate Govt. of India authority and some
other transactions would require RBI permission if they exceed a certain ceiling.
According to Schedule I, Remittance out of lottery winnings is prohibited.
Hence, Mr. Sunder cannot remit the amount of lottery won by him to Mr. Victor in
USA.
(C) According to Regulations on Acquisition and Transfer of Immovable Property
outside India, a person resident in India may acquire immovable property outside
India, jointly with a relative who is a person resident outside India, provided there is
no outflow of funds from India.
In the instant case, Mr. Sunder wants to remit money to meet his obligation of 50%
in the immovable property in USA under joint ownership with his son Mr. Victor.
Hence, as per the regulations, Mr. Sunder cannot remit amount to buy immovable
property in USA.
(D) Period within which export value of goods/software/ services to be realized
[Foreign Exchange Management (Export of Goods and Services) Regulations, 2015]

© The Institute of Chartered Accountants of India


PAPER – 6D: ECONOMIC LAWS 27

(1) The amount representing the full export value of goods / software/ services
exported shall be realised and repatriated to India within nine months from the
date of export, provided
(a) that where the goods are exported to a warehouse established outside
India with the permission of the Reserve Bank, the amount representing
the full export value of goods exported shall be paid to the authorised
dealer as soon as it is realised and in any case within fifteen months from
the date of shipment of goods;
(b) further that the Reserve Bank, or subject to the directions issued by that
Bank in this behalf, the authorised dealer may, for a sufficient and
reasonable cause shown, extend the period of nine months or fifteen
months, as the case may be.
(2) (a) Where the export of goods / software / services has been made by Units
in Special Economic Zones (SEZ) / Status Holder exporter / Export
Oriented Units (EOUs) and units in Electronics Hardware Technology
Parks (EHTPs), Software Technology Parks (STPs) and Bio-Technology
Parks (BTPs) as defined in the Foreign Trade Policy in force, then, the
amount representing the full export value of goods or software shall be
realised and repatriated to India within nine months from the date of
export.
Provided further that the Reserve Bank, or subject to the directions
issued by the Bank in this behalf, the authorised dealer may, for a
sufficient and reasonable cause shown, extend the period of nine months.
(b) The Reserve Bank may for reasonable and sufficient cause direct that the
said exporter/s shall cease to be governed by sub-regulation (2);
Provided that no such direction shall be given unless the unit has been
given a reasonable opportunity to make a representation in the matter.
(c) On such direction, the said exporter/s shall be governed by the provisions
of sub-regulation (1), until directed otherwise by the Reserve Bank.'
Explanation—For the purpose of this regulation, the “date of export” in
relation to the export of software in other than physical form, shall be
deemed to be the date of invoice covering such export.
(E) Period for surrender of received/ realised/ unspent/ unused foreign exchange by
Resident individuals [Regulation 5 of Foreign Exchange Management (Realisation,
repatriation and surrender of foreign exchange) Regulations, 2000]:A Person being
an individual resident in India shall surrender the received/realised/unspent/ unused
foreign exchange whether in the form of currency notes, coins and travellers
cheques, etc. to an authorised person within a period of 180 days from the date of

© The Institute of Chartered Accountants of India


28 FINAL (NEW) EXAMINATION: NOVEMBER, 2018

such receipt/realisation/purchase/acquisition or date of his return to India, as the


case may be.
(F) Exemption from realisation and repatriation in certain cases [Section 9 of
FEMA, 1999]
The provisions of sections 4 and 8 shall not apply to the following, namely:
(a) possession of foreign currency or foreign coins by any person up to such limit
as the Reserve Bank may specify;
(b) foreign currency account held or operated by such person or class of persons
and the limit up to which the Reserve Bank may specify;
(c) foreign exchange acquired or received before the 8th day of July, 1947 or any
income arising or accruing there on which is held outside India by any person
in pursuance of a general or special permission granted by the Reserve Bank;
(d) foreign exchange held by a person resident in India up to such limit as the
Reserve Bank may specify, if such foreign exchange was acquired by way of
gift or inheritance from a person referred to in clause (c), including any income
arising there from;
(e) foreign exchange acquired from employment, business, trade, vocation,
service, honorarium, gifts, inheritance or any other legitimate means up to
such limit as the Reserve Bank may specify; and
(f) such other receipts in foreign exchange as the Reserve Bank may specify.
(3.12) (A) Money laundering is a process: It is the process by which illegal funds and assets
are converted into legitimate funds and assets. In other words, it is basically the
process of converting illegal or black money of a person in a legal or white money.
It is the process used by criminals to wash their “tainted” money to make it “clean.”
Money laundering is a single process however; its cycle can be broken down into
three distinct stages
1. Placement: It is the first and the initial stage when the crime money is injected
into the formal financial System.
2. Layering: Then under the second stage, money injected into the system is
layered and moved or spread over various transactions in different accounts
and different countries. Thus, it will become difficult to detect the origin of the
money.
3. Integration: Under the third and final stage, money enters the financial system
in such a way that original association with the crime is sought to be
obliterated so that the money can then be used by the offender or person
receiving as clean money.

© The Institute of Chartered Accountants of India


PAPER – 6D: ECONOMIC LAWS 29

(B) Multiple methods of money laundering: There are multiple methods through
which money can be laundered and huge profit is being made, some of them are:
• Cash Smuggling: Moving cash from one location to another or depositing the
cash in Swiss Bank Account;
• Structuring: Cash is broken down into formal receipts to buy money orders
etc., smaller amounts are hard to detect;
• Laundering via Real Estate: Buying a land for money and then selling it making
the profits legal.
• Stock Markets scams
• By creating bogus companies.
• Drug Trafficking;
• Bribery and Corruption;
• Kidnapping and Extortion.
(C) Section 2(1)(u) defines "proceeds of crime"as any property derived or obtained,
directly or indirectly, by any person as a result of criminal activity relating to a
scheduled offence or the value of any such property or where such property is
taken/held outside the country, then the property equivalent in value held within the
country.
(D) Letter of Request to a Contracting State in Certain Cases [Section 57 of the
PMLA, 2002]
1. If, in the course of an investigation into an offence or other proceedings under
this Act, an application is made to a Special Court by the Investigating Officer
or any officer superior in rank to the Investigating Officer that any evidence is
required in connection with investigation into an offence or proceedings under
this Act and he is of the opinion that such evidence may be available in any
place in a contracting State, and the Special Court, on being satisfied that
such evidence is required in connection with the investigation into an offence
or proceedings under this Act, may issue a letter of request to a court or an
authority in the contracting State competent to deal with such request to-
(i) examine facts and circumstances of the case,
(ii) take such steps as the Special Court may specify in such letter of
request, and
(iii) forward all the evidence so taken or collected to the Special Court issuing
such letter of request.
2. The letter of request shall be transmitted in such manner as the Central

© The Institute of Chartered Accountants of India


30 FINAL (NEW) EXAMINATION: NOVEMBER, 2018

Government may specify in this behalf.


3. Every statement recorded or document or thing received shall be deemed to
be the evidence collected during the course of investigation.
(E) Power of authority to make survey [Section 16(1) of the PMLA]: Where an
authority, on the basis of material in his possession, has reason to believe (the
reasons for such belief to be recorded in writing) that an offence under section 3
has been committed, he may enter any place—
(i) within the limits of the area assigned to him; or
(ii) in respect of which he is authorised for the purposes of this section by such
other authority, who is assigned the area within which such place is situated,
at which any act constituting the commission of such offence is carried on, and may
require any proprietor, employee or any other person who may at that time and
place be attending in any manner to, or helping in, such act so as to,-
(i) afford him the necessary facility to inspect such records as he may require and
which may be available at such place;
(ii) afford him the necessary facility to check or verify the proceeds of crime or any
transaction related to proceeds of crime which may be found therein; and
(iii) furnish such information as he may require as to any matter which may be
useful for, or relevant to, any proceedings under this Act.
Explanation. - For the purposes of this sub-section, a place, where an act which
constitutes the commission of the offence is carried on, shall also include any other
place, whether any activity is carried on therein or not, in which the person carrying
on such activity states that any of his records or any part of his property relating to
such act are or is kept.
Therefore, Authority under the Act can make survey as per the above stated section.

© The Institute of Chartered Accountants of India


DISCLAIMER
This Suggested Answers hosted on the website do not constitute the basis for

evaluation of the student’s answers in the examination. The answers are

prepared by the Faculty of the Board of Studies with a view to assist the

students in their education. While due care is taken in preparation of the

answers, if any errors or omissions are noticed, the same may be brought to

the attention of the Director of Studies. The Council of the Institute is not in

anyway responsible for the correctness or otherwise of the answers published

herein.

Further, in the Elective Papers which are Case Study based, the solutions

have been worked out on the basis of certain assumptions/views derived from

the facts given in the question or language used in the question. It may be

possible to work out the solution to the case studies in a different manner

based on the assumption made or view taken.

© The Institute of Chartered Accountants of India


2 FINAL EXAMINATION: MAY 2018

PAPER 6D: ECONOMIC LAWS

NOTE: There are three case study questions in the question paper. Candidates are
required to answer all the questions of any two case study questions.
Citation of case laws, sections, subsections, rules may not form part of the answer.

Case Study No. 1


Question
(A) A complaint was made by a complainant (Informant) to the Competition Commission of
India (CCI) against the practices adopted by certain Insurance Companies in
implementation of the Insurance scheme, Country Peoples Plan (CPP) by an imaginary
State Government 'Z' in India.
The CCI after going through the complaint, on merit, ordered a detailed investigation by the
Director General of Investigation under the Competition Act, 2002 (as amended in 2007,
briefly referred to hereinafter as the "Act"). The facts of the case are mentioned as under:
(i) CPP is the health insurance scheme introduced by the Central Government for
below poverty line (BPL) families. The task of implementation of this scheme was
entrusted to the respective State Governments of the country with the Central
Government bearing 75% of the expenses incurred in relation to the annual
premiums.
(ii) A tender was floated by a State Government 'Z' through its agency ULTRA (on
1.11.2009) for selecting and insurance service provider for the implementation of
the CPP for the year beginning 2010-11 for a period of three years. The State
Government 'Z' issued a tender for the implementation of CPP scheme for the
selection of the insurance provider. In this regard, bids were invited from: (a)
insurance companies licensed and registered with the Insurance Regulatory and
Development Authority; and (b) agencies enabled by any central legislation to
undertake health insurance related activities. The last date for submission of the-
tender was 31.1.2010.
(iii) Four Public Sector Insurance Companies A, B, C & D Insurance Company, each
submitted their offer in response to the above tender before its last date of
submission. All these companies formed an Insurance Facilitation Group (IFG) with
the objective of a common cause of furtherance and Development of insurance
business in India and all these companies were members of the IFG. Before
submitting their bids against the above tender, officials of these companies attended
a meeting of IPG as per their practice, held on 27.12.2009 at XYZ place in the State
'Z' with the sole agenda to discuss the Tender Notice on CPP dated 1.11.2009 of the
State Government 'Z'. They agreed on a business sharing model of sharing the
business in the ratio of 55% by the winning company and 15% each by the

© The Institute of Chartered Accountants of India


PAPER – 6D: ECONOMIC LAWS 3

remaining companies of the total business generated. They also agreed on the
premiums to be quoted by each of them in response to the tender. The minu tes of
the meeting signed by officials of aforementioned companies stated to share the
business among the four Insurance Companies with insurance Company with 55%
and other Companies with 15% each. D Insurance Company will be L1 and other
three insurance companies will be L-2 to L-4 in the quotation being submitted on
28th December, 2009 as per the decision taken in the above meeting.
(iv) Seven insurance companies including the A, B, C, & D Insurance Company
submitted the tender documents. The Technical Evaluation Committee (TEC) formed
by the State Government ‘Z’ evaluated the bids on the basis of a scoring system.
The TEC decided that the companies which scored 50 marks and above (a
benchmark set by the TEC through ratings) would be declared successful i n the
technical rounds. As such, only C and D insurance Company were declared
successful and their financial bids were opened in the presence of the
representatives of the respective insurance companies. TEC recommended
acceptance of D Insurance Company’s bid for implementation of CPP scheme being
the lowest in the State ‘Z’ for a period of three years subject to yearly basis
renewals. D Insurance Company was awarded the tender on the basis of
comparative bids mentioned as under:
Details of Price Bids relating to the Tender dated 1.11.2009 for 2010-11.
S. Participating Whether Marks Awarded Premium Amt. as
No. Insurance Technically in Technical stated in Bid (`)
company Qualified Evaluation
Without With ST
S.T @ 10.3%
1 D Yes 76 521 575
2 C Yes 63 597 658
3 E No 49 509 561
4 F No 45 599 652
5 B No 49 590 651
6 A No 47 580 640
7 G No 48 775 854
(v) Accordingly, D Insurance Company won the tender for 2010-11 and later on shared
its business with A, B & C Insurance Company in their agreed mutual model sharing
ratio. The tender was issued for a period of three years. However, towards the end
of the first year of the contract, D Insurance Company sought for an upward revision
of premium to ` 1,000/- per family. When this request of D Insurance Company was
turned down by the State Government 'Z'; D Insurance Company invoked the exit
clause of the contract. As a result of this action, the State Government retendered.

© The Institute of Chartered Accountants of India


4 FINAL EXAMINATION: MAY 2018

(vi) Post Retendering Scenario: It was found that the price rise effected by the
Insurance companies - A, B, C & D Insurance Company could not have been based
on any rational business justification as the retender for the year
2011- 12 and 2012-13 was won by E Insurance Company at a much lower premium
of ` 840/- per family. The awarded contract was even extended with the same
premium for the year 2012-13, 2013-14 and 2014-15 i.e. for a period of three years
and this contract was renewed for the year 2014-15 at the same price. E Insurance
Company confirmed that the company was not incurring any losses for providing
health insurance services under CPP scheme. The details of rates of these
Insurance companies in relation to the tenders of 2010-11 to 2012-13 are mentioned
as under:
Details of Insurance companies rates bids in relation to tenders of 2010 -11
to 2013-14
Price Bids (` )
S. Name of 2010-11 2011-12 2012-13 2013-14
No. Insurance
company
Without With Without With Without With Without With
S.T ST S.T ST S.T ST S.T ST
1 2 3 4 5 6 7 8 9 10
1 A 580 640 850 938 1700 1875 900 994
2 B 590 651 850 938 1250 1392 1100 1214
3 C 597 658 910 1004 1400 1546 920 1016
4 D 521 575 1000 1104 1000 1104 1000 1104
5 E **509 561 840 927 840 927 840 927
** Not technically qualified
(vii) It was observed that the State Government entrusted its agency named ULTRA to
implement CPP scheme in letter and spirit in the State and this agency had actually
facilitated continuance of D Insurance Company as the insurer under these
schemes by employing an arbitrary practices. A, B, C & D Insurance Companies
have claimed that until 2002, all of them were owned by General Insurance
Company.
It was also submitted that pursuant to the enactment of the General Insurance
Business (Nationalization) Amendment Act, 2002, Government of India holds 100%
shares of each of them and controls the management and affairs of the companies
through Department of Financial Services (Insurance Division), Ministry of Finance.
In this regard, a reference may be had to the policy reforms introduced by the
Government of India in 1991 which led to the de-regulation of the Indian economy.

© The Institute of Chartered Accountants of India


PAPER – 6D: ECONOMIC LAWS 5

With the commencement of private participation, a need was felt to modify the
existing market structure of certain select sectors, including, the insurance sector so
as to promote orderly growth of these sectors.
In this regard, the Government of India established a committee in the year 1993
under the chairmanship of Shri R. N. Malhotra (former Governor of the Reserve
Bank of India) to propose reforms for the insurance sector. Pursuant to the
recommendations of the Malhotra Committee, two major regulatory changes were
introduced, including, ending the monopoly of General Insurance Company in the
general insurance business and ending the control exercised by General Insurance
Company over its wholly owned subsidiaries.
These regulatory changes were ushered in to allow the public sector insurance
companies to act independently and to compete with the private players to offer
better services to consumers.
(viii) Further, A, B, C & D Insurance Companies submitted that all decisions relating to
submission of bids, determination of bid amounts, business sharing arrangements,
etc. were taken internally at company level without any ex ante approval/ dire ctions
from Ministry of Finance. Even the decisions taken by the companies were not
notified ex post to the Ministry. These companies participated in the above said
tenders, independent of Ministry of Finance.
(ix) Details of Business Sharing Arrangement among A, B, C & D Insurance Companies
relating to the Tender dated 1.11.2009 are tabulated as under:
Details of Business Sharing Arrangement relating to the Tender dated
1.11.2009
Total Business Generated for D Insurance Company: ` 92,94,65,400/-
S. Name of Insurance Business Sharing Business sharing (in
No. Company (in term of %) term of revenue ( `)
1 A 15 13,94,19,810.00
2 B 15 13,94,19,810.00
3 C 15 13,94,19,810.00
4 D 55 51,12,05,970.00

(x) Turnover of the A,.B,C & D Insurance Companies in the last three financial years
based on the financial statements were as under :
S. Name of the Insurance Annual Turnover (` In crore)
No. Company
2010-11 2011-12 2012-13
1 A 6000 7660 9575

© The Institute of Chartered Accountants of India


6 FINAL EXAMINATION: MAY 2018

2 B 5400 6745 7853


3 C 7600 7500 8765
4 D 6745 7352 7872
You are required to analyse, with reference to the Competition Act Provisions.
Q.1 Whether the public sector insurance companies i.e., A, B, C & D Insurance
Company constitute a single economic entity? Explain. (5 Marks)
Q.2 Examine whether the A, B, C & D Insurance Companies by their conduct have
entered into an agreement and have contravened any of the provisions of the
Competition Act. Explain. (10 Marks)
Q.3 The State Government 'Z' has now desired to include a specific clause in the bid
document to prevent abuse of the Competition Act. What key clauses would you
recommend? Please draft your reply within a total of 100-200 words. (5 Marks)
Q.4 Assume a situation where the agreement and the meeting of IFG took place outside
India. Explain whether the provisions of the Act still be applicable. (4 Marks)
Q.5 Chairman of the Competition Commission of India, based upon the facts of the
above case, has requested you as an officer of the Commission to draft a brief show
cause notice that should be issued to the insurance companies alleged to be in
default. Your notice should cover the following aspects namely Authority issuing the
notice, Defendant details, Alleged contraventions, Facts as available and Time line
for the response by the defendant. Also include the relevant provisions which
empower such notices to be issued. (6 Marks)
(B) You are the Chairman of Competition Commission of India (CCI) under the Competition
Act, 2002 (hereafter, the Act) as amended in 2007 and subsequently you are chairing t he
Bench to deal with information filed under section 19(1) (a) of the Act relating to the radio
taxi market, alleging abuse of dominance and predatory pricing. You do not own a car.
For official journeys, you are provided with an office vehicle. For private use, you
generally avail of the facility available in the market of radio taxis, fitted with GPS
instruments. Therefore, you are fully aware of the radio taxis available in the market and
exposed to the methodology of requisitioning a taxi for personal use and of paying for the
service.
Informants A and B are engaged in the business of providing radio taxi services in a
certain city XXX in South India under the brand names “Press and Hail a Taxi” and “Taxi
before you blink”, A large Radio Taxi provider C is also in the market competing with
Radio Taxi providers A and B and some others too. Informants A and B filed before the
CCI separate information under Section 19 (1) (a) of the Act alleging that Radio Taxi
provider C had abused its dominant position by engaging in predatory pricing in the
relevant market by offering heavy discounts to passengers and incentives to cab drivers,
in contravention of Section 4 (2) (a)(ii) of the Act. Radio Taxi provider C was in the habit

© The Institute of Chartered Accountants of India


PAPER – 6D: ECONOMIC LAWS 7

of having oral agreements with customers thus practising an opaque behaviour


prejudicing the interests of A and B.
Informants alleged that C controlled over 50% of a highly concentrated market,
demonstrating C's dominance. The Informants also alleged that there were considerable
entry barriers present which had made it difficult for a new player to effectively compete.
Consistent payment of high incentives and discounts along with exclusivity clauses in
agreements with drivers allowed C to thwart effective competition, lock -in drivers and
create a wide base of customers.
Additionally, the Informants alleged that the presence of an extensive network of C
across the city XXX had acted as a sufficient detriment to any countervailing buying
power available with consumers. They alleged that the presence of a large network of C
had restricted the power of consumers to negotiate and had substantially restricted
competition in the market for other Radio Taxis in the city XXX.
Based on the high market share of C, the Commission arrived at the prima facie view that
C held a dominant position in the relevant market of "Radio Taxi services" in city XXX and
directed the Director General ("DG") to conduct a detailed investigation into the matter.
Findings of the DG
The DG recognized the different business models prevailing in the radio taxi service
industry i.e. asset-owned model, aggregator model and hybrid model. He noted that while
C functioned under the aggregator model, its services were functionally substitutable with
those provided by other taxis operating under the different business models.
Accordingly, the DG concluded that the relevant product market would be the "market for
radio taxi services" and the relevant geographic market would be the city of XXX.
The DG compared the number of trips/rides undertaken by different players in the
relevant market between 2012 and 2016 to observe that while C did grow at a meager
rate of 63% between January and September of 2015, Informant A's trip size registered a
phenomenal growth of 1200% in the same period. He noted that A was an aggressive
player in the market and that the rise of A as a healthy competitor defeated the argument
of the presence of entry barriers. The DG concluded that C was not in a dominant
position, given these facts.
Informants had alleged that C had access to funds and had availed of the same in big
measure, thwarting the other operators to avail of funds. This, according to them, was an
entry barrier. DG found that no evidence had been supplied by the Informants to
substantiate this entry barrier allegation. DG dismissed the allegation as not proved.
Answer the following 10 Multiple Choice Questions by selecting the most appropriate
answer from the options given for each question. Write a few lines justifying your stance.
(10 x 2= 20 Marks)

© The Institute of Chartered Accountants of India


8 FINAL EXAMINATION: MAY 2018

(i) The oral agreements between Radio Taxi provider C and some customers, falling
within Section 2(b) of the Act ___________.
(a) are not legally enforceable
(b) are legally enforceable
(c) are not anti-competitive
(d) are not actions in concert
(ii) Dominance under the Act should be determined on the basis of ______________.
(a) market share
(b) price leadership.
(c) profitability
(d) ability to operate independently of competitive forces in the relevant market
(iii) Relevant market is made up of ______________.
(a) relevant geographic market
(b) relevant product market
(c) relevant geographic market and relevant product market
(d) market structure and size alone
(iv) Abuse of dominance by a dominant enterprise arises ___________.
(a) if the enterprise imposes unfair or discriminatory condition in purchase or sale
of goods or service
(b) if the enterprise imposes discriminatory condition or price to meet competition
(c) if the enterprise makes a sizeable profit in its activities
(d) if the enterprise is a price leader
(v) Predatory pricing arises when an enterprises __________.
(a) prices its product very high
(b) prices its product just below the prevalent market price
(c) prices its product to clear inventory
(d) prices its product below its cost of production with a view to reducing
competition or eliminating competitors
(vi) Two Enterprises ____________.
(a) can be in a dominant position at the same time
(b) cannot be in a dominant position at the same time

© The Institute of Chartered Accountants of India


PAPER – 6D: ECONOMIC LAWS 9

(c) can be dominant only if they merge


(d) can be dominant only if one acquires the other
(vii) Abuse of dominance does not arise if___________.
(a) the enterprise limits or restricts production of goods or provision of services.
(b) the enterprise limits or restricts technical and scientific development relating to
goods or services to the prejudice of consumers.
(c) the enterprise does not indulge in practices resulting in denial of market
access.
(d) the enterprise uses its dominance in one relevant market to enter into other
relevant market.
(viii) CCI cannot make enquiry into alleged contravention of the provisions in Section 3
and 4 ______.
(a) on unfounded rumours
(b) on its own motion
(c) on receipt of information from consumers or trade associations
(d) on receipt of a reference from the Central Government or State Government
(ix) The parties requesting for confidentiality of information or documents submitted
during the investigation shall have to satisfy the conditions laid down in regulation
_____ of the Competition Commission of India (General) Regulations, 2009.
(a) 42
(b) 39
(c) 35
(d) None of the above
(x) Relevant product market will have to reckon
(a) regulatory trade barriers
(b) physical characteristics or end-use of goods
(c) national procurement policies
(d) transport costs
Answers to Part (A) of Case study 1

Answer 1
Yes, the Public insurance companies, A, B, C & D Insurance company constitute a single
economic entity, which means that companies associated with each other through the virtue of

© The Institute of Chartered Accountants of India


10 FINAL EXAMINATION: MAY 2018

common control operate. These Companies formed an Insurance Facilitation Group (IFG) with
the objective of a common cause of furtherance and development of insurance business in
India and all these companies were members of IFG.
This common control operate can be considered as cartel defined in section 2(c) of the
competition Act, 2002.
"Cartel" includes an association of producers, sellers, distributors, traders or service providers
who, by agreement amongst themselves, limit, control or attempt to control the production,
distribution, sale or price of, or, trade in goods or provision of services.
Alternative answer
It is observed that although the public sector insurance companies namely A, B, C and D
Insurance company are presently under the supervision of the Central Government, each of
them placed a separate bid in response to the tenders issued by the State Government for
implementation of the CPP scheme.
Further, the Insurance companies themselves have submitted that all decisions relating to
submission of bids, determination of bid amounts, business sharing arrangements, etc. were
taken internally at company level without any ex ante approval/ directions from the Ministry of
Finance. Thus, it is apparent that these companies participated in the impugned tenders
independent of Ministry of Finance.
In view of the above, it is concluded that bid offers submitted by the A, B, C and D Insurance
companies in response to the Tender issued by the State Government ‘Z’ in relation to the
CPP were based on their own volition and the Ministry of Finance had no role to play. The
Ministry of Finance did not exercise any de facto or de jure control over business decisions of
these companies in submitting bids for impugned tenders. As such, these insurance
companies do not constitute a single economic unit.
Answer 2
Yes, A, B, C, & D insurance companies have entered into an agreement for sharing the
business on a basis of business sharing model in the ratio of 55% by the winning company
and 15% each by the remaining companies of the total business generated.
Insurance companies through an agreement between them quoted the bids rate, which has
the effect of eliminating or reducing competition for bids or adversely affecting or manipulating
the process for bidding.
These are anti-competitive agreements defined under section 3 of the Competition Act, 2002.
According to the section, it shall not be lawful for any enterprise or association of enterprises
or person or association of persons to 'enter' into an agreement in respect of production,
supply, storage, distribution, acquisition or control of goods or provision of services, which
causes or is likely to cause an appreciable adverse effect on competition within India. All such
agreements entered into in contravention of the aforesaid prohibition shall be void.

© The Institute of Chartered Accountants of India


PAPER – 6D: ECONOMIC LAWS 11

Any agreement entered into between enterprises or associations of enterpr ises or persons or
associations of persons or between any person and enterprise or practice carried on, or
decision taken by, any association of enterprises or association of persons, including cartels,
engaged in identical or similar trade of goods or provision of services, shall be presumed to
have an appreciable adverse effect on competition, in the following manner, where it—
(a) directly or indirectly determines purchase or sale prices;
(b) limits or controls production, supply, markets, technical development, investment or
provision of services;
(c) shares the market or source of production or provision of services by way of allocation of
geographical area of market, or type of goods or services, or number of customers in the
market or any other similar way;
(d) directly or indirectly results in bid rigging or collusive bidding.
Accordingly, in the given case, the agreement between them A, B,C,& D insurance companies
results in the anti-competitive agreements, and thus contravened the provisions of the
Competition Act.
Alternative Answer
“In the given case, these insurance companies had held a meeting under the auspices of IFG
on 27.12.2009 at XYZ place in the State 'Z' with the sole agenda to discuss the ‘Tender Notice
on CPP dated 1.11.2009 of the State Government 'Z', The meeting was held to discuss about
sharing of business and submission of quotation for the above business", The minutes of the
meeting of IFG signed by officials of aforementioned companies indicated that a decision was
taken ‘to share the business among the four PSUs with D Insurance Ltd. with 55% and other
Companies with 15% each …D Insurance Company will be LI and other three insurance
companies will be L-2 to L-4 in the quotation being submitted on 28th December, 2009'.
It is a fact that the decision taken by these companies in the above mentioned IFG meeting
was implemented by them. It is clear that the price quoted by these companies in their price
bids was in accordance with the decision taken in the IFG meeting held on 27.12.2009. In line
with the decision taken in the IFG meeting, D Insurance Company was the L -l bidder.
In terms of the provisions contained in section 3(1) of the Act, no enterprise or association of
enterprises or person or association of persons can enter into any agreement in respect of
production, supply, distribution, storage, acquisition or control or goods or provision of
services, which causes or is likely to cause an appreciable adverse effect on competition
within India. Section 3(2) of the Act declares that any agreement entered into in contravention
of the provisions contained in sub-section (1) shall be void.
By virtue of the presumption contained in subsection (3), any agreement entered into between
enterprises or associations of enterprises or persons or associations of persons or between
any person and enterprise or practice carried on, or decision taken by, any association of
enterprises or association of persons, including cartels, engaged in identical or similar trade of

© The Institute of Chartered Accountants of India


12 FINAL EXAMINATION: MAY 2018

goods or provision of services, which-(a) directly or indirectly determines purchase or sale


prices; (b) limits or controls production, supply, markets, technical development, investment or
provision of services; (c) shares the market or source of production or provision of se rvices by
way of allocation of geographical area of market, or type of goods or services, or number of
customers in the market or any other similar way; (d) directly or indirectly results in bid rigging
or collusive bidding, shall be presumed to have an appreciable adverse effect on competition.
It may also be pointed out that explanation appended to section 3(3) of the Act defines 'bid
rigging' as any agreement, between enterprises or persons engaged in identical or similar
production or trading of goods or provision of services; which has the effect of eliminating or
reducing competition for bids or adversely affecting or manipulating the process for bidding.
In view of the above, it is concluded that conduct of A, B, C & D Insurance Companies have
resulted in manipulation of the bidding process initiated by the State Government in
contravention of the provisions of section 3(1) read with section 3(3)( d) of the Act. In case of
agreements as listed in section 3(3) of the Act, once it is established that suc h an agreement
exists, it will be presumed that the agreement has an appreciable adverse effect on
competition; the onus to rebut the presumption would lie upon the opposite parties".
Further, the insurance companies A, B, C & D have entered into an agreement (in writing as
per the minutes of IFG meeting) to manipulate the tendering process initiated by Z State
Government/ULTRA for implementation of the scheme for the years 2010 -11, 2011-12, 2012-
13 in accordance with the provisions of section 2(b) of the Act. It is clearly and unequivocally
established. Section 2(b) of the Act defines the term 'Agreement'. Accordingly, the term
Agreement includes arrangement or understanding or action in concert (i) whether or not, such
arrangement, understanding or action is formal or in writing (ii) whether or not such
arrangement, understanding or action is intended to be enforceable by legal proceedings .”
Answer 3
To prevent abuse of Competition Act. It is advised that the following clauses be included by
the State Government “Z” to prohibits abuse of dominant position by any enterprise or group.
An enterprise or a group, does not-
(a) directly or indirectly, imposes unfair or discriminatory condition in purchase or sale of
goods or services; or price in purchase or sale (including predatory price) of goods or
service, or
(b) limits or restricts the production of goods or provision of services or market therefor; or
technical or scientific development relating to goods or services to the prejudice of
consumers; or
(c) indulges in practice or practices resulting in denial of market access in any manner; or
(d) makes conclusion of contracts subject to acceptance by other parties of supplementa ry
obligations which, by their nature or according to commercial usage, have no connection
with the subject of such contracts; or

© The Institute of Chartered Accountants of India


PAPER – 6D: ECONOMIC LAWS 13

(e) uses its dominant position in one relevant market to enter into, or protect, other relevant
market.
Answer 4
As per section 32 of the Competition Act, 2002, where-
(a) an agreement referred to in section 3 has been entered into outside India; or
(b) any party to such agreement is outside India; or
(c) any enterprise abusing the dominant position is outside India; or
(d) a combination has taken place outside India; or
(e) any party to combination is outside India; or
(f) any other matter or practice or action arising out of such agreement or dominant position
or combination is outside India;
The Commission shall, have power to inquire under the various provisions of the Act into such
agreement or abuse of dominant position or combination if such agreement or dominant
position or combination has, or is likely to have, an appreciable adverse effect on competition
in the relevant market in India and pass such orders as it may deem fit in accordance with the
provisions of this Act.
Though the agreement and the meeting of IFG took place outside India, but have an
appreciable adverse effect on competition in the relevant market in India, so the provisions of
the Competition Act are applicable.
Answer 5
Drafting of show cause notice
To,
A, B, C, & D Insurance Companies
-----------------------
New Delhi-110014
12th May, 2018
Subject: Show cause notice for entering into anti-competitive agreement or combination
of an enterprise for abusing of dominant position
The Chairman, CCI, has noticed that an agreement / combination of the A, B, C, & D
Insurance Companies in response to the tender issued by the State government (Z), for
selection of the insurance service provider for implementation of the CPP, insurance, is likely
to cause, or has caused an appreciable adverse effect on competition and abuse of dominant
provision under section 3 and 4, within the relevant market in India.

© The Institute of Chartered Accountants of India


14 FINAL EXAMINATION: MAY 2018

All the service providers as aforesaid, are required to respond within thirty days of the receipt
of the notice, as to why investigation in respect of such an agreement/combination should not
be conducted under section 29 of the Competition Act, 2002.
Chairman
CCI

Answers to Part B of Case study 1


(i) Answer (a): Are not legally enforceable
Reasoning: The oral agreements between radio taxi provider C and customers are
presumed to have an appreciable adverse effect on competition. These agreements
are void, so they are not legally enforceable.
(ii) Answer (d): Ability to operate independently of competitive forces in the relevant
market
Reasoning: Dominant position means a position of strength, enjoyed by an
enterprise, in the relevant market, in India, which enables it to —
(a) operate independently of competitive forces prevailing in the relevant market; or
(b) affect its competitors or consumers or the relevant market in its favour.
[Explanation to section 4]
(iii) Answer (c): Relevant geographic market and relevant product market
Reasoning: "Relevant Market" means the market, which may be determined by the
Commission with reference to the relevant product market or the relevant geographic market
or with reference to both the markets; [Section 2(r)]
(iv) Answer (a): If the enterprise imposes unfair or discriminatory condition in purchase or
sale of goods or service
Reasoning: According to section 4 of the Competition Act, 2002, there shall be abuse of
dominant position if an enterprise or a group, directly or indirectly, imposes unfair or
discriminatory condition in purchase or sale of goods or services; or price in purchase or
sale (including predatory price) of goods or service.
(v) Answer (d): prices its product below its cost of production with a view to reducing
competition or eliminating competitors
Reasoning: "predatory price" means the sale of goods or provision of services, at a
price which is below the cost, as may be determined by regulations, of production of
the goods or provision of services, with a view to reduce competition or eliminate the
competitors.

© The Institute of Chartered Accountants of India


PAPER – 6D: ECONOMIC LAWS 15

(vi) Answer (b): cannot be in a dominant position at the same time


Reasoning: Dominant position can be enjoyed by an enterprise, in the relevant
market, in India, which enables it to operate independently of competitive forces
prevailing in the relevant market. Therefore two enterprises cannot be in a dominant
position at the same time.
(vii) Answer (c): the enterprise does not indulge in practices resulting in denial of market
access
Reasoning: According to Section 4(2)(c) of the Competition Act, 2002, there shall be abuse
of dominant position if an enterprise or a group indulges in practice or practices resulting in
denial of market access in any manner. Therefore non indulgences in practices resulting in
denial of market access by the enterprise is not a abuse of dominance.
(viii) Answer (a): on unfounded rumours
Reasoning: Section 19 of the Competition Act, 2002, lays down the procedure for any inquiry
which can be initiated suo motu by the Commission, on receipt of a reference from the
Central Government or a State Government and on the on receipt of an information from
consumers or trade associations.
(ix) Answer (c): 35
According to section 30(3), the parties requesting for confidentiality shall file an affidavit
as specified in regulation 42 of the Competition Commission of India (General)
Regulations, 2009 stating that the conditions prescribed in regulation 35 of the
Competition Commission of India (General) Regulations, 2009 are satisfied.
(x) Answer (b): Physical characteristics or end use of goods
Reasoning: "Relevant Product Market" means a market comprising all those products or
services which are regarded as interchangeable or substitutable by the consumer, by
reason of characteristics of the products or services, their prices and intended use;
[Section 2(t)]

Case study No. 2


(A) A Corporate Insolvency Resolution process, under the Insolvency and Bankruptcy Code
2016 was initiated by M/s A Limited as a Corporate Debtor. The company was in default
to its creditors and the assets were insufficient to meet the liabilities of the company.
Attempts to resolve the insolvency of the corporate debtors failed and in the last, it was
decided to go for liquidation of the company. The balance sheet and additional
information of A Ltd. are given below:

© The Institute of Chartered Accountants of India


16 FINAL EXAMINATION: MAY 2018

Key Financial Information:


Data Amount Data Amount
(` In crore) (` In
crore)
Equity Share Capital 11,000 Land & Building 16,500
Preference Share Capital 3,800 Fixtures & Fittings 1,000
Term Loan 1,500 Stocks 640
Working Capital Loan 1,200 Debtors 550
Unsecured Financial
1,000 Other current Assets 625
Creditors
Government dues 400 Cash 175
Workman dues 240 Accumulated Losses 2,350
Employee Liability 300
Operational Creditors 2,400
21,840 21,840

Additional Information:
Creditors
(1) Term loan is secured against fixed charge on land & building and fixtures & fittings.
Bank A with an ` 800 crore term loan outstanding has first charge on the assets and
Bank B with ` 700 crore outstanding has second charge on the assets.
(2) Working capital loan is provided by Bank C and secured against a floating charge
on debtors stock of the company.
(3) Unsecured financial creditors include a Director X who owns 3% of the share capital
of M/s A Limited with an outstanding loan due to him of ` 50 crores.
Other Liabilities:
(1) Workman dues represents amount payable for the period of 24 months preceding
the liquidation commencement date.
(2) Employee liability includes ` 25 crore is outstanding for employees for a period of
12 months.
(3) Last three years of tax assessment pending total demand raised by the department
is ` 1200 crore. This has not been included in the balance sheet, but reflected as a
contingent liability only. However the liquidator has managed to get an assessment
completion certificate and agreed to a final liability of ` 300 crore.

© The Institute of Chartered Accountants of India


PAPER – 6D: ECONOMIC LAWS 17

Fixed Assets & Other Assets :


(1) Land & Building realized 70% of book value and there would be a cost of ` 175
crore in realizing the assets.
(2) Fixtures & fittings would realize 30% of book value, net of any realization cost.
Stock, debtors & other current assets would realize 65% of book value.
Other information:
(1) There was a pending insurance claim filled by the company for a quality breach by a
supplier, which was not recorded in the books. The liquidator has managed to
recover ` 150 crore from the insurance company.
(2) Lease for the office premises had a lock in period of 10 years, out of which three
years have expired. The landlord has submitted a claim of ` 120 crore for the
remaining seven years of the lease period.
(3) Based on the amount realized & distributed, the cost of liquidation is computed to
be ` 140 crores.
(4) The pending insolvency period cost was ` 80 crore, mainly including interim
funding, remuneration of the IP and other such costs as permitted under the Code.
(5) The secured creditors have decided to relinquish their security interest to the
liquidation estate and receive proceeds from the sale of the liquidation assets by the
liquidator as per provisions laid under the Insolvency and Bankruptcy Code, 2016.
You are required to find out following with reference to the relevant provisions laid under
the Insolvency and Bankruptcy Code, 2016:
Q.1 What would have been the constitution of the Committee of Creditors and what
would have been the voting share of each of the members of the committee?
(2+3 = 5 Marks)
Q.2 Total value realized by liquidator. (4 Marks)
Q.3 Order of Priority with Notes indicating the relevant section of the Code.
(8 Marks)
Q.4 You have been appointed as the Interim Resolution professional of A Ltd. Draft a
public notice as required under the Act and Regulations. (5 Marks)
Q.5 The application before NCLT was filed on 5 th January, 2018. The case was admitted
on 20th January, 2018. The IRP who was appointed on 20 th January, 2018, received
the order on the same day and issued public notice on 23 rd January, 2018 seeks
your guidance on the various time lines to be compiled with. Prepare a checklist for
his ready reference. (5 Marks)
Q.6 In the said case, assume that A Ltd. has transferred an amount of ` 500 crore to its
subsidiary abroad. The subsidiary has acquired assets for its business purposes.

© The Institute of Chartered Accountants of India


18 FINAL EXAMINATION: MAY 2018

How will you, as the liquidator treat the assets of the subsidiary and the shares held
in the subsidiary? (3 Marks)
(B) You are a Chartered Accountant specialising in FEMA related matters. You are back in
office after a short trip and your assistant has compiled all clients' queries on which your
opinion is requested. Choose the most appropriate reply and write a few lines justifying
your stance.
(i) Mr. Patel's mother requires to travel to USA for a complicated brain surgery. The
estimate given by the hospital in USA is USD 3,00,000 over and above Mr. Patel
would need USD 50,000 towards lodging boarding and other incidental expenses.
Mr. Patel had already spent USD 2,00,000 during the concerned Financial Year. Mr.
Patel can remit from India _________.
(a) USD 2,50,000
(b) USD 3,00,000
(c) USD 3,50,000
(d) USD 1,00,000
(ii) Mr. Smith is deputed to India by his company to develop a strategic software for a
period of five years from 1 st January, 2015. He is paid salary to his Indian bank
account. On 1 st May, 2017 he wants to remit his entire salaries ended till 30 th April,
2017 to his home country USA. Mr. Smith can __________.
(a) remit the salary after payment of applicable taxes and contribution to
applicable social security schemes
(b) cannot remit any amount as salary is credited to his bank account in India
(c) remit gross salary before taxes and can make payment of taxes at the year
end
(d) remit salary only upon completion of assignment after payment of taxes and
filing of Income tax return
(iii) Mr. John, an Australian citizen of non-Indian origin is engaged in construction of
farm houses in Australia. He intends to take 50% stake in an Indian company which
is engaged in construction of residential premises in Jammu. Mr. John __________.
(a) cannot make any investment in the Real Estate Sector
(b) can invest through his company in Australia
(c) can make direct investment for construction of residential premises
(d) Both (a) and (b) above
(iv) Mr. Mehra intends to return to India for good after 30 years of stay in USA.
Mr. Mehra needs to ____________.

© The Institute of Chartered Accountants of India


PAPER – 6D: ECONOMIC LAWS 19

(a) close all his bank accounts in USA and remit funds to India
(b) liquidate all his investments before returning to India
(c) bring minimum of USD 2,50,000 to India for his survival
(d) can retain his money, bank accounts, investments etc. abroad without any
restrictions
(v) Mr. Kale migrated to UK 20 year ago. He later on acquired UK citizenship. He
inherited 50 acres of agricultural land in Maharashtra which has an inbuilt Farm
House. Mr. Kale intends to gift or sell this property to his only son who has UK
citizenship, but settled in India. Mr. Kale ___________.
(a) can gift this property to his son but cannot sale it
(b) can neither gift nor sale this property to his son
(c) can sale this property to his son but cannot gift it
(d) can do both, gift as well as sale this property to his son
(vi) Mr. lyer an Indian resident acquired a residential flat in Malaysia in contravention of
FEMA regulations. Fearing actions, he intends to gift the same to his nephew Mr.
Kartik, who is a resident of India at present but will soon be migrating to Malaysia
for higher studies. Mr. Kartik _________.
(a) can acquire the flat from his uncle by way of gift
(b) cannot acquire the flat from his uncle by way of gift
(c) can acquire the flat by way of inheritance but not as a gift
(d) can acquire the flat by way of sale, gift or inheritance
(vii) M/s Charming Garments has a warehouse in Amsterdam to which goods worth
` 10 crore are exported. The firm needs to realise the proceeds of exports
_________
(a) as soon as exports are made
(b) within nine months from the date of export
(c) as soon as goods are sold or within fifteen months from the date of shipment
of goods whichever is earlier.
(d) within twelve months from the date of shipment of goods
(viii) Mr. Gotad travelled to Germany for attending a conference. He acquired USD 5,000
from his travel agent in India, out of which he saved currency notes worth USD
2,500. Upon his return to India, Mr. Gotad _________.
(a) needs to surrender USD 2,500 to his Authorised Dealer (AD) within six months
of date of return

© The Institute of Chartered Accountants of India


20 FINAL EXAMINATION: MAY 2018

(b) needs to surrender USD 2,500 to his AD within ninety days of date of return
(c) can retain USD 2,000 and surrender USD 500 within 90 days of his return to
India
(d) can retain USD 2,500 for his next trip
(ix) For any contravention of FEMA Regulations under section 13 of the Act, where the
sum involved is quantifiable, the quantum of penalty would be __________.
(a) three times of sum involved
(b) rupees two lacs only
(c) upto Rupees five thousand per day of the offence in continue
(d) Both (a) and (c) above
(x) The time limit for compounding of offences under section 13 of FEMA by the
Directorate of Enforcement is
(a) Nine months from the date of application
(b) Six months from the date of committing such contravention
(c) 180 days from the date of receipt of application by the Directorate of
Enforcement
(d) 180 days from the date of application to the Directorate of Enforcement
Answers to Part (A) of Case study 2
Answer 1
In the given case, the committee of creditors will be constituted as per section 21 of the
Insolvency and Bankruptcy Code, 2016.
The members of the committee will comprise all financial creditors excluding related party who
will not have right of representation, participation or voting in the meeting of the committee of
creditors.
Accordingly, the committee of creditors and their voting share will be as under:
S.No. Members Loan Amount Voting Share %
(Rs. Crores)
1 Bank A 800 21.92
2 Bank B 700 19.18
3 Bank C 1200 32.88
4 Unsecured unrelated financial creditors 950 26.02
3650 100

© The Institute of Chartered Accountants of India


PAPER – 6D: ECONOMIC LAWS 21

The director X who is an unsecured financial creditor with ` 50 crores, since related party of
the corporate debtor, shall not have any right of representation, participation or voting in the
committee of creditors.
Answer 2
Total Assets that can be realized by the Liquidator of M/S A Limited will be as follows:
Land & Building realized 70% of book value = ` 11,550 Crore
Less: Cost of realization = ` 175 Crore
Net value = ` 11375 Crore
Fixtures & Fittings realize 30 % of book value = ` 300 Crore
Stock, debtor & other current assets would realize 65% of book value = ` 1179.75 Crore
Insurance claim recovered by the liquidator from insurance company = ` 150 Crore
Total value realized by liquidator = ` 13,004.75 Crore.
[Note: Answer may also be given on the assumption of inclusion of amount of cash
available in the amount of total value released by liquidator. In such case total value
released will be 13,179.75 Crore].
Answer 3
Section 53 of the Code lays the provisions related to distribution of assets or the proceeds
from the sale of the liquidation assets.
Distribution of proceeds from the sale of the liquidation assets: The proceeds from the
sale of the liquidation assets shall be distributed in the following order of priority —
(a) the insolvency resolution process costs and the liquidation costs paid in full;
(b) the following debts which shall rank equally between and among the following : —
(i) workmen's dues for the period of twenty-four months preceding the liquidation
commencement date; and
(ii) debts owed to a secured creditor in the event such secured creditor has
relinquished security in the manner set out in section 52;
(c) wages and any unpaid dues owed to employees other than workmen for the period of
twelve months preceding the liquidation commencement date;
(d) financial debts owed to unsecured creditors;
(e) the following dues shall rank equally between and among the following: —
(i) any amount due to the Central Government and the State Government including the
amount to be received on account of the Consolidated Fund of India and the
Consolidated Fund of a State, if any, in respect of the whole or any part of the
period of two years preceding the liquidation commencement date;

© The Institute of Chartered Accountants of India


22 FINAL EXAMINATION: MAY 2018

(ii) debts owed to a secured creditor for any amount unpaid following the enforcement
of security interest;
(f) any remaining debts and dues;
(g) preference shareholders, if any; and
(h) equity shareholders or partners, as the case may be.
Fees to liquidator: The fees payable to the liquidator shall be deducted proportionately from
the proceeds payable to each class of recipients, and the proceeds to the relevant recipient
shall be distributed after such deduction.
Particulars Amount (` in Crores)
Value Realized by Liquidator 13,004.75
Add: Cash 175.00
Total Amount of Funds Available 13,179.75
Less: Section 53(1)(a)
insolvency resolution process costs and the liquidation costs.
(i) Cost of Liquidation 140.00
(ii) Insolvency Professional related costs* 80.00 220.00
Balance Available 12,959.75
Less: Section (53)(1)(b)
(i) Workmen's dues for the period of 24 months
preceding the liquidation commencement date 240.00
(ii) Debt owed to a secured creditors
(a) Term loans 1500.00
(b) Working capital loan 1200.00 2940.00
Balance Available 10,019.75
Less: Section(53)(1)(c)
Wages and any unpaid dues owed to employees other than 25.00
workmen for the period of twelve months preceding the
liquidation commencement date
Balance available 9994.75
Less: Section(53)(1)(d)
Financial debts owed to unsecured financial creditors 1000.00
Balance Available 8994.75
Less: Section(53)(1)(e)
Amount due to the Central Government and the State
Government

© The Institute of Chartered Accountants of India


PAPER – 6D: ECONOMIC LAWS 23

(i) Government dues 400.00


(ii) Income Tax Liability 300.00 700.00
Balance Available 8294.75
Less: Section(53)(1)(f)
(i) Employee liability (300-25) 275.00 2675.00
(ii) Operational Creditors 2400.00 5619.75
Balance Available
Less: Section(53)(1)(g)
Amount to be given to Preference Shareholders 3800.00
Balance Available 1819.75
Less: Section(53)(1)(h)
Amount to be given to Equity Shareholders 1819.75
Balance Available NIL
[Note 1: Rent claim for unexpired lease period has been considered at nil value as
based on the relevant provisions, payment of periodic nature can only be claimed till
the time order for liquidation is passed
*Note 2: It is assumed that ‘pending insolvency cost of ` 80 crores has not been paid in
full before and now being paid in full].
Answer 4
Draft Public notice to the Creditors of A Ltd., the corporate debtor is as under:
Form A
PUBLIC ANNOUNCEMENT
(Under Regulation 6 of the Insolvency and Bankruptcy Board of India (Insolvency
Resolution Process for Corporate Persons) Regulation, 2016.)
FOR THE ATTENTION OF THE CREDITORS OF A LIMITED
RELEVANT PARTICULARS
1. NAME OF CORPORATE DEBTOR A LIMITED
2. DATE OF INCORPORATION OF CORPORATE DEBTOR
3. AUTHORITY UNDER WHICH CORPORATE DEBTOR IS INCORPORATED /
REGISTERED
4. CORPORATE IDENTITY NUMBER / LIMITED LIABILITY IDENTIFICATION
NUMBER OF CORPORATE DEBTOR
5. ADDRESS OF THE REGISTERED OFFICE AND PRINCIPAL OFFICE (IF ANY) OF
CORPORATE DEBTOR
6. INSOLVENCY COMMENCEMENT DATE IN RESPECT OF CORPORATE DEBTOR

© The Institute of Chartered Accountants of India


24 FINAL EXAMINATION: MAY 2018

7. ESTIMATED DATE OF CLOSURE OF INSOLVENCY RESOLUTION PROCESS


8. NAME AND REGISTRATION NUMBER OF THE INSOLVENCY PROFESSIONAL
ACTING AS INTERIM RESOLUTION PROFESSIONAL
9. ADDRESS AND E -MAIL OF THE INTERIM RESOLUTION PROFESSIONAL, AS
REGISTERED WITH THE BOARD
10 ADDRESS AND E -MAIL TO BE USED FOR CORRESPONDENCE WITH THE
. INTERIM RESOLUTION PROFESSIONAL, IF DIFFERENT FROM THOSE GIVEN
AT SL. NO.9.
11 LAST DATE FOR SUBMISSION OF CLAIMS
.
Notice is hereby given that the National Company Law Tribunal has ordered the
commencement of a corporate insolvency resolution process against the M/S A Ltd. on ---------
-------- [insolvency commencement date].
The creditors of M/S A Ltd., are hereby called upon to submit a proof of their claims on or
before----------------- [within fourteen days from the appointment of the interim resolution
professional] to the interim resolution professional at the address mentioned against item 8.
The financial creditors shall submit their proof of claims by electronic means only. The
operational creditors including workmen and employees may submit the proof of claims by in
person, by post or by electronic means.
Submission of false or misleading proofs of claim shall attract penalties.
Name and Signature of Interim Resolution Professional:
Date and Place:
Answer 5
Checklist for ready reference of various time lines to be complied by IRP within the
Insolvency and Bankruptcy Code are:
Sl.No Process of Insolvency process Timelines
1. Filing of application before NCLT 5th January 2018
2. Admission of application 20th January, 2018
3. Appointment of Interim Resolution 20th January 2018 (within 14 days from the
Professional(IRP)- Actual date commencement date)
4. Public announcement -Actual date Uptil 23rd January, 2018(within 3 days from
the date of appointment of the Interim
Resolution Professional)
5. Collation of claims Within 14 days of the date of appointment of
Interim Resolution Professional

© The Institute of Chartered Accountants of India


PAPER – 6D: ECONOMIC LAWS 25

6. Verification of claims Within 7 date from last date of submission


of claims
7. Constitution of Committee of Immediate after verification of claims
Creditors
8. Holding first meeting of Committee Within 7 days of constitution of Committee
of Creditors of Creditors
9. Filing of report to Adjudicating Before 30 th day of appointment of IRP
Authority
10. Moratorium 180 days from the date of admission of
application i.e. 18 th July, 2018.
Answer 6
According to section 36 of the code, for the purposes of liquidation, the liquidator shall form an
estate of the assets, which will be called the liquidation estate in relation to the corporate
debtor. The liquidation estate shall comprise all liquidation estate assets whi ch shall include
any depository recording securities of the corporate debtor or by any other means as may be
specified by the Board, including shares held in any subsidiary of the corporate debtor.
However, as per the Insolvency and Bankruptcy Code, 2016, assets of any Indian or foreign
subsidiary of the corporate debtor shall not be included in the liquidation estate assets and
shall not be used for recovery in the liquidation.
So, according to the above provision, the assets of the foreign subsidiary of A Ltd., is
excluded for recovery in the liquidation.
Answers to Part (B) of Case study 2

(i) Answer (c): USD 3,50,000


Reasoning: As per Schedule III of the FEM (Current Account Transactions) Rules, 2000,
Individuals can avail of foreign exchange facility within the limit of USD 2,50,000 only. Any
additional remittance in excess of the said limit shall require prior approval of the Reserve
Bank of India. However, for the purposes of expenses in connection with medical treatment
abroad, the individual may avail of exchange facility for an amount in excess of the limit
prescribed if it is so required by a medical institute offering treatment. Mr. Patel can remit from
India 3,00,000+ 50,000= USD 3,50,000.
(ii) Answer (a): remit the salary after payment of applicable taxes and contribution to
applicable social security schemes
Reasoning: As per Schedule III of the FEM (Current Account Transactions) Rules, 2000, a
person who is resident but not permanently resident in India, who is on deputation to the
office or branch of a foreign company or subsidiary or joint venture in India of such foreign
company, may make remittance up to his net salary, after deduction of taxes, contribution to

© The Institute of Chartered Accountants of India


26 FINAL EXAMINATION: MAY 2018

provident fund and other deductions. Accordingly, Mr. Smith can remit the salary after
payment of taxes and contributions related to social security schemes.
(iii) Answer (c): can make direct investment for construction of residential premises
Reasoning: As per the FEM (Permissible Capital Account Transactions) Regulations,
2000, the person resident outside India is prohibited from making investments in India
in any form, in any company, or partnership firm or proprietary concern or any entity whether
incorporated or not which is engaged or proposes to engage in real estate business, or
construction of farm houses. In “real estate business” the term shall not include shall not
include development of townships, construction of residential /commercial premises, roads or
bridges and Real Estate Investment Trusts (REITs) registered and regulated under the SEBI
(REITs) Regulations 2014.
(iv) Answer (d): can retain his money, bank accounts, investments etc. abroad without any
restrictions
Reasoning: As per the Foreign Exchange Management (Foreign Currency Accounts by a
person resident in India) Regulations, 2015, a citizen of a foreign state resident in India may
open, hold and maintain a foreign currency account with a bank outside India.
[Note: This regulation does not form part of the study material. Correct answer given in
common parlance, may be taken into consideration]
(v) Answer (a): can gift this property to his son but cannot sale it
Reasoning: As per the FEM (Acquisition and transfer of immovable property in India)
Regulation, a person of Indian origin resident outside India may transfer any immovable
property in India other than agricultural land/farm house/plantation property, by way of sale to
a person resident in India. Since in the question it an agricultural land, so it will fall in
exception for transfer of property by the way of sale to a person resident in India.
(vi) Answer (b): cannot acquire the flat from his uncle by way of gift
Reasoning: A person resident in India may acquire immovable property outside India, a
person resident in India may acquire immovable property outside India, by way of inheritance
or gift from a person resident in India who has acquired such property in accordance with the
foreign exchange provisions in force at the time of such acquisition. Since in the given case
there was contravention of FEMA regulations, so Mr. Kartik cannot acquire the flat.
(vii) Answer (c): as soon as goods are sold or within fifteen months from the date of
shipment of goods whichever is earlier.
Reasoning: As per FEM (Export of goods and services) Regulation, the amount
representing the full export value of goods / software/ services exported shall be realised and
repatriated to India within nine months from the date of export, provided that where the
goods are exported to a warehouse established outside India with the permission of
the Reserve Bank, the amount representing the full export value of goods exported
shall be paid to the authorised dealer as soon as it is realised and in any case within

© The Institute of Chartered Accountants of India


PAPER – 6D: ECONOMIC LAWS 27

fifteen months from the date of shipment of goods.


(viii) Answer (c): can retain USD 2,000 and surrender USD 500 within 90 days of his return to
India
Reasoning: According to Foreign Exchange Management (Possession and Retention
of Foreign Currency) Regulations, 2015, a person resident in India can retain foreign
currency notes, bank notes and foreign currency traveller’s cheques not exceeding
USD 2,000 or its equivalent in aggregate, provided that such foreign exchange in the
form of currency notes, bank notes and travellers cheques represents unspent
amount of foreign exchange acquired by him from an authorised perso n for travel
abroad.
(ix) Answer (d): Both (a) and (c) above i.e., three times of sum involved and up to rupees
five thousand per day of the offence in continue
Reasoning: According to section 13 of the Foreign Exchange Management Act, 1999,
if any person contravenes any provisions of this Act, or contravenes any rule,
regulation, notification, direction or order issued in exercise of the powers under this
Act, or contravenes any condition subject to which an authorisation is issued by the
Reserve Bank, he shall, upon adjudication, be liable to a penalty up to thrice the sum
involved in such contravention where such amount is quantifiable, or up to two lakh
rupees where the amount is not quantifiable, and where such contravention is a
continuing one, further penalty which may extend to five thousand rupees for every
day after the first day which the contravention continues.
(x) Answer (c): 180 days from the date of receipt of application by the Directorate of
Enforcement
Reasoning: According to section 15 of the Foreign Exchange Management Act,
1999, any contravention under section 13 may, on an application made by the person
committing such contravention, be compounded within one hundred and eighty days
from the date of receipt of application by the Director of Enforcement or such other
officers of the Directorate of Enforcement and Officers of the Reserve Bank as may
be authorised in this behalf by the Central Government in such manner as may be
prescribed.
Case Study No. 3
Everbullish Inc. USA has a subsidiary in Singapore, namely Everbullish Steel Asia Pvt. Ltd.
(ESA) looking after the entire south east Asia, including India.
ESA has following entities operating under it.
(i) A branch in China for manufacturing of steel
(ii) A liaison office in India for marketing of steel exported by ESA directly to Indian
customers.

© The Institute of Chartered Accountants of India


28 FINAL EXAMINATION: MAY 2018

(iii) A project office in Afghanistan


(iv) A commission agent in Bangladesh
(v) A warehouse in Srilanka
ESA upgraded its Liaison Office (LO) in India to a full fledged subsidiary as 1 st April, 2016 and
transferred all its balances to the newly formed subsidiary, name Everbullish Indian Steel Pvt.
Ltd. (EISPL)
Note In each of the above situations, you are required to give relevant 'FEMA' and 'Prohibition
of Benami Property Transaction Act, 1988 and references options or steps to regularize the
contraventions, if any.
(A) ESA was advised that since it has a permission to operate as a LO till 31.3.2018, there is
no need to obtain separate approval from RBI for converting or upgrading the same into
a subsidiary. Hence No permission was taken by ESA or EISPL. Incorporation expenses
were spent by the Indian LO out of funds remitted by ESA. EISPL started local trading in
India. The LO was not closed by the ESA and no intimation was filed with RBI till

31-10-2018. (5 Marks)
Question
Are there any FEMA violations in the above transactions, and if so, then what is the way
out? (5 Marks)
(B) Sensing something wrong, EISPL decided to undergo voluntary FEMA compliance audit.
EISPL has appointed you as a FEMA auditor. In the process of audit, you discover
several transactions where FEMA regulations were not adhered to, or compliances
pending. You are required to give your expert opinion on following matters as to what are
the contraventions under FEMA and how they can be regularized?
Question 1
Receipt of Share application money from ESA amounting to ` One crore on 1 st April
2017. No compliances are made in this respect as the company was advised that
activities of the EISPL falls under the automatic route of RBI. (5 Marks)
Question 2
ESA had bought a large commercial property on 1 st January, 2016 which was then leased
to EISPL w.e.f, 1 st April 2016 and part of the premises was leased to an unrelated Indian
company w.e.f. 1 st April, 2017. (5 Marks)
Question 3
ESA had sent an adhoc amount of ` two crore to EISPL for its day to day requirements.
The funds have been received by the EISPL on 1 st January, 2018. Again no FEMA
compliances are made in this respect. (4 Marks)


date 31-10-2018 will be taken as 31.03.2018

© The Institute of Chartered Accountants of India


PAPER – 6D: ECONOMIC LAWS 29

Question 4
EISPL has exported steel worth ` 10 crore to solid steel Gmbh an unrelated German
Company on 1 st January 2017. Solid steel has run into financial trouble and therefore
refused to pay. Despite best efforts, EISPL is unable to recover the sum. The directors of
EISPL used to follow up for recovery over phone only and therefore no documentary
evidence is available.
(i) Assuming that the total exports of EISPL for the year ended 31 st March 2017 is
likely to cross ` 50 crore, can it write off this sum? (3 Marks)
(ii) Assuming that EISPL has imported steel ingots from solid steel amounting to
` 11 crore, in Dec. 2016, which is still outstanding. Can it net off and make the
payment for the balance of `1 crore only? (3 Marks)
(iii) Will your answer change if the import and export transactions would have happened
in December, 2017 and January, 2018 respectively? (2 Marks)
Question 5
EISPL remitted ` one crore to the project office of the ESA in Afganistan in February,
2018. Is it permissible? Will your answer be different if instead of money, steel worth of
` one crore is exported to the Afganistan P.O.? (3 Marks)
Question 6
EISPL exported goods to Srilanka. For that purpose it hired the warehouse of ESA and
paid warehousing charges. Is it permissible? What is the time limit for realising goods
exported by EISPL to its Srilankan Warehouse? (5 Marks)
Question 7
EISPL wants to remit commission to the agent of ESA for exports made by Bangalore.
However the Agent has requested to pay ` one crore extra, as advance to be adjusted
against future commission. Looking at the present business scenario, it may take 5 years
to adjust the advance commission paid to the Bangladesh Agent. Is it okay from FEMA
perspective? (5 Marks)
Question 8
One of the directors, of the EISPL is a person of India origin with US citizenship. He
wants to acquire a commercial premises in India and then lease it to the company. Is this
permissible under FEMA? Will your answer be different if that director is a US citizen of
non-Indian origin? (5 Marks)
Question 9
In the process of audit it is observed that one of the directors Mr. Valia of EISPL who,
recently joined company has acquired a large bunglow in Bangalore in the name of his
son who has settled in USA. He purchased the same by paying ` 10 crore. However, his
son is still studying and has not disclosed this property in his US tax returns. Upon

© The Institute of Chartered Accountants of India


30 FINAL EXAMINATION: MAY 2018

enquiry Mr. Valia’s son denies of holding any such property. What are the consequences
in this case under the provisions of the “Prohibition of Benami Property Transaction Act,
1988”. (5 Marks)
Answer (A)
According to the Foreign Exchange Management (Establishment of a Branch Office or a
Liaison office or a project office or any other place of business) Regulations, 2016:
1. Liaison Office (LO) means a place of business to act as a channel of communication
between the principal place of business or Head office or by whatever name called and
entities in India but which does not undertake any commercial/ trading / industrial activity,
directly or indirectly, and maintains itself out of inward remittances received from abroad
through normal banking channel.
2. The validity period of an LO is generally for three years, except in the case of Non -
Banking Finance Companies (NBFCs) and those entities engaged in construction and
development sectors, for whom the validity period is two years only. No further extension
would be considered for liaison offices of entities which are Non-Banking Finance
Companies and those engaged in construction and development sectors (excluding
infrastructure development companies). Upon expiry of the validity period, the offices
shall have to either close down or be converted into a Joint Venture / Wholly Owned
Subsidiary in conformity with the extant Foreign Direct Investment policy.
The question states that ESA has the permission to operate as a LO till 31.3.2018.
Hence, we can deduce that ESA must have got the permission to operate as a LO on
1.4.2016. The facts of the case study also states that ESA upgraded its LO in India to full
fledged subsidiary on 1.4.2016.
From the definition of LO, it can be inferred that trading is not included in the permissible
operation of a LO. As per the question ESA has got the permission to operate as a LO
and not as a subsidiary, hence, the decision to operate in the nature of subsidiary without
informing the concerned authority is incorrect.
In every financial year, liaison office have to submit the annual activity certificate
confirming the activities undertaken along with the Audited financial statements, including
the receipt and payment of account on or before 30 th September of the Year.
Failure to comply the above, will attract penalty as provided in the Foreign Exchange
Management Act, 1999.
[Note: The question has provided that LO is liaisoning for steel business, hence it has
been taken to be in the categories of those engaged in construction and development
sectors.]
Alternative answer
ESA was wrongly advised that it can form a subsidiary without any compliances under FEMA.
RBI grants permission for the Liaison office (LO) office for a Special duration and for specified

© The Institute of Chartered Accountants of India


PAPER – 6D: ECONOMIC LAWS 31

activities only. A LO is supposed to adhere to all names under FEMA and comply wi th
conditions mentioned in the permission from RBI.
So, ESA needs to set right things as follows:
(i) Intimate RBI about closure of LO and transfer of all its assets and liabilities in the new
formed subsidiary EISPL.
(ii) File all pending returns of LO with the Income tax authority and audited accounts with
ROC as well as activity certificate under FEMA with its authorized dealer for the onward
submission to RBI.
(iii) Spending funds on incorporation of a company by a LO is in violation of conditions
attached to the activities of the LO and utilization of funds.
(iv) It is given that EISPL started local trading in India. EISPL can do local trading only in
respect of Cash & Carry wholesale Trading under automatic route of RBI. For any other
category it requires prior approval of RBI.
(v) For various offences/contravention mentioned above ESA needs to approach RBI for
compounding of offences. EISPL shall ensure that its activities remain within the purview
of FEMA reputations. For allotment of share to ESA against the balances transferred from
the LO as well as incorporation expenses. If EISPL intends to remit fund to ESA instead
of allotments of shares, they still it needs to obtain RBI approval.
Answer B.
Answer 1
According to Schedule II to the Foreign Exchange Management (Permissible Capital Account
Transactions) Regulations, 2000, investment in India by a person resident outside India,
through issue of securities by a body corporate or an entity in India and investment therein by
a person resident outside India, is a permissible transaction.
Further, according to the Master Directions on Foreign Investment in India-
An Indian company issuing shares /convertible debentures under FDI Scheme to a person
resident outside India shall receive the amount of consideration required to be paid for
such shares /convertible debentures by:
(i) inward remittance through normal banking channels.
(ii) debit to NRE / FCNR account of a person concerned maintained with an AD
category I bank.
(iii) conversion of royalty / lump sum / technical know how fee due for payment /import of
capital goods by units in SEZ or conversion of ECB, shall be treated as consideration
for issue of shares.

© The Institute of Chartered Accountants of India


32 FINAL EXAMINATION: MAY 2018

(iv) conversion of import payables / pre incorporation expenses / share swap can be
treated as consideration for issue of shares with the approval of FIPB. (Now Line
Ministry as FIPB is abolished on 17 th April, 2017.)
(v) debit to non-interest bearing Escrow account in Indian Rupees in India which is
opened with the approval from AD Category – I bank and is maintained with the AD
Category I bank on behalf of residents and non-residents towards payment of share
purchase consideration.
If the shares or convertible debentures are not issued within 180 days from the date
of receipt of the inward remittance or date of debit to NRE / FCNR(B) / Escrow
account, the amount of consideration shall be refunded.
It can be regularized on an application filed to RBI where amount outstanding
towards issue of security is beyond the period of 180 days from the date of receipt.
Alternative answer
Compliances on Shares allotment
Two stages compliance is required in respect of receipt of funds and allotment of shares
under FEMA:
(i) Form ARF needs to be submitted to the authorized dealer AD bank of the company within
30 days of receipt of remittance towards equity shares KYC and Foreign Inward
remittance certificate (FIRC) need to be submitted alongwith form ARF.
(ii) Form ECGPR needs to be filed with AD bank within 30 days of allotment of shares. This
form should be certified by a company Secretary certifying all compliances under the
Companies Act, 2013 and a valuation certificate from a Chartered Accountant certifying
the valuation of shares as per the pricing guidelines under FEMA.
(iii) The FEMA regulation provides that the allotment of shares needs to be com pleted within
six months of the receipt of funds. Under the companies Act, the shares needs to be
allotted in 3 months. Since share are not allotted within the time frame nor intimation
filed, therefore EISPL need to obtain RBI permission for allotment of shares and apply for
compounding of office.
(iv) It may be noted that automatic route of RBI is available only in respect of compliances
made within the prescribed time frame.
Answer 2
According to the Foreign Exchange Management (Acquisition & Transfer o f Immovable
Property in India) Regulations, 2000, provides that a person resident outside India cannot
lease/ rent any part of the property acquired by him.

© The Institute of Chartered Accountants of India


PAPER – 6D: ECONOMIC LAWS 33

Hence, ESA cannot lease the said commercial property to EISPL & to an unrelated Indian
Company.
Alternative answer
Foreign companies are allowed buy immovable property in India for the purpose of carrying on
its own business. Form IP is to be filed with RBI for intimating the purchase of property.
However, remittance of sale proceed needs prior approval of RBI. If ESA has ceased its
activities as LO, it cannot continue to hold and lease property to others. Recently RBI has
permitted to lease additional place to related enterprises.
Under the circumstances, ESA needs to regularize the leasing of premises to EISPL. As
LO cannot earn any income in India, a question would arise for the leasing income.
Step to be taken by ESA
(i) Approach RBI with facts of the case
(ii) Obtain specific approval for lease of premises or sale its subsidiary EISPL.
(iii) Apply for compounding of offence as per advice from RBI.
Answer 3
Schedule I to the Foreign Exchange Management (Permissible Capital Account
Transactions) Regulations, 2000, allows loans and overdrafts (borrowing) by a person
resident in India from a person outside India subject to the compliance of guidelines
issued by RBI in this regard. Hence, ESA is advised to comply with the Newspaper
guidelines.
Alternative answer
EISPL has received on adhoc amount of ` 2 crore for its day to day requirements for ESA
on 1 st January, 2018.
EISPL can take external commercial borrowing (ECB) from its parent company subject to
conditions prescribe in the ECB regulations.
However, any loan under ECB regulations can be drawn only after obtaining loan
Registrations Number (i.e. LRN). In the instance case EISPL has already received the
funds from ESA on 1st January, 2018. It would be better to treat these funds towards
subscription of compulsory convertible debentures (CCDs). Authorized Dealer Bank may
be approached for necessary changes in the FIRC. ESA & EISPL can pass necessary
resolution in this behalf. CCDs are treated at par with equity shares. Hence, EISPL needs
to comply with necessary formalities under the Foreign Direct Investment (FDI)
regulations.
RBI may levy nominal compounding fees for delay in intimation of receipt of funds.
Answer 4
(i) Section 7 of FEMA deals with provisions of Export of Goods and Services.

© The Institute of Chartered Accountants of India


34 FINAL EXAMINATION: MAY 2018

It is the duty of the exporter to see that foreign exchange is realized within the
prescribed time limit. The normal time limit for realization of exports is nine months
from the date of export. If for any reason export proceeds are not realized in time,
the AD/RBI bank may be informed and requested to external the time limit.
As per Master Direction – Export of Goods and Services;
An exporter who has not been able to realize the outstanding export dues despite
best efforts, may either self-write off or approach the AD Category – I banks, who
had handled the relevant shipping documents, with appropriate supporting
documentary evidence. The limits prescribed for write-offs of unrealized export bills
are as under:
Self “write off” by an exporter (Other than status holder exporter) 5%*
Self “write off” by status holder exporter 10%*
“Write off” by AD Bank 10%*
* of the total export proceeds realized during the previous calendar year
The above limits will be related to total export proceeds realized during the previous
calendar year and will be cumulatively available in a year.
Thus, EISPL can write off the amount to the extent as prescribed in the above provisions.
Alternative answer
Section 7 of FEMA deals with provisions of Export of Goods and Services.
It is the duty of the exporter to see that foreign exchange is realized within the
prescribed time limit. The normal time limit for realization of exports is nine months
from the date of export. If for any reason export proceeds are not realized in time,
the AD/RBI bank may be informed and requested to extend the time limit.
Exporter needs to main robust documentations of steps taken to realize the
outstanding dues. In the instant case the directions followed up for payment only
over phone and therefore would land up in trouble as they will not be able to prove
that all reasonable efforts were put in to realize the export proceeds. Under the
circumstances, the company may face stringent actions from the Enforcement
Directorate.
Self-write off of exports is permitted upto 10% of the average annual realization of
exports in past 3 years subject to fulfilment of certain other conditions. As EISPL
does not fall into this category specific approval from RBI is advisable.
(ii) As per Master Direction – Export of Goods and Services;
EISPL can set off the amount and make payment for 1 crore only by following the
conditions:

© The Institute of Chartered Accountants of India


PAPER – 6D: ECONOMIC LAWS 35

AD category –I banks may deal with the cases of set-off of export receivables against
import payables, subject to following terms and conditions:
(1) The import is as per the Foreign Trade Policy in force.
(2) Invoices/Bills of Lading/Airway Bills and Exchange Control copies of Bills of
Entry for home consumption have been submitted by the importer to the
Authorized Dealer bank.
(3) Payment for the import is still outstanding in the books of the importer.
(4) Both the transactions of sale and purchase may be reported separately in R -
Returns and FETERS (Foreign Exchange Transactions- Electronic Reporting
System).
(5) The relative EDF (Export Declaration Form) will be released by the AD bank
only after the entire export proceeds are adjusted / received.
(6) The set-off of export receivables against import payments should be in respect
of the same overseas buyer and supplier and that consent for set -off has been
obtained from him.
(7) The export / import transactions with ACU countries should be kept outside the
arrangement.
(8) All the relevant documents are submitted to the concerned AD bank who should
comply with all the regulatory requirements relating to the transactions.
Alternative answer
Netting off export of goods receivable and import payable from same party is
permitted under the automatic route, provided the outstanding amounts are within the
time frame prescribed in FEMA. In this case both are overdue and hence specific
approval from RBI would be required.
(iii) The position as stated above in part (ii) will not change even if the import and export
transactions would have happened in December 2017 and January 2018 respectively.
Answer 5
According to Foreign Exchange Management (Export and import of currency) Regulations,
2015, any person resident in India may take outside India (other than to Nepal and Bhutan)
currency notes of Government of India and Reserve Bank of India up to an amount not
exceeding `25,000 (Rupees twenty five thousand only).
Hence, EISPL cannot remit amount of ` 1 crore to the project office of ESA in Afghanistan.
However, EISPL can export steel worth ` 1 crore to project office of ESA in Afghanistan (by
following the guidelines as issued by RBI).

© The Institute of Chartered Accountants of India


36 FINAL EXAMINATION: MAY 2018

Answer 6
Remittance of funds for the warehouse rent falls within the current account transactions and
therefore EISPL can freely remit warehouse charges to Sri Lanka.
According to Foreign Exchange Management (Export of Goods and Services) Regulations,
2015, where goods are exported to a warehouse established outside India with the permission
of the Reserve Bank of India, the amount representing the full export value of goods exported
shall be paid to the authorised dealer as soon as it is realised and in any case within fifteen
months from the date of shipment of goods;
The Reserve Bank of India, or subject to the directions issued by that Bank in this behalf, the
authorised dealer may, for a sufficient and reasonable cause shown, extend the period of
fifteen months.
Hence, EISPL can send goods to the warehouse in Sri Lanka. Also, the amount representing
the full export value of goods exported shall be paid to the authorised dealer as soon as it is
realised and in any case within fifteen months from the date of shipment of goods. However,
this period can be extended as mentioned above.
Answer 7
Payment of export commission to an overseas agent is a current account transaction and
hence freely permitted. However, payment of advance commission, lasting for five years
would be regarded as capital account transaction and therefore would require prior approval of
RBI.
EISPL is well advised to approach RBI for remitting advance commission which is in the
nature of loan.
Alternative answer
The Foreign Exchange Management Act, 1999 does not provide for a prohibition for
payment of commission to an agent provided it does not exceed 12.50% of the invoice
value. Hence, EISPL can remit commission to agent of ESA for exports made by
Bangladesh within the above limit.
In view of above, the request to pay ` One crore extra, as advance to be adjusted against
future commission cannot be accepted and is not okay from FEMA perspective as the relative
shipment has not been made.
Answer 8
According to Acquisition and transfer of immovable property in India, Regulations,
A person of Indian origin and resident outside India may acquire immovable property in India
other than an agricultural property, plantation, or a farm house:
Provided that in case of acquisition of immovable property, payment of purchase price, if any,
shall be made out of (i) funds received in India through normal banking channels by way of

© The Institute of Chartered Accountants of India


PAPER – 6D: ECONOMIC LAWS 37

inward remittance from any place outside India or (ii) funds held in a ny non-resident account
maintained in accordance with the provisions of the Act and the regulations made by the
Reserve Bank of India:
Provided further that no payment of purchase price for acquisition of immovable property shall
be made either by traveller’s cheque or by currency notes of any foreign country or any mode
other than those specifically permitted by this clause.
Thus, in the given situation, the said director who is a person of Indian origin with US
citizenship can acquire the commercial premises in India.
According to section 6(3) of the Foreign Exchange Management Act, 1999, a person
resident outside India can acquire or transfer the immovable property in India, other than
a lease not exceeding five years. Thus, the director can lease the sai d commercial
premises but not for a period exceeding 5 years.
If the director would have been a US citizen of non Indian origin then he will not be
allowed to acquire the property in India.
Answer 9
In the given instant, a director Mr. Valia of EISPL has acquired bungalow in Bangalore in the
name of his son who has settled in USA. Upon enquiry Mr. Valia's son denies of holding any
such property and has also not disclosed in his US tax returns.
The given situation falls within the purview of section 2(9) of the Prohibition of benami
Property Transaction Act, 1988. According to the section benami transaction "means a
transaction or an arrangement (a) where a property is transferred to, or is held by, a person,
and the consideration for such property has been provided, or paid by, another person; and
(b) the property is held for the immediate or future benefit, direct or indirect, of the person who
has provided the consideration.
As per the exception to the above clause, Mr. Valia can hold the property in the n ame of his
son provided the consideration is paid out of the known sources of the Mr. Valia. This source
is also not disclosed so it is assumed that it is an unauthorized source.
Further, on enquiry, denial of Mr. Valia's Son of holding of any such property, is known in
respect of such property, as a benami transaction.
As of consequential holding of benami transactions, section 3 states that no person shall enter
into any benami transaction. Whoever enters into any benami transaction shall be punishable
with imprisonment for a term which may extend to three years or with fine or with both.
So, Mr. Valia shall be liable under the Prohibition of Benami Property Transaction Act, 1988.

© The Institute of Chartered Accountants of India

You might also like